CAT Previous Year Paper Session-II 2019

CAT 2019 Session-II 

Verbal Ability 

Instructions 

Comprehension: 

For two years, I tracked down dozens of . . . Chinese in Upper Egypt [who were] selling lingerie. In a deeply conservative region, where Egyptian families rarely allow women to work or own businesses, the Chinese flourished because of their status as outsiders. They didn’t gossip, and they kept their opinions to themselves. In a New Yorker article entitled “Learning to Speak Lingerie,” I described the Chinese use of Arabic as another non-threatening characteristic. I wrote, “Unlike Mandarin, Arabic is inflected for gender, and Chinese dealers, who learn the language strictly by ear, often pick up speech patterns from female customers. I’ve come to think of it as the lingerie dialect, and there’s something disarming about these Chinese men speaking in the feminine voice.” . . . 

When I wrote about the Chinese in the New Yorker, most readers seemed to appreciate the unusual perspective. But as I often find with topics that involve the Middle East, some people had trouble getting past the black-and-white quality of a byline. “This piece is so orientalist I don’t know what to do,” Aisha Gani, a reporter who worked at The Guardian, tweeted. Another colleague at the British paper, Iman Amrani, agreed: “I wouldn’t have minded an article on the subject written by an Egyptian woman—probably would have had better insight.” . . . 

As an MOL (man of language), I also take issue with this kind of essentialism. Empathy and understanding are not inherited traits, and they are not strictly tied to gender and race. An individual who wrestles with a difficult language can learn to be more sympathetic to outsiders and open to different experiences of the world. This learning process— the embarrassments, the frustrations, the gradual sense of understanding and connection—is invariably transformative. In Upper Egypt, the Chinese experience of struggling to learn Arabic and local culture had made them much more thoughtful. In the same way, I was interested in their lives not because of some kind of voyeurism, but because I had also experienced Egypt and Arabic as an outsider. And both the Chinese and the Egyptians welcomed me because I spoke their languages. My identity as a white male was far less important than my ability to communicate. 

And that easily lobbed word—“Orientalist”—hardly captures the complexity of our interactions. What exactly is the dynamic when a man from Missouri observes a Zhejiang native selling lingerie to an Upper Egyptian woman? . . . If all of us now stand beside the same river, speaking in ways we all understand, who’s looking east and who’s looking west? Which way is Oriental? 

For all of our current interest in identity politics, there’s no corresponding sense of identity linguistics. You are what you speak—the words that run throughout your mind are at least as fundamental to your selfhood as is your ethnicity or your gender. And sometimes it’s healthy 

to consider human characteristics that are not inborn, rigid, and outwardly defined. After all, you can always learn another language and change who you are. 

Q. 1 According to the passage, which of the following is not responsible for language’s ability to change us? 

A The ups and downs involved in the course of learning a language. 

B Language’s ability to mediate the impact of identity markers one is born with. 

C The twists and turns in the evolution of language over time. 

D Language’s intrinsic connection to our notions of self and identity. 

Answer: C 

Explanation: 

” This learning process—the embarrassments, the frustrations, the gradual sense of understanding and connection—is invariably transformative.” From this sentence, the option A can be inferred. Hence it is incorrect. 

” After all, you can always learn another language and change who you are.” From this line, option B can be inferred. Hence it is incorrect. 

“You are what you speak—the words that run throughout your mind are at least as fundamental to your selfhood as is your ethnicity or your gender” From this option D can be inferred. Hence it is incorrect. 

The author makes no mention about the inherent ability of language to evolve over time to change a person. Hence, it is not responsible for language’s ability to change us. Option C is the correct answer. 

 

Q. 2 A French ethnographer decides to study the culture of a Nigerian tribe. Which of the following is most likely to be the view of the author of the passage? 

A The author would discourage the ethnographer from conducting the study as Nigerian ethnographers can better understand the tribe. 

B The author would encourage the ethnographer, but ask him/her to first learn the language of the Nigerian tribe s/he wishes to study. 

C The author would encourage the ethnographer, but ask him/her to be mindful of his/her racial and gender identity in the process. 

D The author would encourage the ethnographer and recommend him/her to hire a good translator for the purpose of holding interviews. 

Answer: B 

Explanation: 

The author is of the opinion that learning the language of local cultures would help bridge cultural barriers. Option D is against the author’s point of view. Hence it is definitely incorrect. 

Option A is incorrect. The author is of the opinion that the ability to communicate is far more important than the racial divide between two people. Hence it is unlikely to be the view of the author. 

Option C is incorrect as the author, in the passage, is much more concerned about the ability to communicate that racial and gender identity of the person. 

Option B falls in line with the viewpoint of the author. Hence it is the correct answer. 

 

Q. 3 Which of the following can be inferred from the author’s claim, “Which way is Oriental?” 

A Globalisation has mitigated cultural hierarchies and barriers. 

B Orientalism is a discourse of the past, from colonial times, rarely visible today. 

C Goodwill alone mitigates cultural hierarchies and barriers. 

D Learning another language can mitigate cultural hierarchies and barriers. 

Answer: D 

Explanation: 

“And that easily lobbed word—“Orientalist”—hardly captures the complexity of our interactions. What exactly is the dynamic when a man from Missouri observes a Zhejiang native selling lingerie to an Upper Egyptian woman? . . . If all of us now stand beside the same river, speaking in ways we all understand, who’s looking east and who’s looking west? Which way is Oriental?” 

From the above passage, it is clear that the author consider the word Orientalist an easily lobbied word that does not capture the complex nature of interactions between people of different cultures. The author is of the opinion that if people in different parts of the world all speak in tongues that all of them understand, then the east west divide would be broken. 

The author is of the opinion that learning new languages would help bridge the east west divide. There is no information provided in the passage that globalization has enabled people learn more languages and thereby mitigated cultural hierarchies and barriers. Hence, option A is incorrect. 

Option B is incorrect. The author never makes the claim that Orientalism has disappeared for the most part. The author makes no claim about goodwill. Hence option C is incorrect. 

Option D correctly encapsulates the arguments made by the author. Hence it is the correct answer.

 

Q. 4 The author’s critics would argue that: 

A Language is insufficient to bridge cultural barriers. 

B Empathy can overcome identity politics. 

C Linguistic politics can be erased. 

D Orientalism cannot be practiced by Egyptians. 

Answer: A 

Explanation: 

The major idea put forth by the author is that cultural barriers can be broken down and an outsider can ingrain himself with the local culture by learning the language of the culture. 

The author himself says that an individual who wrestles with a difficult language would learn to be more sympathetic to outsiders. He also says that empathy is not tied to gender and race, and therefore an individual who learns languages is usually empathetic to different races in the world. Thus option B can be inferred from the passage and is incorrect. 

The passage makes no mention of linguistic politics. Also he is of the opinion that a person’s characteristics can be changed for the good by learning another language. Hence option C can be inferred from the author’s argument and is incorrect. 

The word orientalism itself means looking down upon middle eastern countries by the US and European countries. Hence, option D does not make sense. 

Option A is directly in conflict with the author’s main point and that would be the major criticism by the author’s critics. Hence it is the correct answer. 

 

Instructions 

Comprehension: 

British colonial policy . . . went through two policy phases, or at least there were two strategies between which its policies actually oscillated, sometimes to its great advantage. At first, the new colonial apparatus exercised caution, and occupied India by a mix of military power and subtle diplomacy, the high ground in the middle of the circle of circles. This, however, pushed them into contradictions. For, whatever their sense of the strangeness of the country and the thinness of colonial presence, the British colonial state represented the great conquering discourse of Enlightenment rationalism, entering India precisely at the moment of its greatest unchecked arrogance. As inheritors and representatives of this discourse, which carried everything before it, this colonial state could hardly adopt for long such a self-denying attitude. It had restructured everything in Europe—the productive system, the political regimes, the moral and cognitive orders—and would do the same in India, particularly as some empirically inclined theorists of that generation considered the colonies a massive laboratory of utilitarian or other theoretical experiments. Consequently, the colonial state could not settle simply for eminence at the cost of its marginality; it began to take initiatives to introduce the logic of modernity into Indian society. But this modernity did not enter a passive society. 

Sometimes, its initiatives were resisted by pre-existing structural forms. At times, there was a more direct form of collective resistance. Therefore the map of continuity and discontinuity that this state left behind at the time of independence was rather complex and has to be traced with care. 

Most significantly, of course, initiatives for . . . modernity came to assume an external character. The acceptance of modernity came to be connected, ineradicably, with subjection. This again points to two different problems, one theoretical, the other political. 

Theoretically, because modernity was externally introduced, it is explanatorily unhelpful to apply the logical format of the ‘transition process’ to this pattern of change. Such a logical format would be wrong on two counts. First, however subtly, it would imply that what was proposed to be built was something like European capitalism. (And, in any case, historians have forcefully argued that what it was to replace was not like feudalism, with or without modificatory adjectives.) But, more fundamentally, the logical structure of endogenous change does not apply here. 

Here transformation agendas attack as an external force. This externality is not something that can be casually mentioned and forgotten. It is inscribed on every move, every object, every proposal, every legislative act, each line of causality. It comes to be marked on the epoch itself. This repetitive emphasis on externality should not be seen as a nationalist initiative that is so well rehearsed in Indian social science. . . . 

Quite apart from the externality of the entire historical proposal of modernity, some of its contents were remarkable. . . . Economic reforms, or rather alterations . . . did not foreshadow the construction of a classical capitalist economy, with its necessary emphasis on extractive and transport sectors. What happened was the creation of a degenerate version of capitalism —what early dependency theorists called the ‘development of underdevelopment’. 

 

Q. 5 “Consequently, the colonial state could not settle simply for eminence at the cost of its marginality; it began to take initiatives to introduce the logic of modernity into Indian society.” Which of the following best captures the sense of this statement? 

A The cost of the colonial state’s eminence was not settled; therefore, it took the initiative of introducing modernity into Indian society. 

B The colonial enterprise was a costly one; so to justify the cost it began to take initiatives to introduce the logic of modernity into Indian society. 

C The colonial state’s eminence was unsettled by its marginal position; therefore, it developed Indian society by modernising it. 

D The colonial state felt marginalised from Indian society because of its own modernity; therefore, it sought to address that marginalisation by bringing its modernity to change Indian society. 

Answer: D 

Explanation: 

From the passage it can be inferred that though the British enjoyed political eminence in India, they felt that they were still marginalised from Indian society, and hence, to bring the Indian state to the same footing, they sought to introduce modernity, which they felt was the next logical step into Indian society. 

It cannot be inferred from the passage that the British introduced modernity because they believed that the cost of their eminence was not settled. Hence, option A is incorrect. 

The colonial enterprise tried to introduce the logic of modernity because it felt marginalized, rather than to justify the cost of colonization. Hence option B is incorrect. 

Option C states that the introduction of modernity developed Indian society. However, the last paragraph states that the exercise proved counterproductive, and there was a development of underdevelopment. Option C is incorrect. 

Option D best explains the reason for the author introducing the statement mentioned in the question. Hence, option D is the correct answer. 

 

Q. 6 All of the following statements, if true, could be seen as supporting the arguments in the passage, EXCEPT: 

A throughout the history of colonial conquest, natives have often been experimented on by the colonisers. B modernity was imposed upon India by the British and, therefore, led to underdevelopment. C the change in British colonial policy was induced by resistance to modernity in Indian society. 

D the introduction of capitalism in India was not through the transformation of feudalism, as happened in Europe. 

Answer: C 

Explanation: 

“…..empirically inclined theorists of that generation considered the colonies a massive laboratory of utilitarian or other theoretical experiments.” From the aforementioned lines, option A can be inferred. 

“What happened was the creation of a degenerate version of capitalism —what early dependency theorists called the 

‘development of underdevelopment’.” From these lines it can be inferred that, because modernity was imposed upon India by the British, it led to the development of underdevelopment. Option B can be inferred. 

From the passage, it can be understood that feudalism underwent a transformative process into capitalism, unlike the Indian transition which happened inorganically through external factors. Hence, option D can be inferred as well. 

The change in British colonial policy was not induced by resistance to modernity in Indian society, but due to the perception that the British were marginalised in the context of the Indian society. Hence, option C, which cannot be inferred is the correct answer. 

 

Q. 7 All of the following statements about British colonialism can be inferred from the first paragraph, EXCEPT that it: 

A allowed the treatment of colonies as experimental sites. 

B faced resistance from existing structural forms of Indian modernity. 

C was at least partly an outcome of Enlightenment rationalism. 

D was at least partly shaped by the project of European modernity. 

Answer: B 

Explanation: 

“…..empirically inclined theorists of that generation considered the colonies a massive laboratory of utilitarian or other theoretical experiments” From these lines option A can be inferred. 

Consider the lines, “e British colonial state represented the great conquering discourse of Enlightenment rationalism, entering India precisely at the moment of its greatest unchecked arrogance. . As inheritors and representatives of this discourse, which carried everything before it, this colonial state could hardly adopt for long such a self-denying attitude.” Option C can be inferred from it. 

Consider the lines , ” It had restructured everything in Europe—the productive system, the political regimes, the moral and cognitive orders—and would do the same in India, ” Option D can be inferred from these lines. 

It is nowhere mentioned in the passage, that British colonialism faces resistence from the existing structural forms of Indian modernity. Hence , option B is the correct answer. 

 

Q. 8 Which one of the following 5-word sequences best captures the flow of the arguments in the passage? 

A Colonial policy — Enlightenment—external modernity—subjection — underdevelopment. 

B Military power—colonialism—restructuring—feudalism—capitalism. 

C Military power—arrogance—laboratory—modernity—capitalism. 

D Colonial policy—arrogant rationality—resistance—independence—development. 

Answer: A 

Explanation: 

The first part of the passage talks about British colonial policy, which went through two policy phases. Hence, the options B and C which have military power as the introductory idea are incorrect. 

The second idea mentioned in the passage is about Enlightenment rationalism, of which the British colonizers were inheritors and representatives of. 

The subsequent ideas are about how modernity was inorganically injected into India by subjecting it to external forces. The passage further talks about how these economic alterations did not give rise to the construction of a classical capitalist economy, but rather led to the development of underdevelopment. 

Option A mentions all the ideas correctly and hence it is the correct answer. 

 

Q. 9 Which of the following observations is a valid conclusion to draw from the author’s statement that “the logical structure of endogenous change does not apply here. Here transformation agendas attack as an external force”? 

A Colonised societies cannot be changed through logic; they need to be transformed with external force. 

B The transformation of Indian society did not happen organically, but was forced by colonial agendas. 

C The endogenous logic of colonialism can only bring change if it attacks and transforms external forces. 

D Indian society is not endogamous; it is more accurately characterised as aggressively exogamous. 

Answer: B 

Explanation: 

“. Theoretically,because modernity was externally introduced, it is explanatorily unhelpful to apply the logical format of the ‘transition process’ to this pattern of change.” 

From the given lines it can be understood that the general endogeneous method of the process of transition could not be accepted to British colonisation of India, because modernity did not occur naturally but was externally introduced. 

The passage only states that initiatives for modernity were introduced to India through external sources. It does not state that all colonised societies cannot be changed by logic. Hence this option is incorrect. 

In the case of India, the transformational agents themselves are inorganic external forces. Hence, option C cannot be inferred. 

The passage nowhere states that Indian society is exogamous. Hence option D is incorrect. Option B best describes the conclusion that can be drawn from the author’s statement. Hence it is the correct answer. 

 

Instructions 

Comprehension: 

Around the world, capital cities are disgorging bureaucrats. In the post-colonial fervour of the 20th century, coastal capitals picked by trade-focused empires were spurned for “regionally neutral” new ones . . . . But decamping wholesale is costly and unpopular; governments these days prefer piecemeal dispersal. The trend reflects how the world has changed. In past eras, when information travelled at a snail’s pace, civil servants had to cluster together. But now desk-workers can ping emails and video-chat around the world. Travel for face-to-face meetings may be unavoidable, but transport links, too, have improved. . . . 

Proponents of moving civil servants around promise countless benefits. It disperses the risk that a terrorist attack or natural disaster will cripple an entire government. Wonks in the sticks will be inspired by new ideas that walled-off capitals cannot conjure up. Autonomous regulators perform best far from the pressure and lobbying of the big city. Some even hail a cure for ascendant cynicism and populism. The unloved bureaucrats of faraway capitals will become as popular as firefighters once they mix with regular folk. 

Beyond these sunny visions, dispersing central-government functions usually has three specific aims: to improve the lives of both civil servants and those living in clogged capitals; to save money; and to redress regional imbalances. The trouble is that these goals are not always realised. 

The first aim—improving living conditions—has a long pedigree. After the second world war Britain moved thousands of civil servants to “agreeable English country towns” as London was rebuilt. But swapping the capital for somewhere smaller is not always agreeable. Attrition 

rates can exceed 80%. . . . The second reason to pack bureaucrats off is to save money. Office space costs far more in capitals. . . . Agencies that are moved elsewhere can often recruit better workers on lower salaries than in capitals, where well-paying multinationals mop up talent. 

The third reason to shift is to rebalance regional inequality. . . . Norway treats federal jobs as a resource every region deserves to enjoy, like profits from oil. Where government jobs go, private ones follow. . . . Sometimes the aim is to fulfil the potential of a country’s second-tier cities. Unlike poor, remote places, bigger cities can make the most of relocated government agencies, linking them to local universities and businesses and supplying a better-educated workforce. The decision in 1946 to set up America’s Centres for Disease Control in Atlanta rather than Washington, D.C., has transformed the city into a hub for health-sector research and business. 

The dilemma is obvious. Pick small, poor towns, and areas of high unemployment get new jobs, but it is hard to attract the most qualified workers; opt for larger cities with infrastructure and better-qualified residents, and the country’s most deprived areas see little benefit. . . 

Others contend that decentralisation begets corruption by making government agencies less accountable. . . . A study in America found that state-government corruption is worse when the state capital is isolated—journalists, who tend to live in the bigger cities, become less watchful of those in power. 

 

Q. 10 According to the passage, colonial powers located their capitals: 

A based on political expediency. 

B to promote their trading interests. 

C where they had the densest populations. 

D to showcase their power and prestige. 

Answer: B 

Explanation: 

“In the post-colonial fervour of the 20th century, coastal capitals picked by trade-focused empires were spurned for “regionally neutral” new ones”. 

From these lines, it can be inferred that the colonial empires had their capitals in the coasts as the empires were mostly focused on trade. It goes on to say that, post – colonisation, empires had their capitals changed to regionally neutral areas. 

Hence, it can be directly inferred that colonies had capitals in coasts to promote their trading interests. Option B is the correct answer. 

 

Q. 11 According to the author, relocating government agencies has not always been a success for all of the following reasons EXCEPT: 

A a rise in pollution levels and congestion in the new locations. 

B the difficulty of attracting talented, well-skilled people in more remote areas. 

C increased avenues of corruption away from the capital city. 

D high staff losses, as people may not be prepared to move to smaller towns. 

Answer: A 

Explanation: 

Option B is a problem of relocating government agencies and it can be inferred from this line ” Pick small, poor towns, and areas of high unemployment get new jobs, but it is hard to attract the most qualified workers”. Hence, option B is incorrect. 

Option C is true with respect to the passage. It can be inferred from the line ” Others contend that decentralisation begets corruption by making government agencies less accountable .” Hence it is incorrect. 

Option D is also mentioned in the passage. Qualified workers do not want to live in smaller cities. Hence, it is also a reason for relocation not being a success. 

Option A is not mentioned in the passage and hence it is the correct answer. 

 

Q. 12 The “long pedigree” of the aim to shift civil servants to improve their living standards implies that this move: 

A has become common practice in several countries worldwide. 

B is supported by politicians and the ruling elites. 

C takes a long time to achieve its intended outcomes. 

D is not a new idea and has been tried in the past. 

Answer: D 

Explanation: 

The word pedigree has a meaning, ” history of an idea or an activity”. The term long pedigree indicates that the idea has been touted with a lot of times in the past. 

Option D is the only option that conveys this meaning and hence it is the correct answer. 

 

Q. 13 People who support decentralising central government functions are LEAST likely to cite which of the following reasons for their view? 

A It could weaken the nexus between bureaucrats and media in the capital. 

B More independence could be enjoyed by regulatory bodies located away from political centres. 

C Policy makers may benefit from fresh thinking in a new environment. 

D It reduces expenses as infrastructure costs and salaries are lower in smaller cities. 

Answer: A 

Explanation: 

The passage states that regulators perform best if they are far from the lobbying of a big city. Hence, the people who support decentralizing central government functions are likely to cite the above reason for their view. Option B and C are incorrect for this reason. 

Option D is incorrect as the passage states that infrastructure costs and salaries would become lower in smaller cities. The argument is used in the passage. Hence it is correct. 

The nexus between bureaucrats and media is not mentioned in the passage. Hence the argument is least likely to be used by people who support the decentralising of central government functions. 

Option A is the correct answer. 

 

Q. 14 The “dilemma” mentioned in the passage refers to: 

A Keeping government agencies in the largest city with good infrastructure or moving them to a remote area with few amenities. 

B Relocating government agencies to boost growth in remote areas with poor amenities or to relatively larger cities with good amenities. 

C Encouraging private enterprises to relocate to smaller towns or not incentivising them in order to keep government costs in those towns low. 

D Concentrating on decongesting large cities or focusing on boosting employment in relatively larger cities. 

Answer: B 

Explanation: 

“The dilemma is obvious. Pick small, poor towns, and areas of high unemployment get new jobs, but it is hard to attract 

the most qualified workers; opt for larger cities with infrastructure and better-qualified residents, and the country’s most deprived areas see little benefit” 

Option A is incorrect. The passage makes no mention about having the government agencies in the “largest” city. It talks about having them in “larger cities”. Hence it is incorrect. 

Option C talks about relocation of private enterprises. This is not mentioned in the passage as the passage is primarily about the relocation of government bureaucrats. 

Option D makes no mention about allotting highly qualified workers to smaller cities. Hence it is incorrect. 

Option B makes the right comparison. It compares the hard task of relocating qualified workers to smaller towns, to allocating workers to larger cities , which would result in smaller towns receiving little benefit. 

Option B is the correct answer. 

 

Instructions 

Comprehension: 

The magic of squatter cities is that they are improved steadily and gradually by their residents. To a planner’s eye, these cities look chaotic. I trained as a biologist and to my eye, they look organic. Squatter cities are also unexpectedly green. They have maximum density—1 million people per square mile in some areas of Mumbai—and have minimum energy and material use. People get around by foot, bicycle, rickshaw, or the universal shared taxi. 

Not everything is efficient in the slums, though. In the Brazilian favelas where electricity is stolen and therefore free, people leave their lights on all day. But in most slums recycling is literally a way of life. The Dharavi slum in Mumbai has 400 recycling units and 30,000 ragpickers. Six thousand tons of rubbish are sorted every day. In 2007, the Economist reported that in Vietnam and Mozambique, “Waves of gleaners sift the sweepings of Hanoi’s streets, just as Mozambiquan children pick over the rubbish of Maputo’s main tip. Every city in Asia and Latin America has an industry based on gathering up old cardboard boxes.” . . . 

In his 1985 article, Calthorpe made a statement that still jars with most people: “The city is the most environmentally benign form of human settlement. Each city dweller consumes less land, less energy, less water, and produces less pollution than his counterpart in settlements of lower densities.” “Green Manhattan” was the inflammatory title of a 2004 New Yorker article by David Owen. “By the most significant measures,” he wrote, “New York is the greenest community in the United States, and one of the greenest cities in the world . . . 

The key to New York’s relative environmental benignity is its extreme compactness. . . . Placing one and a half million people on a twenty – three – square-mile island sharply reduces their opportunities to be wasteful.” He went on to note that this very compactness forces people to live in the world’s most energy-efficient apartment buildings. . . . 

Urban density allows half of humanity to live on 2.8 per cent of the land. . . . Consider just the infrastructure efficiencies. 

According to a 2004 UN report: “The concentration of population and enterprises in urban areas greatly reduces the unit cost of piped water, sewers, drains, roads, electricity, garbage collection, transport, health care, and schools.” . . . 

The nationally subsidised city of Manaus in northern Brazil “answers the question” of how to stop deforestation: give people decent jobs. Then they can afford houses, and gain security. One hundred thousand people who would otherwise be deforesting the jungle around Manaus are now prospering in town making such things as mobile phones and televisions. . . . 

Of course, fast-growing cities are far from an unmitigated good. They concentrate crime, pollution, disease and injustice as much as business, innovation, education and entertainment. . . . 

But if they are overall a net good for those who move there, it is because cities offer more than just jobs. They are transformative: in the slums, as well as the office towers and leafy suburbs, the progress is from hick to metropolitan to cosmopolitan . . . 

 

Q. 15 We can infer that Calthorpe’s statement “still jars” with most people because most people: 

A do not consider cities to be eco-friendly places. 

B consider cities to be very crowded and polluted. 

C do not regard cities as good places to live in. 

D regard cities as places of disease and crime. 

Answer: A 

Explanation: 

“The city is the most environmentally benign form of human settlement. Each city dweller consumes less land, less energy, less water, and produces less pollution than his counterpart in settlements 

of lower densities.” 

Calthrope’s major contention is that cites are eco-friendly as they consume less resources than people living in places that have lower population densities. 

Options B,C,D does not directly contradict Calthrope’s statement. Hence , they cannot be the reason why the statement that jars with most people. 

Option A is directly opposed to Calthrope’s viewpoints. Hence, this option is most likely to jar with most people. Option A i the correct answer. 

 

Q. 16 In the context of the passage, the author refers to Manaus in order to: 

A explain how urban areas help the environment. 

B describe the infrastructure efficiencies of living in a city. 

C promote cities as employment hubs for people. 

D explain where cities source their labour for factories. 

Answer: A 

Explanation: 

The author gives the example of Manaus to show how an entire community of people whose major job was deforestation of the jungle have now been able to prosper by making things such as mobile phones and televisions. 

Option A is the correct answer. It is the major reason for the author giving out the example of Manaus. Options D is incorrect and is not mentioned in the passage. 

Option B and C are the pros of being in a squatter city, but it is not the reason why the author gives the example of Manaus. 

Option A is the correct answer. 

 

Q. 17 According to the passage, squatter cities are environment-friendly for all of the following reasons EXCEPT: 

A They recycle material. 

B their transportation is energy efficient. 

C their streets are kept clean. 

D they sort out garbage. 

Answer: C 

Explanation: 

Option A would help keep squatter cities environment friendly, as recycling material would reduce the amount of non biodegradable materials present in the environment. 

Option B would also help squatter cities be more environment friendly by reducing pollution. 

Option D would also help squatter cities be more environment friendly as sorting garbage and treating them would go a long way in preventing soil and water pollution. 

Option C is incorrect. This is because, keeping the streets clean would mean that the wastes are somewhere in dumped in the environment near the local community. 

Hence, option C cannot be inferred from the passage and is the correct answer. 

 

Q. 18 Which one of the following statements would undermine the author’s stand regarding the greenness of cities? 

A Sorting through rubbish contributes to the rapid spread of diseases in the slums. 

B The high density of cities leads to an increase in carbon dioxide and global warming. 

C The compactness of big cities in the West increases the incidence of violent crime. 

D Over the last decade the cost of utilities has been increasing for city dwellers. 

Answer: B 

Explanation: 

The rapid spread of diseases in the slum would only affect the people in the slums and not the greenness of the cities. Hence it is incorrect. 

Option C is incorrect as the incidence of crime in the West would not impact the greenness of the cities. Increasing cost of utilities , in the same way, would not affect the flora of the cities. 

An increase in carbon-di-oxide and global warming, however, would contribute greatly to the change in climate. A change in climate would adversely affect the greenery in the cities. Hence, this would greatly undermine the author’s stand that cities are indeed green. 

 

Q. 19 From the passage it can be inferred that cities are good places to live in for all of the following reasons EXCEPT that they: 

A help prevent destruction of the environment. 

B contribute to the cultural transformation of residents. 

C offer employment opportunities. 

D have suburban areas as well as office areas. 

Answer: D 

Explanation: 

From the sentence, “One hundred thousand people who would otherwise be deforesting the jungle around Manaus are now prospering in town making such things as mobile phones and televisions” given in the passage, option A can be inferred. 

“But if they are overall a net good for those who move there, it is because cities offer more than just jobs. They are transformative” From this line, option B can be inferred. 

The entire second paragraph of the passage mentions how multiple people have got jobs in squatter cities. Hence, option C can be inferred. 

Option D is not a reason for cities being a good place to live in. Hence, this option cannot be inferred and is the correct answer. 

 

Instructions 

Comprehension: 

War, natural disasters and climate change are destroying some of the world’s most precious cultural sites. Google is trying to help preserve these archaeological wonders by allowing users access to 3D images of these treasures through its site. But the project is raising questions about Google’s motivations and about who should own the digital copyrights. Some critics call it a form of “digital colonialism.” When it comes to archaeological treasures, the losses have been ounting. ISIS blew up parts of the ancient city of Palmyra in Syria and an earthquake hit Bagan, an ancient city in Myanmar, damaging dozens of temples, in 2016. In the past, all archaeologists and historians had for restoration and research were photos, drawings, remnants and intuition. But that’s changing. Before the earthquake at Bagan, many of the temples on the site were scanned. . . . 

[These] scans . . . are on Google’s Arts & Culture site. The digital renditions allow viewers to virtually wander the halls of the temple, look up-close at paintings and turn the building over, to look up at its chambers. . . . 

[Google Arts & Culture] works with museums and other nonprofits . . . to put high-quality images online. The images of the temples in Bagan are part of a collaboration with CyArk, a nonprofit that creates the 3D scanning of historic sites. . . . Google . . . says [it] doesn’t make money off this website, but it fits in with Google’s mission to make the world’s information available and useful. 

Critics say the collaboration could be an attempt by a large corporation to wrap itself in the sheen of culture. Ethan Watrall, an archaeologist, professor at Michigan State University and a member of the Society for American Archaeology, says he’s not comfortable with the arrangement between CyArk and Google. . . . Watrall says this project is just a way for Google to promote Google. “They want to make this material accessible so people will browse it and be filled with wonder by it,” he says. “But at its core, it’s all about advertisements and driving traffic.” Watrall says these images belong on the site of a museum or educational institution, where there is serious scholarship and a very different mission. . . . [There’s] another issue for some archaeologists and art historians. CyArk owns the copyrights of the scans — not the countries where these sites are located. That means the countries need CyArk’s permission to use these images for commercial purposes. 

Erin Thompson, a professor of art crime at John Jay College of Criminal Justice in New York City, says it’s the latest example of a Western nation appropriating a foreign culture, a centuries-long battle. . . . CyArk says it copyrights the scans so no one can use them in an inappropriate way. The company says it works closely with authorities during the process, even training local people to help. But critics like Thompson are not persuaded. . . . She would prefer the scans to be owned by the countries and people where these sites are located. 

 

Q. 20 Which of the following, if true, would most strongly invalidate Dr. Watrall’s objections? 

A Google takes down advertisements on its website hosting CyArk’s scanned images. 

B There is a ban on CyArk scanning archeological sites located in other countries. 

C CyArk uploads its scanned images of archaeological sites onto museum websites only. 

D CyArk does not own the copyright on scanned images of archaeological sites. 

Answer: C 

Explanation: 

“They want to make this material accessible so people will browse it and be filled with wonder by it,” he says. “But at its core, it’s all about advertisements and 

driving traffic.” Watrall says these images belong on the site of a museum or educational institution, where there is serious scholarship and a very different mission”. 

From the above mentioned lines it can be reasonably inferred that Dr. Watrall is not critial if the digitally scanned images are on official museum websites and archaeological sites. 

Option C mentions the case when CyArk uploads the scanned images on museum sites only. This would invalidate the arguments made by Dr.Watrall. 

Option A is incorrect as Dr. Watrall considers the venture as a medium to promote Google itself. Just taking down advertisements would not invalidate the professor’s claim. 

Option B is incorrect as a ban in certain locations would certainly not prevent promotion of and commercialization by Google. The same reason can be attributed to option D. CyArk not owning the copyright of the archaeological sites would not prevent using it for commercial purposes. 

Hence, option C is the correct answer. 

 

Q. 21 By “digital colonialism”, critics of the CyArk-Google project are referring to the fact that: 

A the scanning process can damage delicate frescos and statues at the sites. 

B CyArk and Google have not shared the details of digitisation with the host countries. 

C countries where the scanned sites are located do not own the scan copyrights. 

D CyArk and Google have been scanning images without copyright permission from host countries. Answer: C 

Explanation: 

From the lines, ” [There’s] another issue for some archaeologists and art historians. CyArk owns the copyrights of the scans — not the countries where these sites are located. That means the countries need CyArk’s permission to use these images for commercial purposes”, it can be seen that critics view the Google-CyArk project as one that appropriates the copyrights of the digital scans in such a way even the countries in which the sites are located need CyArk’s permission to use the images. 

Option A, D , B are not mentioned anywhere in the passage. 

Option C is describes perfectly why the critics of the Google-CyArk project term it as digital colonialism and hence it is the correct answer. 

 

Q. 22 Of the following arguments, which one is LEAST likely to be used by the companies that digitally scan cultural sites? 

A It helps preserve precious images in case the sites are damaged or destroyed. 

B It enables people who cannot physically visit these sites to experience them. 

C It provides images free of cost to all users. 

D It allows a large corporation to project itself as a protector of culture. 

Answer: D 

Explanation: 

The option that would not help a company that digitally scans cultural sites would be the given answer. 

Option A is incorrect as preserving images of sites in case they are damaged would be one of the foremost arguments made by these companies. 

Option B and C would surely help the cause of companies that scan cultural sites. Hence,they are incorrect. 

Option D is the correct answer. A company that digitally scans cultural sites would not give the reason of being able to project itself as a protector of culture as a reason to scan the cultural sites. This is a self centered goal such a company and hence is the least likely of the arguments that would be used in this case. 

Hence, option D is the correct answer. 

 

Q. 23 Based on his views mentioned in the passage, one could best characterise Dr. Watrall as being: 

A uneasy about the marketing of archaeological images for commercial use by firms such as Google and CyArk. B dismissive of laypeople’s access to specialist images of archaeological and cultural sites. C critical about the links between a non-profit and a commercial tech platform for distributing archaeological images. 

D opposed to the use of digital technology in archaeological and cultural sites in developing countries. Answer: C 

Explanation: 

From the passage, it can be inferred that Dr. Watrall is not comfortable with the arrangement between Cyark and Google. He is of the opinion that though the material is promoted as a means for people to view the artifacts, the ulterior motive is for advertisements and commercial purposes. 

Option A is incorrect. The professor is uneasy about the arrangement between a non-profit organisation and a commercial organisation, whose values are, in reality, different from what they portray. 

Option B is incorrect. The professor is not in dissmissive of laypeople’s access to specialist images and such information is not given in the passage. 

Option D is incorrect. The professor is only dismissive of the commercial agreement between two organisations that portray themselves to be involved in non-profit work. He is not against the use of digital technology in archaeological and cultural sites in developing countries. 

Option C is correct and it correctly represents the views of professor Dr. Watrall. 

 

Q. 24 In Dr. Thompson’s view, CyArk owning the copyright of its digital scans of archaeological sites is akin to: 

A tourists uploading photos of monuments onto social media. 

B the seizing of ancient Egyptian artefacts by a Western museum. 

C the illegal downloading of content from the internet. 

D digital platforms capturing users’ data for market research. 

Answer: B 

Explanation: 

After reading the lines, “Erin Thompson, a professor of art crime at John Jay College of Criminal Justice in New York City, says it’s the latest example of a Western nation appropriating a foreign culture, a centuries-long battle.” it can be inferred that professor accuses CyArk of appropriating a foreign culture. 

The only option present that is an example of a western nation appropriating a foreign culture would be the seizing of ancient Egyptian artefacts by a Western museum. Hence, option B is the correct answer. 

Instructions 

The four sentences (labelled 1, 2, 3, 4) given below, when properly sequenced would yield a coherent paragraph. Decide on the proper sequence of the order of the sentences and key in the sequence of the four numbers as your answer. 

 

Q. 25 Decide on the proper sequence of the order of the sentences

1. To the uninitiated listener, atonal music can sound like chaotic, random noise. 

2. Atonality is a condition of music in which the constructs of the music do not ‘live’ within the confines of a particular key signature, scale, or mode. 

3. After you realize the amount of knowledge, skill, and technical expertise required to compose or perform it, your tune may change, so to speak. 

4. However, atonality is one of the most important movements in 20th century music. 

Answer:2143 

Explanation: 

After reading all the sentences , it can be inferred that though atonal music can sometimes sound random and chaotic, there is a lot of knowledge and skill that is required to perform atonal music. 

Sentence 2 talks about what exactly atonal music is. Hence, this sentence must be the first sentence of the paragraph. Sentence 1 talks about the misconceptions of atonal music that the uninitiated make. This sentence must be the second sentence of the passage. Sentence 4, now clears the misconception about atonality and states that it is one of the most important movements in music. Sentence 3 gives the reason why atonality is music is so difficult to attain and says that the untrained listener would change his mind when he understands the amount of knowledge and skill required to produce atonality. Therefore, sentences 4 and 3 form a block. 

Therefore the correct sequence of sentences is 2-1-4-3. 

 

Q. 26 Decide on the proper sequence of the order of the sentences

1. Living things—animals and plants—typically exhibit correlational structure. 

2. Adaptive behaviour depends on cognitive economy, treating objects as equivalent. 

3. The information we receive from our senses, from the world, typically has structure and order, and is not arbitrary. 

4. To categorize an object means to consider it equivalent to other things in that category, and different— along some salient dimension—from things that are not. 

Answer:2431 

Explanation: 

After reading all the sentences, it can be inferred that the passage talks about how comparisons are made between objects in different aspects, and how such comparisons are important facets of cognitive ability and consequently our adaptive behaviour. 

Sentence 2 introduces how adaptive behavior depends on cognitive economy. Hence, it is the first sentence of the paragraph. 

Sentence 4 elaborates on how different objects are compared. This sentence logically follows sentence 2. Sentence 3 shows how such comparisons have structure and order, and how they are not arbitary. Hence, sentence 3 follows sentence 4. Sentence 1 is completes the passage in a way that elucidates how animals and plats are equivalent to each other by exhibiting correlatoinal structure. 

The correct sequence is 2-4-3-1. 

 

Q. 27 Decide on the proper sequence of the order of the sentences

1. Conceptualisations of ‘women’s time’ as contrary to clock-time and clock-time as synonymous with economic rationalism are two of the deleterious results of this representation. 

2. While dichotomies of ‘men’s time’, ‘women’s time’, clock-time, and caring time can be analytically useful, this article argues that everyday caring practices incorporate a multiplicity of times; and both men and women can engage in these multiple-times 

3. When the everyday practices of working sole fathers and working sole mothers are carefully examined to explore conceptualisations of gendered time, it is found that caring time is often more focused on the clock than generally theorised. 

4. Clock-time has been consistently represented in feminist literature as a masculine artefact representative of a ‘time is money’ perspective. 

Answer:4132 

Explanation: 

After reading all the lines it can be seen that the paragraph talks about the deleterious results of introducing the concept of women’s time, and how the everyday practices of both men and women incorporate a multiplicity of times. 

Sentence 4 introduces the concept of clock time which has been represented in feminist literature and it is the introductory sentence of the paragraph. The passage goes on to say that the concept of ‘women’s time’ as is deleterious.Hence, sentence 1 follows sentence 4. Sentence 3 explains how the concept of clock time actually works out for working sole fathers and working sole mothers. It follows sentence 1. Sentence 2 gives a summary of the paragraph which states that the everyday caring practices involve a multiplicity of times hence it would be the last sentence of the paragraph. 

The correct order of sequence is 4-1-3-2. 

 

Instructions 

The passage given below is followed by four alternate summaries. Choose the option that best captures the essence of the passage. 

Q. 28 Choose the option that best captures the essence of the passage

Language is an autapomorphy found only in our lineage, and not shared with other branches of our group such as primates. We also have no definitive evidence that any species other than Homo sapiens ever had language. However, it must be noted straightaway that ‘language’ is not a monolithic entity, but rather a 

complex bundle of traits that must have evolved over a significant time frame…. Moreover, language crucially draws on aspects of cognition that are long established in the primate lineage, such as memory: the language faculty as a whole comprises more than just the uniquely linguistic features. 

A Language evolved with linguistic features building on features of cognition such as memory. 

B Language, a derived trait found only in humans, has evolved over time and involves memory. 

C Language is not a single, uniform entity but the end result of a long and complex process of linguistic evolution. 

D Language is a distinctively human feature as there is no evidence of the existence of language in any other species. 

Answer: A 

Explanation: 

The passage states that language is only found in humans and not among any other primates. The passage also states that language is a bundle of traits such as memory that evolved over a period of time. 

The passage does not talk about language that has evolved over time. Hence, option B is incorrect. Option C does not completely summarise the passage. Hence, it incorrect. 

Option D talks only about language being a distinctive human feature. It does not talk about language having evolved from a complex bundle of traits. Hence it is incorrect. 

Option A correctly summarises the main idea of the passage and is the correct answer. 

 

Q. 29 Choose the option that best captures the essence of the passage

Privacy-challenged office workers may find it hard to believe, but open-plan offices and cubicles were invented by architects and designers who thought that to break down the social walls that divide people, you had to break down the real walls, too. Modernist architects saw walls and rooms as downright fascist. The spaciousness and flexibility of an open plan would liberate homeowners and office dwellers from the confines of boxes. But companies took up their idea less out of a democratic ideology than a desire to pack in as many workers as they could. The typical open-plan office of the first half of the 20th century was a white-collar assembly line. Cubicles were interior designers’ attempt to put some soul back in. 

A Wall-free office spaces could have worked out the way their utopian inventors intended had companies cared for workers’ satisfaction. 

B Wall-free office spaces did not quite work out as desired and therefore cubicles came into being. 

C Wall-free office spaces did not quite work out the way their utopian inventors intended, as they became tools for exploitation of labor. 

D Wall-free office spaces did not quite work out as companies don’t believe in democratic ideology. 

Answer: C 

Explanation: 

After reading the entire paragraph, it can be inferred that the main idea of the passage is that while the inventors of the open-plan offices had the liberation of office dwellers from boxes in mind, the companies used it to pack as much people as possible inside. 

Option A is incorrect as the passage makes no mentions about workers satisfaction. 

Option B is incorrect as it misrepresents the timeline given in the passage. Cubicles existed earlier and only later were wall free office spaces invented. 

Option D is incorrect as it could not be inferred from the passage that the companies that did not believe in democratic ideology 

Option C correctly encapsulates the main idea of the passage and hence it is the correct answer. 

 

Q. 30 Choose the option that best captures the essence of the passage

Social movement organizations often struggle to mobilize supporters from allied movements in their efforts to achieve critical mass. Organizations with hybrid identities—those whose organizational identities span the boundaries of two or more social movements, issues, or identities—are vital to mobilizing these constituencies. Studies of the post-9/11 U.S. antiwar movement show that individuals with past involvement in non-anti-war movements are more likely to join hybrid organizations than are individuals without involvement in non-anti-war movements. In addition, they show that organizations with hybrid identities occupy relatively more central positions in inter-organizational contact networks within the antiwar movement and thus recruit significantly more participants in demonstrations than do non hybrid organizations. 

A Movements that work towards social change often find it difficult to mobilize a critical mass of supporters. 

B Organizations with hybrid identities are able to mobilize individuals with different points of view. 

C Post 9/11 studies show that people who are involved in non anti-war movements are likely to join hybrid organizations. 

D Hybrid organizations attract individuals that are deeply involved in anti-war movements. 

Answer: B 

Explanation: 

After reading all the lines of the paragraph, it is evident that social movement organisations depend on organisations with hybrid identities because , within them they contain individuals with multiple points of views on different movements and issues. The example shows how people who are campaigning for non-anti-war movements are more likely to join hybrid organisations. It also says that organisations with hybrid identities occupy more central positions within organisations that are involved in anti-war movements. 

Option A is incorrect. The passage talks more about why hybrid organisations are vital to providing participants in social movements. 

Option C is just an illustration of the idea that the paragraph is trying to make. Hence, it would not be the most appropriate summary of the paragraph. 

Option D is incorrect. From the paragraph we can infer that hybrid organisations attract individuals that are deeply involved in non-anti-war movements. 

Option B is an appropriate summary of the passage and hence it is the correct answer. 

Instructions 

For the following questions answer them individually 

 

Q. 31 Five sentences related to a topic are given below. Four of them can be put together to form a meaningful and coherent short paragraph. Identify the odd one out. Choose its number as your answer and key it in. 

1. Ocean plastic is problematic for a number of reasons, but primarily because marine animals eat it. 

2. The largest numerical proportion of ocean plastic falls in small size fractions. 

3. Aside from clogging up the digestive tracts of marine life, plastic also tends to adsorb pollutants from the water column. 

4. Plastic in the oceans is arguably one of the most important and pervasive environmental problems today. 

5. Eating plastic has a number of negative consequences such as the retention of plastic particles in the gut for longer periods than normal food particles. 

Answer:2 

Explanation: 

After reading all the sentences it can be inferred that the passage talks about the impact of ocean plastic on marine organisms. 

Sentence 4 introduces the important and pervasive environmental problem of having plastic in the oceans.Hence it is the first sentence of the passage. Sentence 1 gives the reason behind why ocean plastic is problematic, elucidating that marine animals eat it. Hence it should follow sentence 4. Sentence 5 gives the consequences of marine animals eating ocean plastic. Sentence 3 further elaborates sentence 5 and follows it. 

Hence the correct order of sentences is 4-1-5-3. Sentence 2 does not fit in with the rest of the passage and is the correct option. 

 

Q. 32 The four sentences (labelled 1, 2, 3, 4) given below, when properly sequenced would yield a coherent paragraph. Decide on the proper sequence of the order of the sentences and key in the sequence of the four numbers as your answer. 

1. Such a belief in the harmony of nature requires a purpose presumably imposed by the goodness and wisdom of a deity. 

2. These parts, all fit together into an integrated, well-ordered system that was created by design. 

3. Historically, the notion of a balance of nature is part observational, part metaphysical, and not scientific in any way. 

4. It is an example of an ancient belief system called teleology, the notion that what we call nature has a predetermined destiny associated with its component parts. 

Answer:3421 

Explanation: 

After reading all the sentences it can be reasonably inferred that the passage talks about the ancient belief system called telelogy which describes how nature has a predetermined destiny associated with its component parts, and how these parts fit together into an ordered system. 

Sentence 3 introduces the idea of the notion of balance of nature. Hence, this would serve as the introductory sentence. Sentence 4 gives an example of a system that tries to explain the balance in nature called telelogy. Hence, this sentence follows sentence 3. Sentence, 2 follows sentence 4 as it furthers the idea given in sentence 4. It explains about how the component parts explained in sentence 4 fit together. Sentence 1 would serve as the concluding sentence as it gives the necessary condition required to have a belief that has been explained in sentences 4 and 2. 

Hence, the correct ordering of the sentences is 3-4-2-1. 

 

Q. 33 Five sentences related to a topic are given below in a jumbled order. Four of them form a coherent and unified paragraph. Identify the odd sentence that does not go with the four. Key in the number of the option that you choose. 

1. Socrates told us that ‘the unexamined life is not worth living’ and that to ‘know thyself’ is the path to true wisdom 

2. It suggests that you should adopt an ancient rhetorical method favored by the likes of Julius Caesar and known as ‘illeism’ – or speaking about yourself in the third person. 

3. Research has shown that people who are prone to rumination also often suffer from impaired decision making under pressure and are at a substantially increased risk of depression. 

4. Simple rumination – the process of churning your concerns around in your head – is not the way to achieve self-realization. 

5. The idea is that this small change in perspective can clear your emotional fog, allowing you to see past your biases. 

Answer:1 

Explanation: 

After reading all the sentences, it can be reasonably inferred that the passage talks about how rumination is not the way to achieve-self realization, but another method favored by Caesar, ‘illeism’ would help a person see past his biases. 

Sentence 4 introduces the idea of rumination. Hence this sentence would serve as the introductory sentence. Sentence 3 follows sentence 4 because it indicates the results of the research done on rumination. Sentence 2 would be the next logical progression, since it puts forth an alternative to rumination, known as illeism. Sentence 2 and sentence 5 would form a block as sentence 5 explains the consequences of adopting the ancient method of illeism, mentioned in sentence 2. 

4-3-2-5 would be the correct ordering of the sentences. Sentence 1 does not fit in with the passage and hence, it is the correct answer. 

 

Q. 34 Five sentences related to a topic are given below. Four of them can be put together to form a meaningful and coherent short paragraph. Identify the odd one out.Choose its number as your answer and key it in. 1. A particularly interesting example of inference occurs in many single panel comics. 2. It’s the creator’s participation and imagination that makes the single-panel comic so engaging and so rewarding. 

3. Often, the humor requires you to imagine what happened in the instant immediately before or immediately after the panel you’re being shown. 

4. To get the joke, you actually have to figure out what some of these missing panels must be. 5. It is as though the cartoonist devised a series of panels to tell the story and has chosen to show you only one – and typically not even the funniest. 

Answer:2 

Explanation: 

After reading all the sentences it is clear that the paragraph talks about how to understand the humor behind the single panel comics. 

The sentence 1 sets the platform for the author to explain how to infer humour in a single panel comic. Consider the sentences 3,4,5. Those sentences are aimed at the reader. 

“3. Often, the humor requiresyou to imagine what happened in the instant immediately before or immediately after the panel you’re being shown.” 

“4.To get the joke, you actually have to figure out what some of these missing panels must be.” “5. It is as though the cartoonist devised a series of panels to tell the story and has chosen to show you only one – and typically not even the funniest.” 

Sentence 2 on the other hand is a stand alone sentence that does not fit in with the rest of the paragraph. Hence, it is the odd sentence. 

DILR 

Instructions 

Comprehension: 

Ten players, as listed in the table below, participated in a rifle shooting competition comprising of 10 rounds. Each round had 6 participants. Players numbered 1 through 6 participated in Round 1, players 2 through 7 in Round 2,…, players 5 through 10 in Round 5, players 6 through 10 and 1 in Round 6, players 7 through 10, 1 and 2 in Round 7 and so on. The top three performances in each round were awarded 7, 3 and 1 points respectively. There were no ties in any of the 10 rounds. The table below gives the total number of points obtained by the 10 players after Round 6 and Round 10. 

The following information is known about Rounds 1 through 6: 

1. Gordon did not score consecutively in any two rounds. 

2. Eric and Fatima both scored in a round. 

The following information is known about Rounds 7 through 10: 

1. Only two players scored in three consecutive rounds. One of them was Chen. No other player scored in any two consecutive rounds. 

2. Joshin scored in Round 7, while Amita scored in Round 10. 

3. No player scored in all the four rounds. 

Q. 35 What were the scores of Chen, David, and Eric respectively after Round 3? 

A 3, 6, 3 

B 3, 3, 3 

C 3, 3, 0 

D 3, 0, 3 

Answer: B 

Explanation: 

From the condition given in the premise, we can make the following table: 

The information known about Rounds 1 through 6: 

1. Gordon(G) did not score consecutively in any two rounds. 

2. Eric(E) and Fatima(F) both scored in a round. 

By observing the table: 

1. Jordan(J) scored 7 points in both the rounds 5th & 6th. 

2. Amita (A) scored 1,7 points then she scored 7 in the first round. 

3. Bala (B) scored 1 point in both the rounds 1st and 2nd. 

4. Ikea (I) scored 1 point in the round 4th and 5th. 

5. Gordon(G- 7,7,3 ) did not score consecutively in any two rounds so it scored in 2nd, 4th and 6th rounds respectively. We can make the following table from the details given in the question. 

T: Total after the sixth round and TT: Total after the 10th round. 

1. Only two players scored in three consecutive rounds. One of them was Chen. So He scored 1 point in the rounds 8th, 9th and 10th. 

2. Ikea scored 15 points (1,7,7) in three rounds respectively. 

3. Eric scored 7 in round 10. 

4. Amita will score 3 in round 10, and 7 in round 7. 

We can make the following table: 

Hence option B is correct. 

Q. 36 Which three players were in the last three positions after Round 4? 

A Bala, Ikea, Joshin 

B Bala, Hansa, Ikea 

C Bala, Chen, Gordon 

D Hansa, Ikea, Joshin 

Answer: D 

Explanation: 

From the condition given in the premise, we can make the following table: 

The information known about Rounds 1 through 6: 

1. Gordon(G) did not score consecutively in any two rounds. 

2. Eric(E) and Fatima(F) both scored in a round. 

By observing the table: 

1. Jordan(J) scored 7 points in both the rounds 5th & 6th. 

2. Amita (A) scored 1,7 points then she scored 7 in the first round. 

3. Bala (B) scored 1 point in both the rounds 1st and 2nd. 

4. Ikea (I) scored 1 point in the round 4th and 5th. 

5. Gordon(G- 7,7,3 ) did not score consecutively in any two rounds so it scored in 2nd, 4th and 6th rounds respectively.

We can make the following table from the details given in the question. 

T: Total after the sixth round and TT: Total after the 10th round. 

1. Only two players scored in three consecutive rounds. One of them was Chen. So He scored 1 point in the rounds 8th, 9th and 10th. 

2. Ikea scored 15 points (1,7,7) in three rounds respectively. 

3. Eric scored 7 in round 10. 

4. Amita will score 3 in round 10, and 7 in round 7. 

We can make the following table: 

Hence option D is correct. 

Q. 37 Which player scored points in maximum number of rounds? 

A Joshin 

B Chen 

C Amita 

D Ikea 

Answer: D 

Explanation: 

From the condition given in the premise, we can make the following table: 

The information known about Rounds 1 through 6: 

1. Gordon(G) did not score consecutively in any two rounds. 

2. Eric(E) and Fatima(F) both scored in a round. 

By observing the table: 

1. Jordan(J) scored 7 points in both the rounds 5th & 6th. 

2. Amita (A) scored 1,7 points then she scored 7 in the first round. 

3. Bala (B) scored 1 point in both the rounds 1st and 2nd. 

4. Ikea (I) scored 1 point in the round 4th and 5th. 

5. Gordon(G- 7,7,3 ) did not score consecutively in any two rounds so it scored in 2nd , 4th and 6th rounds respectively. We can make the following table from the details given in the question. 

T: Total after the sixth round and TT: Total after the 10th round. 

1. Only two players scored in three consecutive rounds. One of them was Chen. So He scored 1 point in the rounds 8th, 9th and 10th. 

2. Ikea scored 15 points (1,7,7) in three rounds respectively. 

3. Eric scored 7 in round 10. 

4. Amita will score 3 in round 10, and 7 in round 7. 

We can make the following table: 

Hence option D is correct. 

Q. 38 Which players scored points in the last round? 

A Amita, Eric, Joshin 

B Amita, Chen, David 

C Amita, Bala, Chen 

D Amita, Chen, Eric 

Answer: D 

Explanation: 

From the condition given in the premise, we can make the following table: The information known about Rounds 1 through 6: 

1. Gordon(G) did not score consecutively in any two rounds. 

2. Eric(E) and Fatima(F) both scored in a round. 

By observing the table: 

1. Jordan(J) scored 7 points in both the rounds 5th & 6th. 

2. Amita (A) scored 1,7 points then she scored 7 in the first round. 

3. Bala (B) scored 1 point in both the rounds 1st and 2nd. 

4. Ikea (I) scored 1 point in the round 4th and 5th. 

5. Gordon(G- 7,7,3 ) did not score consecutively in any two rounds so it scored in 2nf , 4th and 6th rounds respectively. We can make the following table from the details given in the question. 

T: Total after the sixth round and TT: Total after the 10th round. 

1. Only two players scored in three consecutive rounds. One of them was Chen. So He scored 1 point in the rounds 8th, 9th and 10th. 

2. Ikea scored 15 points (1,7,7) in three rounds respectively. 

3. Eric scored 7 in round 10. 

4. Amita will score 3 in round 10, and 7 in round 7. 

We can make the following table: 

Hence option D is correct. 

 

Instructions 

Comprehension: 

To compare the rainfall data, India Meteorological Department (IMD) calculated the Long Period Average (LPA) of rainfall during period June-August for each of the 16 states. The figure given below shows the actual rainfall (measured in mm) during June-August, 2019 and the percentage deviations from LPA of respective states in 2018. Each state along with its actual rainfall is presented in the figure. 

Q. 39 If a ‘Heavy Monsoon State’ is defined as a state with actual rainfall from June-August, 2019 of 900 mm or more, then approximately what percentage of ‘Heavy Monsoon States’ have a negative deviation from respective LPAs in 2019? 

A 42.86 

B 75.00 

C 57.14 

D 14.29 

Answer: A 

Explanation: 

The states which satisfy the condition given in the question: 

Maharashtra, Mizoram, Sikkim, Goa, Arunachal, Kerla, Meghalaya…..7 states 

The ‘Heavy Monsoon States’ have a negative deviation: Arunachal, Kerla, Meghalaya 

= 3/7×100=42.86% 

Option A 

 

Q. 40 If a ‘Low Monsoon State’ is defined as a state with actual rainfall from June-August, 2019 of 750 mm or less, then what is the median ‘deviation from LPA’ (as defined in the Y-axis of the figure) of ‘Low Monsoon States’? 

A -10% 

B 10% 

C -20% 

D -30% 

Answer: A 

Explanation: 

All the states which satisfy the condition for ‘ Low monsoon state’ are Gujrat (+25%), Karnataka (+20%), Rajasthan (+15), MP (+10%), Assam (-10%), WB (-30%), Jharkhand (-35%), Delhi (-40%) and Manipur (-60%). 

The median of all the deviation is -10% Assam. 

Q. 41 What is the average rainfall of all states that have actual rainfall of 600 mm or less in 2019 and have a negative deviation from LPA? 

A 367 mm 

B 500 mm 

C 450 mm 

D 460 mm 

Answer: D 

Explanation: 

The states Assam, WB, Jharkhand, Delhi and Manipur satisfy the conditions given in the question. The actual rainfall of all these states in 2019 are 600,600,400,300,400 

Average of these states= 2300/5=460mm 

 

Q. 42 The LPA of a state for a year is defined as the average rainfall in the preceding 10 years considering the period of June-August. For example, LPA in 2018 is the average rainfall during 2009-2018 and LPA in 2019 is the average rainfall during 2010-2019. It is also observed that the actual rainfall in Gujarat in 2019 is 20% more than the rainfall in 2009. The LPA of Gujarat in 2019 is closest to 

A 475 mm 

B 505 mm 

C 490 mm 

D 525 mm 

Answer: C 

Explanation: 

The actual rainfall in Gujarat in 2019 is 20% more than the rainfall in 2009. 

So If the actual rainfall in 2009 = x mm 

Then the actual rainfall in 2019= 1.2x mm 

Actual rainfall in 2019= 600mm 

Then, actual rainfall in 2009 = 500mm 

As deviation is +25% so average 2009 – 2018 is 600/1.25 = 480 

LPA 2019 = (480×10 – 500 + 600 )/10 = 490mm 

Answer C 

Instructions 

Comprehension: 

The first year students in a business school are split into six sections. In 2019 the Business Statistics course was taught in these six sections by Annie, Beti, Chetan, Dave, Esha, and Fakir. All six sections had a common midterm (MT) and a common endterm (ET) worth 100 marks each. ET contained more questions than MT. Questions for MT and ET were prepared collectively by the six faculty members. Considering MT and ET together, each faculty member prepared the same number of questions. 

Each of MT and ET had at least four questions that were worth 5 marks, at least three questions that were worth 10 marks, and at least two questions that were worth 15 marks. In both MT and ET, all the 5-mark questions preceded the 10-mark questions, and all the 15- mark questions followed the 10-mark questions. 

The following additional facts are known. 

i. Annie prepared the fifth question for both MT and ET. For MT, this question carried 5 marks. 

ii. Annie prepared one question for MT. Every other faculty member prepared more than one questions for MT. 

iii. All questions prepared by a faculty member appeared consecutively in MT as well as ET. 

iv. Chetan prepared the third question in both MT and ET; and Esha prepared the eighth question in both. 

v. Fakir prepared the first question of MT and the last one in ET. Dave prepared the last question of MT and the first one in ET. 

Q. 43 The second question in ET was prepared by: 

A Chetan 

B Beti 

C Esha 

D Dave 

Answer: D 

Explanation: 

All six sections had a common midterm (MT) and a common end term (ET) worth 100 marks each. 

Each of MT and ET had at least four questions that were worth 5 marks, at least three questions that were worth 10 marks, and at least two questions that were worth 15 marks. 

5×4=20, 10×3=30, 15×2=30 

The total possible with considering the minimum number of questions of each type = 20+30+30=80 marks. Rest 20 marks are possible by the following cases: {5,5,5,5} {5,5,10} {10,10} {5,15} 

ET contained more questions than MT. 

Thus MT cannot consider the case {5,5,5,5} 

The number of questions in each case: 

1) {5,5,5,5} = 9+4 =13 questions 

2) {5,5,10} = 9+3 =12 questions 

3) {10,10} = 9+2 =11 questions 

4) {5,15} = 9+2 =11 questions 

Considering MT and ET together, each faculty member prepared the same number of questions. The total number of questions should be multiple of 6, thus the total number of questions will be 24. 

For ET and MT, there are 2 cases : 

{5,5,5,5}{5,15} 

{5,5,5,5}{10,10} 

According to the statement (i), Annie prepared the fifth question for both MT and ET. For MT, this question carried 5 marks. Thus {10,10} case is not possible. 

MT {5,5,5,5,5,10,10,10,15,15,15} 

ET {5,5,5,5,5,5,5,5,10,10,10,15,15} 

From statement (i),(ii),(iv),(v), every other faculty member prepared two questions for MT. we can create the following table: 

{ Annie(A), Beti(B), Chetan(C), Dave (D), Fakir(F) } 

There are 24 questions in total so each faculty will make 4 questions. 

We can create the following table for ET. 

Hence the correct option is D 

Q. 44 How many 5‐mark questions were there in MT and ET combined? 

A 13 

B 12 

C 10 

D Cannot be determined 

Answer: A 

Explanation: 

All six sections had a common midterm (MT) and a common end term (ET) worth 100 marks each. 

Each of MT and ET had at least four questions that were worth 5 marks, at least three questions that were worth 10 marks, and at least two questions that were worth 15 marks. 

5×4=20, 10×3=30, 15×2=30 

The total possible with considering the minimum number of questions of each type = 20+30+30=80 marks. Rest 20 marks are possible by the following cases: {5,5,5,5} {5,5,10} {10,10} {5,15} 

ET contained more questions than MT. 

Thus MT cannot consider the case {5,5,5,5} 

The number of questions in each case: 

1) {5,5,5,5} = 9+4 =13 questions 

2) {5,5,10} = 9+3 =12 questions 

3) {10,10} = 9+2 =11 questions 

4) {5,15} = 9+2 =11 questions 

Considering MT and ET together, each faculty member prepared the same number of questions. The total number of questions should be multiple of 6, thus the total number of questions will be 24. 

For ET and MT, there are 2 cases : 

{5,5,5,5}{5,15} 

{5,5,5,5}{10,10} 

According to the statement (i), Annie prepared the fifth question for both MT and ET. For MT, this question carried 5 marks. Thus {10,10} case is not possible. 

MT {5,5,5,5,5,10,10,10,15,15,15} 

ET {5,5,5,5,5,5,5,5,10,10,10,15,15} 

From statement (i),(ii),(iv),(v), every other faculty member prepared two questions for MT. we can create the following table: 

{ Annie(A), Beti(B), Chetan(C), Dave (D), Fakir(F) } 

There are 24 questions in total so each faculty will make 4 questions. 

We can create the following table for ET. 

Hence the correct option is A 

Q. 45 Who prepared 15-mark questions for MT and ET? 

A Only Beti, Dave, Esha and Fakir 

B Only Dave and Fakir 

C Only Esha and Fakir 

D Only Dave, Esha and Fakir 

Answer: D 

Explanation: 

All six sections had a common midterm (MT) and a common end term (ET) worth 100 marks each. 

Each of MT and ET had at least four questions that were worth 5 marks, at least three questions that were worth 10 marks, and at least two questions that were worth 15 marks. 

5×4=20, 10×3=30, 15×2=30 

The total possible with considering the minimum number of questions of each type = 20+30+30=80 marks. Rest 20 marks are possible by the following cases: {5,5,5,5} {5,5,10} {10,10} {5,15} 

ET contained more questions than MT. 

Thus MT cannot consider the case {5,5,5,5} 

The number of questions in each case: 

1) {5,5,5,5} = 9+4 =13 questions 

2) {5,5,10} = 9+3 =12 questions 

3) {10,10} = 9+2 =11 questions 

4) {5,15} = 9+2 =11 questions 

Considering MT and ET together, each faculty member prepared the same number of questions. The total number of questions should be multiple of 6, thus the total number of questions will be 24. 

For ET and MT, there are 2 cases : 

{5,5,5,5}{5,15} 

{5,5,5,5}{10,10} 

According to the statement (i), Annie prepared the fifth question for both MT and ET. For MT, this question carried 5 marks. Thus {10,10} case is not possible. 

MT {5,5,5,5,5,10,10,10,15,15,15} 

ET {5,5,5,5,5,5,5,5,10,10,10,15,15} 

From statement (i),(ii),(iv),(v), every other faculty member prepared two questions for MT. we can create the following table: 

{ Annie(A), Beti(B), Chetan(C), Dave (D), Fakir(F) } 

There are 24 questions in total so each faculty will make 4 questions. 

We can create the following table for ET. 

Hence the correct option is D 

Q. 46 Which of the following questions did Beti prepare in ET? 

A Seventh question 

B Fourth question 

C Ninth question 

D Tenth question 

Answer: D 

Explanation: 

All six sections had a common midterm (MT) and a common end term (ET) worth 100 marks each. 

Each of MT and ET had at least four questions that were worth 5 marks, at least three questions that were worth 10 marks, and at least two questions that were worth 15 marks. 

5×4=20, 10×3=30, 15×2=30 

The total possible with considering the minimum number of questions of each type = 20+30+30=80 marks. Rest 20 marks are possible by the following cases: {5,5,5,5} {5,5,10} {10,10} {5,15} 

ET contained more questions than MT. 

Thus MT cannot consider the case {5,5,5,5} 

The number of questions in each case: 

1) {5,5,5,5} = 9+4 =13 questions 

2) {5,5,10} = 9+3 =12 questions 

3) {10,10} = 9+2 =11 questions 

4) {5,15} = 9+2 =11 questions 

Considering MT and ET together, each faculty member prepared the same number of questions. The total number of questions should be multiple of 6, thus the total number of questions will be 24. 

For ET and MT, there are 2 cases : 

{5,5,5,5}{5,15} 

{5,5,5,5}{10,10} 

According to the statement (i), Annie prepared the fifth question for both MT and ET. For MT, this question carried 5 marks. Thus {10,10} case is not possible. 

MT {5,5,5,5,5,10,10,10,15,15,15} 

ET {5,5,5,5,5,5,5,5,10,10,10,15,15} 

From statement (i),(ii),(iv),(v), every other faculty member prepared two questions for MT. we can create the following table: 

{ Annie(A), Beti(B), Chetan(C), Dave (D), Fakir(F) } 

There are 24 questions in total so each faculty will make 4 questions. 

We can create the following table for ET. 

Hence the correct option is D 

Instructions 

Comprehension: 

Three pouches (each represented by a filled circle) are kept in each of the nine slots in a 3 × 3 grid, as shown in the figure. Every pouch has a certain number of one-rupee coins. The minimum and maximum amounts of money (in rupees) among the three pouches in each of the nine slots are given in the table. 

For example, we know that among the three pouches kept in the second column of the first row, the minimum amount in a pouch is Rs. 6 and the maximum amount is Rs. 8. 

There are nine pouches in any of the three columns, as well as in any of the three rows. It is known that the average amount of money (in rupees) kept in the nine pouches in any column or in any row is an integer. It is also known that the total amount of money kept in the three pouches in the first column of the third row is Rs. 4. 

Q. 47 What is the total amount of money (in rupees) in the three pouches kept in the first column of the second row? 

Answer:13 

Explanation: 

We can make the following table from “the total amount of money kept in the three pouches in the first column of the third row is Rs. 4.” 

If the minimum and maximum value are 1, then the sum of the three pouches in the middle will be Rs 3.

If we calculate the maximum and minimum value possible for each slot in column 1. For the slot, column 1 and row 1, the maximum value possible is 10{2,4,4} while the minimum value possible is 8{2,2,4}. 

Similarly, for the slot, column 1 and row 2, the maximum value possible is 13{3,5,5} while the minimum value possible is 11{3,3,5}. 

It is known that the average amount of money (in rupees) kept in the nine pouches in any column or in any row is an integer. Thus the sum of coins in a row or column must be a multiple of 9. 

So, we can iterate that 10,13,4 …{27} is the only sum possible for the slots of column 1. 

We now know two elements of row 2, thus we can iterate from the maximum and the minimum value possible for the slot {cloumn 3, row 2} that 38 is the only value possible for the slot. 

We can make the following table: 

Similarly, we can find the amount for Column 2. 

For the slot, column 2 and row 1, the maximum value possible is 22{6,8,8} while the minimum value possible is 20{6,6,8}. 

For the slot, column 2 and row 3, the maximum value possible is 5{1,2,3} while the minimum value possible is 4{1,1,2}. Thus {20,3,4} is the only solution possible. 

We can similarly make the following table for the last column. 

The total amount of money (in rupees) in the three pouches kept in the first column of the second row=13 Correct answer 13 

Q. 48 How many pouches contain exactly one coin? 

Answer:8 

Explanation: 

We can make the following table from “the total amount of money kept in the three pouches in the first column of the third row is Rs. 4.” 

If the minimum and maximum value are 1, then the sum of the three pouches in the middle will be Rs 3.

If we calculate the maximum and minimum value possible for each slot in column 1. For the slot, column 1 and row 1, the maximum value possible is 10{2,4,4} while the minimum value possible is 8{2,2,4}. 

Similarly, for the slot, column 1 and row 2, the maximum value possible is 13{3,5,5} while the minimum value possible is 11{3,3,5}. 

It is known that the average amount of money (in rupees) kept in the nine pouches in any column or in any row is an integer. Thus the sum of coins in a row or column must be a multiple of 9. 

So, we can iterate that 10,13,4 …{27} is the only sum possible for the slots of column 1. 

We now know two elements of row 2, thus we can iterate from the maximum and the minimum value possible for the slot {cloumn 3, row 2} that 38 is the only value possible for the slot. 

We can make the following table: 

Similarly, we can find the amount for Column 2. 

For the slot, column 2 and row 1, the maximum value possible is 22{6,8,8} while the minimum value possible is 20{6,6,8}. 

For the slot, column 2 and row 3, the maximum value possible is 5{1,2,3} while the minimum value possible is 4{1,1,2}. Thus {20,3,4} is the only solution possible. 

We can similarly make the following table for the last column. 

Answer 8 

Q. 49 What is the number of slots for which the average amount (in rupees) of its three pouches is an integer? 

Answer:2 

Explanation: 

We can make the following table from “the total amount of money kept in the three pouches in the first column of the third row is Rs. 4.” 

If the minimum and maximum value are 1, then the sum of the three pouches in the middle will be Rs 3.

If we calculate the maximum and minimum value possible for each slot in column 1. For the slot, column 1 and row 1, the maximum value possible is 10{2,4,4} while the minimum value possible is 8{2,2,4}. 

Similarly, for the slot, column 1 and row 2, the maximum value possible is 13{3,5,5} while the minimum value possible is 11{3,3,5}. 

It is known that the average amount of money (in rupees) kept in the nine pouches in any column or in any row is an integer. Thus the sum of coins in a row or column must be a multiple of 9. 

So, we can iterate that 10,13,4 …{27} is the only sum possible for the slots of column 1. 

We now know two elements of row 2, thus we can iterate from the maximum and the minimum value possible for the slot {cloumn 3, row 2} that 38 is the only value possible for the slot. 

We can make the following table: 

Similarly, we can find the amount for Column 2. 

For the slot, column 2 and row 1, the maximum value possible is 22{6,8,8} while the minimum value possible is 20{6,6,8}. 

For the slot, column 2 and row 3, the maximum value possible is 5{1,2,3} while the minimum value possible is 4{1,1,2}. Thus {20,3,4} is the only solution possible. 

We can similarly make the following table for the last column. 

Answer 2 

Q. 50 The number of slots for which the total amount in its three pouches strictly exceeds Rs. 10 is 

Answer:3 

Explanation: 

We can make the following table from “the total amount of money kept in the three pouches in the first column of the third row is Rs. 4.” 

If the minimum and maximum value are 1, then the sum of the three pouches in the middle will be Rs 3.

If we calculate the maximum and minimum value possible for each slot in column 1. For the slot, column 1 and row 1, the maximum value possible is 10{2,4,4} while the minimum value possible is 8{2,2,4}. 

Similarly, for the slot, column 1 and row 2, the maximum value possible is 13{3,5,5} while the minimum value possible is 11{3,3,5}. 

It is known that the average amount of money (in rupees) kept in the nine pouches in any column or in any row is an integer. Thus the sum of coins in a row or column must be a multiple of 9. 

So, we can iterate that 10,13,4 …{27} is the only sum possible for the slots of column 1. 

We now know two elements of row 2, thus we can iterate from the maximum and the minimum value possible for the slot {cloumn 3, row 2} that 38 is the only value possible for the slot. 

We can make the following table: 

Similarly, we can find the amount for Column 2. 

For the slot, column 2 and row 1, the maximum value possible is 22{6,8,8} while the minimum value possible is 20{6,6,8}. 

For the slot, column 2 and row 3, the maximum value possible is 5{1,2,3} while the minimum value possible is 4{1,1,2}. Thus {20,3,4} is the only solution possible. 

We can similarly make the following table for the last column. 

Answer 3 

Instructions 

Comprehension: 

Three doctors, Dr. Ben, Dr. Kane and Dr. Wayne visit a particular clinic Monday to Saturday to see patients. Dr. Ben sees each patient for 10 minutes and charges Rs. 100/-. Dr. Kane sees each patient for 15 minutes and charges Rs. 200/-, while Dr. Wayne sees each patient for 25 minutes and charges Rs. 300/-. The clinic has three rooms numbered 1, 2 and 3 which are assigned to the three doctors as per the following table. 

The clinic is open from 9 a.m. to 11.30 a.m. every Monday to Saturday. 

On arrival each patient is handed a numbered token indicating their position in the queue, starting with token number 1 every day. As soon as any doctor becomes free, the next patient in the queue enters that emptied room for consultation. If at any time, more than one room is free then the waiting patient enters the room with the smallest number. For example, if the next two patients in the queue have token numbers 7 and 8 and if rooms numbered 1 and 3 are free, then patient with token number 7 enters room number 1 and patient with token number 8 enters room number 3. 

Q. 51 What is the maximum number of patients that the clinic can cater to on any single day? 

A 12 

B 30 

C 31 

D 15 

Answer: C 

Explanation: 

If all the doctors served the patients one after the other, then in 2.5 hrs, Ben will serve 15 patients, Kane will serve 10 patients and Wayne will serve 6 patients. 

A total of 31 patients can be served on a particular day. 

Q. 52 The queue is never empty on one particular Saturday. Which of the three doctors would earn the maximum amount in consultation charges on that day? 

A Dr. Wayne 

BDr. Kane 

C Dr. Ben 

D Both Dr. Wayne and Dr. Kane 

Answer: B 

Explanation: 

If all the doctors served the patients one after the other, then in 2.5 hrs, Ben will serve 15 patients, Kane will serve 10 patients and Wayne will serve 6 patients. 

Ben will earn = 15*100=Rs 1500 

Kane will earn = 10*200=Rs 2000 

Wayne will earn = 6*300=Rs 1800 

So Kane will earn the maximum amount in consultation charges on that day. 

Option B 

Q. 53 Mr. Singh visited the clinic on Monday, Wednesday, and Friday of a particular week, arriving at 8:50 a.m. on each of the three days. His token number was 13 on all three days. On which day was he at the clinic for the maximum duration? 

A Same duration on all three days 

B Friday 

C Monday 

D Wednesday 

Answer: C 

Explanation: 

Mr Singh is 13th in the sequence on all the three days. 

The following table will show the sequence for Monday, Wednesday and Friday. 

He will stay the longest when the 13th guy is served by Doctor Wayne. 

From the table, on Monday he had to wait at the clinic for the maximum duration: till 10:15. Option C 

Q. 54 On a slow Thursday, only two patients are waiting at 9 a.m. After that two patients keep arriving at exact 15-minute intervals starting at 9:15 a.m. — i.e. at 9:15 a.m., 9:30 a.m., 9:45 a.m. etc. Then the total duration in minutes when all three doctors are simultaneously free is 

A 30 

B 10 

C 15 

D 0 

Answer: D 

Explanation: 

On Thursday, the preference order for the patients is Wayne, Ben and Kane. 

The first two customers will be served by Wayne and Ben. While Kane will be empty for the first 15 mins. Then he and Ben will serve the next two customers and Wayne will be empty for 5 minutes as shown in the figure below. 

As we can see that the cycle will repeat after every 30 mins. 

So all three doctors are never simultaneously free. 

Option D 

Instructions 

Comprehension: 

In the table below the check marks indicate all languages spoken by five people: Paula, Quentin, Robert, Sally and Terence. For example, Paula speaks only Chinese and English. 

These five people form three teams, Team 1, Team 2 and Team 3. Each team has either 2 or 3 members. A team is said to speak a particular language if at least one of its members speak that language. 

The following facts are known. 

(1) Each team speaks exactly four languages and has the same number of members. 

(2) English and Chinese are spoken by all three teams, Basque and French by exactly two teams and the other languages by exactly one team. 

(3) None of the teams include both Quentin and Robert. 

(4) Paula and Sally are together in exactly two teams. 

(5) Robert is in Team 1 and Quentin is in Team 3. 

Q. 55 Who among the following four is not a member of Team 2? 

A Paula 

B Terence 

C Quentin 

D Sally 

Answer: C 

Explanation: 

From statement 1 and 2,Each team speaks exactly four languages. English and Chinese are spoken by all three teams, Basque and French by exactly two teams and the other languages by exactly one team, multiple options are possible. 

In the following tables: A, B, C can be any team among Team 1,Team 2,Team 3. 

From the data given in the question, the person who speaks Arabic also speaks French. Thus the only option possible is ‘Table 2’. 

According to statement 4, “Paula and Sally are together in exactly two teams.” 

Sally knows Basque, thus, she will be in group A and B, with Paula. 

According to statement 5, Robert(Arabic) is in Team 1 and Quentin(Dutch) is in Team 3. 

Thus, Group C is Team 1 and Group A is Team 3. 

From the table, the correct option is C. 

Q. 56 Who among the following four people is a part of exactly two teams? 

A Paula 

B Quentin 

C Sally 

D Robert 

Answer: C 

Explanation: 

From statement 1 and 2,Each team speaks exactly four languages. English and Chinese are spoken by all three teams, Basque and French by exactly two teams and the other languages by exactly one team, multiple options are possible. 

In the following tables: A, B, C can be any team among Team 1,Team 2,Team 3. 

From the data given in the question, the person who speaks Arabic also speaks French. Thus the only option possible is ‘Table 2’. 

According to statement 4, “Paula and Sally are together in exactly two teams.” 

Sally knows Basque, thus, she will be in group A and B, with Paula. 

According to statement 5, Robert(Arabic) is in Team 1 and Quentin(Dutch) is in Team 3. 

Thus, Group C is Team 1 and Group A is Team 3. 

From the table, the correct option is C. 

Q. 57 Who among the five people is a member of all teams? 

A Terence 

B Sally 

C Paula 

D No one 

Answer: C 

Explanation: 

From statement 1 and 2,Each team speaks exactly four languages. English and Chinese are spoken by all three teams, Basque and French by exactly two teams and the other languages by exactly one team, multiple options are possible. 

In the following tables: A, B, C can be any team among Team 1,Team 2,Team 3. 

From the data given in the question, the person who speaks Arabic also speaks French. Thus the only option possible is ‘Table 2’. 

According to statement 4, “Paula and Sally are together in exactly two teams.” 

Sally knows Basque, thus, she will be in group A and B, with Paula. 

According to statement 5, Robert(Arabic) is in Team 1 and Quentin(Dutch) is in Team 3. 

Thus, Group C is Team 1 and Group A is Team 3. 

From the table, the correct option is C. 

Q. 58 Apart from Chinese and English, which languages are spoken by Team 1? 

A Arabic and French 

B Basque and French 

C Arabic and Basque 

D Basque and Dutch 

Answer: A 

Explanation: 

From statement 1 and 2,Each team speaks exactly four languages. English and Chinese are spoken by all three teams, Basque and French by exactly two teams and the other languages by exactly one team, multiple options are possible. 

In the following tables: A, B, C can be any team among Team 1,Team 2,Team 3. 

From the data given in the question, the person who speaks Arabic also speaks French. Thus the only option possible is ‘Table 2’. 

According to statement 4, “Paula and Sally are together in exactly two teams.” 

Sally knows Basque, thus, she will be in group A and B, with Paula. 

According to statement 5, Robert(Arabic) is in Team 1 and Quentin(Dutch) is in Team 3. 

Thus, Group C is Team 1 and Group A is Team 3. 

From the table, the correct option is A. 

Instructions 

Comprehension: 

A large store has only three departments, Clothing, Produce, and Electronics. The following figure shows the percentages of revenue and cost from the three departments for the years 2016, 2017 and 2018. The dotted lines depict percentage levels. So for example, in 2016, 50% of the store’s revenue came from its Electronics department while 40% of its costs were incurred in the Produce department. 

In this setup, Profit is computed as (Revenue – Cost) and Percentage Profit as Profit/Cost × 100%. It is known that 

1. The percentage profit for the store in 2016 was 100%. 

2. The store’s revenue doubled from 2016 to 2017, and its cost doubled from 2016 to 2018. 

3. There was no profit from the Electronics department in 2017. 

4. In 2018, the revenue from the Clothing department was the same as the cost incurred in the Produce department. 

 

Q. 59 What was the percentage profit of the store in 2018? 

Answer:25 

Explanation: 

We can make the following table from the web chart given in the question:

If we consider the total cost in the year 2016 as 100, then according to Statement 1, the total revenue in 2016 must be 200. 

The store’s revenue doubled from 2016 to 2017, thus the total revenue in the year 2017 = 400. We can find the revenue for the individual department in the year 2017, from the table. There was no profit from the Electronics department in 2017, thus, we can find the total cost in 2017= 300 Considering the statement 4, we can find the total revenue in 2018 and tabulate the following table. 

The percentage profit of the store in 2018= (250-200)/200= 25% 

Q. 60 What was the ratio of revenue generated from the Produce department in 2017 to that in 2018? 

A 16 : 9 

B 4 : 3 

C 9 : 16 

D 8 : 5 

Answer: D 

Explanation: 

We can make the following table from the web chart given in the question: 

If we consider the total cost in the year 2016 as 100, then according to Statement 1, the total revenue in 2016 must be 200. The store’s revenue doubled from 2016 to 2017, thus the total revenue in the year 2017 = 400. We can find the revenue for the individual department in the year 2017, from the table. There was no profit from the Electronics department in 2017, thus, we can find the total cost in 2017= 300 Considering the statement 4, we can find the total revenue in 2018 and tabulate the following table.

The ratio of revenue generated from the Produce department in 2017 to that in 2018 = 160:100= 8:5 

Q. 61 What percentage of the total profits for the store in 2016 was from the Electronics department? 

Answer:70 

Explanation: 

We can make the following table from the web chart given in the question: 

If we consider the total cost in the year 2016 as 100, then according to Statement 1, the total revenue in 2016 must be 200. The store’s revenue doubled from 2016 to 2017, thus the total revenue in the year 2017 = 400. We can find the revenue for the individual department in the year 2017, from the table. There was no profit from the Electronics department in 2017, thus, we can find the total cost in 2017= 300 Considering the statement 4, we can find the total revenue in 2018 and tabulate the following table. 

Profit in 2016 = 200-100= 100 

Profit in the electronic department in 2016= 100-30=70 

The total profits= for the store in 2016 were from the Electronics department 70% 

Q. 62 What was the approximate difference in profit percentages of the store in 2017 and 2018? 

A 15.5 

B 25.0 

C 8.3 

D 33.3 

Answer: C 

Explanation: 

We can make the following table from the web chart given in the question: 

If we consider the total cost in the year 2016 as 100, then according to Statement 1, the total revenue in 2016 must be 200. The store’s revenue doubled from 2016 to 2017, thus the total revenue in the year 2017 = 400. We can find the revenue for the individual department in the year 2017, from the table. There was no profit from the Electronics department in 2017, thus, we can find the total cost in 2017= 300 Considering the statement 4, we can find the total revenue in 2018 and tabulate the following table. 

Profit percentage in 2017= (400-300)/300 %= 33.33% 

Profit percentage in 2018= (250-200)/200 %= 25% 

The approximate difference in profit percentages of the store in 2017 and 2018= (33.33-25)%= 8.33% Option C. 

Instructions 

Comprehension: 

Students in a college are discussing two proposals — 

A: a proposal by the authorities to introduce dress code on campus, and 

B: a proposal by the students to allow multinational food franchises to set up outlets on college campuses. A student does not necessarily support either of the two proposals. 

In an upcoming election for student union president, there are two candidates in fray: 

Sunita and Ragini. Every student prefers one of the two candidates. 

A survey was conducted among the students by picking a sample of 500 students. The following information was noted from this survey. 

1. 250 students supported proposal A and 250 students supported proposal B. 

2. Among the 200 students who preferred Sunita as student union president, 80% supported proposal A. 

3. Among those who preferred Ragini, 30% supported proposal A. 

4. 20% of those who supported proposal B preferred Sunita. 

5. 40% of those who did not support proposal B preferred Ragini. 

6. Every student who preferred Sunita and supported proposal B also supported proposal A. 

7. Among those who preferred Ragini, 20% did not support any of the proposals. 

Q. 63 Among the students surveyed who supported proposal A, what percentage preferred Sunita for student union president? 

Answer:64 

Explanation: 

Total number of students surveyed= 500 

Every student prefers one of the two candidates. Ragini(R) and Sunita(S). 

Thus, R+S=500. 

According to statement 2, “Among the 200 students who preferred Sunita as student union president, 80% supported proposal A.” 

The number of students who support Sunita(S)=200 

The number of students who supported Ragini(R)=300 

According to statements 2 and 3, 160 students who supported Sunita also supported the proposal A & 90 students who supported Ragini also supported proposal A. 

According to statements 4 and 6, we can make the following Venn diagram for Sunita.

According to statement 5 and 7, we can make the following Venn diagram. 

The number of students who preferred Sunita and the proposal A=160 

=160/250= 64% 

Q. 64 What percentage of the students surveyed who did not support proposal A preferred Ragini as student union president? 

Answer:84 

Explanation: 

Total number of students surveyed= 500 

Every student prefers one of the two candidates. Ragini(R) and Sunita(S). 

Thus, R+S=500. 

According to statement 2, “Among the 200 students who preferred Sunita as student union president, 80% supported proposal A.” 

The number of students who support Sunita(S)=200 

The number of students who supported Ragini(R)=300 

According to statements 2 and 3, 160 students who supported Sunita also supported the proposal A & 90 students who supported Ragini also supported proposal A. 

According to statements 4 and 6, we can make the following Venn diagram for Sunita. 

According to statement 5 and 7, we can make the following Venn diagram. 

The percentage of the students surveyed who did not support proposal A preferred Ragini as student union president = 210/250=84% 

Answer 84 

Q. 65 What percentage of the students surveyed who supported both proposals A and B preferred Sunita as student union president? 

A 40 

B 25 

C 20 

D 50 

Answer: D 

Explanation: 

Total number of students surveyed= 500 

Every student prefers one of the two candidates. Ragini(R) and Sunita(S). 

Thus, R+S=500. 

According to statement 2, “Among the 200 students who preferred Sunita as student union president, 80% supported proposal A.” 

The number of students who support Sunita(S)=200 

The number of students who supported Ragini(R)=300 

According to statements 2 and 3, 160 students who supported Sunita also supported the proposal A & 90 students who supported Ragini also supported proposal A. 

According to statements 4 and 6, we can make the following Venn diagram for Sunita. 

According to statement 5 and 7, we can make the following Venn diagram. 

According to the Venn diagram, the students surveyed who supported both proposals A and B preferred Sunita as 50 student union president 50/50+50 % =50% 

Q. 66 How many of the students surveyed supported proposal B, did not support proposal A and preferred Ragini as student union president? 

A 150 

B 210 

C 200 

D 40 

Answer: A 

Explanation: 

Total number of students surveyed= 500 

Every student prefers one of the two candidates. Ragini(R) and Sunita(S). 

Thus, R+S=500. 

According to statement 2, “Among the 200 students who preferred Sunita as student union president, 80% supported proposal A.” 

The number of students who support Sunita(S)=200 

The number of students who supported Ragini(R)=300 

According to statements 2 and 3, 160 students who supported Sunita also supported the proposal A & 90 students who supported Ragini also supported proposal A. 

According to statements 4 and 6, we can make the following Venn diagram for Sunita. 

According to statement 5 and 7, we can make the following Venn diagram. 

From the diagram, we can understand that option A is correct. 

Quantitative Aptitude 

Instructions 

For the following questions answer them individually 

Q. 67 The average of 30 integers is 5. Among these 30 integers, there are exactly 20 which do not exceed 5. What is the highest possible value of the average of these 20 integers? 

A 3.5 

B 5 

C 4.5 

D 4 

Answer: C 

Explanation: 

It is given that the average of the 30 integers = 5 

Sum of the 30 integers = 30*5=150 

There are exactly 20 integers whose value is less than 5. 

To maximise the average of the 20 integers, we have to assign minimum value to each of the remaining 10 integers So the sum of 10 integers = 10*6=60 

The sum of the 20 integers = 150-60= 90 

Average of 20 integers =90/20 = 4.5 

Q. 68 Amal invests Rs 12000 at 8% interest, compounded annually, and Rs 10000 at 6% interest, compounded semi-annually, both investments being for one year. Bimal invests his money at 7.5% simple interest for one year. If Amal and Bimal get the same amount of interest, then the amount, in Rupees, invested by Bimal is 

Answer:20920 

Explanation: 

The amount with Amal at the end of 1 year = 12000*1.08+10000*1.03*1.03=23569 

Interest received by Amal = 23569-22000=1569 

Let the amount invested by Bimal = 100b 

Interest received by Bimal = 100b*7.5*1/100=7.5b 

It is given that the amount of interest received by both of them is the same 

7.5b=1569 

b=209.2 

Amount invested by Bimal = 100b = 20920 

Q. 69 What is the largest positive integer n such that (n2 +7n+12)/(n2n−12) is also a positive integer?

A 6 

B 16 

C 8 

D 12 

Answer: D 

Explanation: 

= (n2 +3n+4n+12) /(n2 −4n+3n−12)

= [n(n+3)+4(n+3)]/[n(n−4)+3(n−4)]

=[(n+4)(n+3)]/[(n−4)(n+3)]

=( n+4)/(n−4)

=( n−4)+8 / (n−4)

= 1 + 8/(n−4) which will be maximum when n-4 =8 

n=12 

D is the correct answer. 

Q. 70 How many pairs (m, n) of positive integers satisfy the equation m2 + 105 = n2

Answer:4 

Explanation: 

n2 m2 = 105 

(n-m)(n+m) = 1*105, 3*35, 5*21, 7*15, 15*7, 21*5, 35*3, 105*1. 

n-m=1, n+m=105 ==> n=53, m=52 

n-m=3, n+m=35 ==> n=19, m=16 

n-m=5, n+m=21 ==> n=13, m=8 

n-m=7, n+m=15 ==> n=11, m=4 

n-m=15, n+m=7 ==> n=11, m=-4 

n-m=21, n+m=5 ==> n=13, m=-8 

n-m=35, n+m=3 ==> n=19, m=-16 

n-m=105, n+m=1 ==> n=53, m=-52 

Since only positive integer values of m and n are required. There are 4 possible solutions. 

Q. 71 Two ants A and B start from a point P on a circle at the same time, with A moving clockwise and B moving anti-clockwise. They meet for the first time at 10:00 am when A has covered 60% of the track. If A returns to P at 10:12 am, then B returns to P at 

A 10:25 am 

B 10:45 am 

C 10:18 am 

D 10:27 am 

Answer: D 

Explanation: 

When A and B met for the first time at 10:00 AM, A covered 60% of the track. 

So B must have covered 40% of the track. 

It is given that A returns to P at 10:12 AM i.e A covers 40% of the track in 12 minutes 

60% of the track in 18 minutes 

B covers 40% of track when A covers 60% of the track. 

B covers 40% of the track in 18 minutes. 

B will cover the rest 60% in 27 minutes, hence it will return to B at 10:27 AM 

Q. 72 Let a1, a2, … be integers such that a1a2 + a3a4 + …. + (−1)n−1an = n, for all  n ≥ 1.Then a51 + a52 + …. + a1023  equals 

A 0 

B 1 

C 10 

D -1 

Answer: B 

Explanation: 

a1a2 + a3a4 + …. + (−1)n−1an =

It is clear from the above equation that when n is odd, the coefficient of a is positive otherwise negative. 

a1a2 = 2 

a1 = a2 + 2 

a1a2 + a3 = 3 

On substituting the value of a1 in the above equation, we get 

a3 = 1 

a1a2 + a3a4 = 4 

On substituting the values of a1, a3 in the above equation, we get 

a4 = -1

a1a2 + a3a4 + a5 = 5 

On substituting the values of a1, a3, a4 in the above equation, we get 

a5 = 1

So we can conclude that a3, a5, a7….an+1= 1 and a2, a4, a6.a2n = -1 

Now we have to find the value of a51 + a52 + …. + a1023 

Number of terms = 1023=51+(n-1)1 

n=973 

There will be 486 even and 487 odd terms, so the value of a51 + a52 + …. + a1023 = 486*-1+487*1=1 

Q. 73 Two circles, each of radius 4 cm, touch externally. Each of these two circles is touched externally by a third circle. If these three circles have a common tangent, then the radius of the third circle, in cm, is 

A 1/√2

B π/3

C √2 

D 1 

Answer: D 

Explanation: 

Let ‘h’ be the height of the triangle ABC 

Area of triangle ABC =√[( 8 + r) × 4 × 4 × r] =½  × (4 + 4) × height 

Height (h) = √(8 + r) r

√[(8 + r) r] + r = 4

√[(8 + r) r] = 4 – r

16r=16 , r=1 

Q. 74 Let A be a real number. Then the roots of the equation x2 − 4x log2A = 0 

 are real and distinct if and only if 

A A > 1/16

B  A < 1/16

C A > 1/8

D A < 1/8 

Answer: A 

Explanation: 

The roots of x2 − 4x log2A = 0 will be real and distinct if and only if the discriminant is greater than zero 

16+4*log2A > 0 

log2A > -4 

A > 1/16 

Q. 75 The quadratic equation x2 + bx + c = 0 has two roots 4a and 3a, where a is an integer. Which of the following is a possible value of b2 + c

A 3721 

B 361 

C 427 

D 549 

Answer: D 

Explanation: 

Given, 

The quadratic equation x2 + bx + c = 0  has two roots 4a and 3a 

7a=-b 

12a2 = c 

We have to find the value of b2 + c = 49a2  + 12a2  =61a2  

Now lets verify the options 

61a2  = 3721 ==> a= 7.8 which is not an integer 

61a2  = 361 ==> a= 2.42 which is not an integer 

61a2  = 427 ==> a= 2.64 which is not an integer 

61a2  = 3721 ==> a= 3 which is an integer 

Q. 76 The base of a regular pyramid is a square and each of the other four sides is an equilateral triangle, the length of each side being 20 cm. The vertical height of the pyramid, in cm, is 

A 12 

B 10√2

C 8√3

D 5√5

Answer: B 

Explanation: 

It is given that the base of the pyramid is square and each of the four sides are equilateral triangles. Length of each side of the equilateral triangle = 20cm 

Since the side of the triangle will be common to the square as well, the side of the square = 20cm

Let h be the vertical height of the pyramid ie OA 

OB = 10 since it is half the side of the square 

AB is the height of the equilateral triangle i.e 10√3 

AOB is a right angle, so applying the Pythagorean formula, we get OA2 + OB2 = AB2 

h2 + 100= 300

h=10√2

Q. 77 Let ABC be a right-angled triangle with hypotenuse BC of length 20 cm. If AP is perpendicular on BC, then the maximum possible length of AP, in cm, is 

A 10 

B 5 

C 8√2

D 6√2

Answer: A 

Explanation: 

Let p be the length of AP. 

It is given that ∠ BAC = 900 and ∠ APC = 900 

Let ∠ ABC = θ , then ∠ BAP = 900θ and ∠ BCA = 900θ 

So ∠ PAC = θ 

Triangles BPA and APC are similar 

p2 = x (20 − x

We have to maximize the value of p, which will be maximum when x=20-x 

x=10 

Q. 78 If x is a real number, then √[loge (4xx2)/3] is areal number if and only if 

A 1 ≤ x ≤ 3

B 1 ≤ x ≤ 2

C −1 ≤ x ≤ 3

D −3 ≤ x ≤ 3 

Answer: A 

Explanation: 

√[loge (4xx2)/3] will be real if loge(4xx2) /3 ≥ 0

(4xx2) /3 > 1

4x x2 − 3 > 0 

x2 − 4x + 3 < 0 

1< x< 3 

Q. 79 If 5x − 3y = 13438 and 5x−1 + 3y+1 = 9686, then x + y equals 

Answer:13 

Explanation: 

5x − 3y = 13438 and 5x−1 + 3y+1 = 9686 

5x + 3y ∗ 15 = 9686 ∗ 5 

5x + 3y ∗ 15 = 48430 

16* 3y =34992 

3y = 2187 

y = 7 

5x =13438+2187=15625 

x=6 

x+y = 13 

Q. 80 The strength of a salt solution is p% if 100 ml of the solution contains p grams of salt. Each of three vessels A, B, C contains 500 ml of salt solution of strengths 10%, 22%, and 32%, respectively. Now, 100 ml of the solution in vessel A is transferred to vessel B. Then, 100 ml of the solution in vessel B is transferred to vessel C. Finally, 100 ml of the solution in vessel C is transferred to vessel A. The strength, in percentage, of the resulting solution in vessel A is 

A 15 

B 13 

C 12 

D 14 

Answer: D 

Explanation: 

Each of three vessels A, B, C contains 500 ml of salt solution of strengths 10%, 22%, and 32%, respectively. The amount of salt in vessels A, B, C = 50 ml, 110 ml, 160 ml respectively. 

The amount of water in vessels A, B, C = 450 ml, 390 ml, 340 ml respectively. 

In 100 ml solution in vessel A, there will be 10ml of salt and 90 ml of water 

In 100 ml of solution in vessel B, there will be 22 ml of salt and 78 ml of water. 

In 100 ml of solution in vessel C, there will be 32 ml of salt and 68 ml of water. 

Now, 100 ml of the solution in vessel A is transferred to vessel B. Then, 100 ml of the solution in vessel B is transferred to vessel C. Finally, 100 ml of the solution in vessel C is transferred to vessel A i.e after the first transfer, the amount of salt in vessels A, B, C = 40, 120, 160 ml respectively. after the second transfer, the amount of salt in vessels A, B, C =40, 98, 182 ml respectively. After the third transfer, the amount of salt in vessels A, B, C = 72, 98, 150 respectively. 

Percentage of salt in vessel A =72/500 × 100=14.4 

Q. 81 A cyclist leaves A at 10 am and reaches B at 11 am. Starting from 10:01 am, every minute a motorcycle leaves A and moves towards B. Forty-five such motorcycles reach B by 11 am. All motorcycles have the same speed. If the cyclist had doubled his speed, how many motorcycles would have reached B by the time the cyclist reached B? 

A 22 

B 23 

C 15 

D 20 

Answer: C 

Explanation: 

It is given that starting from 10:01 am, every minute a motorcycle leaves A and moves towards B. Forty-five such motorcycles reach B by 11 am. 

It means that the forty-fifth motorcycle starts at 10:45 AM at A and reaches B by 11:00 AM i.e 15 minutes. Since the speed of all the motorcycles is the same, all the motorcycles will take the same duration i.e 15 minutes. 

If the cyclist doubles the speed, then he will reach B by 10:30 AM. (Since if the speed is doubled, time is reduced by half) 

Since each motorcycle takes 15 minutes to reach B, 15 motorcycles would have reached B by the time the cyclist reaches B 

Q. 82 A man makes complete use of 405 cc of iron, 783 cc of aluminium, and 351 cc of copper to make a number of solid right circular cylinders of each type of metal. These cylinders have the same volume and each of these has radius 3 cm. If the total number of cylinders is to be kept at a minimum, then the total surface area of all these cylinders, in sq cm, is 

A 1026(1 + π)

B 8464π

C 928π

D 1044(4 + π

Answer: A 

Explanation: 

It is given that the volume of all the cylinders is the same, so the volume of each cylinder = HCF of (405, 783, 351) =27 

The number of iron cylinders =405/27 = 15 

The number of aluminium cylinders = 783/27= 29 

The number of copper cylinders =351/27 = 13 

15*π r2h = 405 

15*π 9 ∗ h = 405 

π h = 3 

Now we have to calculate the total surface area of all the cylinders 

Total number of cylinders = 15+29+13 = 57 

Total surface area of the cylinder = 57*(2π rh + 2π r2

=57(2*3*3 + 2*9*π  ) 

= 1026(1 + π)

 

Q. 83 The real root of the equation 26x + 23x+2 − 21 = 0 is 

A log2 9

B (log2 3)/3

C log2 27

D (log2 7)/3 

Answer: B 

Explanation: 

Let 23x = v 

26x + 23x+2 − 21 = 0 

= v2 + 4v − 21 = 0 

=(v+7)(v-3)=0 

v=3, -7 

23x = 3 or 23x = -7(This can be negated) 

3x= log2 3

x= log2 3 /3 

 

Q. 84 How many factors of 24 × 35 × 104 are perfect squares which are greater than 1? Answer:44 

Explanation: 

24 × 35 × 104 

= 24 × 35 × 24∗ 54

=28 × 35 × 54 

For the factor to be a perfect square, the factor should be even power of the number. 

In 28  , the factors which are perfect squares are 20 , 22 , 24, 26 , 28= 5 

Similarly, in 35, the factors which are perfect squares are 30, 32, 34= 3 

In 54, the factors which are perfect squares are 50 , 52 , 54 = 3 

Number of perfect squares greater than 1 = 5*3*3-1 

=44 

 

Q. 85 In a six-digit number, the sixth, that is, the rightmost, digit is the sum of the first three digits, the fifth digit is the sum of first two digits, the third digit is equal to the first digit, the second digit is twice the first digit and the fourth digit is the sum of 

fifth and sixth digits. Then, the largest possible value of the fourth digit is 

Answer:7 

Explanation: 

Let the six-digit number be ABCDEF 

F = A+B+C, E= A+B, C=A, B= 2A, D= E+F 

D=2A+2B+C=6A+C, we have to find the largest possible value of D 

The only possible value of A is 1 because if A = 2, D = 12+C even if C is 0 D is two digit number If A= 1, then C = 1 

D = 6*1+1=7 

 

Q. 86 John jogs on track A at 6 kmph and Mary jogs on track B at 7.5 kmph. The total length of tracks A and B is 325 metres. While John makes 9 rounds of track A, Mary makes 5 rounds of track B. In how many seconds will Mary make one round of track A? 

Answer:48 

Explanation: 

Speed of John = 6kmph 

Speed of Mary = 7.5 kmph 

Lengths of tracks A and B = 325 m 

Let the length of track A be a, then the length of track B = 325-a 

9 rounds of John on track A = 5 rounds of Mary on track B 

(9× a)/(6 × 5/18) = 5⋅(325−a) / 7.5× 5/18

On solving we get , 13a=1300 

a=100 

The length of track A = 100m, track B = 225m 

Mary makes one round of track A =100 / 7.5× 5/18= 48 sec 

 

Q. 87 In 2010, a library contained a total of 11500 books in two categories – fiction and nonfiction. In 2015, the library contained a total of 12760 books in these two categories. During this period, there was a 10% increase in the fiction category while there was a 12% increase in the non-fiction category. How many fiction books were in the library in 2015? 

A 6160 

B 6600 

C 6000 

D 5500 

Answer: B 

Explanation: 

Let the number of fiction and non-fiction books in 2010 = 100a, 100b respectively 

It is given that the total number of books in 2010 = 11500 

100a+100b = 11500 ——-Eq 1 

The number of fiction and non-fiction books in 2015 = 110a, 112b respectively 

110a+112b = 12760 ——-Eq 2 

On solving both the equations we get, b=55, a= 60 

The number of fiction books in 2015 = 110*60=6600 

 

Q. 88 John gets Rs 57 per hour of regular work and Rs 114 per hour of overtime work. He works all together 172 hours and his income from overtime hours is 15% of his income from regular hours. Then, for how many hours did he work overtime? 

Answer:12 

Explanation: 

It is given that John works altogether 172 hours i.e including regular and overtime hours. 

Let a be the regular hours, 172-a will be the overtime hours 

John’s income from regular hours = 57*a 

John’s income for working overtime hours = (172-a)*144 

It is given that his income from overtime hours is 15% of his income from regular hours 

a*57*0.15 = (172-a)*114 

a=160 

The number of hours for which he worked overtime = 172-160=12 hrs 

 

Q. 89 If (2n + 1) + (2n + 3) + (2n + 5) + … + (2n + 47) = 5280, then What is the value of 1 + 2 + 3 + .. + n?

Answer:4851 

Explanation: 

Let us first find the number of terms 

47=1+(n-1)2 

n=24 

24*2n+1+3+5+….47=5280 

48n+576=5280 

48n=4704 

n=98 

Sum of first 98 terms = 98*99/2 

=4851 

 

Q. 90 A shopkeeper sells two tables, each procured at cost price p, to Amal and Asim at a profit of 20% and at a loss of 20%, respectively. Amal sells his table to Bimal at a profit of 30%, while Asim sells his table to Barun at a loss of 30%. If the amounts paid by Bimal and Barun are x and y, respectively, then (x − y) / p equals 

A 1 

B 1.2 

C 0.50 

D 0.7 

Answer: A 

Explanation: 

CP of the table at which the shopkeeper procured each table = p 

It is given that shopkeeper sold the tables to Amal and Asim at a profit of 20% and at a loss of 20%, respectively The selling price of the tables = 1.2p and 0.8p to Amal and Asim respectively. 

Amal sells his table to Bimal at a profit of 30% 

So, CP of the table by Bimal (x)= 1.2p*1.3 = 1.56p 

Asim sells his table to Barun at a loss of 30% 

So, CP of the table by Barun (y)= 0.7*0.8p = 0.56p 

(x-y)/p = (1.56p-0.56p)/p = p/p=1 

 

Q. 91 In a triangle ABC, medians AD and BE are perpendicular to each other, and have lengths 12 cm and 9 cm, respectively. Then, the area of triangle ABC, in sq cm, is 

A 78 

B 80 

C 72 

D 68 

Answer: C 

Explanation: 

It is given that AD and BE are medians which are perpendicular to each other. 

The lengths of AD and BE are 12cm and 9cm respectively. 

It is known that the centroid G divides the median in the ratio of 2:1 

Area of △ABC = 2* Area of the triangle ABD 

Area of △ABD = Area of △AGB + Area of △BGD 

AGB = ∠ BGD = 90 

Since (Given) 

Area of △AGB = ½ ×8 × 6= 24 

Area of △BGD = ½× 6 × 4= 12 

Area of △ABD = 24 + 12 = 36

Area of △ ABC = 2 × 36 = 72 

 

Q. 92 The number of common terms in the two sequences: 15, 19, 23, 27, . . . . , 415 and 14, 19, 24, 29, . . . , 464 is 

A 21 

B 20 

C 18 

D 19 

Answer: B 

Explanation: 

A: 15, 19, 23, 27, . . . . , 415 

B: 14, 19, 24, 29, . . . , 464 

Here the first common term = 19 

Common difference = LCM of 5, 4=20 

19+(n-1)20 ≤ 415 

(n-1)20 ≤ 396

(n-1) ≤ 19.8 

n=20 

 

Q. 93 Let a, b, x, y be real numbers such that a2 + b2 = 25, x2 + y2 = 169, and ax + by = 65. If k = ay bx, then 

A 0 < k 5/13

B k 5/13

C k = 5/13

D k = 0 

Answer: D 

 

Q. 94 Mukesh purchased 10 bicycles in 2017, all at the same price. He sold six of these at a profit of 25% and the remaining four at a loss of 25%. If he made a total profit of Rs. 2000, then his purchase price of a bicycle, in Rupees, was 

A 6000 

B 8000 

C 4000 

D 2000 

Answer: C 

Explanation: 

Let the cost of each bicycle= 100b 

CP of 10 bicycles = 1000b 

It is given that he sold six of these at a profit of 25% and the remaining four at a loss of 25% 

SP of 10 bicycles = 125b*6+75b*4 

=1050b 

Profit = 1050b-1000b =50b 

50b=2000 

CP = 100b = 4000 

 

Q. 95 In an examination, the score of A was 10% less than that of B, the score of B was 25% more than that of C, and the score of C was 20% less than that of D. If A scored 72, then the score of D was 

Answer:80 

Explanation: 

Let the score of D = 100d 

The score of C = 20% less than that of D = 80d 

The score of B = 25% more than C = 100d 

The score of A = 10% less than B =90d 

90d=72 

100d= 72*100/90 

= 80 

 

Q. 96 The salaries of Ramesh, Ganesh and Rajesh were in the ratio 6:5:7 in 2010, and in the ratio 3:4:3 in 2015. If Ramesh’s salary increased by 25% during 2010-2015, then the percentage increase in Rajesh’s salary during this period is closest to 

A 10 

B 7 

C 9 

D 8 

Answer: B 

Explanation: 

Let the salaries of Ramesh, Ganesh and Rajesh in 2010 be 6x, 5x, 7x respectively 

Let the salaries of Ramesh, Ganesh and Rajesh in 2015 be 3y, 4y, 3y respectively 

It is given that Ramesh’s salary increased by 25% during 2010-2015,3y = 1.25*6x 

y=2.5x 

Percentage increase in Rajesh’s salary = 7.5-7/7=0.07 

=7% 

 

Q. 97 

Let A and B be two regular polygons having a and b sides, respectively. If b = 2a and each interior angle of 3/2 B is times each interior angle of A, then each interior angle, in degrees, of a regular polygon with a + b sides is 

Answer:150 

Explanation: 

Each interior angle in an n-sided polygon = (n−2)180/

It is given that each interior angle of B is 3/2 times each interior angle of A and b = 2a 

(b−2)180 / b = 3/2 ×(a−2)180 / a

2 × (b − 2) × a = 3 × (a − 2) ×

2(ab-2a) = 3(ab-2b) 

ab-6b+4a=0 

a*2a-12a+4a=0 

2a2 − 8a = 0 

a(2a-8) = 0 

a cannot be zero so 2a=8 

a=4, b = 4*2=8 

a+b = 12 

Each interior angle of a regular polygon with 12 sides =[(12−2)× 180] /12=150 

 

Q. 98 Let f be a function such that f (mn) = f (m) f (n) for every positive integers m and n. If f (1), f (2) and f (3) are positive integers, f (1) < f (2), and f (24) = 54, then f (18) equals 

Answer:12 

Explanation: 

Given, f(mn) = f(m)f(n) 

when m= n= 1, f(1) = f(1)*f(1) ==> f(1) = 1 

when m=1, n= 2, f(2) = f(1)*f(2) ==> f(1) = 1 

when m=n= 2, f(4) = f(2)*f(2) ==> f(4) = [f(2)]2 

Similarly f(8) = f(4)*f(2) = [f(2)]3 

f(24) = 54 

[f(2)]3 * [f(3)] = 33 ∗ 2

On comparing LHS and RHS, we get 

f(2) = 3 and f(3) = 2 

Now we have to find the value of f(18) 

f(18) = [f(2)] * [f(3)]2 

= 3*4=12 

 

Q. 99 Anil alone can do a job in 20 days while Sunil alone can do it in 40 days. Anil starts the job, and after 3 days, Sunil joins him. Again, after a few more days, Bimal joins them and they together finish the job. If Bimal has done 10% of the job, then in how many days was the job done? 

A 12 

B 13 

C15 

D 14 

Answer: B 

Explanation: 

Let the total work be LCM of 20, 40 = 40 units 

Efficiency of Anil and Sunil is 2 units and 1 unit per day respectively. 

Anil works alone for 3 days, so Anil must have completed 6 units. 

Bimal completes 10% of the work while working along with Anil and Sunil. 

Bimal must have completed 4 units. 

The remaining 30 units of work is done by Anil and Sunil 

Number of days taken by them 30/3=10 

The total work is completed in 3+10=13 days 

 

Q. 100 In an examination, Rama’s score was one-twelfth of the sum of the scores of Mohan and Anjali. After a review, the score of each of them increased by 6. The revised scores of Anjali, Mohan, and Rama were in the ratio 11:10:3. Then Anjali’s score exceeded Rama’s score by 

A 26 

B 32 

C 35 

D 24 

Answer: B 

Explanation: 

Let the scores of Rama, Anjali and Mohan be r, a, m. 

It is given that Rama’s score was one-twelfth of the sum of the scores of Mohan and Anjali 

r= (m+a)/12 

The scores of Rama, Anjali and Mohan after review = r+6, a+6, m+6 

a+6:m+6:r+6 = 11:10:3 

a-r=8x 

3x-6= (21x−12)/12 

12(3x-6) = 21x-12 

x=4 

Anjali’s score exceeds Rama’s score by 8x=32 

CAT Previous Year Paper Session-I 2019

CAT 2019 Session-I 

Verbal Ability 

Instructions 

Scientists recently discovered that Emperor Penguins—one of Antarctica’s most celebrated species—employ a particularly unusual technique for surviving the daily chill. As detailed in an article published today in the journal Biology Letters, the birds minimize heat loss by keeping the outer surface of their plumage below the temperature of the surrounding air. At the same time, the penguins’ thick plumage insulates their body and keeps it toasty. . . . 

The researchers analyzed thermographic images . . . taken over roughly a month during June 2008. During that period, the average air temperature was 0.32 degrees Fahrenheit. At the same time, the majority of the plumage covering the penguins’ bodies was even colder: the surface of their warmest body part, their feet, was an average 1.76 degrees Fahrenheit, but the plumage on their heads, chests and backs were -1.84, -7.24 and -9.76 degrees Fahrenheit respectively. Overall, nearly the entire outer surface of the penguins’ bodies was below freezing at all times, except for their eyes and beaks. The scientists also used a computer simulation to determine how much heat was lost or gained from each part of the body – and discovered that by keeping their outer surface below air temperature, the birds might paradoxically be able to draw very slight amounts of heat from the air around them. The key to their trick is the difference between two different types of heat transfer: radiation and convection. 

The penguins do lose internal body heat to the surrounding air through thermal radiation, just as our bodies do on a cold day. Because their bodies (but not surface plumage) are warmer than the surrounding air, heat gradually radiates outward over time, moving from a warmer material to a colder one. To maintain body temperature while losing heat, penguins, like all warm-blooded animals, rely on the metabolism of food. The penguins, though, have an additional strategy. Since their outer plumage is even colder than the air, the simulation showed that they might gain back a little of this heat through thermal convection—the transfer of heat via the movement of a fluid (in this case, the air). As the cold Antarctic air cycles around their bodies, slightly warmer air comes into contact with the plumage and donates minute amounts of heat back to the penguins, then cycles away at a slightly colder temperature. 

Most of this heat, the researchers note, probably doesn’t make it all the way through the plumage and back to the penguins’ bodies, but it could make a slight difference. At the very least, the method by which a penguin’s plumage wicks heat from the bitterly cold air that surrounds it helps to cancel out some of the heat that’s radiating from its interior. And given the Emperors’ unusually demanding breeding cycle, every bit of warmth counts. . . . Since [penguins trek as far as 75 miles to the coast to breed and male penguins] don’t eat anything during [the incubation period of 64 days], conserving calories by giving up as little heat as possible is absolutely crucial. 

Q. 1 Which of the following can be responsible for Emperor Penguins losing body heat? 

A Food metabolism. 

B Plumage. 

C Reproduction process. 

D Thermal convection. 

Answer: C 

Explanation: 

Option A: It has been mentioned that food metabolism is used to maintain body temperature. But it cannot be inferred that heat is lost due to food metabolism. 

Option B: The colder temperature of plumage results in slight heat gain from the surrounding air. Hence this option is incorrect. 

Option C: In the last paragraph of the passage, it has been mentioned that heat is very important for the breeding of Emperor Penguins. So it can be inferred that this conserved heat might be used in the reproductive process of Emperor Penguins. Hence C is the answer. 

Option D: Consider the line: “Since their outer plumage is……………………..thermal convection—the transfer of heat via the movement of a fluid (in this case, the air).” It is clear that the process of thermal convection is responsible for heat gain and not heat loss. Hence D is incorrect. 

 

Q. 2 All of the following, if true, would negate the findings of the study reported in the passage EXCEPT: 

A The penguins’ plumage were made of a material that did not allow any heat transfer through convection or radiation. 

B The average temperature of the feet of penguins in the month of June 2008 were found to be 2.76 degrees Fahrenheit. 

C The average air temperature recorded during the month of June 2008 in the area of study was -10 degrees Fahrenheit. 

D The temperature of the plumage on the penguins’ heads, chests and backs were found to be 1.84, 7.24 and 9.76 degrees Fahrenheit respectively. 

Answer: B 

Explanation: 

The primary findings of the study conclude that Emperor Penguins reduce the heat loss by keeping the temperature of the outer surface of their plumage lower than the surrounding air. In fact, they gain a little heat from the surrounding air through thermal convection. 

Option A: If the plumage did not allow thermal convection, it would contradict the findings of the study. Hence A is not the answer. 

Option B: Since the transfer of heat takes place through the plumage, variation in the average temperature of feet will not affect the conclusions of the study. Hence B is the answer. 

Option C: The average temperature of plumage should be lower than that of the air. It has been mentioned in the passage that the temperatures of the plumage on their heads, chests and backs were -1.84, -7.24 and -9.76 degrees Fahrenheit respectively. If the temperature of the air is -10 degrees Fahrenheit, Penguins would not be able to gain the heat. Hence, this will negate the study findings. 

Option D: All the temperatures mentioned in this option are higher than the temperature of the air, but the study assumes the surrounding air temperature to be higher. This option will also negate the study findings. 

 

Q. 3 Which of the following best explains the purpose of the word “paradoxically” as used by the author? 

A Keeping their body colder helps penguins keep their plumage warmer. 

B Heat gain through radiation happens despite the heat loss through convection. 

C Heat loss through radiation happens despite the heat gain through convection. 

D Keeping a part of their body colder helps penguins keep their bodies warmer. 

Answer: D 

Explanation: 

The word “paradoxically” has been used by the author to indicate the two contradictory characteristics mentioned in the statement. 

Option A: This option states the exact opposite conclusion mentioned in the passage. As per the passage, penguins keep their plumage colder to keep their body warmer. Hence A is incorrect. 

Option B: 

Option C: 

Option D: 

 

Q. 4 In the last sentence of paragraph 3, “slightly warmer air” and “at a slightly colder temperature” refer to ______ AND ______ respectively: 

A The cold Antarctic air whose temperature is higher than that of the plumage AND the fall in temperature of the Antarctic air after it has transmitted some heat to the plumage. 

B The cold Antarctic air which becomes warmer because of the heat radiated out from penguins’ bodies AND the fall in temperature of the surrounding air after thermal convection. 

C The air trapped in the plumage which is warmer than the Antarctic air AND the fall in temperature of the trapped plumage air after it radiates out some heat. 

D The air inside penguins’ bodies kept warm because of metabolism of food AND the fall in temperature of the body air after it transfers some heat to the plumage. 

Answer: A 

Explanation: 

Option A: Consider the sentence: “As the cold Antarctic air cycles around their bodies, slightly warmer air comes into contact with the plumage and donates minute amounts of heat back to the penguins, then cycles away at a slightly colder temperature.” It has been mentioned in the passage that the plumage temperature is lower than the surrounding air temperature. Hence, “slightly warmer air” refers to the Antarctica air that surrounds the plumage and “at a slightly colder temperature” refers to the fall in temperature due to heat loss. 

Option B: The process of convections and not radiation is involved in this case. Hence the first part of the option is incorrect. B is not the answer. 

Option C: The passage does not mention air trapped in plumage. Hence this option is rejected. 

Option D: “slightly warmer air” refers to the Antarctica air and not the air inside the penguins’ bodies. Hence D is incorrect. 

 

Instructions 

Contemporary internet shopping conjures a perfect storm of choice anxiety. Research has consistently held that people who are presented with a few options make better, easier decisions than those presented with many. . . . Helping consumers figure out what to buy amid an endless sea of choice online has become a cottage industry unto itself. Many brands and retailers now wield marketing buzzwords such as curation, differentiation, and discovery as they attempt to sell an assortment of stuff targeted to their ideal customer. Companies find such shoppers through the data gold mine of digital advertising, which can catalog people by gender, income level, personal interests, and more. Since Americans have lost the ability to sort through the sheer volume of the consumer choices available to them, a ghost now has to be in the retail machine, whether it’s an algorithm, an influencer, or some snazzy ad tech to help a product follow you around the internet. Indeed, choice fatigue is one reason so many people gravitate toward lifestyle influencers on Instagram—the relentlessly chic young moms and perpetually vacationing 20-somethings—who present an aspirational worldview, and then recommend the products and services that help achieve it. . . . 

For a relatively new class of consumer-products start-ups, there’s another method entirely. Instead of making sense of a sea of existing stuff, these companies claim to disrupt stuff as Americans know it. Casper (mattresses), Glossier (makeup), Away (suitcases), and many others have sprouted up to offer consumers freedom from choice: The companies have a few aesthetically pleasing and supposedly highly functional options, usually at mid-range prices. They’re selling nice things, but maybe more importantly, they’re selling confidence in those things, and an ability to opt out of the stuff rat race. . . . 

One-thousand-dollar mattresses and $300 suitcases might solve choice anxiety for a certain tier of consumer, but the companies that sell them, along with those that attempt to massage the larger stuff economy into something navigable, are still just working within a consumer market that’s broken in systemic ways. The presence of so much stuff in America might be more valuable if it were more evenly distributed, but stuff’s creators tend to focus their energy on those who already have plenty. As options have expanded for people with disposable income, the opportunity to buy even basic things such as fresh food or quality diapers has contracted for much of America’s lower classes. 

For start-ups that promise accessible simplicity, their very structure still might eventually push them toward overwhelming variety. Most of these companies are based on hundreds of millions of dollars of venture capital, the investors of which tend to expect a steep growth rate that can’t be achieved by selling one great mattress or one great sneaker. Casper has expanded into bedroom furniture and bed linens. Glossier, after years of marketing itself as no-makeup makeup that requires little skill to apply, recently launched a full line of glittering color cosmetics. There may be no way to opt out of stuff by buying into the right thing. 

Q. 5 Which one of the following best sums up the overall purpose of the examples of Casper and Glossier in the passage? 

A They are exceptions to a dominant trend in consumer markets. 

B They are increasing the purchasing power of poor Americans. 

C They might transform into what they were exceptions to. 

D They are facilitating a uniform distribution of commodities in the market. 

Answer: C 

Explanation: 

Option A: The startups Casper and Glossier are certainly breaking the trend of choice anxiety. Yet, the author argues that they are turning into something that they intended to disrupt. Hence, this does not capture the purpose of the author. 

Option B: The author argues that even these startups are targeting select few mid-range customers rather than the lower classes. Hence, this option directly contradicts the author’s claim. 

Option C: These startups initially started as an exception to offering a wide variety of choices. Yet, due to limited customers, and want of steep growth, they might transform into a type of company that they intended to disrupt. Hence, this option correctly resounds the author’s fear and captures his purpose of argument. Hence C is correct 

Option D: This option is largely vague and can have multiple interpretations. One interpretation can be that these startups are targeting a selected band of customers and do not have offering for lower-class customers. Hence, there is no uniform distribution. 

Q. 6 All of the following, IF TRUE, would weaken the author’s claims EXCEPT: 

A The annual sale of companies that hired lifestyle influencers on Instagram for marketing their products were 40% less than those that did not. 

B Product options increased market competition, bringing down the prices of commodities, which, in turn, increased purchasing power of the poor. 

C The empowerment felt by purchasers in buying a commodity were directly proportional to the number of options they could choose from. 

D The annual sales growth of companies with fewer product options were higher than that of companies which curated their products for target consumers. 

Answer: D 

Explanation: 

Option A: Paragraph 1 says “choice fatigue is one reason so many people gravitate toward lifestyle influencers on Instagram”. Hence, as per the passage, a company with a wide range of products and a lifestyle influencer is likely to perform better than a company with only a wide range of products. Hence, this statement negates the claim of the author. 

Option B: “As options have expanded for people with disposable income, the opportunity to buy even basic things such as fresh food or quality diapers has contracted for much of America’s lower classes.” The author argues that a variety of products are offered only for a certain class of consumers other than the lower class. If a variety of options indeed helped the poor, then his argument is weakened. 

Option C: “Research has consistently held that people who are presented with a few options make better, easier decisions than those presented with many”. “Americans have lost the ability to sort through the sheer volume”. Clearly, people are overwhelmed by options and prefer lesser variety. Hence, option C is contradictory. 

Option D: This option is largely vague and leaves unanswered questions behind. Also, the author doesn’t make any comparison between the growth of these two type of companies. The author only says that, as the company targets only few consumers, for the want of growth they are likely to expand to variety of products. As there is no information about their growths, this option neither strengthens nor weakens the claim. 

Q. 7 Based on the passage, all of the following can be inferred about consumer behaviour EXCEPT that: 

A having too many product options can be overwhelming for consumers. 

B too many options have made it difficult for consumers to trust products. 

C consumers tend to prefer products by start-ups over those by established companies. 

D consumers are susceptible to marketing images that they see on social media. 

Answer: C 

Explanation: 

Option A: Paragraph 1 says “Since Americans have lost the ability to sort through the sheer volume of the consumer choices available to them” Since the product options are overwhelming, they are unable to sort through the options. Hence, option A can be inferred from the passage. 

Option B: Paragraph 1 says “Research has consistently ….. industry unto itself.” As people experience choice anxiety due to overwhelming options, they are unable to trust products while selecting. Hence, they look-out for celebrities and curators to make a decision. 

Option C: There is no such comparison in the passage that shows people’s preference towards products by startups. Hence, option C cannot be inferred. 

Option D: Paragraph 1 says “a ghost now has to be in the retail machine, whether it’s an algorithm, an influencer, or some snazzy ad tech to help a product follow you around the internet”. Due to our inability to sort, we depend on influencers or we are vulnerable to snazzy ads to purchase products. Hence, D can be inferred. 

Q. 8 A new food brand plans to launch a series of products in the American market. Which of the following product plans is most likely to be supported by the author of the passage? 

A A range of 10 products priced between $5 and $10. 

B A range of 25 products priced between $5 and $10. 

C A range of 10 products priced between $10 and $25. 

D A range of 25 products priced between $10 and $25. 

Answer: A 

Explanation: 

The author principally argues for lesser choices. He says that choice anxiety is overwhelming and people make better decisions with lesser choices. 

He is also critical about companies targeting only certain band of well-off customers and critiques them for not offering products for consumers of lower classes. 

Hence, a product group with lesser variety, and targeted to lower class customers would be most acceptable to the author. 

Q. 9 Which of the following hypothetical statements would add the least depth to the author’s prediction of the fate of start-ups offering few product options? 

A An exponential surge in their sales enables start-ups to meet their desired profit goals without expanding their product catalogue. 

B Start-ups with few product options are no exception to the American consumer market that is deeply divided along class lines. 

C With Casper and Glossier venturing into new product ranges, their regular customers start losing trust in the companies and their products. 

D With the motive of promoting certain rival companies, the government decides to double the tax-rates for these start-ups. 

Answer: A 

Explanation: 

By “Depth”, the author suggests a scenario that adds value or supplies additional information which supports his claim. 

Option A: If the startup products grow exponentially and are self-sufficient and do not expand to other products, this scenario directly contradicts the author’s probable prediction of these companies. Hence, it would add the least depth to the author’s argument. A is the correct answer. 

Option B: Lets consider that startups with few product options already exist. In such a case, these startups are no exceptions. For the sake of steep growth and surviving, they might have to expand into different product categories. Hence it adds some depth to the author’s prediction. 

Option C: “There may be no way to opt-out of stuff by buying into the right thing.” The author is clearly displeased with startups ending up with overwhelming variety. Losing regular customers for better growth further invigorates the author’s claim against numerous choices. Hence, it adds some value to his criticism. 

Option D: If the government doubles their tax rates, as these startups are dependent on select customers for income, they might have to venture into other products and varieties to accentuate their returns and keep the company afloat. Hence, their fate would likely end up the way author predicted it to be. 

 

Instructions 

As defined by the geographer Yi-Fu Tuan, topophilia is the affective bond between people and place. His 1974 book set forth a wide-ranging exploration of how the emotive ties with the material environment vary greatly from person to person and in intensity, subtlety, and mode of expression. Factors influencing one’s depth of response to the environment include cultural background, gender, race, and historical circumstance, and Tuan also argued that there is a biological and sensory element. Topophilia might not be the strongest of human emotions— indeed, many people feel utterly indifferent toward the environments that shape their lives – but when activated it has the power to elevate a place to become the carrier of emotionally charged events or to be perceived as a symbol. 

Aesthetic appreciation is one way in which people respond to the environment. A brilliantly colored rainbow after gloomy afternoon showers, a busy city street alive with human interaction—one might experience the beauty of such landscapes that had seemed quite ordinary only moments before or that are being newly discovered. This is quite the opposite of a second topophilia bond, namely that of the acquired taste for certain landscapes and places that one knows well. When a place is home, or when a space has become the locus of memories or the means of gaining a livelihood, it frequently evokes a deeper set of attachments than those predicated purely on the visual. A third response to the environment 

also depends on the human senses but may be tactile and olfactory, namely a delight in the feel and smell of air, water, and the earth. 

Topophilia—and it’s very close conceptual twin, sense of place—is an experience that, however elusive, has inspired recent architects and planners. Most notably, new urbanism seeks to counter the perceived placelessness of modern suburbs and the decline of central cities through neo-traditional design motifs. Although motivated by good intentions, such attempts to create places rich in meaning are perhaps bound to disappoint. As Tuan noted, purely aesthetic responses often are suddenly revealed, but their intensity rarely is long lasting. Topophilia is difficult to design for and impossible to quantify, and its most articulate interpreters have been self-reflective philosophers such as Henry David Thoreau, evoking a marvelously intricate sense of place at Walden Pond, and Tuan, describing his deep affinity for the desert. 

Topophilia connotes a positive relationship, but it often is useful to explore the darker affiliations between people and place. Patriotism, literally meaning the love of one’s terra patria or homeland, has long been cultivated by governing elites for a range of nationalist projects, including war preparation and ethnic cleansing. Residents of upscale residential developments have disclosed how important it is to maintain their community’s distinct identity, often by casting themselves in a superior social position and by reinforcing class and racial differences. And just as a beloved landscape is suddenly revealed, so too may landscapes of fear cast a dark shadow over a place that makes one feel a sense of dread or anxiety—or topophobia. 

Q. 10 In the last paragraph, the author uses the example of “Residents of upscale residential developments” to illustrate the: 

A manner in which environments are designed to minimise the social exclusion of their clientele. 

B introduction of nationalist projects by such elites to produce a sense of dread or topophobia. 

C social exclusivism practised by such residents in order to enforce a sense of racial or class superiority. 

D sensitive response to race and class problems in upscale residential developments. 

Answer: C 

Explanation: 

“Residents of upscale residential developments have disclosed how important it is to maintain their community’s distinct identity, often by casting themselves in a superior social position and by reinforcing class and racial differences.” 

Option A: The option implies that the clients are made to feel at home. While the phrase “Residents of upscale residential developments” is used to capture the intent of social dominance of a particular class. Hence this option is incorrect. 

Option B: The option implies that jingoism of a certain class might lead to topophobia. The option is yet again unrelated. 

Option C: Residents of upscale residential developments intend to promote their community by reinforcing sectarian differences. This exclusivism(Practice of being exclusive/important) is clearly captured in the option. Hence C is correct. 

Option D: Sensitive response indicates a considerate response where other’s sentiments are considered. While these residents are inconsiderate and consider themself superior. Also, the option doesn’t capture the purpose clearly. Hence, incorrect 

 

Q. 11 Which one of the following comes closest in meaning to the author’s understanding of topophilia? 

A Scientists have found that most creatures, including humans, are either born with or cultivate a strong sense of topography. 

B The tendency of many cultures to represent their land as “motherland” or “fatherland” may be seen as an expression of their topophilia 

C Nomadic societies are known to have the least affinity for the lands through which they traverse because they tend to be photophobic. 

D The French are not overly patriotic, but they will refuse to use English as far as possible, even when they know it well. 

Answer: B 

Explanation: 

Option A: The entire passage deals with “TOPOPHILIA” and “TOPOGRAPHY” is unrelated. Also, the author says that we experience topophilia in three forms and that we are not born with it. 

Option C: An illustration of topophobia doesn’t represent the author’s view on topophilia 

Option D: The option speaks about glossophilia(Love of language) and is unrelated to topophilia 

Option B: “Topophilia connotes a positive relationship, but it often is useful to explore the darker affiliations between people and place. Patriotism, literally meaning the love of one’s terra patria or homeland”. 

Despite a negative tone, the author says that one form of topophilia is patriotism. Even though not wholesome, it comes 

“closest” to the author’s understanding of topophilia among the given options. Hence B is correct. 

 

Q. 12 Which one of the following best captures the meaning of the statement, “Topophilia is difficult to design for and impossible to quantify . . .”? 

A The deep anomie of modern urbanisation led to new urbanism’s intricate sense of place. 

B Architects have to objectively quantify spaces and hence cannot be topophilic. 

C Philosopher-architects are uniquely suited to develop topophilic design 

D People’s responses to their environment are usually subjective and so cannot be rendered in design. 

Answer: D 

Explanation: 

“As Tuan noted, purely aesthetic responses often are suddenly revealed, but their intensity rarely is long lasting. Topophilia is difficult to design for and impossible to quantify”. The author says that people’s response to aesthetics is shortlived and usually subsides overtime. Hence, it is difficult to design or quantify. 

Option A: “Amomie” means lack of morals or ethics. It is unrelated to the passage. 

Option B: An objective analysis by architects does not explain the reason as to why it is difficult to quantify topophilia. Option C: This statement is in the form of an opinion and does not explain the above statement. 

Option D: Since every person has different topophilic attractions and have different responses to aesthetics. Capturing topophilia in the form of design is impossible. This option elaborates and explains the reason for quantifying topophilia. Hence option D is correct. 

 

Q. 13 The word “topophobia” in the passage is used: 

A to represent a feeling of dread towards particular spaces and places. 

B to signify the fear of studying the complex discipline of topography. 

C to signify feelings of fear or anxiety towards topophilic people. 

D as a metaphor expressing the failure of the homeland to accommodate non-citizens. 

Answer: A 

Explanation: 

“And just as a beloved landscape is suddenly revealed, so too may landscapes of fear cast a dark shadow over a place that makes one feel a sense of dread or anxiety—or topophobia.” 

Option B speaks about topography, while Option C speaks about dread towards people. 

Option D is unrelated to topophobia. Hence, all of them are incorrect 

Option A clearly captures the essence of the last sentence in the passage. 

 

Q. 14 Which of the following statements, if true, could be seen as not contradicting the arguments in the passage? 

A New Urbanism succeeded in those designs where architects collaborated with their clients. 

B Generally speaking, in a given culture, the ties of the people to their environment vary little in significance or intensity. 

C The most important, even fundamental, response to our environment is our tactile and olfactory response. 

D Patriotism, usually seen as a positive feeling, is presented by the author as a darker form of topophilia. 

Answer: D 

Explanation: 

Option A: “new urbanism seeks to… Although motivated by good intentions, such attempts to create places rich in meaning are perhaps bound to disappoint.” The author says new urbanism that tries to induce sense of place is bound to fail. Since there is no mention of clients, irrespectively new urbanism is going to fail. Hence, it is contradicting the author. 

Option B: “His 1974 book set forth a wide-ranging exploration of how the emotive ties with the material environment vary greatly from person to person and in intensity, subtlety, and mode of expression.” This option is contradicting the passage yet again. 

Option C: The author lists out three ways of experiencing topophilia but doesn’t emphasize about any one way. Hence, even though not contradictory, this option is factually misquoting the passage. 

Option D: “Topophilia connotes a positive relationship, but it often is useful to explore the darker affiliations between people and place. Patriotism, literally meaning the love of one’s terra patria or homeland..” Clearly, the author has a negative intonation when he says “darker affiliation”. He presents patriotism as a darker manifestation of topophilia. Hence, this statement is correct and does not contradict the author. Hence option D is correct. 

 

Instructions 

“Free of the taint of manufacture” – that phrase, in particular, is heavily loaded with the ideology of what the Victorian socialist William Morris called the “anti-scrape”, or an anticapitalist conservatism (not conservatism) that solaced itself with the vision of a preindustrial golden age. In Britain, folk may often appear a cosy, fossilised form, but when you look more closely, the idea of folk – who has the right to sing it, dance it, invoke it, collect it, belong to it or appropriate it for political or cultural ends – has always been contested territory. . . . 

In our own time, though, the word “folk” . . . has achieved the rare distinction of occupying fashionable and unfashionable status simultaneously. Just as the effusive floral prints of the radical William Morris now cover genteel sofas, so the revolutionary intentions of many folk historians and revivalists have led to music that is commonly regarded as parochial and conservative. And yet – as newspaper columns periodically rejoice – folk is hip again, influencing artists, clothing and furniture designers, celebrated at music festivals, awards ceremonies and on TV, reissued on countless record labels. Folk is a sonic “shabby chic”, containing elements of the uncanny and eerie, as well as an antique veneer, a whiff of Britain’s heathen dark ages. The very obscurity and anonymity of folk music’s origins open up space for rampant imaginative fancies. . . . 

[Cecil Sharp, who wrote about this subject, believed that] folk songs existed in constant transformation, a living example of an art form in a perpetual state of renewal. “One man sings a song, and then others sing it after him, changing what they do not like” is the most concise summary of his conclusions on its origins. He compared each rendition of a ballad to an acorn falling from an oak tree; every subsequent iteration sows the song anew. But there is tension in newness. In the late 1960s, purists were suspicious of folk songs recast in rock idioms. Electrification, however, comes in many forms. For the early-20th-century composers such as Vaughan Williams and Holst, there were thunderbolts of inspiration from oriental mysticism, angular modernism and the body blow of the first world war, as well as input from the rediscovered folk tradition itself. 

For the second wave of folk revivalists, such as Ewan MacColl and AL Lloyd, starting in the 40s, the vital spark was communism’s dream of a post-revolutionary New Jerusalem. For their younger successors in the 60s, who thronged the folk clubs set up by the old guard, the lyrical freedom of Dylan and the unchained melodies of psychedelia created the conditions for folk rock’s own golden age, a brief Indian summer that lasted from about 1969 to 1971. . . . Four decades on, even that progressive period has become just one more era ripe for fashionable emulation and pastiche. The idea of a folk tradition being exclusively confined to oral transmission has become a much looser, less severely guarded concept. Recorded music and television, for today’s metropolitan generation, are where the equivalent of folk memories are seeded. . . . 

 

Q. 15 At a conference on folk forms, the author of the passage is least likely to agree with which one of the following views? 

A Folk forms, in their ability to constantly adapt to the changing world, exhibit an unusual poise and homogeneity with each change. 

B The plurality and democratising impulse of folk forms emanate from the improvisation that its practitioners bring to it. 

C The power of folk resides in its contradictory ability to influence and be influenced by the present while remaining rooted in the past. 

D Folk forms, despite their archaic origins, remain intellectually relevant in contemporary times. 

Answer: A 

Q. 16 The primary purpose of the reference to William Morris and his floral prints is to show: 

A that despite its archaic origins, folk continues to remain a popular tradition. 

B the pervasive influence of folk on contemporary art, culture, and fashion. 

C that what is once regarded as radical in folk, can later be seen as conformist. 

D that what was once derided as genteel is now considered revolutionary. 

Answer: C 

Q. 17 The author says that folk “may often appear a cosy, fossilised form” because: 

A folk is a sonic “shabby chic” with an antique veneer. 

B of its nostalgic association with a pre-industrial past. 

C it has been arrogated for various political and cultural purposes. 

D the notion of folk has led to several debates and disagreements. 

Answer: B 

Q. 18 Which of the following statements about folk revivalism of the 1940s and 1960s cannot be inferred from the passage? 

A Electrification of music would not have happened without the influence of rock music. 

B Even though it led to folk-rock’s golden age, it wasn’t entirely free from critique. 

C It reinforced Cecil Sharp’s observation about folk’s constant transformation. 

D Freedom and rebellion were popular themes during the second wave of folk revivalism. Answer: A 

Q. 19 All of the following are causes for plurality and diversity within the British folk tradition EXCEPT: 

A that British folk continues to have traces of pagan influence from the dark ages. 

B paradoxically, folk forms are both popular and unpopular. 

C the fluidity of folk forms owing to their history of oral mode of transmission. 

D that British folk forms can be traced to the remote past of the country. 

Answer: B 

 

Instructions 

In the past, credit for telling the tale of Aladdin has often gone to Antoine Galland . . . the first European translator of . . . Arabian Nights [which] started as a series of translations of an incomplete manuscript of a medieval Arabic story collection. . . But, though those tales were of medieval origin, Aladdin may be a more recent invention. Scholars have not found a manuscript of the story that predates the version published in 1712 by Galland, who wrote in his diary that he first heard the tale from a Syrian storyteller from Aleppo named Hanna Diyab. . . 

Despite the fantastical elements of the story, scholars now think the main character may actually be based on a real person’s real experiences. . . . Though Galland never credited Diyab in his published translations of the Arabian Nights stories, Diyab wrote something of his own: a travelogue penned in the mid-18th century. In it, he recalls telling Galland the story of Aladdin [and] describes his own hard-knocks upbringing and the way he marveled at the extravagance of Versailles. The descriptions he uses were very similar to the descriptions of the lavish palace that ended up in Galland’s version of the Aladdin story. [Therefore, author Paulo Lemos] Horta believes that “Aladdin might be the young Arab Maronite from Aleppo, marveling at the jewels and riches of Versailles.” . . . 

For 300 years, scholars thought that the rags-to-riches story of Aladdin might have been inspired by the plots of French fairy tales that came out around the same time, or that the story was invented in that 18th century period as a byproduct of French Orientalism, a fascination with stereotypical exotic Middle Eastern luxuries that was prevalent then. The idea that Diyab might have based it on his own life — the experiences of a Middle Eastern man encountering the French, not vice-versa — flips the script. [According to Horta,] “Diyab was ideally placed to embody the overlapping world of East and West, blending the storytelling traditions of his homeland with his youthful observations of the wonder of 18th-century France.” . . . 

To the scholars who study the tale, its narrative drama isn’t the only reason storytellers keep finding reason to return to Aladdin. It reflects not only “a history of the French and the Middle East, but also [a story about] Middle Easterners coming to Paris and that speaks to our world today,” as Horta puts it. “The day Diyab told the story of Aladdin to Galland, there were riots due to food shortages during the winter and spring of 1708 to 1709, and Diyab was sensitive to those people in a way that Galland is not. When you read this diary, you see this solidarity among the Arabs who were in Paris at the time. . . . 

There is little in the writings of Gall and that would suggest that he was capable of developing a character like Aladdin with sympathy, but Diyab’s memoir reveals a narrator adept at capturing the distinctive psychology of a young protagonist, as well as recognizing the kinds of injustices and opportunities that can transform the path of any youthful adventurer.” 

Q. 20 All of the following serve as evidence for the character of Aladdin being based on Hanna Diyab EXCEPT: 

A Diyab’s narration of the original story to Galland. 

B Diyab’s humble origins and class struggles, as recounted in his travelogue. 

C Diyab’s description of the wealth of Versailles in his travelogue. 

D Diyab’s cosmopolitanism and cross-cultural experience. 

Answer: A 

Q. 21 Which of the following is the primary reason for why storytellers are still fascinated by the story of Aladdin? 

A The traveller’s experience that inspired the tale of Aladdin resonates even today. 

B The archetype of the rags-to-riches story of Aladdin makes it popular even today. 

C The tale of Aladdin documents the history of Europe and Middle East. 

D The story of Aladdin is evidence of the eighteenth century French Orientalist attitude. 

Answer: A 

Q. 22 Which of the following does not contribute to the passage’s claim about the authorship of Aladdin? 

A The narrative sensibility of Diyab’s travelogue. 

B Galland’s acknowledgment of Diyab in his diary. 

C The story-line of many French fairy tales of the 18th century. 

D The depiction of the affluence of Versailles in Diyab’s travelogue. 

Answer: C 

Q. 23 The author of the passage is most likely to agree with which of the following explanations for the origins of the story of Aladdin? 

A Basing it on his own life experiences, Diyab transmitted the story of Aladdin to Galland who included it in Arabian Nights. 

B Galland derived the story of Aladdin from Diyab’s travelogue in which he recounts his fascination with the wealth of Versailles. 

C The story of Aladdin has its origins in an undiscovered, incomplete manuscript of a medieval Arabic collection of stories. 

D Galland received the story of Aladdin from Diyab who, in turn, found it in an incomplete medieval manuscript. 

Answer: A 

Q. 24 Which of the following, if true, would invalidate the inversion that the phrase “flips the script” refers to? 

A Diyab’s travelogue described the affluence of the French city of Bordeaux, instead of Versailles. 

B The French fairy tales of the eighteenth century did not have rags-to-riches plot lines like that of the tale of Aladdin. 

C The description of opulence in Hanna Diyab’s and Antoine Galland’s narratives bore no resemblance to each other. 

D Galland acknowledged in the published translations of Arabian Nights that he heard the story of Aladdin from Diyab. 

Answer: C 

 

Instructions 

For the following questions answer them individually 

Q. 25 Five sentences related to a topic are given below in a jumbled order. Four of them form a coherent and unified paragraph. Identify the odd sentence that does not go with the four. Key in the number of the options that you choose. 

1. ‘Stat’ signaled something measurable, while ‘matic’ advertised free labour; but ‘tron’, above all, indicated control. 

2. It was a totem of high modernism, the intellectual and cultural mode that decreed no process or phenomenon was too complex to be grasped, managed and optimized. 

3. Like the heraldic shields of ancient knights, these morphemes were painted onto the names of scientific technologies to proclaim one’s history and achievements to friends and enemies alike. 4. The historian Robert Proctor at Stanford University calls the suffix ‘-tron’, along with ‘-matic’ and ‘- stat’, embodied symbols. 

5. To gain the suffix was to acquire a proud and optimistic emblem of the electronic and atomic age. 

Answer: 2 

Q. 26 The four sentences (labelled 1, 2, 3, 4) given below, when properly sequenced would yield a coherent paragraph. Decide on the proper sequence of the order of the sentences and key in the sequence of the four numbers as your answer. 

1. People with dyslexia have difficulty with print-reading, and people with autism spectrum disorder have difficulty with mind-reading. 

2. An example of a lost cognitive instinct is mind-reading: our capacity to think of ourselves and others as having beliefs, desires, thoughts and feelings. 

3. Mind-reading looks increasingly like literacy, a skill we know for sure is not in our genes, since scripts have been around for only 5,000-6,000 years. 

4. Print-reading, like mind-reading varies across cultures, depends heavily on certain parts of the brain, and is subject to developmental disorders. 

Answer:2341 

Q. 27 The four sentences (labelled 1, 2, 3, 4) given below, when properly sequenced would yield a coherent paragraph. Decide on the proper sequence of the order of the sentences and key in the sequence of the four numbers as your answer. 

1. Metaphors may map to similar meanings across languages, but thei r subtle differences can have a profound effect on our understanding of the world. 

2. Latin scholars point out carpe diem is a horticultural metaphor that, particularly seen in the context of its source, is more accurately translated as “plucking the day,” evoking the plucking and gathering of ripening fruits or flowers, enjoying a moment that is rooted in the sensory experience of nature, unrelated to the force implied in seizing. 

3. The phrase carpe diem, which is often translated as “seize the day and its accompanying philosophy, has gone on to inspire countless people in how they live their lives and motivates us to see the world a little differently from the norm 

4. It’s an example of one of the more telling ways that we mistranslate metaphors from one language to another, revealing in the process our hidden assumptions about what we really value. 

Answer:3241 

Q. 28 The four sentences (labelled 1, 2, 3, 4) given below, when properly sequenced would yield a coherent paragraph. Decide on the proper sequence of the order of the sentences and key in the sequence of the four numbers as your answer. 

1. We’ll all live under mob rule until then, which doesn’t help anyone. 

2. Perhaps we need to learn to condense the feedback we receive online so that 100 replies carry the same weight as just one. 

3. As we grow more comfortable with social media conversations being part of the way we interact every day, we are going to have to learn how to deal with legitimate criticism. 

4. A new norm will arise where it is considered unacceptable to reply with the same point that dozens of others have already. 

Answer:3241 

Q. 29 The passage given below is followed by four alternate summaries. Choose the option that best captures the essence of the passage. 

Vance Packard’s The Hidden Persuaders alerted the public to the psychoanalytical techniques used by the advertising industry. Its premise was that advertising agencies were using depth interviews to identify hidden consumer motivations, which were then used to entice consumers to buy goods. Critics and reporters often wrongly assumed that Packard was writing mainly about subliminal advertising. Packard never mentioned the word subliminal, however, and devoted very little space to discussions of “subthreshold” effects. Instead, his views largely aligned with the notion that individuals do not always have access to their conscious thoughts and can be persuaded by supraliminal messages without their knowledge. 

A Packard held that advertising as a ‘hidden persuasion’ understands the hidden motivations of consumers and works at the supraliminal level, though the people targeted have no awareness of being persuaded. 

B Packard held that advertising as a ‘hidden persuasion’ builds on peoples’ conscious thoughts and awareness, by understanding the hidden motivations of consumers and works at the subliminal level. 

C Packard argued that advertising as a ‘hidden persuasion’ works at the supraliminal level, wherein the people targeted are aware of being persuaded, after understanding the hidden motivations of consumers and works. 

D Packard argued that advertising as a ‘hidden persuasion’ understands the hidden motivations of consumers and works at the subliminal level, on the subconscious level of the awareness of the people targeted. 

Answer: A 

Q. 30 The passage given below is followed by four alternate summaries. Choose the option that best captures the essence of the passage. 

A distinguishing feature of language is our ability to refer to absent things, known as displaced reference. A speaker can bring distant referents to mind in the absence of any obvious stimuli. Thoughts, not limited to the here and now, can pop into our heads for unfathomable reasons. This ability to think about distant things necessarily precedes the ability to talk about them. Thought precedes meaningful referential communication. A prerequisite for the emergence of human-like meaningful symbols is that the mental categories they relate to can be invoked even in the absence of immediate stimuli. 

A Displaced reference is particular to humans and thoughts pop into our heads for no real reason. 

B Thoughts precede all speech acts and these thoughts pop up in our heads even in the absence of any stimulus. 

C Thoughts are essential to communication and only humans have the ability to think about objects not present in their surroundings. 

D The ability to think about objects not present in our environment precedes the development of human communication. 

Answer: D 

Q. 31 The four sentences (labelled 1, 2, 3, 4) given below, when properly sequenced would yield a coherent paragraph. Decide on the proper sequence of the order of the sentences and key in the sequence of the four numbers as your answer. 

1. If you’ve seen a little line of text on websites that says something like “cus tomers who bought this also enjoyed that” you have experienced this collaborative filtering firsthand. 

2. The problem with these algorithms is that they don’t take into account a host of nuances and circumstances that might interfere with their accuracy. 

3. If you just bought a gardening book for your cousin, you might get a flurry of links to books about gardening, recommended just for you! – the algorithm has no way of knowing you hate gardening and only bought the book as a gift. 

4. Collaborative filtering is a mathematical algorithm by which correlations and cooccurrences of behaviors are tracked and then used to make recommendations. 

Answer:4123 

Q. 32 Five sentences related to a topic are given below. Four of them can be put together to form a meaningful and coherent short paragraph. Identify the odd one out. 

Choose its number as your answer and key it in. 

1. His idea to use sign language was not a compl etely new idea as Native Americans used hand gestures to communicate with other tribes. 

2. Ancient Greek philosopher Aristotle, for example, observed that men who are deaf are incapable of speech. 

3. People who were born deaf were denied the right to sign a will as they were “presumed to understand nothing; because it is not possible that they have been able to learn to read or write.” 4. Pushback against this prejudice began in the 16th century when Pedro Ponce de León created a formal sign language for the hearing impaired. 

5. For millennia, people with hearing impairments encountered marginalization because it was believed that language could only be learned by hearing the spoken word. 

Answer:2 

Q. 33 The passage given below is followed by four alternate summaries. Choose the option that best captures the essence of the passage. 

Physics is a pure science that seeks to understand the behavior of matter without regard to whether it will afford any practical benefit. Engineering is the correlative applied science in which physical theories are put to some specific use, such as building a bridge or a nuclear reactor. Engineers obviously rely heavily on the discoveries of physicists, but an engineer’s knowledge of the world is not the same as the physicist’s knowledge. In fact, an engineer’s know-how will often depend on physical theories that, from the point of view of pure physics, are false. There are some reasons for this. First, theories that are false in the purest and strictest sense are still sometimes very good approximations to the true ones, and often have the added virtue of being much easier to work with. Second, sometimes the true theories apply only under highly idealized conditions which can only be created under controlled experimental situations. The engineer finds that in the real world, theories rejected by physicists yield more accurate predictions than the ones that they accept. 

A Though engineering draws heavily from pure science, it contributes to knowledge, by incorporating the constraints and conditions in the real world. 

B Engineering and physics fundamentally differ on matters like building a bridge or a nuclear reactor. 

C The relationship between pure and applied science is strictly linear, with the pure science directing applied science, and never the other way round. 

D The unique task of the engineer is to identify, understand, and interpret the design constraints to produce a successful result. 

Answer: A 

Q. 34 Five sentences related to a topic are given below. Four of them can be put together to form a meaningful and coherent short paragraph. Identify the odd one out. 

Choose its number as your answer and key it in. 

1. One argument is that actors that do not fit within a single, well-defined category may suffer an “illegitimacy discount”. 

2. Others believe that complex identities confuse audiences about an organization’s role or purpose. 3. Some organizations have complex and multidimensional identities that span or combine categories, while other organizations possess narrow identities. 

4. Identity is one of the most important features of organizations, but there exist opposing views among sociologists about how identity affects organizational performance. 

5. Those who think that complex identities are beneficial point to the strategic advantages of ambiguity, and organizations’ potential to differentiate themselves from competitors. 

Answer:1 

DILR 

Instructions 

Comprehension: 

A supermarket has to place 12 items (coded A to L) in shelves numbered 1 to 16. Five of these items are types of biscuits, three are types of candies and the rest are types of savouries. Only one item can be kept in a shelf. Items are to be placed such that all items of same type are clustered together with no empty shelf between items of the same type and at least one empty shelf between two different types of items. At most two empty shelves can have consecutive numbers. 

The following additional facts are known. 

1. A and B are to be placed in consecutively numbered shelves in increasing order. 

2. I and J are to be placed in consecutively numbered shelves both higher numbere d than the shelves in which A and B are kept. 

3. D, E and F are savouries and are to be placed in consecutively numbered shelves in increasing order after all the biscuits and candies. 

4. K is to be placed in shelf number 16. 

5. L and J are items of the same type, w hile H is an item of a different type. 

6. C is a candy and is to be placed in a shelf preceded by two empty shelve s. 

7. L is to be placed in a shelf preceded by exactly one empty shelf. 

Q. 35 In how many different ways can the items be arranged on the shelves? 

A 8 

B 4 

C 2 

D 1 

Answer: A 

Explanation: 

The total number of biscuits = 5, the total number of candies =3 and the total number of savouries = 12-(3+5)=4 

Representing the candies as C, biscuits as B and savories as S. K is to be placed in shelf number 16. D, E and F are savouries and are to be placed in consecutively numbered shelves in increasing order after all the biscuits and candies. Since there is no empty shelf between the items of same type, D,E,F and K are savouries and placed at 13,14,15 and 16 respectively. This can be tabulated as follows: 

The shelf 12 will be empty. 

It is given that items are to be placed such that all items of same type are clustered together. From 1, A and B are to be placed in consecutively numbered shelves in increasing order. 

From 6, C is a candy and is to be placed in a shelf preceded by two empty shelves and from 7, L is to be placed in a shelf preceded by exactly one empty shelf. 

Hence C and L are items of different types. Since C is a candy, L will be a biscuit. 

From 5, L and J are items of the same type, while H is an item of a different type. 

Since I and J are clustered together, I, J and L are biscuits and H is a candy. 

So C,H are candies and I,J,L are biscuits. It is given that A, B are place consecutively. Hence A and B are items of same types. So A, B should be biscuits because if they are candies, there will be 4 candies. 

Hence, I,J,L,A,B are biscuits and C,H and G are candies. 

Now there are two empty shelves before C and exactly one empty shelf before L, then the different cases can be tabulated as follows: 

Case 1: 

Case 2: 

The number of arrangements for the first case = 2*2=4 

The number of arrangements for the second case = 2*2=4 

The total number of arrangements = 4+4=8 

Q. 36 Which of the following items is not a type of biscuit? 

A L 

B A 

C B 

D G 

Answer: D 

Explanation: 

The total number of biscuits = 5, the total number of candies =3 and the total number of savouries = 12-(3+5)=4 

Representing the candies as C, biscuits as B and savories as S. K is to be placed in shelf number 16. D, E and F are savouries and are to be placed in consecutively numbered shelves in increasing order after all the biscuits and candies. Since there is no empty shelf between the items of same type, D,E,F and K are savouries and placed at 13,14,15 and 16 respectively. This can be tabulated as follows: 

The shelf 12 will be empty. 

It is given that items are to be placed such that all items of same type are clustered together. From 1, A and B are to be placed in consecutively numbered shelves in increasing order. 

From 6, C is a candy and is to be placed in a shelf preceded by two empty shelves and from 7, L is to be placed in a shelf preceded by exactly one empty shelf. 

Hence C and L are items of different types. Since C is a candy, L will be a biscuit. 

From 5, L and J are items of the same type, while H is an item of a different type. 

Since I and J are clustered together, I, J and L are biscuits and H is a candy. 

So C,H are candies and I,J,L are biscuits. It is given that A, B are place consecutively. Hence A and B are items of same types. So A, B should be biscuits because if they are candies, there will be 4 candies. 

Hence, I,J,L,A,B are biscuits and C,H and G are candies. 

Now there are two empty shelves before C and exactly one empty shelf before L, then the different cases can be tabulated as follows: 

Case 1: 

Case 2: 

G is a candy. Hence D is the answer. 

Q. 37 Which of the following can represent the numbers of the empty shelves in a possible arrangement? 

A 1, 7, 11, 12 

B 1, 5, 6, 12 

C 1, 2, 6, 12 

D 1, 2, 8, 12 

Answer: C 

Explanation: 

The total number of biscuits = 5, the total number of candies =3 and the total number of savouries = 12-(3+5)=4 

Representing the candies as C, biscuits as B and savories as S. K is to be placed in shelf number 16. D, E and F are savouries and are to be placed in consecutively numbered shelves in increasing order after all the biscuits and candies. Since there is no empty shelf between the items of same type, D,E,F and K are savouries and placed at 13,14,15 and 16 respectively. This can be tabulated as follows: 

The shelf 12 will be empty. 

It is given that items are to be placed such that all items of same type are clustered together. From 1, A and B are to be placed in consecutively numbered shelves in increasing order. 

From 6, C is a candy and is to be placed in a shelf preceded by two empty shelves and from 7, L is to be placed in a shelf preceded by exactly one empty shelf. 

Hence C and L are items of different types. Since C is a candy, L will be a biscuit. 

From 5, L and J are items of the same type, while H is an item of a different type. 

Since I and J are clustered together, I, J and L are biscuits and H is a candy. 

So C,H are candies and I,J,L are biscuits. It is given that A, B are place consecutively. Hence A and B are items of same types. So A, B should be biscuits because if they are candies, there will be 4 candies. 

Hence, I,J,L,A,B are biscuits and C,H and G are candies. 

Now there are two empty shelves before C and exactly one empty shelf before L, then the different cases can be tabulated as follows: 

Case 1: 

Case 2: 

From the table(case 2), only 1,2,6 and 12 are empty in the same arrangement. Hence, C is the answer. 

Q. 38 Which of the following statements is necessarily true? 

A  All biscuits are kept before candies. 

B There are two empty shelves between the biscuits and the candies. 

 C All candies are kept before biscuits. 

D There are at least four shelves between items B and C. 

Answer: D 

Explanation: 

The total number of biscuits = 5, the total number of candies =3 and the total number of savouries = 12-(3+5)=4 

Representing the candies as C, biscuits as B and savories as S. K is to be placed in shelf number 16. D, E and F are savouries and are to be placed in consecutively numbered shelves in increasing order after all the biscuits and candies. Since there is no empty shelf between the items of same type, D,E,F and K are savouries and placed at 13,14,15 and 16 respectively. This can be tabulated as follows: 

The shelf 12 will be empty. 

It is given that items are to be placed such that all items of same type are clustered together. From 1, A and B are to be placed in consecutively numbered shelves in increasing order. 

From 6, C is a candy and is to be placed in a shelf preceded by two empty shelves and from 7, L is to be placed in a shelf preceded by exactly one empty shelf. 

Hence C and L are items of different types. Since C is a candy, L will be a biscuit. 

From 5, L and J are items of the same type, while H is an item of a different type. 

Since I and J are clustered together, I, J and L are biscuits and H is a candy. 

So C,H are candies and I,J,L are biscuits. It is given that A, B are place consecutively. Hence A and B are items of same types. So A, B should be biscuits because if they are candies, there will be 4 candies. 

Hence, I,J,L,A,B are biscuits and C,H and G are candies. 

Now there are two empty shelves before C and exactly one empty shelf before L, then the different cases can be tabulated as follows: 

Case 1: 

Case 2: 

Option A and C are wrong as candies can come before biscuits and vice versa. B is not necessarily true as there can be one empty shelf too as shown in the table. Option D is true as there are at least 4 shelves between B and C. Hence D is the answer. 

 

Instructions 

Comprehension: 

Six players – Tanzi, Umeza, Wangdu, Xyla, Yonita and Zeneca competed in an archery tournament. The tournament had three compulsory rounds, Rounds 1 to 3. In each round every player shot an arrow at a target. Hitting the centre of the target (called bull’s eye) fetched the highest score of 5. The only other possible scores that a player could achieve were 4, 3, 2 and 1. Every bull’s eye score in the first three rounds gave a player one additional chance to shoot in the bonus rounds, Rounds 4 to 6. The possible scores in Rounds 4 to 6 were identical to the first three. 

A player’s total score in the tournament was the sum of his/her scores in all rounds played by him/her. The table below presents partial information on points scored by the players after completion of the tournament. In the table, NP means that the player did not participate in that round, while a hyphen means that the player participated in that round and the score information is missing.

The following facts are also known. 

1.Tanzi, Umeza and Yonita had the same total score. 

2.Total scores for all players, except one, were in mul tiples of three. 

3.The highest total score was one more than double of the lowest to tal score. 

4.The number of players hitting bull’s eye in Round 2 was double of that in Ro und 3. 

5.Tanzi and Zeneca had the same score in Round 1 but different scores in Round 3. 

Q. 39 What was the highest total score? 

A 25 

B 21 

C 24 

D 23 

Answer: A 

Explanation: 

It is given that every bull’s eye score in the first three rounds gave a player one additional chance to shoot in the bonus rounds, Rounds 4 to 6, which means Tanzi scored Bull’s eye only once in the first 3 rounds because she participated only once in round 4 to 6. Similarly, Umeza scored Bull’s eye exactly 2 times in the first 3 rounds. Wangdu did not score Bull’s eye in the first three rounds and so on. 

Now from 1, Tanzi, Umeza and Yonita had the same total score. 

So, Total score of Tanzi will be 4+5+5+a=14+a, (She scored Bull’s eye(a score of 5) in exactly one round and a is the unknown score) 

Total score of Umeza = 1+2+5+5+b = 13+b (She scored Bull’s eye(a score of 5) in exactly 2 rounds and b is the unknown score) 

Total score of Yonita = 3+5+5+c=13+c (She scored Bull’s eye(a score of 5) in exactly one round and c is the unknown score) 

Now 14+a=13+b=13+c, 

Also it is given that total scores for all players, except one, were in multiples of three, so these three will have to be a multiple of 3. 

So, (a,b,c) can be either (1,2,2) or (4,5,5) in the same order. But the value (5,5) for b and c is not possible. (Umeza scored Bull’s eye in exactly 2 rounds and Yonita in exactly 1 round) 

Hence, a=1,b=2 and c=2. So each of Tanzi, Umeza and Yonita had total score of 15. 

Tabulating the data, we have 

From 5, Tanzi and Zeneca had the same score in Round 1 but different scores in Round 3. 

Zeneca score Bull’s eye 2 times in round 1 to 3. If Tanzi scored 1 in round 1, then Zeneca also has to score 1 in round 1, which means both Tanzi and Zeneca scores in round 3 will be 5, which violates 5. Hence Tanzi scored 5 in round 1 and Zeneca also scored the same in round 1.So the new table is: 

From 4, the number of players hitting bull’s eye in Round 2 was double of that in Round 3. 

So, in round 3 either 1 or 2 Bull’s eye can be scored and in round 2, 2 or 4 Bull’s eye can be scored. 

Case 1: If only 1 Bull’s eye is scored in the round 3, then in round 3 Umeza will score 2 and Zeneca will score 2/3/4 in round 3, which means both will score 5 in round 2. So minimum Bull’s eye in round 2 will be 3. (Umeza, Zeneca and Xyla) 

Hence this case is rejected. 

Case 2: 2 Bull’s eye were scored in round 3 and 4 Bull’s eye were scored in round 2. So in round 2 Umeza, Yonita and Zeneca scored 5. This can be tabulated as: 

In round 3, 2 Bull’s eye can only be scored by Xyla and Umeza. 

The highest scorer can be either Xyla or Zeneca. The lowest scorer will be Wangdu. 

1.Consider Zeneca is the highest scorer. 

From 3, the highest total score was one more than double of the lowest total score. So the only possible score for Zeneca is 23 and that for Wangdu is 11. (11*2+1=23) 

But this will violate condition 2, since both Zeneca and Wangdu do not have their scores as multiples of three in this case. 

Hence, Xyla will be the highest scorer. The only possible total score for Xyla will be 25, and that for Wangdu is 12(4+4+4). (12*2+1=25) 

Since Xyla already has non-multiple of 3 as total score. Zeneca will have 24 as the total score. The complete table is:

The highest score is 25. 

Q. 40 What was Zeneca’s total score? 

A 21 

B 22 

C 23 

D 24 

Answer: D 

Explanation: 

It is given that every bull’s eye score in the first three rounds gave a player one additional chance to shoot in the bonus rounds, Rounds 4 to 6, which means Tanzi scored Bull’s eye only once in the first 3 rounds because she participated only once in round 4 to 6. Similarly, Umeza scored Bull’s eye exactly 2 times in the first 3 rounds. Wangdu did not score Bull’s eye in the first three rounds and so on. 

Now from 1, Tanzi, Umeza and Yonita had the same total score. 

So, Total score of Tanzi will be 4+5+5+a=14+a, (She scored Bull’s eye(a score of 5) in exactly one round and a is the unknown score) 

Total score of Umeza = 1+2+5+5+b = 13+b (She scored Bull’s eye(a score of 5) in exactly 2 rounds and b is the unknown score) 

Total score of Yonita = 3+5+5+c=13+c (She scored Bull’s eye(a score of 5) in exactly one round and c is the unknown score) 

Now 14+a=13+b=13+c, 

Also it is given that total scores for all players, except one, were in multiples of three, so these three will have to be a multiple of 3. 

So, (a,b,c) can be either (1,2,2) or (4,5,5) in the same order. But the value (5,5) for b and c is not possible. (Umeza scored Bull’s eye in exactly 2 rounds and Yonita in exactly 1 round) 

Hence, a=1,b=2 and c=2. So each of Tanzi, Umeza and Yonita had total score of 15. 

Tabulating the data, we have 

From 5, Tanzi and Zeneca had the same score in Round 1 but different scores in Round 3. 

Zeneca score Bull’s eye 2 times in round 1 to 3. If Tanzi scored 1 in round 1, then Zeneca also has to score 1 in round 1, which means both Tanzi and Zeneca scores in round 3 will be 5, which violates 5. Hence Tanzi scored 5 in round 1 and Zeneca also scored the same in round 1.So the new table is: 

From 4, the number of players hitting bull’s eye in Round 2 was double of that in Round 3. So, in round 3 either 1 or 2 Bull’s eye can be scored and in round 2, 2 or 4 Bull’s eye can be scored. 

Case 1: If only 1 Bull’s eye is scored in the round 3, then in round 3 Umeza will score 2 and Zeneca will score 2/3/4 in round 3, which means both will score 5 in round 2. So minimum Bull’s eye in round 2 will be 3. (Umeza, Zeneca and Xyla) 

Hence this case is rejected. 

Case 2: 2 Bull’s eye were scored in round 3 and 4 Bull’s eye were scored in round 2. So in round 2 Umeza, Yonita and Zeneca scored 5. This can be tabulated as: 

In round 3, 2 Bull’s eye can only be scored by Xyla and Umeza. 

The highest scorer can be either Xyla or Zeneca. The lowest scorer will be Wangdu. 

1.Consider Zeneca is the highest scorer. 

From 3, the highest total score was one more than double of the lowest total score. So the only possible score for Zeneca is 23 and that for Wangdu is 11. (11*2+1=23) 

But this will violate condition 2, since both Zeneca and Wangdu do not have their scores as multiples of three in this case. 

Hence, Xyla will be the highest scorer. The only possible total score for Xyla will be 25, and that for Wangdu is 12(4+4+4). (12*2+1=25) 

Since Xyla already has non-multiple of 3 as total score. Zeneca will have 24 as the total score. The complete table is:

Zeneca total score is 24. 

Q. 41 Which of the following statements is true? 

A Xyla’s score was 23. 

B Zeneca’s score was 23. 

C Zeneca was the highest scorer. 

D Xyla was the highest scorer. 

Answer: D 

Explanation: 

It is given that every bull’s eye score in the first three rounds gave a player one additional chance to shoot in the bonus rounds, Rounds 4 to 6, which means Tanzi scored Bull’s eye only once in the first 3 rounds because she participated only once in round 4 to 6. Similarly, Umeza scored Bull’s eye exactly 2 times in the first 3 rounds. Wangdu did not score Bull’s eye in the first three rounds and so on. 

Now from 1, Tanzi, Umeza and Yonita had the same total score. 

So, Total score of Tanzi will be 4+5+5+a=14+a, (She scored Bull’s eye(a score of 5) in exactly one round and a is the unknown score) 

Total score of Umeza = 1+2+5+5+b = 13+b (She scored Bull’s eye(a score of 5) in exactly 2 rounds and b is the unknown score) 

Total score of Yonita = 3+5+5+c=13+c (She scored Bull’s eye(a score of 5) in exactly one round and c is the unknown score) 

Now 14+a=13+b=13+c, 

Also it is given that total scores for all players, except one, were in multiples of three, so these three will have to be a multiple of 3. 

So, (a,b,c) can be either (1,2,2) or (4,5,5) in the same order. But the value (5,5) for b and c is not possible. (Umeza scored Bull’s eye in exactly 2 rounds and Yonita in exactly 1 round) 

Hence, a=1,b=2 and c=2. So each of Tanzi, Umeza and Yonita had total score of 15. 

Tabulating the data, we have 

From 5, Tanzi and Zeneca had the same score in Round 1 but different scores in Round 3. 

Zeneca score Bull’s eye 2 times in round 1 to 3. If Tanzi scored 1 in round 1, then Zeneca also has to score 1 in round 1, which means both Tanzi and Zeneca scores in round 3 will be 5, which violates 5. Hence Tanzi scored 5 in round 1 and Zeneca also scored the same in round 1.So the new table is: 

From 4, the number of players hitting bull’s eye in Round 2 was double of that in Round 3. 

So, in round 3 either 1 or 2 Bull’s eye can be scored and in round 2, 2 or 4 Bull’s eye can be scored. 

Case 1: If only 1 Bull’s eye is scored in the round 3, then in round 3 Umeza will score 2 and Zeneca will score 2/3/4 in round 3, which means both will score 5 in round 2. So minimum Bull’s eye in round 2 will be 3. (Umeza, Zeneca and Xyla) 

Hence this case is rejected. 

Case 2: 2 Bull’s eye were scored in round 3 and 4 Bull’s eye were scored in round 2. So in round 2 Umeza, Yonita and Zeneca scored 5. This can be tabulated as: 

In round 3, 2 Bull’s eye can only be scored by Xyla and Umeza. 

The highest scorer can be either Xyla or Zeneca. The lowest scorer will be Wangdu. 

1.Consider Zeneca is the highest scorer. 

From 3, the highest total score was one more than double of the lowest total score. So the only possible score for Zeneca is 23 and that for Wangdu is 11. (11*2+1=23) 

But this will violate condition 2, since both Zeneca and Wangdu do not have their scores as multiples of three in this case. 

Hence, Xyla will be the highest scorer. The only possible total score for Xyla will be 25, and that for Wangdu is 12(4+4+4). (12*2+1=25) 

Since Xyla already has non-multiple of 3 as total score. Zeneca will have 24 as the total score. The complete table is: 

Xyla was the highest scorer. 

Q. 42 What was Tanzi’s score in Round 3? 

A 4 

B 5 

C 3 

D 1 

Answer: D 

Explanation: 

It is given that every bull’s eye score in the first three rounds gave a player one additional chance to shoot in the bonus rounds, Rounds 4 to 6, which means Tanzi scored Bull’s eye only once in the first 3 rounds because she participated only once in round 4 to 6. Similarly, Umeza scored Bull’s eye exactly 2 times in the first 3 rounds. Wangdu did not score Bull’s eye in the first three rounds and so on. 

Now from 1, Tanzi, Umeza and Yonita had the same total score. 

So, Total score of Tanzi will be 4+5+5+a=14+a, (She scored Bull’s eye(a score of 5) in exactly one round and a is the unknown score) 

Total score of Umeza = 1+2+5+5+b = 13+b (She scored Bull’s eye(a score of 5) in exactly 2 rounds and b is the unknown score) 

Total score of Yonita = 3+5+5+c=13+c (She scored Bull’s eye(a score of 5) in exactly one round and c is the unknown score) 

Now 14+a=13+b=13+c, 

Also it is given that total scores for all players, except one, were in multiples of three, so these three will have to be a multiple of 3. 

So, (a,b,c) can be either (1,2,2) or (4,5,5) in the same order. But the value (5,5) for b and c is not possible. (Umeza scored Bull’s eye in exactly 2 rounds and Yonita in exactly 1 round) 

Hence, a=1,b=2 and c=2. So each of Tanzi, Umeza and Yonita had total score of 15. 

Tabulating the data, we have 

From 5, Tanzi and Zeneca had the same score in Round 1 but different scores in Round 3. 

Zeneca score Bull’s eye 2 times in round 1 to 3. If Tanzi scored 1 in round 1, then Zeneca also has to score 1 in round 1, which means both Tanzi and Zeneca scores in round 3 will be 5, which violates 5. Hence Tanzi scored 5 in round 1 and Zeneca also scored the same in round 1.So the new table is: 

From 4, the number of players hitting bull’s eye in Round 2 was double of that in Round 3. 

So, in round 3 either 1 or 2 Bull’s eye can be scored and in round 2, 2 or 4 Bull’s eye can be scored. 

Case 1: If only 1 Bull’s eye is scored in the round 3, then in round 3 Umeza will score 2 and Zeneca will score 2/3/4 in round 3, which means both will score 5 in round 2. So minimum Bull’s eye in round 2 will be 3. (Umeza, Zeneca and Xyla) 

Hence this case is rejected. 

Case 2: 2 Bull’s eye were scored in round 3 and 4 Bull’s eye were scored in round 2. So in round 2 Umeza, Yonita and Zeneca scored 5. This can be tabulated as: 

In round 3, 2 Bull’s eye can only be scored by Xyla and Umeza. 

The highest scorer can be either Xyla or Zeneca. The lowest scorer will be Wangdu. 

1.Consider Zeneca is the highest scorer. 

From 3, the highest total score was one more than double of the lowest total score. So the only possible score for Zeneca is 23 and that for Wangdu is 11. (11*2+1=23) 

But this will violate condition 2, since both Zeneca and Wangdu do not have their scores as multiples of three in this case. 

Hence, Xyla will be the highest scorer. The only possible total score for Xyla will be 25, and that for Wangdu is 12(4+4+4). (12*2+1=25) 

Since Xyla already has non-multiple of 3 as total score. Zeneca will have 24 as the total score. The complete table is:

Tanzi scored 1 in round 3. 

Instructions 

The following table represents addition of two six-digit numbers given in the first and the second rows, while the sum is given in the third row. In the representation, each of the digits 0, 1, 2, 3, 4, 5, 6, 7, 8, 9 has been coded with one letter among A, B, C, D, E, F, G, H, J, K, with distinct letters representing distinct digits. 

Q. 43 Which digit does the letter A represent? 

Answer:1 

Explanation: 

The value of F can only be 0 as F+F=F can only hold if F=0. 

Also, A can only be 1(in the second column) because to get a carry of more than 1, B has to be a double-digit number which is not possible. (A carry is a digit that is transferred from one column of digits to another column of more significant digits.) 

So the data can be tabulated as follows: 

Since the last row in the third column is 0, the carry to the second column must have been 1, Hence B+1+1=11 => B=9 

In the 4th column, H+H = 10 since a carry 1 has gone to the 3rd column. Hence H=5. 

G+K must be 11 and the carry 1 goes to the next column, so C=1+1=2. 

Now, G,K can take values (3,8), (4,7) and (5,6) in any order. 

From 5th column G=J+1 => J=G-1 

Case: G=3 and K=8, here J =2 which is not possible as C =2 

Case: G=8 and K=3, J=7, a possible case. 

Case: G=4 and K=7, J=3 possible 

Case: G=7 and K=4, J=6 possible 

Case: G=5 and K=6, J=4 not possible as H =5. 

Case: G=6 and K=5, J=5 both J and K are same, not possible. 

Hence the cases can be tabulated as follows: 

The letter A represents 1. 

Q. 44 Which digit does the letter B represent? 

Answer:9 

Explanation: 

The value of F can only be 0 as F+F=F can only hold if F=0. 

Also, A can only be 1(in the second column) because to get a carry of more than 1, B has to be a double-digit number which is not possible. (A carry is a digit that is transferred from one column of digits to another column of more significant digits.) 

So the data can be tabulated as follows: 

Since the last row in the third column is 0, the carry to the second column must have been 1, Hence B+1+1=11 => B=9 

In the 4th column, H+H = 10 since a carry 1 has gone to the 3rd column. Hence H=5. 

G+K must be 11 and the carry 1 goes to the next column, so C=1+1=2. 

Now, G,K can take values (3,8), (4,7) and (5,6) in any order. 

From 5th column G=J+1 => J=G-1 

Case: G=3 and K=8, here J =2 which is not possible as C =2 

Case: G=8 and K=3, J=7, a possible case. 

Case: G=4 and K=7, J=3 possible 

Case: G=7 and K=4, J=6 possible 

Case: G=5 and K=6, J=4 not possible as H =5. 

Case: G=6 and K=5, J=5 both J and K are same, not possible. 

Hence the cases can be tabulated as follows: 

The letter B represents 9. 

Q. 45 Which among the digits 3, 4, 6 and 7 cannot be represented by the letter D? 

Answer:7 

Explanation: 

The value of F can only be 0 as F+F=F can only hold if F=0. 

Also, A can only be 1(in the second column) because to get a carry of more than 1, B has to be a double-digit number which is not possible. (A carry is a digit that is transferred from one column of digits to another column of more significant digits.) 

So the data can be tabulated as follows: 

Since the last row in the third column is 0, the carry to the second column must have been 1, Hence B+1+1=11 => B=9 

In the 4th column, H+H = 10 since a carry 1 has gone to the 3rd column. Hence H=5. 

G+K must be 11 and the carry 1 goes to the next column, so C=1+1=2. 

Now, G,K can take values (3,8), (4,7) and (5,6) in any order. 

From 5th column G=J+1 => J=G-1 

Case: G=3 and K=8, here J =2 which is not possible as C =2 

Case: G=8 and K=3, J=7, a possible case. 

Case: G=4 and K=7, J=3 possible 

Case: G=7 and K=4, J=6 possible 

Case: G=5 and K=6, J=4 not possible as H =5. 

Case: G=6 and K=5, J=5 both J and K are same, not possible. 

Hence the cases can be tabulated as follows: 

In all possible cases 7 is already represented by a letter other than D. Hence 7 is the answer. 

Q. 46 Which among the digits 4, 6, 7 and 8 cannot be represented by the letter G? 

Answer:6 

Explanation: 

The value of F can only be 0 as F+F=F can only hold if F=0. 

Also, A can only be 1(in the second column) because to get a carry of more than 1, B has to be a double-digit number which is not possible. (A carry is a digit that is transferred from one column of digits to another column of more significant digits.) 

So the data can be tabulated as follows: 

Since the last row in the third column is 0, the carry to the second column must have been 1, Hence B+1+1=11 => B=9 

In the 4th column, H+H = 10 since a carry 1 has gone to the 3rd column. Hence H=5. 

G+K must be 11 and the carry 1 goes to the next column, so C=1+1=2. 

Now, G,K can take values (3,8), (4,7) and (5,6) in any order. 

From 5th column G=J+1 => J=G-1 

Case: G=3 and K=8, here J =2 which is not possible as C =2 

Case: G=8 and K=3, J=7, a possible case. 

Case: G=4 and K=7, J=3 possible 

Case: G=7 and K=4, J=6 possible 

Case: G=5 and K=6, J=4 not possible as H =5. 

Case: G=6 and K=5, J=5 both J and K are same, not possible. 

Hence the cases can be tabulated as follows: 

From the table it is clear that 6 cannot be represented by G. 

Instructions 

Comprehension: 

Five vendors are being considered for a service. The evaluation committee evaluated each vendor on six aspects – Cost, Customer Service,Features, Quality, Reach, and Reliability. Each of these evaluations are on a scale of 0 (worst) to 100 (perfect). The evaluation scores on these aspects are shown in the radar chart. For example, Vendor 1 obtains a score of 52 on Reliability, Vendor 2 obtains a score of 45 on Features and Vendor 3 obtains a score of 90 on Cost. 

Q. 47 On which aspect is the median score of the five vendors the least? 

A Customer Service 

B Cost 

C Reliability 

D Quality 

Answer: A 

Explanation: 

The data can be tabulated as follows(approximately): 

Customer Services: 28,41,50,55,70 (The median is 50) 

Cost: 50,71,77,81,90 (The median is 77) 

Reliability: 26, 40, 52, 60, 75 (The median is 52) 

Quality: 40, 48, 62, 69, 72 (The median is 62) 

Hence the customer services has the lowest median. 

Q. 48 A vendor’s final score is the average of their scores on all six aspects. Which vendor has the highest final score? 

A Vendor 4 

B Vendor 2 

C Vendor 1 

D Vendor 3 

Answer: D 

Explanation: 

The data can be tabulated as follows(approximately): 

The average of the vendors will be the highest which has the highest total score. Hence vendor 3 has the highest average. 

Q. 49 List of all the vendors who are among the top two scorers on the maximum number of aspects is: 

A Vendor 2, Vendor 3 and Vendor 4 

B Vendor 1 and Vendor 5 

C Vendor 2 and Vendor 5 

D Vendor 1 and Vendor 2 

Answer: B 

Explanation: 

The data can be tabulated as follows(approximately): 

Top 3 on Reliability: Vendor 3, Vendor 5 

Top 3 on Reach: Vendor 1, Vendor 5 

Top 3 on Quality: Vendor 1, Vendor 2 

Top 3 on Features: Vendor 4, Vendor 5 

Top 3 on Customer Services: Vendor 4, Vendor 1 

Top 3 on Cost: Vendor 3, Vendor 2 

Vendor 1: 3 times Vendor 2: Only once Vendor 3: 2 times Vendor 4: 2 times Vendor 5: 3 times 

Here 1 and 5 comes 3 times. Hence B is the answer. 

Q. 50 List of all the vendors who are among the top three vendors on all six aspects is: 

A Vendor 1 and Vendor 3 

B None of the Vendors 

C Vendor 3 

D Vendor 1 

Answer: C 

Explanation: 

The data can be tabulated as follows(approximately): 

Top 3 on Reliability: Vendor 3, Vendor 5, Vendor 1 

Top 3 on Reach: Vendor 1, Vendor 5, Vendor 3 

Top 3 on Quality: Vendor 1, Vendor 2, Vendor 3 

Top 3 on Features: Vendor 4, Vendor 5, Vendor 3 

Top 3 on Customer Services: Vendor 4, Vendor 1, Vendor 3 

Top 3 on Cost: Vendor 3, Vendor 2, Vendor 1 

Only Vendor 3 ranks among top 3 in all the six parameters. 

 

Instructions 

Comprehension: 

The Ministry of Home Affairs is analysing crimes committed by foreigners in different states and union territories (UT) of India. All cases refer to the ones registered against foreigners in 2016. 

The number of cases – classified into three categories: IPC crimes, SLL crimes and other crimes – for nine states/UTs are shown in the figure below. These nine belong to the top ten states/UTs in terms of the total number of cases registered. The remaining state (among top ten) is West Bengal, where all the 520 cases registered were SLL crimes. 

The table below shows the ranks of the ten states/UTs mentioned above among ALL states/UTs of India in terms of the number of cases registered in each of the three category of crimes. A state/UT is given rank r for a category of crimes if there are (r‐1) states/UTs having a larger number of cases registered in that category of crimes. For example, if two states have the same number of cases in a category, and exactly three other states/UTs have larger numbers of cases registered in the same category, then both the states are given rank 4 in that category. Missing ranks in the table are denoted by *. 

Q. 51 What is the rank of Kerala in the ‘IPC crimes’ category? 

Answer:5 

Explanation: 

The data can be tabulated as follows(approximately): 

Rank of Delhi in IPC crimes category = 1, The rank of Karnataka and Maharashtra is 3(from table), then the rank of Goa can only be 2. 

The rank of Telangana is 6 which has less |IPC crimes than Kerala, which means the rank of Kerala can be less than or equal to 5. 

Now, there are two states with 3 ranks, so there will be no rank 4, there can only be rank 5 which is Kerala. 

Q. 52 In the two states where the highest total number of cases are registered, the ratio of the total number of cases in IPC crimes to the total number in SLL crimes is closest to 

A 3 : 2 

B 19 : 20 

C 11 : 10 

D 1 : 9 

Answer: D 

Explanation: 

The data can be tabulated as follows(approximately): 

The highest cases are registered in West Bengal and Delhi. 

The total number of IPC crimes = 63-64 

The total number of SLL crimes = 520+35-36 = 555-556 

Hence the ratio = (63-64)/(555-556) = 0.11 (Approximately) = 1:9 

Q. 53 Which of the following is DEFINITELY true about the ranks of states/UT in the ‘other crimes’ category? i) Tamil Nadu: 2 

ii) Puducherry: 3 

A both i) and ii) 

B only ii) 

C neither i) , nor ii) 

D only i) 

Answer: A 

Explanation: 

The data can be tabulated as follows(approximately): 

From the table, the rank of Tamilnadu in other crimes is 2. The states which are not in the table will have crimes less than Telangana(i.e 24-25) 

From the table the rank of Pudducherry in other crimes is 3. 

Q. 54 What is the sum of the ranks of Delhi in the three categories of crimes? 

Answer:5 

Explanation: 

The data can be tabulated as follows(approximately): 

The data can be tabulated as follows(approximately): 

The rank of Delhi in IPC crimes should be 1 because the states which are not in table cannot crime more than that of Telangana which is 24-25. 

Similarly Delhi Rank in Other crimes will be 1. 

Now in SLL crimes clearly West Bengal has rank 1. It is given that Karnataka has rank 2. The rank 3 can go to either Goa, Delhi and Maharashtra but Goa and Maharashtra already have rank 4. So Delhi will have rank 3. Also no state outside of the table can be ranked 3 in SLL crimes as the maximum number of crimes should be less than that of Telangana(24-25). Here the number of SLL crimes is 35-36. 

Hence the sum of the ranks = 1+3+1=5 

Instructions 

Comprehension: 

The figure below shows the street map for a certain region with the street intersections marked from a through l. A person standing at an intersection can see along straight lines to other intersections that are in her line of sight and all other people standing at these intersections. For example, a person standing at intersection g can see all people standing at intersections b, c, e, f, h, and k. In particular, the person standing at intersection g can see the person standing at intersection e irrespective of whether there is a person standing at intersection f. 

Six people U, V, W, X, Y, and Z, are standing at different intersections. No two people are standing at the same intersection. 

The following additional facts are known. 

1. X, U, and Z are standing at the three corners of a triangle formed by three street segments. 2. X can see only U and Z. 

3. Y can see only U and W. 

4. U sees V standing in the next intersection behind Z. 

5. W cannot see V or Z. 

6. No one among the six is standing at intersection d. 

Q. 55 Who is standing at intersection a? 

A W 

B Y 

C No one 

D V 

Answer: C 

Explanation: 

From 1, X, U, and Z are standing at the three corners of a triangle formed by three street segments. From 2, X can see only U and Z. 

From 4, U sees V standing in the next intersection behind Z. Also, no one among the six is standing at intersection d. Only cases possible are: 

1. 

W cannot see V or Z. So W can only be at the intersection a. Since Y can see only U and W, Y can only be at c where X can see him. Hence this case is rejected. 

2. 

Y can only see U and W. Y cannot be placed anywhere. Hence this case is also rejected. 

3. 

Y can only see U and W. Y cannot be placed anywhere. Hence this case is also rejected. 

4. 

W cannot see V or Z. W can only be placed at i. Y can see only U and W. Y can only be placed at j or e, where he can see more people than U and W. Hence this case is also rejected. 

5. 

W cannot see V or Z. Y can only see U and W. Hence W and Y can only be placed as shown:

No one is standing at the intersection A. Hence C is the answer. 

Q. 56 Who can V see? 

A Z only 

B U, W and Z only 

C U and Z only 

D U only 

Answer: C 

Explanation: 

From 1, X, U, and Z are standing at the three corners of a triangle formed by three street segments. From 2, X can see only U and Z. 

From 4, U sees V standing in the next intersection behind Z. Also, no one among the six is standing at intersection d. Only cases possible are: 

1. 

W cannot see V or Z. So W can only be at the intersection a. Since Y can see only U and W, Y can only be at c where X can see him. Hence this case is rejected. 

2. 

Y can only see U and W. Y cannot be placed anywhere. Hence this case is also rejected. 

3. 

Y can only see U and W. Y cannot be placed anywhere. Hence this case is also rejected. 

4. 

W cannot see V or Z. W can only be placed at i. Y can see only U and W. Y can only be placed at j or e, where he can see more people than U and W. Hence this case is also rejected. 

5. 

W cannot see V or Z. Y can only see U and W. Hence W and Y can only be placed as shown:

V can see U and Z only. Hence C is the answer. 

Q. 57 What is the minimum number of street segments that X must cross to reach Y? 

A 1 

B 4 

C 2 

D 3 

Answer: C 

Explanation: 

From 1, X, U, and Z are standing at the three corners of a triangle formed by three street segments. From 2, X can see only U and Z. 

From 4, U sees V standing in the next intersection behind Z. Also, no one among the six is standing at intersection d. Only cases possible are: 

1. 

W cannot see V or Z. So W can only be at the intersection a. Since Y can see only U and W, Y can only be at c where X can see him. Hence this case is rejected. 

2. 

Y can only see U and W. Y cannot be placed anywhere. Hence this case is also rejected. 

3. 

Y can only see U and W. Y cannot be placed anywhere. Hence this case is also rejected. 

4. 

W cannot see V or Z. W can only be placed at i. Y can see only U and W. Y can only be placed at j or e, where he can see more people than U and W. Hence this case is also rejected. 

5. 

W cannot see V or Z. Y can only see U and W. Hence W and Y can only be placed as shown:

To reach Y, X has to go from b to g and g to k, i.e. 2 streets. 

Q. 58 Should a new person stand at intersection d, who among the six would she see? 

A W and X only 

B U and W only 

C U and Z only 

D V and X only 

Answer: A 

Explanation: 

From 1, X, U, and Z are standing at the three corners of a triangle formed by three street segments. From 2, X can see only U and Z. 

From 4, U sees V standing in the next intersection behind Z. Also, no one among the six is standing at intersection d. Only cases possible are: 

1. 

W cannot see V or Z. So W can only be at the intersection a. Since Y can see only U and W, Y can only be at c where X can see him. Hence this case is rejected. 

2. 

Y can only see U and W. Y cannot be placed anywhere. Hence this case is also rejected. 

3. 

Y can only see U and W. Y cannot be placed anywhere. Hence this case is also rejected. 

4. 

W cannot see V or Z. W can only be placed at i. Y can see only U and W. Y can only be placed at j or e, where he can see more people than U and W. Hence this case is also rejected. 

5. 

W cannot see V or Z. Y can only see U and W. Hence W and Y can only be placed as shown: 

If a new person stands at d(left down corner), they can see W and X only. Hence A is the answer. 

Instructions 

Comprehension: 

Princess, Queen, Rani and Samragni were the four finalists in a dance competition. Ashman, Badal, Gagan and Dyu were the four music composers who individually assigned items to the dancers. Each dancer had to individually perform in two dance items assigned by the different composers. The first items performed by the four dancers were all assigned by different music composers. No dancer performed her second item before the performance of the first item by any other dancers. The dancers performed their second items in the same sequence of their performance of their first items. 

The following additional facts are known. 

i) No composer who assigned item to Princess, assigned any item to Queen. 

ii) No composer who assigned item to Rani, assigned any item to Samragni. 

iii) The first performance was by Princess; this item was assigned by Badal. 

iv) The last performance was by Rani; this item was assigned by Gagan. 

v) The items assigned by Ashman were performed consecutive ly. The number of performances between items assigned by each of the remaining composers was the same. 

Q. 59 Which of the following is true? 

A The second performance was composed by Dyu. 

B The third performance was composed by Dyu. 

C The third performance was composed by Ashman. 

D The second performance was composed by Gagan. 

Answer: A 

Explanation: 

Since the dancers performed their second items in the same sequence of their performance of their first items. The table will be as follows: 

The items assigned by Ashman were performed consecutively. The number of performances between items assigned by each of the remaining composers was the same. 

Also, the first items performed by the four dancers were all assigned by different music composers. Badal can come only at the place as shown in the table. 

Then Ashman can only compose for the following performances. 

Hence Dyu will compose for the following performances: 

From (i) No composer who assigned item to Princess, assigned any item to Queen. 

From (ii) No composer who assigned item to Rani, assigned any item to Samragni. 

Hence Dyu will compose for Samragni 1st Performance and Gagan will compose for Queen 1st Performance. Also, Badal will compose for Samragni 2nd Performance and Dyu will compose for Queens 2nd Performance. 

Hence, the complete table is as follows: 

The second performance was composed by Dyu. Hence A is the answer. 

Q. 60 Which of the following is FALSE? 

A Samragni did not perform in any item composed by Ashman. 

B Princess did not perform in any item composed by Dyu. 

C Rani did not perform in any item composed by Badal. 

D Queen did not perform in any item composed by Gagan. 

Answer: D 

Explanation: 

Since the dancers performed their second items in the same sequence of their performance of their first items. The table will be as follows: 

The items assigned by Ashman were performed consecutively. The number of performances between items assigned by each of the remaining composers was the same. 

Also, the first items performed by the four dancers were all assigned by different music composers. Badal can come only at the place as shown in the table. 

Then Ashman can only compose for the following performances. 

Hence Dyu will compose for the following performances: 

From (i) No composer who assigned item to Princess, assigned any item to Queen. 

From (ii) No composer who assigned item to Rani, assigned any item to Samragni. 

Hence Dyu will compose for Samragni 1st Performance and Gagan will compose for Queen 1st Performance. Also, Badal will compose for Samragni 2nd Performance and Dyu will compose for Queens 2nd Performance. 

Hence, the complete table is as follows: 

Option A: Samragni did not perform in any item composed by Ashman. This statement is true. Option B: Princess did not perform in any item composed by Dyu. This is also true. 

Option C: Rani did not perform in any item composed by Badal. This statement is true. 

Option D: Queen did not perform in any item composed by Gagan. This statement is false. 

Hence D is the answer. 

Q. 61 The sixth performance was composed by: 

A Badal 

B Dyu 

C Ashman 

D Gagan 

Answer: A 

Explanation: 

Since the dancers performed their second items in the same sequence of their performance of their first items. The table will be as follows: 

The items assigned by Ashman were performed consecutively. The number of performances between items assigned by each of the remaining composers was the same. 

Also, the first items performed by the four dancers were all assigned by different music composers. Badal can come only at the place as shown in the table. 

Then Ashman can only compose for the following performances. 

Hence Dyu will compose for the following performances: 

From (i) No composer who assigned item to Princess, assigned any item to Queen. 

From (ii) No composer who assigned item to Rani, assigned any item to Samragni. 

Hence Dyu will compose for Samragni 1st Performance and Gagan will compose for Queen 1st Performance. Also, Badal will compose for Samragni 2nd Performance and Dyu will compose for Queens 2nd Performance. 

Hence, the complete table is as follows: 

The sixth performance was composed by Badal. Hence C is the answer. 

Q. 62 Which pair of performances were composed by the same composer? 

A The first and the seventh 

B The third and the seventh 

C The second and the sixth 

D The first and the sixth 

Answer: D 

Explanation: 

Since the dancers performed their second items in the same sequence of their performance of their first items. The table will be as follows: 

The items assigned by Ashman were performed consecutively. The number of performances between items assigned by each of the remaining composers was the same. 

Also, the first items performed by the four dancers were all assigned by different music composers. Badal can come only at the place as shown in the table. 

Then Ashman can only compose for the following performances. 

Hence Dyu will compose for the following performances: 

From (i) No composer who assigned item to Princess, assigned any item to Queen. 

From (ii) No composer who assigned item to Rani, assigned any item to Samragni. 

Hence Dyu will compose for Samragni 1st Performance and Gagan will compose for Queen 1st Performance. Also, Badal will compose for Samragni 2nd Performance and Dyu will compose for Queens 2nd Performance. 

Hence, the complete table is as follows: 

The first and the sixth items were composed by Badal. Hence D is the answer. 

Instructions 

Comprehension: 

A new game show on TV has 100 boxes numbered 1, 2, . . . , 100 in a row, each containing a mystery prize. The prizes are items of different types, a, b, c, . . . , in decreasing order of value. The most expensive item is of type a, a diamond ring, and there is exactly one of these. You are told that the number of items at least doubles as you move to the next type. For example, there would be at least twice as many items of type b as of type a, at least twice as many items of type c as of type b and so on. There is no particular order in which the prizes are placed in the boxes. 

Q. 63 What is the minimum possible number of different types of prizes? 

Answer:2 

Explanation: 

It is given that the most expensive item is a diamond ring of type a and there is exactly one of these. Since the item b should be at least twice. The minimum number of items will be obtained when a=1 and b=99, which means there are only two different types of items. 

Q. 64 What is the maximum possible number of different types of prizes? 

Answer:6 

Explanation: 

It is given that the most expensive item is a diamond ring of type a and there is exactly one of these. Since the number of items of type b should be at least twice of that of a and the number of items of type c should be at least twice of that of b and so on. So the maximum number of different types of items of a, b and c will be obtained when a=1, b=2, c=4, d=8, e=16, f=69. Hence the maximum number of different types of items will be 6. 

If the number of items is 7, then the minimum number of prizes should be 1+2+4+8+16+32+64=127 which is more than 100. 

Hence 6 is the answer. 

Q. 65 Which of the following is not possible? 

A There are exactly 75 items of type e. 

B There are exactly 30 items of type b. 

C There are exactly 45 items of type c. 

D There are exactly 60 items of type d. 

Answer: C 

Explanation: 

Option A: There are exactly 75 items of type e. 

a=1,b=2,c=4,d=8, e=85. Here the maximum value of e= 85. Hence it can take the value 75. An example of such case is a=1,b=2,c=4,d=18, e=75 

Option B: There are exactly 30 items of type b. 

a=1 b=30 and c=69. Hence this case is also possible. 

Option C: There are exactly 45 items of type c. 

Since the value of d should be at least 90, it means that d is not present because 45+90 will be more than 100(maximum number of items). Only a,b and c are present. 

The maximum value of b = 22 and a =1, but 45+22+1=68, which is less than 100. So this case is not possible. Option D: There are exactly 60 items of type d. 

d=60, c=30, b=9 and a=1. a+b+c+d=100. Hence this case is possible. 

C is the answer. 

Q. 66 You ask for the type of item in box 45. Instead of being given a direct answer, you are told that there are 31 items of the same type as box 45 in boxes 1 to 44 and 43 items of the same type as box 45 in boxes 46 to 100. 

What is the maximum possible number of different types of items? 

A 5 

B 6 

C 4 

D 3 

Answer: A 

Explanation: 

The total number of items from 1 to 100, which are of same type as in box 45 = 31+1+43=75 Now to maximize the number of items, a=1, b=2, c=4, d=18 and e=75(given) 

There can be maximum 5 types of items. 

If we consider number of items to be 6, then minimum number of items of 5th type will be 16, 1+2+4+8+16+75=106 which is more than 100. 

 

Quantitative Aptitude 

Instructions 

For the following questions answer them individually 

Q. 67 Two cars travel the same distance starting at 10:00 am and 11:00 am, respectively, on the same day. They reach their common destination at the same point of time. If the first car travelled for at least 6 hours, then the highest possible value of the percentage by which the speed of the second car could exceed that of the first car is 

A 20 

B 30 

C 25 

D 10 

Answer: A 

Explanation: 

Let the speed of cars be a and b and the distance =d 

Minimum time taken by 1st car = 6 hours, 

For maximum difference in time taken by both of them, car 1 has to start at 10:00 AM and car 2 has to start at 11:00 AM. 

Hence, car 2 will take 5 hours. 

Hence a=d/6 and b = d/5

Hence the speed of car 2 will exceed the speed of car 1 by(d/5d/6)/ d/6×100 = (d/30)/(d/6) ×100 = 20 

Q. 68  If a1, a2, …… are in A.P., then, 1/(√a1+√a2)+1/(√a2+√a3)+…….+1/ (√an+√an+1

is equal to 

A n/(√an+√an+1)

B n-1/(√an+√an-1)

C n-1/(√a1+√an)

D n/(√a1-√an+1)

Answer: A  

Q. 69 AB is a diameter of a circle of radius 5 cm. Let P and Q be two points on the circle so that the length of PB is 6 cm, and the length of AP is twice that of AQ. Then the length, in cm, of QB is nearest to 

A 9.3 

B 7.8 

C 9.1 

D 8.5 

Answer: C 

Explanation: 

Since AB is a diameter, AQB and APB will be at right angles. 

In right triangle APB, AP = √(102 − 62) = 8

Now, 2AQ=AP => AQ= 8/2=4 

In right triangle AQB, AP =√(102 − 42) =  9.165 =9.1 (Approx) 

Q. 70 If (5.55)x = (0.555)y = 1000, then the value of 1/x1/y is 

A ⅓

B 3

C 1 

D ⅔

Answer: A 

Explanation: 

We have, (5.55)x = (0.555)y = 1000 

Taking log in base 10 on both sides, 

x(log10 555 -2) = y( log10 555-3) = 3 

Then, x( log10 555-2) = 3…..(1) 

y(log10 555 -3) = 3 …..(2) 

From (1) and (2) 

=> log10 555=3/x +2 = 3/y +3

=> 1/x1/y = ⅓

Q. 71 The income of Amala is 20% more than that of Bimala and 20% less than that of Kamala. If Kamala’s income goes down by 4% and Bimala’s goes up by 10%, then the percentage by which Kamala’s income would exceed Bimala’s is nearest to 

A 31 

B 29 

C 28 

D 32 

Answer: A 

Explanation: 

Assuming the income of Bimla = 100a, then the income of Amala will be 120a. 

And the income of Kamala will be 120a*100/80=150a 

If Kamala’s income goes down by 4%, then new income of Kamala = 150a-150a(4/100) = 150a-6a=144a If Bimla’s income goes up by 10 percent, her new income will be 100a+100a(10/100)=110a => Hence the Kamala income will exceed Bimla income by (144a-110a)*100/110a=31 

Q. 72 The wheels of bicycles A and B have radii 30 cm and 40 cm, respectively. While traveling a certain distance, each wheel of A required 5000 more revolutions than each wheel of B. If bicycle B traveled this distance in 45 minutes, then its speed, in km per hour, was 

A 18π

B 14π

C 16π

D 12π 

Answer: C Explanation: 

Distance covered by A in 1 revolution = 2π *30 = 60π 

Distance covered by B in 1 revolution = 2π *40 = 80π 

Now, (5000+n)60π = 80π

=> 15000= 4n-3n =>n=15000 

Then distance travelled by B = 15000*80π cm = 12π km 

Hence, the speed =12π× 60/45 = 16 π

Q. 73 The product of the distinct roots of ∣ x2 x − 6 ∣= x + 2 is 

A −16 

B -4 

C -24 

D -8 

Answer: A 

Explanation: 

We have, ∣ x2 x − 6 ∣= x + 2 

=> |(x-3)(x+2)|=x+2 

For x<-2, (3-x)(-x-2)=x+2 

=> x-3=1 =>x=4 (Rejected as x<-2) 

For -2≤ x<3, (3-x)(x+2)=x+2 =>x=2,-2 

For x≥ 3, (x-3)(x+2)=x+2 =>x=4 

Hence the product =4*-2*2=-16 

Q. 74 In a race of three horses, the first beat the second by 11 metres and the third by 90 metres. If the second beat the third by 80 metres, what was the length, in metres, of the racecourse? 

Answer:880 

Explanation: 

Assuming the length of race course = x and the speed of three horses be a,b and c respectively. 

x/a = x−11/b

Hence, ……(1) 

and x/ax−90/c ……(2)

Also x/b = x−80/c ,……(3)

From 1 and 2, we get, x−11/b = x−90/c …..(4)

Dividing (3) by (4), we get, => x−11/x = x−90/x−80 

=> (x−11) (x−80) = x (x−90)

91x-90x=880 

=> x=880 

Q. 75 If the population of a town is p in the beginning of any year then it becomes 3 + 2p in the beginning of the next year. If the population in the beginning of 2019 is 1000, then the population in the beginning of 2034 will be 

A (1003)15 + 6

B  (997)15 − 3 

C (997)214 + 3

D (1003)215 − 3 

Answer: D 

Explanation: 

The population of town at the beginning of 1st year = p 

The population of town at the beginning of 2nd year = 3+2p 

The population of town at the beginning of 3rd year = 2(3+2p)+3 = 2*2p+2*3+3 =4p+3(1+2) 

The population of town at the beginning of 4th year = 2(2*2p+2*3+3)+3 = 8p+3(1+2+4) 2n−1 ( ) 

Similarly population at the beginning of the nth year = 2n−1 p+3(2n−1 −1) = 2n−1(p + 3)-3 The population in the beginning of 2019 is 1000, then the population in the beginning of 2034 will be

(22034−2019 ) (1000 + 3) -3 = 215 (1003)-3

Q. 76 Consider the function f satisfying f (x + y) = f (x) f (y) where x,y are positive integers, and f(1) = 2. If f(a + 1) +f (a + 2) + … + f(a + n) = 16 (2n – 1) then a is equal to 

Answer:3 

Explanation: 

f (x + y) = f (x) f (y) 

Hence, f(2)=f(1+1)=f(1)*f(1)=2*2=4 

f(3)=f(2+1)=f(2)*f(1)=4*2=8 

f(4)=f(3+1)=f(3)*f(1)=8*2=16 

…….=> f(x)=  2x 

Now, f(a + 1) +f (a + 2) + … + f(a + n) = 16 (2n – 1) 

On putting n=1 in the equation we get, f(a+1)=16 => f(a)*f(1)=16 (It is given that f (x + y) = f (x) f (y))

=> 2a*2=16 

=> a=3 

Q. 77 Amala, Bina, and Gouri invest money in the ratio 3 : 4 : 5 in fixed deposits having respective annual interest rates in the ratio 6 : 5 : 4. What is their total interest income (in Rs) after a year, if Bina’s interest income exceeds Amala’s by Rs 250? 

A 6350 

B 6000 

C 7000 

D 7250 

Answer: D 

Explanation: 

Assuming the investment of Amala, Bina, and Gouri be 300x, 400x and 500x, hence the interest incomes will be 300x*6/100=18x, 400x*5/100=20x and 500x*4/100 = 20x 

Given, Bina’s interest income exceeds Amala by 20x-18x=2x=250 => x=125 

Now, total interest income = 18x+20x+20x=58x = 58*125 = 7250 

Q. 78 For any positive integer n, let f(n) = n(n + 1) if n is even, and f(n) = n + 3 if n is odd. If m is a positive integer such that 8f(m + 1) – f(m) = 2, then m equals 

Answer:10 

Explanation: 

Assuming m is even, then 8f(m+1)-f(m)=2 

m+1 will be odd 

So, 8(m+1+3)-m(m+1)=2 

=> 8m+32- m2 m =2 

=> m2 − 7m − 30 = 0

=> m=10,-3 

Rejecting the negative value, we get m=10 

Q. 79 The product of two positive numbers is 616. If the ratio of the difference of their cubes to the cube of their difference is 157:3, then the sum of the two numbers is 

A 58 

B 85 

C 50 

D 95 

Answer: C 

Explanation: 

Assume the numbers are a and b, then ab=616

We have, a3b3/(ab)3  = 157/3

=> 3 (a3 b3) = 157 (a3b3+ 3ab (a b)

=> 154 (a3 b3) + 3 ∗ 157 ∗ ab (a b)= 0 

=> 154 (a3 b3) + 3 ∗ 616 ∗ 157 (a b)= 0 (∵ab=616) 

=> a3 b3 + (3 × 4 × 157 (a b)) (∵154*4=616) 

=> (a b) (a2 + b2 + ab) = 3 × 4 × 157 (a b)

=> a2 + b2 + ab = 3 × 4 × 157 

Adding ab=616 on both sides, we get 

a2 + b2 + ab + ab = 3 × 4 × 157 + 616 

=> (a + b)2 = 3 × 4 × 157 + 616 = 2500

=> a+b=50 

Q. 80 One can use three different transports which move at 10, 20, and 30 kmph, respectively. To reach from A to B, Amal took each mode of transport ⅓  of his total journey time, while Bimal took each mode of transport ⅓ of the total distance. The percentage by which Bimal’s travel time exceeds Amal’s travel time is nearest to 

A 22 

B 20 

C 19 

D 21 

Answer: A 

Explanation: 

Assume time taken by Amal = t 

Then, the distance between A and B = d= t/3(10 + 20 + 30) = 20t

Total time taken by Bimal = 20t/3(1/10 + 1/20 +1/30) = 20t/3 *11/60 = 11t/9

Hence, the time taken by Bimal will exceed Amal by (11t/9-t)/t = 22.22 

Q. 81 Meena scores 40% in an examination and after review, even though her score is increased by 50%, she fails by 35 marks. If her post-review score is increased by 20%, she will have 7 marks more than the passing score. The percentage score needed for passing the examination is 

A 60 

B 80 

C 70 

D 75 

Answer: C 

Explanation: 

Assuming the maximum marks =100a, then Meena got 40a 

After increasing her score by 50%, she will get 40a(1+50/100)=60a 

Passsing score = 60a+35 

Post review score after 20% increase = 60a*1.2=72a 

=>Hence, 60a+35+7=72a 

=>12a=42 =>a=3.5 

=> maximum marks = 350 and passing marks = 210+35=245 

=> Passing percentage = 245*100/350 = 70 

Q. 82 A person invested a total amount of Rs 15 lakh. A part of it was invested in a fixed deposit earning 6% annual interest, and the remaining amount was invested in two other deposits in the ratio 2 : 1, earning annual interest at the rates of 4% and 3%, respectively. If the total annual interest income is Rs 76000 then the amount (in Rs lakh) invested in the fixed deposit was 

Answer:9 

Explanation: 

Assuming the amount invested in the ratio 2:1 was 200x and 100x, then the fixed deposit investment = 1500000-300x Hence, the interest = 200x*4/100 = 8x and 100x*3/100=3x 

Interest from the fixed deposit = (1500000-300x)*6/100 = 90000-18x 

Hence the total interest = 90000-18x+8x+3x=90000-7x =76000 

=> 7x=14000 => x=2000 

Hence, the fixed deposit investment = 1500000-300*2000 = 900000 = 9 lakhs 

Q. 83 A club has 256 members of whom 144 can play football, 123 can play tennis, and 132 can play cricket. Moreover, 58 members can play both football and tennis, 25 can play both cricket and tennis, while 63 can play both football and cricket. If every member can play at least one game, then the number of members who can play only tennis is 

A 38 

B 32 

C 45 

D 43 

Answer: D 

Explanation: 

Assume the number of members who can play exactly 1 game = I 

The number of members who can play exactly 1 game = II 

The number of members who can play exactly 1 game = III 

I+2II+3III=144+123+132=399….(1) 

I+II+III=256……(2) 

=> II+2III=143…..(3) 

Also, II+3III=58+25+63=146 ……(4) 

=> III = 3 (From 3 and 4) 

=> II =137 

=> I = 116 

The members who play only tennis = 123-58-25+3 = 43 

Q. 84 If a1 + a2 + a3 + …. + an = 3(2n+1 − 2) n ≥ 1, for every a11, then equals 

Answer:6144 

Explanation: 

11th term of series = a11= Sum of 11 terms – Sum of 10 terms =3(211+1 − 2) -3 (210+1 − 2)

= 3(212 − 2 − 211 + 2) =3(2 − 1) 211 = 3*(211) = 6144 

Q. 85 The number of the real roots of the equation 2 cos(x(x + 1)) = 2x + 2x is 

A 2 

B 1 

C infinite 

D 0 

Answer: B 

Explanation: 

2 cos(x(x + 1)) = 2x + 2x 

The maximum value of LHS is 2 when cos(x(x + 1)) is 1 and the minimum value of RHS is 2 using AM ≥ GM Hence LHS and RHS can only be equal when both sides are 2. For LHS, cosx(x+1)=1 => x(x+1)=0 => x=0,-1 

For RHS minimum value, x=0 

Hence only one solution x=0 

Q. 86 At their usual efficiency levels, A and B together finish a task in 12 days. If A had worked half as efficiently as she usually does, and B had worked thrice as efficiently as he usually does, the task would have been completed in 9 days. How many days would A take to finish the task if she works alone at her usual efficiency? 

A 36 

B 24 

C 18 

D 12 

Answer: C 

Explanation: 

Assuming A completes a units of work in a day and B completes B units of work in a day and the total work = 1 unit 

Hence, 12(a+b)=1………(1) 

Also, 9(a/2 +3b)=1 ………(2) 

Using both equations, we get, 12(a+b)= 9(a/2 +3b) 

=> 4a+4b= 3a/2+9b 

=> 5a/2=5b 

=> a=2b 

Substituting the value of b in equation (1), 

12(3a/2 )=1 

=> a= 1/18

Hence, the number of days required = 1/( 1/18 )=18 

Q. 87 In a class, 60% of the students are girls and the rest are boys. There are 30 more girls than boys. If 68% of the students, including 30 boys, pass an examination, the percentage of the girls who do not pass is 

Answer:20 

Explanation: 

Assuming the number of students =100x 

Hence, the number of girls = 60x and the number of boys = 40x 

We have, 60x-40x=30 => x=1.5 

The number of girls = 60*1.5=90 

Number of girls that pass = 68x-30=68*1.5-30 = 102-30=72 

The number of girls who do not pass = 90-72=18 

Hence the percentage of girls who do not pass = 1800/90=20 

Q. 88 In a circle of radius 11 cm, CD is a diameter and AB is a chord of length 20.5 cm. If AB and CD intersect at a point E inside the circle and CE has length 7 cm, then the difference of the lengths of BE and AE, in cm, is 

A 2.5 

B 1.5 

C 3.5 

D 0.5 

Answer: D 

Explanation: 

In figure AE*BE=CE*DE (Ptolemy Theorem) 

=> 7*15= x(20.5-x) (Assuming AE=x) 

=> 210=x(41-2x) 

=> 2x2-41x+210=0 

=> x=10 or x=10.5 => AE=10 or AE=10.5 Hence BE = 20.5-10=10.5 or BE = 20.5-10.5=10 Required difference= 10.5-10=0.5 

Q. 89 On selling a pen at 5% loss and a book at 15% gain, Karim gains Rs. 7. If he sells the pen at 5% gain and the book at 10% gain, he gains Rs. 13. What is the cost price of the book in Rupees? 

A 95 

B 85 

C 80 

D 100 

Answer: C 

Explanation: 

Assuming the cost price of pen = 100p and the cost price of book = 100b 

So, on selling a pen at 5% loss and a book at 15% gain, net gain = -5p+15b = 7 ….1 

On selling the pen at 5% gain and the book at 10% gain, net gain = 5p+10b = 13 …..2 

Adding 1 and 2 we get, 25b=20 

Hence 100b= 20*4=80, 

C is the answer. 

Q. 90 A chemist mixes two liquids 1 and 2. One litre of liquid 1 weighs 1 kg and one litre of liquid 2 weighs 800 gm. If half litre of the mixture weighs 480 gm, then the percentage of liquid 1 in the mixture, in terms of volume, is 

A 80 

B 70 

C 85 

D 75 

Answer: A 

Explanation: 

The weight/volume(g/L) for liquid 1 = 1000 

The weight/volume(g/L) for liquid 2 = 800 

The weight/volume(g/L) of the mixture = 480/(1/2) = 960 

Using alligation the ratio of liquid 1 and liquid 2 in the mixture = (960-800)/(1000-960) = 160/40 = 4:1 Hence the percentage of liquid 1 in the mixture = 4*100/(4+1)=80 

Q. 91 Ramesh and Gautam are among 22 students who write an examination. Ramesh scores 82.5. The average score of the 21 students other than Gautam is 62. The average score of all the 22 students is one more than the average score of the 21 students other than Ramesh. The score of Gautam is 

A 53 

B 51 

C 48 

D 49 

Answer: B 

Explanation: 

Assume the average of 21 students other than Ramesh = a 

Sum of the scores of 21 students other than Ramesh = 21a 

Hence the average of 22 students = a+1 

Sum of the scores of all 22 students = 22(a+1) 

The score of Ramesh = Sum of scores of all 22 students – Sum of the scores of 21 students other than Ramesh = 22(a+1)-21a=a+22 = 82.5 (Given) 

=> a = 60.5 

Hence, sum of the scores of all 22 students = 22(a+1) = 22*61.5 = 1353 

Now the sum of the scores of students other than Gautam = 21*62 = 1302 

Hence the score of Gautam = 1353-1302=51 

Q. 92 If m and n are integers such that (√2)19 34 42 9m 8n = 3n 16m ( 4√ 64 )  then m is

A -20 

B -24 

C -12 

D -16 

Answer: C Explanation: 

We have, (√2)1934429m8n = 3n 16m ( 4√ 64 ) 

Converting both sides in powers of 2 and 3, we get 3n 3 2 2 

219/4 34 24 32m 23m =  3n 24m 26/4 

Comparing the power of 2 we get, 19/2+4+3n = 4m+6/4

=> 4m=3n+12 …..(1) 

Comparing the power of 3 we get, 4 + 2m = n

Substituting the value of n in (1), we get 

4m=3(4+2m)+12 

=> m=-12 

Q. 93 Three men and eight machines can finish a job in half the time taken by three machines and eight men to finish the same job. If two machines can finish the job in 13 days, then how many men can finish the job in 13 days? 

Answer:13 

Explanation: 

Consider the work done by a man in a day = a and that by a machine = b 

Since, three men and eight machines can finish a job in half the time taken by three machines and eight men to finish the same job, hence the efficiency will be double. 

=> 3a+8b = 2(3b+8a) 

=> 13a=2b 

Hence work done by 13 men in a day = work done by 2 machines in a day. 

=> If two machines can finish the job in 13 days, then same work will be done 13 men in 13 days. Hence the required number of men = 13 

Q. 94 Corners are cut off from an equilateral triangle T to produce a regular hexagon H. Then, the ratio of the area of H to the area of T is 

A 2 : 3 

B 4 : 5 

C 5 : 6 

D 3 : 4 

Answer: A 

Explanation: 

The given figure can be divided into 9 regions or equilateral triangles of equal areas as shown below, 

Now the hexagon consists of 6 regions and the triangle consists of 9 regions. Hence the ratio of areas = 6/9 =2:3 

Q. 95 Let x and y be positive real numbers such that log5 (x + y) + log5 (x y) = 3, and log2 y − log2 x = 1 − log2 3 . Then xy equals 

A 150 

B 25 

C 100 

D 250 

Answer: A 

Explanation: 

We have, log5 (x + y) + log5 (x y) = 3

=> x2 y2 = 125 ……(1) 

log2 y − log2 x = 1 − log2 3

=> y/x= 2/3

=> 2x=3y => x= 3y/2 

On substituting the value of x in 1, we get 

5x2/4 =125 

=>y=10, x=15 

Hence xy=150 

Q. 96 

Let S be the set of all points (x, y) in the x-y plane such that∣ x ∣ + ∣ y ∣≤ 2 and ∣ x ∣≥ 1. Then, the area, in square units, of the region represented by S equals 

Answer:2 

Explanation: 

Sum of the area of region I and II is the required area. 

Now, required area = 4 × ½× 1× 1 = 2 

Q. 97 With rectangular axes of coordinates, the number of paths from (1, 1) to (8, 10) via (4, 6), where each step from any point (x, y) is either to (x, y+1) or to (x+1, y), is 

Answer:3920 

Explanation: 

The number of paths from (1, 1) to (8, 10) via (4, 6) = The number of paths from (1,1) to (4,6) * The number of paths from (4,6) to (8,10) 

To calculate the number of paths from (1,1) to (4,6), 4-1 =3 steps in x-directions and 6-1=5 steps in y direction (3+5)C

Hence the number of paths from (1,1) to (4,6) = = 56 

To calculate the number of paths from (4,6) to (8,10), 8-4 =4 steps in x-directions and 10-6=4 steps in y direction (4+4)C

Hence the number of paths from (4,6) to (8,10) = = 70 

The number of paths from (1, 1) to (8, 10) via (4, 6) = 56*70=3920 

Q. 98 If the rectangular faces of a brick have their diagonals in the ratio 3 : 2√3 : √15, then the ratio of the length of the shortest edge of the brick to that of its longest edge is 

A √3 : 2

B 1 : √3

C 2 : √3

D √2 : √3

Answer: B

Explanation: 

Assuming the dimensions of the brick are a, b and c and the diagonals are 3, 2√3 and√15 

Hence, a2 + b2 =32  ……(1) 

b2 + c2=(2√3)2 ……(2) 

c2 + a2=(√15)2 ……(3) 

Adding the three equations, 2(a2 + b2 + c2 ) = 9+12+15=36 

=> a2 + b2 + c2= 18……(4) 

Subtracting (1) from (4), we get c2= 9 =>c=3 

Subtracting (2) from (4), we get a2= 6 =>a= √6

Subtracting (3) from (4), we get b2= 3 =>b= √3

The ratio of the length of the shortest edge of the brick to that of its longest edge is =√3/3 =1:√3 

Q. 99 The number of solutions to the equation ∣ x ∣ (6x2 + 1) = 5x2 is 

Answer:5 

Explanation: 

For x <0, -x(6x2 + 1 ) = 5x2 

=> ( 6x2 + 1) = -5x 

=> (6x2 + 5x + 1 ) = 0 

=>(6x2 + 3x + 2x + 1 ) = 0 

=> (3x+1)(2x+1)=0 =>x= -⅓or x= -½

For x=0, LHS=RHS=0 (Hence, 1 solution) 

For x >0, x( 6x2 + 1) = 5x2

=> (6x2 − 5x + 1 ) = 0 

=>(3x-1)(2x-1)=0 =>x=⅓ or x= ½

Hence, the total number of solutions = 5 

Q. 100 Let T be the triangle formed by the straight line 3x + 5y – 45 = 0 and the coordinate axes. Let the circumcircle of T have radius of length L, measured in the same unit as the coordinate axes. Then, the integer closest to L is 

Answer:9 

Explanation: 

In any right triangle, the circumradius is half of the hypotenuse. Here,L=½ * the length of the hypotenuse =½  √(152 + 92) =½ *306 = ½×17.49 = 8.74 

Hence, the integer close to L = 9 

CAT Previous Year Paper Session-II 2018

CAT 2018 Session-II

Verbal Ability 

Instructions 

Read the passage carefully and answer the following questions 

NOT everything looks lovelier the longer and closer its inspection. But Saturn does. It is gorgeous through Earthly telescopes. However, the 13 years of close observation provided by Cassini, an American spacecraft, showed the planet, its moons and its remarkable rings off better and better, revealing finer structures, striking novelties and greater drama. . . . 

By and large the big things in the solar system—planets and moons—are thought of as having been around since the beginning. The suggestion that rings and moons are new is, though, made even more interesting by the fact that one of those moons, Enceladus, is widely considered the most promising site in the solar system on which to look for alien life. If Enceladus is both young and bears life, that life must have come into being quickly. This is also believed to have been the case on Earth. Were it true on Enceladus, that would encourage the idea that life evolves easily when conditions are right. 

One reason for thinking Saturn’s rings are young is that they are bright. The solar system is suffused with comet dust, and comet dust is dark. Leaving Saturn’s ring system (which Cassini has shown to be more than 90% water ice) out in such a mist is like leaving laundry hanging on a line downwind from a smokestack: it will get dirty. The lighter the rings are, the faster this will happen, for the less mass they contain, the less celestial pollution they can absorb before they start to discolour. . . . Jeff Cuzzi, a scientist at America’s space agency, NASA, who helped run Cassini, told the Lunar and Planetary Science Conference in Houston that combining the mass estimates with Cassini’s measurements of the density of comet-dust near Saturn suggests the rings are no older than the first dinosaurs, nor younger than the last of them—that is, they are somewhere between 200m and 70m years old. 

That timing fits well with a theory put forward in 2016, by Matija Cuk of the SETI Institute, in California and his colleagues. They suggest that at around the same time as the rings came into being an old set of moons orbiting Saturn destroyed themselves, and from their remains emerged not only the rings but also the planet’s current suite of inner moons—Rhea, Dione, Tethys, Enceladus and Mimas. . . . 

Dr Cuk and his colleagues used computer simulations of Saturn’s moons’ orbits as a sort of time machine. Looking at the rate at which tidal friction is causing these orbits to lengthen they extrapolated backwards to find out what those orbits would have looked like in the past. They discovered that about 100m years ago the orbits of two of them, Tethys and Dione, would have interacted in a way that left the planes in which they orbit markedly tilted. But their orbits are untitled. The obvious, if unsettling, conclusion was that this interaction never happened—and thus that at the time when it should have happened, Dione and Tethys were simply not there. They must have come into being later. . . . 

Q. 1 Based on information provided in the passage, we can infer that, in addition to water ice, Saturn’s rings might also have small amounts of: 

A. methane and rock particles. 

B. helium and methane. 

C. helium and comet dust. 

D. rock particles and comet dust. 

Answer: D. 

Explanation: 

In the fourth paragraph, it is mentioned that “they suggest that at around the same time as the rings came into being an old set of moons orbiting Saturn destroyed themselves, and from their remains emerged not only the rings……”. From this, we can infer that the rings were formed from the moons. Also, from the third paragraph, it can be inferred that Saturn’s rings consist of comet dust. 

Hence, option D. is the correct answer. 

Q. 2 Based on information provided in the passage, we can conclude all of the following EXCEPT: 

A. none of Saturn’s moons ever had suitable conditions for life to evolve. 

B. Thethys and Dione are less than 100 million years old. 

C. Saturn’s lighter rings discolour faster than rings with greater mass. 

D. Saturn’s rings were created from the remains of older moons. 

Answer: A. 

Explanation: 

In the last paragraph, it is given that about 100m years ago, Thethys and Dione were not there. From the last line of the passage we can conclude that Thethys and Dione are less than 100 million years old. Option B. can be concluded. In the third paragraph, it is mentioned “The lighter the rings are, the faster this will happen”. Option C. can be concluded. From the fourth paragraph, option D. can be concluded. 

Sufficient information has not been provided from which we can conclude that none of Saturn’s moons ever had suitable conditions for life to evolve. 

Hence, option A. is the correct answer. 

Q. 3 The phrase “leaving laundry hanging on a line downwind from a smokestack” is used to explain how the ringed planet’s: 

A. rings lose mass over time. 

B. rings discolour and darken over time. 

C. moons create a gap between the rings. 

D. atmosphere absorbs comet dust. 

Answer: B. 

Explanation: 

The phrase explains how clothes would darken over time if left hanging and facing smokestack. The phrase refers to the darkening of the Saturn’s rings under the influence of comet dust. 

Hence, option B. is the correct answer. 

Q. 4 Data provided by Cassini challenged the assumption that: 

A. new celestial bodies can form from the destruction of old celestial bodies. 

B. all big things in the solar system have been around since the beginning. 

C. there was life on earth when Saturn’s rings were being formed. 

D. Saturn’s ring system is composed mostly of water ice. 

Answer: B. 

Explanation: 

Referring to the first paragraph and first few lines of the second paragraph, it was believed that the celestial bodies had been existing from the beginning. However, the data provided by Cassini gave an insight that the rings and moons of Saturn are newly created. Thus, it challenged the earlier held notion. 

Hence, option B. is the correct answer. 

Q. 5 The main objective of the passage is to: 

A. highlight the beauty, finer structures and celestial drama of Saturn’s rings and moons. 

B. establish that Saturn’s rings and inner moons have been around since the beginning of time. 

C. provide evidence that Saturn’s rings and moons are recent creations. 

D. demonstrate how the orbital patterns of Saturn’s rings and moons change over time. 

Answer: C. 

Explanation: 

Refer to the lines from the passage – “The suggestion that rings and moons are new is,” “One reason for thinking Saturn’s rings are young is that they are bright.”, “Cassini’s measurements of the density of comet-dust near Saturn suggests the rings are no older than the first dinosaurs, nor younger than the last of them.” Throughout the passage, the author has emphasized on the fact that the rings and the moons of Saturn are recent phenomena. Option C. is the most relevant in this context. 

Option A. is not the primary objective of the passage otherwise the author would not have detailed the timeline of the formation of the moons and the rings of Saturn. 

Option B. is factually wrong as per the information given in the passage. 

Option D. is out of context. 

Hence, option C. is the correct answer. 

 

Instructions 

Read the passage carefully and answer the questions given 

More and more companies, government agencies, educational institutions and philanthropic organisations are today in the grip of a new phenomenon: ‘metric fixation’. The key components of metric fixation are the belief that it is possible – and desirable – to replace professional judgment (acquired through personal experience and talent) with numerical indicators of comparative performance based upon standardised data (metrics); and that the best way to motivate people within these organisations is by attaching rewards and penalties to their measured performance. 

The rewards can be monetary, in the form of pay for performance, say, or reputational, in the form of college rankings, hospital ratings, surgical report cards and so on. But the most dramatic negative effect of metric fixation is its propensity to incentivise gaming: that is, encouraging professionals to maximise the metrics in ways that are at odds with the larger purpose of the organisation. If the rate of major crimes in a district becomes the metric according to which police officers are promoted, then some officers will respond by simply not recording crimes or downgrading them from major offences to misdemeanours. Or take the case of surgeons. When the metrics of success and failure are made public – affecting their reputation and income – some surgeons will improve their metric scores by refusing to operate on patients with more complex problems, whose surgical outcomes are more likely to be negative. Who suffers? The patients who don’t get operated upon. 

When reward is tied to measured performance, metric fixation invites just this sort of gaming. But metric fixation also leads to a variety of more subtle unintended negative consequences. These include goal displacement, which comes in many varieties: when performance is judged by a few measures, and the stakes are high (keeping one’s job, getting a pay rise or raising the stock price at the time that stock options are vested), people focus on satisfying those measures – often at the expense of other, more important organisational goals that are not measured. The best-known example is ‘teaching to the test’, a widespread phenomenon that has distorted primary and secondary education in the United States since the adoption of the No Child Left Behind Act of 2001. 

Short-termism is another negative. Measured performance encourages what the US sociologist Robert K Merton in 1936 called ‘the imperious immediacy of interests … where the actor’s paramount concern with the foreseen immediate consequences excludes consideration of further or other consequences’. In short, advancing short-term goals at the expense of long-range considerations. This problem is endemic to publicly traded corporations that sacrifice long-term research and development, and the development of their staff, to the perceived imperatives of the quarterly report. 

To the debit side of the ledger must also be added the transactional costs of metrics: the expenditure of employee time by those tasked with compiling and processing the metrics in the first place – not to mention the time required to actually read them. . . . 

Q. 6 All of the following can be a possible feature of the No Child Left Behind Act of 2001, EXCEPT: 

A. school funding and sanctions are tied to yearly improvement shown on tests. 

B. standardised test scores can be critical in determining a student’s educational future. 

C. assessment is dependent on the teacher’s subjective evaluation of students’ class participation. 

D. the focus is more on test-taking skills than on higher order thinking and problem-solving. 

Answer: C. 

Explanation: 

The author has criticized the No Child Left Behind Act of 2001. So, it should be against what the author has supported in the passage. We know that the author has been critical of metric fixation. Therefore, the No Child Left Behind Act of 2001 must have the features of metric fixation. 

Option C. cannot be a feature of the No Child Left Behind Act of 2001 as it mentions the subjective evaluation of students based on their participation in the class which is against the theory of metric fixation. 

Hence, option C. is the correct answer. 

Q. 7 What main point does the author want to convey through the examples of the police officer and the surgeon? 

A. Some professionals are likely to be significantly influenced by the design of performance measurement systems. 

B. Metrics-linked rewards may encourage unethical behaviour among some professionals. 

C. Critical public roles should not be evaluated on metrics-based performance measures. 

D. The actions of police officers and surgeons have a significantly impact on society. 

Answer: B. 

Explanation: 

In the second paragraph, the author discusses that one of the major drawbacks of metric fixation is the rise in unethical behaviour in order to maximize the metrics. The author, further, goes on to give the examples of the police officer and the surgeon to substantiate his claims. Therefore, option B. is the correct answer. 

Option A. does not mention that the influence would be unethical and harmful in nature. 

Option C. is the underlying message of the author but, he does not explicitly provide the examples of the police officer and the surgeon to prove this. 

Option D. is too broad and has no specifics about the unethical behaviour which could be encouraged by metric fixation. 

Q. 8 Which of the following is NOT a consequence of the ‘metric fixation’ phenomenon mentioned in the passage? 

A. Finding a way to show better results without actually improving performance. 

B. Improving cooperation among employees leading to increased organisational effectiveness in the long run. 

C. Deviating from organisationally important objectives to measurable yet less important objectives. 

D. Short-term orientation induced by frequent measurement of performance. 

Answer: B. 

Explanation: 

From the second paragraph, we can say that metric fixation encourages professionals to maximize the metrics in ways that are at odds with the larger purpose of the organization. Option A. is a consequence of metric fixation. From the third paragraph, we can infer that metric fixation leads to goal displacement. 

The author has stated short-termism as a consequence of metric fixation in the penultimate paragraph. Option B. as a consequence of metric fixation has not been discussed in the paragraph. 

Hence, option B. is the correct answer. 

Q. 9 Of the following, which would have added the least depth to the author’s argument? 

A. Assessment of the pros and cons of a professional judgment-based evaluation system. 

B. An analysis of the reasons why metrics fixation is becoming popular despite its drawbacks. 

C. A comparative case study of metrics- and non-metrics-based evaluation, and its impact on the main goals of an organisation. 

D. More real-life illustrations of the consequences of employees and professionals gaming metrics-based performance measurement systems. 

Answer: D. 

Explanation: 

In the passage, the author has discussed the ill-effects of metric fixation. He has discussed gaming of the metrics-based performance system in detail. By providing more real-life illustrations of the same, the author would not have added any value to the main argument. 

Options A, B. and C. are relevant to the discussion and will surely add weight to the main idea of the passage. Hence, option D. is the correct answer. 

Q. 10 What is the main idea that the author is trying to highlight in the passage? 

A. Performance measurement needs to be precise and cost-effective to be useful for evaluating organisational performance. 

B. Evaluating performance by using measurable performance metrics may misguide organisational goal achievement. 

C. Long-term organisational goals should not be ignored for short-term measures of organisational success. 

D. All kinds of organisations are now relying on metrics to measure performance and to give rewards and punishments. 

Answer: B. 

Explanation: 

The author has criticized the method of metric fixation in the passage. He has stated that metric fixation will lead professionals to adhere to practices that are at odds with the larger purpose of the organization. He has also explained that metric fixation will lead to goal displacement. In this light, option B. is the most relevant. Option A. is incorrect because it is against the author’s view. 

Option C. is narrow as it focuses on short-termism only which is one of the ill-effects of metric fixation as mentioned in the passage. 

Option D. does not state that the author is criticizing the metric fixation method to measure the performance. Hence, option B. is the correct answer. 

 

Instructions 

Read the passage carefully and answer the questions given 

Will a day come when India’s poor can access government services as easily as drawing cash from an ATM? . . . [N]o country in the world has made accessing education or health or policing or dispute resolution as easy as an ATM, because the nature of these activities requires individuals to use their discretion in a positive way. Technology can certainly facilitate this in a variety of ways if it is seen as one part of an overall approach, but the evidence so far in education, for instance, is that just adding computers alone doesn’t make education any better. . . . 

The dangerous illusion of technology is that it can create stronger, top down accountability of service providers in implementation-intensive services within existing public sector organisations. One notion is that electronic management information systems (EMIS) keep better track of inputs and those aspects of personnel that are ‘EMIS visible’ can lead to better services. A. recent study examined attempts to increase attendance of Auxiliary Nurse Midwife (ANMs) at clinics in Rajasthan, which involved high-tech time clocks to monitor attendance. The study’s title says it all: Band-Aids on a 

Corpse . . . e-governance can be just as bad as any other governance when the real issue is people and their motivation. For services to improve, the people providing the services have to want to do a better job with the skills they have. A. study of medical care in Delhi found that even though providers, in the public sector had much better skills than private sector providers their provision of care in actual practice was much worse. 

In implementation-intensive services the key to success is face-to-face interactions between a teacher, a nurse, a policeman, an extension agent and a citizen. This relationship is about power. Amartya Sen’s . . . report on education in West Bengal had a supremely telling anecdote in which the villagers forced the teacher to attend school, but then, when the parents went off to work, the teacher did not teach, but forced the children to massage his feet. . . . As long as the system empowers providers over citizens, technology is irrelevant. 

The answer to successfully providing basic services is to create systems that provide both autonomy and accountability. In basic education for instance, the answer to poor teaching is not controlling teachers more . . . The key . . . is to hire teachers who want to teach and let them teach, expressing their professionalism and vocation as a teacher through autonomy in the classroom. This autonomy has to be matched with accountability for results—not just narrowly measured through test scores, but broadly for the quality of the education they provide. 

A. recent study in Uttar Pradesh showed that if, somehow, all civil service teachers could be replaced with contract teachers, the state could save a billion dollars a year in revenue and double student learning. Just the additional autonomy and accountability of contracts through local groups—even without complementary system changes in information and empowerment—led to that much improvement. The first step to being part of the solution is to create performance information accessible to those outside of the government. . . . 

Q. 11 According to the author, service delivery in Indian education can be improved in all of the following ways EXCEPT through: 

A. access to information on the quality of teaching. 

B. elimination of government involvement. 

C. recruitment of motivated teachers. 

D. use of technology. 

Answer: B. 

Explanation: 

In the last line of the passage, the author mentions about the availability of information which should be the first step towards solving the service delivery in the Indian education system. 

In the penultimate paragraph, the author says that the key is to hire those teachers who want to teach. In other words, the author supports the recruitment of motivated teachers. 

In the first paragraph, the author states that technology can facilitate better service delivery in Indian education. The author has nowhere talked about the elimination of government involvement. He wants that the autonomy and accountability of the teachers should be increased. 

Hence, option B. is the correct answer. 

Q. 12 In the context of the passage, we can infer that the title “Band Aids on a Corpse” (in paragraph 2) suggests that: 

A. the nurses attended the clinics, but the clinics were ill-equipped. 

B. the clinics were better funded, but performance monitoring did not result in any improvement. 

C. the nurses who attended the clinics were too poorly trained to provide appropriate medical care. 

D. the electronic monitoring system was a superficial solution to a serious problem. 

Answer: D. 

Explanation: 

The author has explained the phrase “Band Aids on a Corpse” by stating that ” e-governance can be just as bad as any other governance when the real issue is people and their motivation.” From this, we can infer that the solution was not 

intended to tackle the real cause of the problem which was the motivation of the people. If people are not motivated, forcing them to come on time will act only as a specious way to deal with the issue. 

Hence, option D. is the correct answer. 

Q. 13 The author questions the use of monitoring systems in services that involve face-to-face interaction between service providers and clients because such systems: 

A. do not improve services that need committed service providers. 

B. are ineffective because they are managed by the government. 

C. improve the skills but do not increase the motivation of service providers. 

D. are not as effective in the public sector as they are in the private sector. 

Answer: A. 

Explanation: 

In the third paragraph, the author has given the example of a school where the villagers forced the teachers to come to school, but the teacher instead of teaching indulged in various other non-productive activities. Further, the author also mentions that as long as the system empowers providers over citizens, technology is irrelevant. So, the author wants to convey that commitment and motivation are the primary requirements in systems which involve face-to-face interaction between service providers and clients. Therefore, using technology to monitor in such scenarios will be ineffective. 

Hence, option A. is the correct answer. 

Q. 14 The main purpose of the passage is to: 

A. argue that some types of services can be improved by providing independence and requiring accountability. 

B. analyse the shortcomings of government-appointed nurses and their management through technology. 

C. critique the government’s involvement in educational activities and other implementation-intensive services. 

D. find a solution to the problem of poor service delivery in education by examining different strategies. 

Answer: A. 

Explanation: 

The author has explained in the passage that without increasing the autonomy and accountability of the person involved in a job, monitoring systems will be ineffective in improving the services. So, the author has advocated for making the persons more responsible and give them more independence. Option A. is the most relevant in this context. Option B. is narrow in the sense that the passage does not only focus on the case of nurses. Option C. is irrelevant as the author does not criticize the involvement of government. 

Option D. is incorrect because the author is not trying to find a solution, but he has proposed a solution to deal with the problem. 

Hence, option A. is the correct answer. 

Q. 15 Which of the following, IF TRUE, would undermine the passage’s main argument? 

A. If it were proven that increase in autonomy of service providers leads to an exponential increase in their work ethic and sense of responsibility. 

B. If it were proven that service providers in the private sector have better skills than those in the public sector. 

C. Empowerment of service providers leads to increased complacency and rigged performance results. 

D. If absolute instead of moderate technological surveillance is exercised over the performance of service providers. 

Answer: C. 

Explanation: 

The author has argued in the passage and proposed some ways to increase productivity and to make the systems more effective. Option C. which states “Empowerment of service providers leads to increased complacency and rigged performance results.” will undermine the author’s main argument because if empowerment of the service providers leads to rigged performance results, the whole purpose will be defeated. 

Option A. supports the passage’s main idea. 

Option B. is irrelevant. 

Option D. does not talk about the effect of implementing absolute surveillance on the performance of service providers. Hence, option C. is the correct answer. 

 

Instructions 

Read the passage carefully and answer the questions given 

Grove snails as a whole are distributed all over Europe, but a specific variety of the snail, with a distinctive white-lipped shell, is found exclusively in Ireland and in the Pyrenees mountains that lie on the border between France and Spain. The researchers sampled a total of 423 snail specimens from 36 sites distributed across Europe, with an emphasis on gathering large numbers of the white-lipped variety. When they sequenced genes from the mitochondrial DNA. of each of these snails and used algorithms to analyze the genetic diversity between them, they found that. . . a distinct lineage (the snails with the white-lipped shells) was indeed endemic to the two very specific and distant places in question. 

Explaining this is tricky. Previously, some had speculated that the strange distributions of creatures such as the white lipped grove snails could be explained by convergent evolution—in which two populations evolve the same trait by coincidence—but the underlying genetic similarities between the two groups rules that out. Alternately, some scientists had suggested that the white-lipped variety had simply spread over the whole continent, then been wiped out everywhere besides Ireland and the Pyrenees, but the researchers say their sampling and subsequent DNA. analysis eliminate that possibility too. “If the snails naturally colonized Ireland, you would expect to find some of the same genetic type in other areas of Europe, especially Britain. We just don’t find them,” Davidson, the lead author, said in a press statement. 

Moreover, if they’d gradually spread across the continent, there would be some genetic variation within the white-lipped type, because evolution would introduce variety over the thousands of years it would have taken them to spread from the Pyrenees to Ireland. That variation doesn’t exist, at least in the genes sampled. This means that rather than the organism gradually expanding its range, large populations instead were somehow moved en mass to the other location within the space of a few dozen generations, ensuring a lack of genetic variety. 

“There is a very clear pattern, which is difficult to explain except by involving humans,” Davidson said. Humans, after all, colonized Ireland roughly 9,000 years ago, and the oldest fossil evidence of grove snails in Ireland dates to roughly the same era. Additionally, there is archaeological evidence of early sea trade between the ancient peoples of Spain and Ireland via the Atlantic and even evidence that humans routinely ate these types of snails before the advent of agriculture, as their burnt shells have been found in Stone Age trash heaps. 

The simplest explanation, then? Boats. These snails may have inadvertently traveled on the floor of the small, coast hugging skiffs these early humans used for travel, or they may have been intentionally carried to Ireland by the seafarers as a food source. “The highways of the past were rivers and the ocean-as the river that flanks the Pyrenees was an ancient trade route to the Atlantic, what we’re actually seeing might be the long lasting legacy of snails that hitched a ride…as humans travelled from the South of France to Ireland 8,000 years ago,” Davidson said. 

Q. 16 The passage outlines several hypotheses and evidence related to white-lipped grove snails to arrive at the most convincing explanation for: 

A. why the white-lipped variety of grove snails are found only in Ireland and the Pyrenees. 

B. why the white-lipped variety of grove snails were wiped out everywhere except in Ireland and the Pyrenees. 

C. how the white-lipped variety of grove snails might have migrated from the Pyrenees to Ireland. 

D. how the white-lipped variety of grove snails independently evolved in Ireland and the Pyrenees. 

Answer: A. 

Explanation: 

Throughout the passage, the author has contemplated the reasons why the white-lipped variety of grove snails are found only in Ireland and the Pyrenees. This is also evident from the last line of the first paragraph, the first line of the second paragraph and the first line of the fourth paragraph. 

The author has not discussed the reasons why the snails were wiped out from the other parts of the world. Option B. is incorrect. 

The author has focused neither on migration nor on the evolution of the snails. Option C. and D. are irrelevant. Hence, option A. is the correct answer. 

Q. 17 In paragraph 4, the evidence that “humans routinely ate these types of snails before the advent of agriculture” can be used to conclude that: 

A. white-lipped grove snails may have inadvertently traveled from the Pyrenees to Ireland on the floor of the small, coast-hugging skiffs that early seafarers used for travel. 

B. 9,000 years ago, during the Stone Age, humans traveled from the South of France to Ireland via the Atlantic Ocean. 

C. rivers and oceans in the Stone Age facilitated trade in white-lipped grove snails. 

D. the seafarers who traveled from the Pyrenees to Ireland might have carried white-lipped grove snails with them as edibles. 

Answer: D. 

Explanation: 

In the fourth paragraph, the author states that the appearance of grove snails and the arrival of humans in Ireland coincided. Further, the author proves his point by mentioning about the evidence that humans routinely ate these types of snails before the advent of agriculture. From this, we can infer that people who came to colonize Ireland must have brought snails with them as edibles. Option D. is the most relevant in this context. 

Options B. and C. are out of context. 

Option A. might be factually true, but it cannot be concluded from the given sentence. 

Hence, option D. is the correct answer. 

Q. 18 Which one of the following makes the author eliminate convergent evolution as a probable explanation for why white-lipped grove snails are found in Ireland and the Pyrenees? 

A. The coincidental evolution of similar traits (white-lipped shell) in the grove snails of Ireland and the Pyrenees. 

B. The absence of genetic variation between white-lipped grove snails of Ireland and the Pyrenees. 

C.The absence of genetic similarities between white-lipped grove snails of Ireland and snails from other parts of Europe, especially Britain. 

D. The distinct lineage of white-lipped grove snails found specifically in Ireland and the Pyrenees. 

Answer: B. 

Explanation: 

In the second paragraph, the author mentions convergent evolution in which two populations evolve the same trait by coincidence. In that case, if the traits are similar by mere coincidence, the genetic structure must be different as they are part of two distinct populations. However, in the study, it was found that the two groups of snails have genetic similarities and thus, it cannot be a case of convergent evolution. Option B. states the same. Hence, option B. is the correct answer. 

Q. 19 All of the following evidence supports the passage’s explanation of sea travel/trade EXCEPT: 

A. the oldest fossil evidence of white-lipped grove snails in Ireland dates back to roughly 9,000 years ago, the time when humans colonised Ireland. 

B. archaeological evidence of early sea trade between the ancient peoples of Spain and Ireland via the Atlantic Ocean. 

C. absence of genetic variation within the white-lipped grove snails of Ireland and the Pyrenees, whose genes were sampled. 

D. the coincidental existence of similar traits in the white-lipped grove snails of Ireland and the Pyrenees because of convergent evolution. 

Answer: D. 

Explanation: 

In the second paragraph, the author mentions convergent evolution in which two populations evolve the same trait by coincidence. In that case, if the traits are similar by mere coincidence, the genetic structure must be different as they are part of two distinct populations. However, in the study, it was found that the two groups of snails have genetic similarities and thus, it cannot be a case of convergent evolution. Thus, the author refutes the claim that convergent evolution can explain the similarity in characteristics. Therefore, option C. supports the passage’s explanation of sea travel/trade while option D. rejects. 

Hence, option D. is the correct answer. 

 

Instructions 

Read the passage carefully and answer the given questions 

The complexity of modern problems often precludes any one person from fully understanding them. Factors contributing to rising obesity levels, for example, include transportation systems and infrastructure, media, convenience foods, changing social norms, human biology and psychological factors. . . . The multidimensional or layered character of complex problems also undermines the principle of meritocracy: the idea that the ‘best person’ should be hired. There is no best person. When putting together an oncological research team, a biotech company such as Gilead or Genentech would not construct a multiple-choice test and hire the top scorers, or hire people whose resumes score highest according to some performance criteria. Instead, they would seek diversity. They would build a team of people who bring diverse knowledge bases, tools and analytic skills. . . . 

Believers in a meritocracy might grant that teams ought to be diverse but then argue that meritocratic principles should apply within each category. Thus the team should consist of the ‘best’ mathematicians, the ‘best’ oncologists, and the ‘best’ biostatisticians from within the pool. That position suffers from a similar flaw. Even with a knowledge domain, no test or criteria applied to individuals will produce the best team. Each of these domains possesses such depth and breadth, that no test can exist. Consider the field of neuroscience. Upwards of 50,000 papers were published last year covering various techniques, domains of enquiry and levels of analysis, ranging from molecules and synapses up through networks of neurons. Given that complexity, any attempt to rank a collection of neuroscientists from best to worst, as if they were competitors in the 50-metre butterfly, must fail. What could be true is that given a specific task and the composition of a particular team, one scientist would be more likely to contribute than another. Optimal hiring depends on context. Optimal teams will be diverse. 

Evidence for this claim can be seen in the way that papers and patents that combine diverse ideas tend to rank as high impact. It can also be found in the structure of the so-called random decision forest, a state-of-the-art machine-learning algorithm. Random forests consist of ensembles of decision trees. If classifying pictures, each tree makes a vote: is that a picture of a fox or a dog? A. weighted majority rules. Random forests can serve many ends. They can identify bank 

fraud and diseases, recommend ceiling fans and predict online dating behaviour. When building a forest, you do not select the best trees as they tend to make similar classifications. You want diversity. Programmers achieve that diversity by training each tree on different data, a technique known as bagging. They also boost the forest ‘cognitively’ by training trees on the hardest cases – those that the current forest gets wrong. This ensures even more diversity and accurate forests. 

Yet the fallacy of meritocracy persists. Corporations, non-profits, governments, universities and even preschools test, score and hire the ‘best’. This all but guarantees not creating the best team. Ranking people by common criteria produces homogeneity. . . . That’s not likely to lead to breakthroughs. 

Q. 20 Which of the following conditions, if true, would invalidate the passage’s main argument? 

A. If it were proven that teams characterised by diversity end up being conflicted about problems and take a long time to arrive at a solution. 

B. If a new machine-learning algorithm were developed that proved to be more effective than the random decision forest. 

C. If top-scorers possessed multidisciplinary knowledge that enabled them to look at a problem from several perspectives. 

D. If assessment tests were made more extensive and rigorous. 

Answer: C. 

Explanation: 

Throughout the passage, the author has argued that each field of study has become so vast that diversity in knowledge and skills is required to sail through. Meritocracy is not enough to bring the required variety. This is the main idea presented by the author. 

Option A. is distorted because the author is not concerned about the negative consequences of his proposition and how to deal with them. 

Option B. is too narrow in its scope as it focuses on random decision trees which is not the main idea. Option C. addresses the primary concerns which the author has and thus, weakens the main idea of the passage. Option D. is irrelevant as the author has no problem with the assessment tests. 

Hence, option C. is the correct answer. 

Q. 21 Which of the following best describes the purpose of the example of neuroscience? 

A. In narrow fields of knowledge, a meaningful assessment of expertise has always been possible. 

B. Unlike other fields of knowledge, neuroscience is an exceptionally complex field, making a meaningful assessment of neuroscientists impossible. 

C. In the modern age, every field of knowledge is so vast that a meaningful assessment of merit is impossible. 

D. Neuroscience is an advanced field of science because of its connections with other branches of science like oncology and biostatistics. 

Answer: C. 

Explanation: 

Just before giving the example of neuroscience, the author has mentioned that each of these domains possesses such depth and breadth, that no test can exist. From this we can infer that the purpose behind mentioning neuroscience as an example by the author is to show that each field is so complex now that a meaningful assessment of merit is impossible. Option C. is the most relevant in this case. 

Hence, option C. is the correct answer. 

Q. 22 The author critiques meritocracy for all the following reasons EXCEPT that: 

A. an ideal team comprises of best individuals from diverse fields of knowledge. 

B. diversity and context-specificity are important for making major advances in any field. 

C. modern problems are multifaceted and require varied skill-sets to be solved. 

D. criteria designed to assess merit are insufficient to test expertise in any field of knowledge. 

Answer: A. 

Explanation: 

Option B. is the main idea that the author wants to express through the passage. So, it is one of the main reasons why the author criticizes meritocracy. 

Option C. is also one of the reasons as conveyed by the author through the example of neuroscientists in the second paragraph. 

The author mentions in the second paragraph “each of these domains possesses such depth and breadth, that no test can exist.” From this, we can infer option D. to be a valid reason. 

Option A. is not a reason why the author criticizes meritocracy. 

Hence, option A. is the correct answer. 

Q. 23 Which of the following conditions would weaken the efficacy of a random decision forest? 

A. If the types of decision trees in each ensemble of the forest were doubled. 

B. If a large number of decision trees in the ensemble were trained on data derived from easy cases. 

C. If the types of ensembles of decision trees in the forest were doubled. 

D. If a large number of decision trees in the ensemble were trained on data derived from easy and hard cases. 

Answer: B. 

Explanation: 

In the last two lines of the third paragraph, it has been given that forest is cognitively boosted by training the trees on the hardest cases. So, if a large number of decision trees in the ensemble were trained on data derived from easy cases, the forest will not get a cognitive boost and thus weaken the efficacy of a random decision forest. Hence, option B. is the correct answer. 

Q. 24 On the basis of the passage, which of the following teams is likely to be most effective in solving the problem of rising obesity levels? 

A. A specialised team of nutritionists from various countries, who are also trained in the machine-learning algorithm of random decision forest. 

B. A team comprised of nutritionists, psychologists, urban planners and media personnel, who have each scored a distinction in their respective subject tests. 

C. A specialised team of top nutritionists from various countries, who also possess some knowledge of psychology. 

D. A team comprised of nutritionists, psychologists, urban planners and media personnel, who have each performed well in their respective subject tests. 

Answer: D. 

Explanation: 

According to the author’s main idea, the problem should be tackled by a diverse group of members from different fields. On the basis of this, we can eliminate options A. and C. because, in these options, the expert team consists of only nutritionists. 

Out of options B. and D, option D. is better because it mentions a team of members who have performed well in their respective field. In option B, the members are selected on the parameter of meritocracy, which is not concurrent with the author’s viewpoint. 

Hence, option D. is the correct answer. 

 

Instructions 

For the following questions answer them individually 

Q. 25 The four sentences (labelled 1,2,3,4) given in this question, when properly sequenced, form a coherent paragraph. Each sentence is labelled with a number. Decide on the proper sequence of order of the sentences and key in this sequence of four numbers as your answer: 

1. In the era of smart world, however, ‘Universal Basic Income’ is an ineffective instrument which cannot address the potential breakdown of the social contract when large swathes of the population would effectively be unemployed. 

2. In the era of industrial revolution, the abolition of child labour, poor laws and the growth of trade unions helped families cope with the pressures of mechanised work. 

3. Growing inequality could be matched by a creeping authoritarianism that is bolstered by technology that is increasingly able to peer into the deepest vestiges of our lives. 

4. New institutions emerge which recognise ways in which workers could contribute to and benefit by economic growth when, rather than if, their jobs are automated. 

Answer:4213 

Explanation: 

4 should be the opening sentence since it states that new institutions recognize ways in which workers can contribute to the economy. The other 3 sentences provide examples and elaborate on the same and hence, sentence 4, which introduces the topic of discussion, should be the opening sentence. 

2 talks about the period of industrial revolution. 1 talks about the smart world. Chronologically, 1 should follow 2. Moreover, 2 talks about an example that conformed to the point mentioned in sentence 4. On the other hand, 1 talks about an inadequacy. Therefore, 2 should have preceded 1. 3 should be the last sentence of the paragraph. 

Sentences 4213 form a coherent paragraph and hence, 4213 is the correct answer. 

Q. 26 The passage given below is followed by four summaries. Choose the option that best captures the author’s position. 

The early optimism about sport’s deterrent effects on delinquency was premature as researchers failed to find any consistent relationships between sports participation and deviance. As the initial studies were based upon cross-sectional data and the effects captured were short-term, it was problematic to test and verify the temporal sequencing of events suggested by the deterrence theory. The correlation between sport and delinquency could not be disentangled from class and cultural variables known. Choosing individuals to play sports in the first place was problematic, which became more acute in the subsequent decades as researchers began to document just how closely sports participation was linked to social class indicators. 

A. There is a direct relationship between sport participation and delinquency but it needs more empirical evidence. 

B. Statistical and empirical weaknesses stand in the way of inferring any relationship between sports participation and deviance. 

C. Sports participation is linked to class and cultural variables such as education, income, and social capital. 

D. Contradicting the previous optimism, latter researchers have proved that there is no consistent relationship between sports participation and deviance. 

Answer: B. 

Explanation: 

The main points of the passage are that the relation between sports participation and deviation have not been established and that there is not sufficient evidence to prove the relationship. 

Option A. is distorted because it states that the relationship is already established. 

Option B. mentions all the relevant points. 

Option C. does not talk about the relationship and hence, ruled out. 

Option D. distorts what is given in the paragraph by saying that latter researchers have “proved” there is no consistent relationship. Thus, it is can be eliminated. 

Hence, option B. is the correct answer. 

Q. 27 The four sentences (labelled 1, 2, 3, and 4) given in this question, when properly sequenced, form a coherent paragraph. Decide on the proper order for the sentences and key in this sequence of four numbers as your answer. 

1. Self-management is thus defined as the ‘individual’s ability to manage the symptoms, treatment, physical and psychosocial consequences and lifestyle changes inherent in living with a chronic condition’. 

2. Most people with progressive diseases like dementia prefer to have control over their own lives and health-care for as long as possible. 

3. Having control means, among other things, that patients themselves perform self-management activities. 

4. Supporting people in decisions and actions that promote self-management is called self-management support requiring a cooperative relationship between the patient, the family, and the professionals. 

Answer:2314 

Explanation: 

1 states that ‘self management is ‘thus’ defined as the individual’s ability to manage…’. Therefore, some details about self management should have been provided before sentence 1. 

Sentence 2 states that people with dementia prefer to have ‘control’ in their lives. Sentence 3 states what ‘having control means’. Therefore, sentences 2 and 3 form a mandatory pair. 

Sentence 1 should follow sentence 3 since sentence 3 states that ‘having control means to perform self management activities’ and sentence 1 defined self management activities. Sentence 4 should be the last sentence since it states what self management support is. Self management support can be defined only after defining what self management is. 

Sentences 2314 form a coherent paragraph. Therefore, 2314 is the correct answer. 

Q. 28 The passage given below is followed by four summaries. Choose the option that best captures the author’s position: 

A. Japanese government panel announced that it recommends regulating only genetically modified organisms that have had foreign genes permanently introduced into their genomes and not those whose endogenous genes have been edited. The only stipulation is that researchers and businesses will have to register their modifications to plants or animals with the government, with the exception of microbes cultured in contained environments. Reactions to the decision are mixed. While lauding the potential benefits of genome editing, an editorial opposes across-the-board permission. Unforeseen risks in gene editing cannot be ruled out. All genetically modified products must go through the same safety and labeling processes regardless of method. 

A. Excepting microbes cultured in contained environments from the regulations of genome editing is premature. 

B. Creating categories within genetically modified products in terms of transgenic modification and genome editing advances science but defies laws. 

C. Exempting from regulations the editing of endogenous genes is not desirable as this procedure might be risk-prone. 

D. A government panel in Japan says transgenic modification and genome editing are not the same. 

Answer: C. 

Explanation: 

Let us note down the important points: 

The Japanese government recommends regulating GM organisms in which foreign genes are introduced, not those in which the endogenous genes have been edited. The step has drawn mixed reactions since there are some risks involved in gene editing. 

Option A. shifts the focus on exempting microbes. The central theme of the passage is that endogenous gene editing is not totally risk free. Therefore, we can eliminate option A. 

Option B. states that categorizing GM products advances science but defies laws. No such comparison has been made in the paragraph. The legality of the issue has not been discussed and hence, we can eliminate option B. 

Option D. fails to capture the fact that endogenous gene editing is not devoid of risks. 

Only option C. captures the fact that exempting endogenous gene editing is not desirable due to the risks involved. Therefore, option C. is the right answer. 

Q. 29 Five sentences related to a topic are given below. Four of them can be put together to form a meaningful and coherent short paragraph. Identify the odd one out. 

1. Much has been recently discovered about the development of songs in birds. 

2. Some species are restricted to a single song learned by all individuals, others have a range of songs. 3. The most important auditory stimuli for the birds are the sounds of other birds. 4. For all bird species there is a prescribed path to development of the final song, 5. A. bird begins with the subsong, passes through plastic song, until it achieves the species song. 

Answer:

Explanation: 

On reading the sentences, we can infer that the paragraph talks about the development of songs in birds. 

1 should be the opening sentence since it introduces the topic of discussion, the development of songs in birds. 1 provides a proper introduction to the paragraph by stating that much has been discovered about the development of songs in birds. 

4 should be the sentence that follows 1 since it states that the development of songs in birds follow a template process. 5 explains the mechanism in which the song is developed. Therefore, sentence 5 should follow sentence 4. 2 should be the last sentence since it states that some species restrict themselves to one song while other species have more than one song. 

Sentences 1452 form a coherent paragraph. Sentence 3 talks about the sounds of birds while the other sentences are about how a species develops a song. Therefore, sentence 3 is the one out of context and hence, 3 is the right answer. 

Q. 30 The four sentences (labelled 1, 2, 3, and 4) given in this question, when properly sequenced, form a coherent paragraph. Decide on the proper order for the sentences and key in this sequence of four numbers as your answer. 

1. It was his taxpayers who had to shell out as much as $1.6bn over 10 years to employees of failed companies. 

2. Companies in many countries routinely engage in such activities which means that the employees are left with unpaid entitlements 

3. Deliberate and systematic liquidation of a company to avoid liabilities and then restarting the business is called phoenixing. 

4. The Australian Minister for Revenue and Services discovered in an audit that phoenixing had cost the Australian economy between $2.9bn and $5.1bn last year. 

Answer:3241 

Explanation: 

3 should be the opening sentence since it introduces the concept of phoenixing. Sentence 2 logically continues sentence 1 by stating that companies in many countries engage in such activities (Phoenixing). Sentence 2 should be followed by sentence 4 since it moves to a specific instance (Australian minister’s discovery) from general statements. Sentence 1 should follow sentence 4 since it states that it was ‘his’ taxpayers (Australian Minister’s taxpayers or Australian citizens) who had to foot the bill. 

Sentences 3241 form a coherent paragraph. Therefore, 3241 is the correct answer. 

Q. 31 Five sentences related to a topic are given below. Four of them can be put together to form a meaningful and coherent short paragraph. Identify the odd one out. Choose its number as your answer and key the number in: 

1. Our smartphones can now track our diets, our biological cycles, even our digestive systems and sleep patterns. 

2. Researchers have even coined a new term, “orthosomnia”, to describe the insomnia brought on by paying too much attention to smartphones and sleep-tracking apps. 

3. Sleep, nature’s soft nurse, is a blissful, untroubled state all too easily disturbed by earthly worries or a guilty conscience. 

4. The existence of a market for such apps is unsurprising: shift work, a long-hours culture and blue light from screens have conspired to rob many of us of sufficient rest. 

5. A. new threat to a good night’s rest has emerged – smart-phones, with sleep-tracking apps. 

Answer:

Explanation: 

The use of the word “such apps” in 4 indicates that 4 must be preceded by a line that mentions a type of app. We find this in 5 and 2. So the pairs could be 5-4 or 2-4. If we see the sentences 5 and 1 they introduce the topic under discussion and provide context for the details provided in 2 and 4. Hence, 1 and 5 should come at the start of the paragraph and 2-4 should be the end of the paragraph. Between 5 and 1, 5 provides a better opening line as it introduces the main topic of discussion. Hence, the order of the paragraph should be 5-1-2-4. 

Sentence 3 which talks of “guilty conscience” is out of context with the rest of the paragraph. 

Q. 32 Five sentences related to a topic are given below. Four of them can be put together to form a meaningful and coherent short paragraph. Identify the odd one out. 

1. As India looks to increase the number of cities, our urban planning must factor in potential natural disasters and work out contingencies in advance. 

2. Authorities must revise data and upgrade infrastructure and mitigation plans even if their local area hasn’t been visited by a natural calamity yet. 

3. Extreme temperatures, droughts, and forest fires have more than doubled since 1980. 4. There is no denying the fact that our baseline normal weather is changing. 

5. It is no longer a question of whether we will be hit by nature’s fury but rather when. 

Answer:

Explanation: 

If we read all the sentences together, we see that the author is arguing for India preparing itself in advance for future natural disasters. Sentence 4, which introduces the broader context makes for a good opening line. Sentences 1 and 2 together make the main point that the author is trying make through the paragraph – that India should prepare itself for future natural disasters. Sentence 5 emphasizes the main point by adding that natural disasters will occur in the future and thus makes for a good concluding line. 

Sentence 3, that talks about extreme temperatures does not lead off to any of the other sentences nor does it add to any of the other sentences. Hence, it is the odd one out. 

Q. 33 The four sentences (labelled 1,2,3,4) given in this question, when properly sequenced, form a coherent paragraph. Each sentence is labelled with a number. Decide on the proper sequence of order of the sentences and key in this sequence of four numbers as your answer: 

1. They would rather do virtuous side projects assiduously as long as these would not compel them into doing their day jobs more honourably or reduce the profit margins. 

2. They would fund a million of the buzzwordy programs rather than fundamentally question the rules of their game or alter their own behavior to reduce the harm of the existing distorted, inefficient and unfair rules. 

3. Like the dieter who would rather do anything to lose weight than actually eat less, the business elite would save the world through social-impact-investing and philanthro-capitalism. 

4. Doing the right thing — and moving away from their win-win mentality — would involve real sacrifice; instead, it’s easier to focus on their pet projects and initiatives. 

Answer:3241 

Explanation: 

After reading all the sentences, we know that the paragraph is about the businessmen who, instead of tackling the root causes, focus on superficial solutions. Statement 3 is the opening sentence as it introduces the topic by comparing businessmen with a dieter who is ready to do everything except eating less. Statement 2 mentions the examples of some of the specious solutions mentioned in statement 3. Statement 4 provides the reason why businessmen are hesitant to execute the genuine solutions which will bring real change. Statement 4 mentions the alternative taken by businessmen. The word ‘rather’ in statement 1 connects it with statement 4. Thus, the correct order is 3 – 2 – 4 – 1. Hence, 3241 is the correct answer. 

Q. 34 The passage given below is followed by four summaries. Choose the option that best captures the author’s position. 

Should the moral obligation to rescue and aid persons in grave peril, felt by a few, be enforced by the criminal law? Should we follow the lead of a number of European countries and enact bad Samaritan laws? Proponents of bad Samaritan laws must overcome at least three different sorts of obstacles. First, they must show the laws are morally legitimate in principle, that is, that the duty to aid others is a proper candidate for legal enforcement. Second, they must show that this duty to aid can be defined in a way that can be fairly enforced by the courts. Third, they must show that the benefits of the laws are worth their problems, risks and costs. 

A. A. number of European countries that have successfully enacted bad Samaritan laws may serve as model statutes. 

B. Everyone agrees that people ought to aid others, the only debate is whether to have a law on it. 

C. If bad Samaritan laws are found to be legally sound and enforceable they must be enacted. 

D. Bad Samaritan laws may be desirable but they need to be tested for legal soundness. 

Answer: D. 

Explanation: 

In the given paragraph, the author has discussed about Bad Samaritan laws and whether it is enforceable by law. While answering the question, the author puts forward three points which she deems necessary for the implementation of Bad Samaritan law. Only after crossing the three obstacles mentioned by the author, the law should be enacted. Option D. is the most relevant in this context. 

Option A. is about implementing the law without any conditions, which is not what the author wants to convey. Option B. does not mention anything about the three obstacles. 

Option C. is stated with a firmness which is not the tone of the author. The author says that the law may be enacted, not must be enacted. 

Hence, option D. is the correct answer. 

LRDI 

Instructions 

The base exchange rate of a currency X with respect to a currency Y is the number of units of currency Y which is equivalent in value to one unit of currency X. Currency exchange outlets buy currency at buying exchange rates that are lower than base exchange rates, and sell currency at selling exchange rates that are higher than base exchange rates. 

A. currency exchange outlet uses the local currency L to buy and sell three international currencies A, B, and C, but does not exchange one international currency directly with another. The base exchange rates of A, B. and C. with respect to L are in the ratio 100:120:1. The buying exchange rates of each of A, B, and C. with respect to L are 5% below the corresponding base exchange rates, and their selling exchange rates are 10% above their corresponding base exchange rates. The following facts are known about the outlet on a particular day: 

1. The amount of L used by the outlet to buy C. equals the amount of L it received by selling C. 2. The amounts of L used by the outlet to buy A. and B. are in the ratio 5:3. 

3. The amounts of L the outlet received from the sales of A. and B. are in the ratio 5:9. 

4. The outlet received 88000 units of L by selling A. during the day. 

5. The outlet started the day with some amount of L, 2500 units of A, 4800 units of B, and 48000 units of C. 6. The outlet ended the day with some amount of L, 3300 units of A, 4800 units of B, and 51000 units of C. 

Q. 35 How many units of currency A. did the outlet buy on that day? 

Answer:1200 

Explanation: 

It is given that the base exchange rates of A, B. and C. with respect to L are in the ratio 100:120:1. Let us assume that base exchange rates are ‘100a’, ‘120a’ and ‘a’ in that order. 

It is given that the buying exchange rates of each of A, B, and C. with respect to L are 5% below the corresponding base exchange rates. Therefore, we can say that the buying exchange rates are 95a, 114a, 0.95a. 

It is given that the selling exchange rates of each of A, B, and C. with respect to L are 10% above their corresponding base exchange rates. Therefore, we can say that the selling exchange rates are 110a, 132a, 1.1a. 

We know about the opening and closing units in stock for each currency. Let us draw the table accordingly.

Let ‘p’, ‘q’ and ‘r’ be the number of units of currency A, B. and C. bought by the outlet on that day. 

Then, we can say that the outlet sold ‘p – 800’, ‘q’ and ‘r-3000’ units of currency A, B. and C. respectively. 

It is given that the amount of L used by the outlet to buy C. equals the amount of L it received by selling C. 

⇒ 0.95a*r = 1.1a*(r – 3000) 

⇒ 0.15r = 3300 

⇒ r = 22000 

It is also given that the amounts of L used by the outlet to buy A. and B. are in the ratio 5:3. 

=>(p ∗ 95a)/(q ∗ 114a) = 5/3

=>p = 2q 

Also, the amounts of L the outlet received from the sales of A. and B. are in the ratio 5:9. 

=> [(p − 800) ∗ 110a ] / (q ∗ 132a) = 5/9

=> [(2q − 800) ∗ 110a ] / (q ∗ 132a) = 5/9

=>q = 600 

Therefore, p = 2q = 2*600 = 1200. 

It is given that the outlet received 88000 units of L by selling A. during the day. 

⇒ (p-800)*110a = 88000 

⇒ (1200-800)*110a = 88000 

⇒ 44000a = 88000 

⇒ a = 2 

We can fill the entire table and answer all the questions. 

From the table we can see that the currency outlet bought 1200 units of A. 

Q. 36 How many units of currency C. did the outlet sell on that day? 

A. 22000 

B. 19000 

C. 6000 

D. 3000 

Answer: B. 

Explanation: 

It is given that the base exchange rates of A, B. and C. with respect to L are in the ratio 100:120:1. Let us assume that base exchange rates are ‘100a’, ‘120a’ and ‘a’ in that order. 

It is given that the buying exchange rates of each of A, B, and C. with respect to L are 5% below the corresponding base exchange rates. Therefore, we can say that the buying exchange rates are 95a, 114a, 0.95a. 

It is given that the selling exchange rates of each of A, B, and C. with respect to L are 10% above their corresponding base exchange rates. Therefore, we can say that the selling exchange rates are 110a, 132a, 1.1a. 

We know about the opening and closing units in stock for each currency. Let us draw the table accordingly. 

Let ‘p’, ‘q’ and ‘r’ be the number of units of currency A, B. and C. bought by the outlet on that day. 

Then, we can say that the outlet sold ‘p – 800’, ‘q’ and ‘r-3000’ units of currency A, B. and C. respectively.

It is given that the amount of L used by the outlet to buy C. equals the amount of L it received by selling C. 

⇒ 0.95a*r = 1.1a*(r – 3000) 

⇒ 0.15r = 3300 

⇒ r = 22000 

It is also given that the amounts of L used by the outlet to buy A. and B. are in the ratio 5:3. 

⇒ (p ∗ 95a)/ (q ∗ 114a) = 5/3

p = 2q 

Also, the amounts of L the outlet received from the sales of A. and B. are in the ratio 5:9. 

⇒ [(p − 800) ∗ 110a] / ( q ∗ 132a) = 5/9

⇒ [(2q − 800) ∗ 110a] / ( q ∗ 132a) = 5/9

q = 600 

Therefore, p = 2q = 2*600 = 1200. 

It is given that the outlet received 88000 units of L by selling A. during the day. 

⇒ (p-800)*110a = 88000 

⇒ (1200-800)*110a = 88000 

⇒ 44000a = 88000 

⇒ a = 2 

We can fill the entire table and answer all the questions. 

From the table we can see that the currency outlet sold 19000 units of currency C. Hence, option B. is the correct 

answer. 

Q. 37 What was the base exchange rate of currency B. with respect to currency L on that day ? 

Answer:240 

Explanation: 

It is given that the base exchange rates of A, B. and C. with respect to L are in the ratio 100:120:1. Let us assume that base exchange rates are ‘100a’, ‘120a’ and ‘a’ in that order. 

It is given that the buying exchange rates of each of A, B, and C. with respect to L are 5% below the corresponding base exchange rates. Therefore, we can say that the buying exchange rates are 95a, 114a, 0.95a. 

It is given that the selling exchange rates of each of A, B, and C. with respect to L are 10% above their corresponding base exchange rates. Therefore, we can say that the selling exchange rates are 110a, 132a, 1.1a. 

We know about the opening and closing units in stock for each currency. Let us draw the table accordingly. 

Let ‘p’, ‘q’ and ‘r’ be the number of units of currency A, B. and C. bought by the outlet on that day. 

Then, we can say that the outlet sold ‘p – 800’, ‘q’ and ‘r-3000’ units of currency A, B. and C. respectively.

It is given that the amount of L used by the outlet to buy C. equals the amount of L it received by selling C. 

⇒ 0.95a*r = 1.1a*(r – 3000) 

⇒ 0.15r = 3300 

⇒ r = 22000 

It is also given that the amounts of L used by the outlet to buy A. and B. are in the ratio 5:3. 

⇒ (p ∗ 95a)/ (q ∗ 114a) = 5/3

p = 2q 

Also, the amounts of L the outlet received from the sales of A. and B. are in the ratio 5:9. 

⇒ [(p − 800) ∗ 110a] / ( q ∗ 132a) = 5/9

⇒ [(2q − 800) ∗ 110a] / ( q ∗ 132a) = 5/9

q = 600 

Therefore, p = 2q = 2*600 = 1200. 

It is given that the outlet received 88000 units of L by selling A. during the day. 

⇒ (p-800)*110a = 88000 

⇒ (1200-800)*110a = 88000 

⇒ 44000a = 88000 

⇒ a = 2

We can fill the entire table and answer all the questions.

From the table we can see that the base exchange rate of currency B. with respect to currency L was 240. 

Q. 38 What was the buying exchange rate of currency C. with respect to currency L on that day? 

A. 1.10 

B. 0.95 

C. 2.20 

D. 1.90 

Answer: D. 

Explanation: 

It is given that the base exchange rates of A, B. and C. with respect to L are in the ratio 100:120:1. Let us assume that base exchange rates are ‘100a’, ‘120a’ and ‘a’ in that order. 

It is given that the buying exchange rates of each of A, B, and C. with respect to L are 5% below the corresponding base exchange rates. Therefore, we can say that the buying exchange rates are 95a, 114a, 0.95a. 

It is given that the selling exchange rates of each of A, B, and C. with respect to L are 10% above their corresponding base exchange rates. Therefore, we can say that the selling exchange rates are 110a, 132a, 1.1a. 

We know about the opening and closing units in stock for each currency. Let us draw the table accordingly.

Let ‘p’, ‘q’ and ‘r’ be the number of units of currency A, B. and C. bought by the outlet on that day. 

Then, we can say that the outlet sold ‘p – 800’, ‘q’ and ‘r-3000’ units of currency A, B. and C. respectively.

It is given that the amount of L used by the outlet to buy C. equals the amount of L it received by selling C. 

⇒ 0.95a*r = 1.1a*(r – 3000) 

⇒ 0.15r = 3300 

⇒ r = 22000 

It is also given that the amounts of L used by the outlet to buy A. and B. are in the ratio 5:3. 

⇒ (p ∗ 95a)/ (q ∗ 114a) = 5/3

p = 2q 

Also, the amounts of L the outlet received from the sales of A. and B. are in the ratio 5:9. 

⇒ [(p − 800) ∗ 110a] / ( q ∗ 132a) = 5/9

⇒ [(2q − 800) ∗ 110a] / ( q ∗ 132a) = 5/9

q = 600 

Therefore, p = 2q = 2*600 = 1200. 

It is given that the outlet received 88000 units of L by selling A. during the day. 

⇒ (p-800)*110a = 88000 

⇒ (1200-800)*110a = 88000 

⇒ 44000a = 88000 

⇒ a = 2 

 We can fill the entire table and answer all the questions. 

From the table we can see that the buying exchange rate of currency C. with respect to currency L was 1.9. Hence, we 

can say that option D. is the correct answer. 

 

Instructions 

Each visitor to an amusement park needs to buy a ticket. Tickets can be Platinum, Gold, or Economy. Visitors are classified as Old, Middle-aged, or Young. The following facts are known about visitors and ticket sales on a particular day: 

1. 140 tickets were sold. 

2. The number of Middle-aged visitors was twice the number of Old visitors, while the number of Young visitors was twice the number of Middle-aged visitors. 

3. Young visitors bought 38 of the 55 Economy tickets that were sold, and they bought half the total number of Platinum tickets that were sold. 

4. The number of Gold tickets bought by Old visitors was equal to the number of Economy tickets bought by Old visitors. 

Q. 39 If the number of Old visitors buying Platinum tickets was equal to the number of Middle-aged visitors buying Platinum tickets, then which among the following could be the total number of Platinum tickets sold? 

A. 34 

B. 36 

C. 38 

D. 32 

Answer: D. 

Explanation: 

Let ‘x’ be the the number of Old visitors. Then, the number of middle-aged visitors = 2x. 

Also, the number of Young visitors = 2*2x = 4x 

⇒ x+2x+4x = 140 

⇒ x = 20 

It is given that total of 55 Economy tickets were sold out. 

It is given that Young visitors half the total number of Platinum tickets that were sold. 

Let ‘Y’ be the number of Platinum tickets bought by the Young visitors. 

Then,the number of Platinum tickets sold = 2Y. 

Consequently, we can say that the number of Gold tickets sold = 140 – 55 – 2Y = 85 – 2Y. 

Let us assume that ‘Z’ is the number of Economy tickets bought by the Old visitors. It is given that the number of Gold tickets bought by Old visitors was equal to the number of Economy tickets bought by Old visitors. 

It is given that the number of Old visitors buying Platinum tickets was equal to the number of Middle-aged visitors buying Platinum tickets. 

20 – 2Z = (Y+2Z) – 20 

Y + 4Z = 40 

2Y + 8Z = 80 

2Y = 80 – 8Z 

We can see that Z can take only integer values. Therefore, we can say that the the total number of Platinum tickets sold will be a multiple of 8. Hence, option D. is the correct answer. 

Q. 40 If the number of Old visitors buying Platinum tickets was equal to the number of Middle-aged visitors buying Economy tickets, then the number of Old visitors buying Gold tickets was 

Answer:

Explanation: 

Let ‘x’ be the the number of Old visitors. Then, the number of middle-aged visitors = 2x. 

Also, the number of Young visitors = 2*2x = 4x 

⇒ x+2x+4x = 140 

⇒ x = 20 

It is given that total of 55 Economy tickets were sold out. 

It is given that Young visitors half the total number of Platinum tickets that were sold. 

Let ‘Y’ be the number of Platinum tickets bought by the Young visitors. 

Then,the number of Platinum tickets sold = 2Y. 

Consequently, we can say that the number of Gold tickets sold = 140 – 55 – 2Y = 85 – 2Y. 

Let us assume that ‘Z’ is the number of Economy tickets bought by the Old visitors. It is given that the number of Gold tickets bought by Old visitors was equal to the number of Economy tickets bought by Old visitors. 

It is given that the number of Old visitors buying Platinum tickets was equal to the number of Middle-aged visitors buying Economy tickets. 

20 – 2Z = 17 – Z 

⇒ Z = 3 

Therefore, we can say that the number of Old visitors buying Gold tickets = 3 

Q. 41 If the number of Old visitors buying Gold tickets was strictly greater than the number of Young visitors buying Gold tickets, then the number of Middle-aged visitors buying Gold tickets was 

Answer:

Explanation: 

Let ‘x’ be the the number of Old visitors. Then, the number of middle-aged visitors = 2x. 

Also, the number of Young visitors = 2*2x = 4x 

⇒ x+2x+4x = 140 

⇒ x = 20 

It is given that total of 55 Economy tickets were sold out. 

It is given that Young visitors half the total number of Platinum tickets that were sold. 

Let ‘Y’ be the number of Platinum tickets bought by the Young visitors. 

Then,the number of Platinum tickets sold = 2Y. 

Consequently, we can say that the number of Gold tickets sold = 140 – 55 – 2Y = 85 – 2Y. 

Let us assume that ‘Z’ is the number of Economy tickets bought by the Old visitors. It is given that the number of Gold tickets bought by Old visitors was equal to the number of Economy tickets bought by Old visitors. 

It is given that the number of Old visitors buying Gold tickets was strictly greater than the number of Young visitors buying Gold tickets. 

Z > 42 – Y 

⇒ Z + Y > 42 … (1) 

The number of Middle-aged visitors buying Gold tickets = 43 – (Y+Z) 

Since (Y+Z) > 42, then We can say that (Y+Z)min = 43. 

Hence, the number of Middle-aged visitors buying Gold tickets = 43 – 43 = 0 

Q. 42 Which of the following statements MUST be FALSE? 

A. The numbers of Gold and Platinum tickets bought by Young visitors were equal 

B. The numbers of Middle-aged and Young visitors buying Gold tickets were equal 

C. The numbers of Old and Middle-aged visitors buying Platinum tickets were equal 

D. The numbers of Old and Middle-aged visitors buying Economy tickets were equal 

Answer: D. 

Explanation: 

Let ‘x’ be the the number of Old visitors. Then, the number of middle-aged visitors = 2x. 

Also, the number of Young visitors = 2*2x = 4x 

⇒ x+2x+4x = 140 

⇒ x = 20 

It is given that total of 55 Economy tickets were sold out. 

It is given that Young visitors half the total number of Platinum tickets that were sold. 

Let ‘Y’ be the number of Platinum tickets bought by the Young visitors. 

Then,the number of Platinum tickets sold = 2Y. 

Consequently, we can say that the number of Gold tickets sold = 140 – 55 – 2Y = 85 – 2Y. 

Let us assume that ‘Z’ is the number of Economy tickets bought by the Old visitors. It is given that the number of Gold tickets bought by Old visitors was equal to the number of Economy tickets bought by Old visitors. 

Let us check with the help of options. 

Option (A): The numbers of Gold and Platinum tickets bought by Young visitors were equal. Y = 42 – Y 

⇒ Y = 21. Hence, this statement can be true. 

Option (B): The numbers of Middle-aged and Young visitors buying Gold tickets were equal  43 – (Y+Z) = 42 – Y 

⇒ Z = 1. Hence, this statement can be true. 

Option (C): The numbers of Old and Middle-aged visitors buying Platinum tickets were equal 20 – 2Z = (Y+2Z) – 20 

⇒ Y+4Z = 40. Hence, this statement can be true. 

Option (D): The numbers of Old and Middle-aged visitors buying Economy tickets were equal Z = 17 – Z 

⇒ Z = 8.5. This is not possible as Z has to be an integer. Hence, we can say that this statement is false. 

Instructions 

An agency entrusted to accredit colleges looks at four parameters: faculty quality (F), reputation (R), placement quality (P), and infrastructure (I). The four parameters are used to arrive at an overall score, which the agency uses to give an accreditation to the colleges. In each parameter, there are five possible letter grades given, each carrying certain points: A. (50 points), B. (40 points), C. (30 points), D. (20 points), and F (0 points). The overall score for a college is the weighted sum of the points scored in the four parameters. The weights of the parameters are 0.1, 0.2, 0.3 and 0.4 in some order, but the order is not disclosed. Accreditation is awarded based on the following scheme: 

Eight colleges apply for accreditation, and receive the following grades in the four parameters (F, R, P, and I):

It is further known that in terms of overall scores: 

1. High Q is better than Best Ed; 

2. Best Ed is better than Cosmopolitan; and 

3. Education Aid is better than A-one. 

Q. 43 What is the weight of the faculty quality parameter? 

A. 0.2 

B. 0.3 

C. 0.4 

D. 0.1 

Answer: D. 

Explanation: 

It is given that: High Q > Best Ed > Cosmopolitan and Education Aid > A-one 

We can say that High Q > Cosmopolitan 

We can see that both High Q and Cosmopolitan got same points in reputation (R) and placement quality (P). High Q received more points in infrastructure (I) than Cosmopolitan whereas Cosmopolitan received more points in faculty Quality (F) than High Q. 

Hence, we can say that Infrastructure’s weight should be greater than Faculty quality. i.e. I > F 

Similarly, We can see that both Best Ed and Cosmopolitan got same points in faculty Quality (F) and placement quality (P). Best Ed received more points in reputation (R) than Cosmopolitan whereas Cosmopolitan received more points in infrastructure (I) than Best Ed. 

Hence, we can say that reputation’s weight should be greater than infrastructure. i.e. R > I 

Similarly, We can see that both Education Aid and A-one got same points in faculty Quality (F) and reputation (R). Education Aid received more points in infrastructure (I) than A-one whereas A-one received more points in placement quality (P) than Education Aid. 

Hence, we can say that reputation’s weight should be greater than infrastructure. i.e. I > P So basically there are two possible cases: R > I > P > F or R > I > F > P 

Case 1: Order of weights assigned = R > I > P > F 

R = 0.4, I = 0.3. P = 0.2, F = 0.1 

In this case overall score received by Best Ed = 0.1*40+0.4*30+0.2*20+0.3*20 = 26 

In this case overall score received by High Q = 0.1*30+0.4*20+0.2*20+0.3*40 = 27 

We can see that High Q’s overall score is higher than Best Ed. Hence, this is a possible case. Case 2: Order of weights assigned = R > I > F > P 

R = 0.4, I = 0.3. P = 0.1, F = 0.2 

In this case overall score received by Best Ed = 0.2*40+0.4*30+0.1*20+0.3*20 = 28 

In this case overall score received by High Q = 0.2*30+0.4*20+0.1*20+0.3*40 = 28 

We can see that High Q’s overall score is not greater than the overall score received Best Ed. Hence, this case is not possible. 

Now that we know the weight of each parameter, we can calculate the overall score and accreditation received by each college. 

We can see that weight of the faculty quality parameter = 0.1. Hence,option D. is the correct answer. 

Q. 44 How many colleges receive the accreditation of AAA? 

Answer:

Explanation: 

It is given that: High Q > Best Ed > Cosmopolitan and Education Aid > A-one 

We can say that High Q > Cosmopolitan 

We can see that both High Q and Cosmopolitan got same points in reputation (R) and placement quality (P). High Q received more points in infrastructure (I) than Cosmopolitan whereas Cosmopolitan received more points in faculty Quality (F) than High Q. 

Hence, we can say that Infrastructure’s weight should be greater than Faculty quality. i.e. I > F Similarly, We can see that both Best Ed and Cosmopolitan got same points in faculty Quality (F) and placement quality 

(P). Best Ed received more points in reputation (R) than Cosmopolitan whereas Cosmopolitan received more points in infrastructure (I) than Best Ed. 

Hence, we can say that reputation’s weight should be greater than infrastructure. i.e. R > I 

Similarly, We can see that both Education Aid and A-one got same points in faculty Quality (F) and reputation (R). Education Aid received more points in infrastructure (I) than A-one whereas A-one received more points in placement quality (P) than Education Aid. 

Hence, we can say that reputation’s weight should be greater than infrastructure. i.e. I > P So basically there are two possible cases: R > I > P > F or R > I > F > P 

Case 1: Order of weights assigned = R > I > P > F 

R = 0.4, I = 0.3. P = 0.2, F = 0.1 

In this case overall score received by Best Ed = 0.1*40+0.4*30+0.2*20+0.3*20 = 26 

In this case overall score received by High Q = 0.1*30+0.4*20+0.2*20+0.3*40 = 27 

We can see that High Q’s overall score is higher than Best Ed. Hence, this is a possible case. Case 2: Order of weights assigned = R > I > F > P 

R = 0.4, I = 0.3. P = 0.1, F = 0.2 

In this case overall score received by Best Ed = 0.2*40+0.4*30+0.1*20+0.3*20 = 28 

In this case overall score received by High Q = 0.2*30+0.4*20+0.1*20+0.3*40 = 28 

We can see that High Q’s overall score is not greater than the overall score received Best Ed. Hence, this case is not possible. 

Now that we know the weight of each parameter, we can calculate the overall score and accreditation received by each college. 

From the table, we can see that three received the accreditation of AAA. 

Q. 45 What is the highest overall score among the eight colleges ? 

Answer:48 

Explanation: 

It is given that: High Q > Best Ed > Cosmopolitan and Education Aid > A-one 

We can say that High Q > Cosmopolitan 

We can see that both High Q and Cosmopolitan got same points in reputation (R) and placement quality (P). High Q received more points in infrastructure (I) than Cosmopolitan whereas Cosmopolitan received more points in faculty Quality (F) than High Q. 

Hence, we can say that Infrastructure’s weight should be greater than Faculty quality. i.e. I > F 

Similarly, We can see that both Best Ed and Cosmopolitan got same points in faculty Quality (F) and placement quality (P). Best Ed received more points in reputation (R) than Cosmopolitan whereas Cosmopolitan received more points in infrastructure (I) than Best Ed. 

Hence, we can say that reputation’s weight should be greater than infrastructure. i.e. R > I Similarly, We can see that both Education Aid and A-one got same points in faculty Quality (F) and reputation 

(R). Education Aid received more points in infrastructure (I) than A-one whereas A-one received more points in placement quality (P) than Education Aid. 

Hence, we can say that reputation’s weight should be greater than infrastructure. i.e. I > P So basically there are two possible cases: R > I > P > F or R > I > F > P 

Case 1: Order of weights assigned = R > I > P > F 

R = 0.4, I = 0.3. P = 0.2, F = 0.1 

In this case overall score received by Best Ed = 0.1*40+0.4*30+0.2*20+0.3*20 = 26 

In this case overall score received by High Q = 0.1*30+0.4*20+0.2*20+0.3*40 = 27 

We can see that High Q’s overall score is higher than Best Ed. Hence, this is a possible case. Case 2: Order of weights assigned = R > I > F > P 

R = 0.4, I = 0.3. P = 0.1, F = 0.2 

In this case overall score received by Best Ed = 0.2*40+0.4*30+0.1*20+0.3*20 = 28 

In this case overall score received by High Q = 0.2*30+0.4*20+0.1*20+0.3*40 = 28 

We can see that High Q’s overall score is not greater than the overall score received Best Ed. Hence, this case is not possible. 

Now that we know the weight of each parameter, we can calculate the overall score and accreditation received by each college. 

From the table we can see that Education Aid scored the highest overall score = 48. 

Q. 46 How many colleges have overall scores between 31 and 40, both inclusive? 

A.

B.

C.

D.

Answer: A. 

Explanation: 

It is given that: High Q > Best Ed > Cosmopolitan and Education Aid > A-one 

We can say that High Q > Cosmopolitan 

We can see that both High Q and Cosmopolitan got same points in reputation (R) and placement quality (P). High Q received more points in infrastructure (I) than Cosmopolitan whereas Cosmopolitan received more points in faculty Quality (F) than High Q. 

Hence, we can say that Infrastructure’s weight should be greater than Faculty quality. i.e. I > F 

Similarly, We can see that both Best Ed and Cosmopolitan got same points in faculty Quality (F) and placement quality (P). Best Ed received more points in reputation (R) than Cosmopolitan whereas Cosmopolitan received more points in infrastructure (I) than Best Ed. 

Hence, we can say that reputation’s weight should be greater than infrastructure. i.e. R > I Similarly, We can see that both Education Aid and A-one got same points in faculty Quality (F) and reputation 

(R). Education Aid received more points in infrastructure (I) than A-one whereas A-one received more points in placement quality (P) than Education Aid. 

Hence, we can say that reputation’s weight should be greater than infrastructure. i.e. I > P So basically there are two possible cases: R > I > P > F or R > I > F > P 

Case 1: Order of weights assigned = R > I > P > F 

R = 0.4, I = 0.3. P = 0.2, F = 0.1 

In this case overall score received by Best Ed = 0.1*40+0.4*30+0.2*20+0.3*20 = 26 

In this case overall score received by High Q = 0.1*30+0.4*20+0.2*20+0.3*40 = 27 

We can see that High Q’s overall score is higher than Best Ed. Hence, this is a possible case. Case 2: Order of weights assigned = R > I > F > P 

R = 0.4, I = 0.3. P = 0.1, F = 0.2 

In this case overall score received by Best Ed = 0.2*40+0.4*30+0.1*20+0.3*20 = 28 

In this case overall score received by High Q = 0.2*30+0.4*20+0.1*20+0.3*40 = 28 

We can see that High Q’s overall score is not greater than the overall score received Best Ed. Hence, this case is not possible. 

Now that we know the weight of each parameter, we can calculate the overall score and accreditation received by each college. 

From the table we can see that none of the mentioned colleges received an overall score between 31 and 40, both inclusive. Hence, option A. is the correct answer. 

Instructions 

Fun Sports (FS) provides training in three sports – Gilli-danda (G), Kho-Kho (K), and Ludo (L). Currently it has an enrollment of 39 students each of whom is enrolled in at least one of the three sports. The following details are known: 1. The number of students enrolled only in L is double the number of students enrolled in all the three sports. 2. There are a total of 17 students enrolled in G. 

3. The number of students enrolled only in G is one less than the number of students enrolled only in L. 4. The number of students enrolled only in K is equal to the number of students who are enrolled in both K and L. 5. The maximum student enrollment is in L. 

6. Ten students enrolled in G are also enrolled in at least one more sport. 

Q. 47 What is the minimum number of students enrolled in both G and L but not in K? 

Answer:

Explanation: 

Let ‘x’ be the number of students enrolled in all three sports. Then the number of students enrolled only in L = 2x 

It is given that there are a total of 17 students enrolled in G. Also, ten students enrolled in G are also enrolled in at least one more sport. Hence, the number of students enrolled in only G = 17 – 10 = 7 

The number of students enrolled only in G is one less than the number of students enrolled only in L. Hence, the number of students enrolled only in L = 7+1 

⇒ 2x = 8 

⇒ x = 4 

Let us assume that ‘y’ students are enrolled in K and L but not G. Then, the number of students enrolled only in K = y + 4 

Let us assume that ‘z’ be the the number of students enrolled in G and K but not L. Then, the number of students enrolled G and L bot not K = 10 – 4 – z = 6 – z 

It is given that a total of 39 students in the sports. 

7 + z + 4 + 6 – z + 8 + y + y + 4 = 39 

⇒ y = 5 

Number of students enrolled in G = 17 

Number of students enrolled in K = 9 + 4 + 5 + z = 18 + z 

Number of students enrolled in L = 6 – z + 4 + 5 + 8 = 23 – z 

It is given that the maximum student enrollment is in L. 

⇒ 23 – z > 18 + z 

⇒ 2z < 5 

⇒ z < 2.5 

Therefore, we can say that z can take three values = {0, 1, 2} 

The number of students enrolled in both G and L but not in K = 6 – z. This number will be minimum when ‘z’ is maximum. z_{max} = 2 

Therefore, the minimum number of students enrolled in both G and L but not in K = 6 – 2 = 4 

Q. 48 If the numbers of students enrolled in K and L are in the ratio 19:22, then what is the number of students enrolled in L? 

A. 18 

B. 17 

C. 22 

D. 19 

Answer: C. 

Explanation: 

Let ‘x’ be the number of students enrolled in all three sports. Then the number of students enrolled only in L = 2x 

It is given that there are a total of 17 students enrolled in G. Also, ten students enrolled in G are also enrolled in at least one more sport. Hence, the number of students enrolled in only G = 17 – 10 = 7 

The number of students enrolled only in G is one less than the number of students enrolled only in L. Hence, the number of students enrolled only in L = 7+1 

⇒ 2x = 8 

⇒ x = 4 

Let us assume that ‘y’ students are enrolled in K and L but not G. Then, the number of students enrolled only in K = y + 4 

Let us assume that ‘z’ be the the number of students enrolled in G and K but not L. Then, the number of students enrolled G and L bot not K = 10 – 4 – z = 6 – z 

It is given that a total of 39 students in the sports. 

7 + z + 4 + 6 – z + 8 + y + y + 4 = 39 

⇒ y = 5 

Number of students enrolled in G = 17 

Number of students enrolled in K = 9 + 4 + 5 + z = 18 + z 

Number of students enrolled in L = 6 – z + 4 + 5 + 8 = 23 – z It is given that the maximum student enrollment is in L. 

⇒ 23 – z > 18 + z 

⇒ 2z < 5 

⇒ z < 2.5 

Therefore, we can say that z can take three values = {0, 1, 2} It is given that the numbers of students enrolled in K and L are in the ratio 19:22. 

(18 + z )/ (23 − z) = 19/22

z = 1 which is a possible solution as well. 

In this case the number of students enrolled in L = 23 – z = 23 – 1 = 22. Hence, option C. is the correct answer. 

Q. 49 Due to academic pressure, students who were enrolled in all three sports were asked to withdraw from one of the three sports. After the withdrawal, the number of students enrolled in G was six less than the number of students enrolled in L, while the number of students enrolled in K went down by one. After the withdrawal, how many students were enrolled in both G and K? 

Answer:

Explanation: 

Let ‘x’ be the number of students enrolled in all three sports. Then the number of students enrolled only in L = 2x 

It is given that there are a total of 17 students enrolled in G. Also, ten students enrolled in G are also enrolled in at least one more sport. Hence, the number of students enrolled in only G = 17 – 10 = 7 

The number of students enrolled only in G is one less than the number of students enrolled only in L. Hence, the number of students enrolled only in L = 7+1 

⇒ 2x = 8 

⇒ x = 4 

Let us assume that ‘y’ students are enrolled in K and L but not G. Then, the number of students enrolled only in K = y + 4 

Let us assume that ‘z’ be the the number of students enrolled in G and K but not L. Then, the number of students enrolled G and L bot not K = 10 – 4 – z = 6 – z 

It is given that a total of 39 students in the sports. 

7 + z + 4 + 6 – z + 8 + y + y + 4 = 39 

⇒ y = 5 

Number of students enrolled in G = 17 

Number of students enrolled in K = 9 + 4 + 5 + z = 18 + z 

Number of students enrolled in L = 6 – z + 4 + 5 + 8 = 23 – z 

It is given that the maximum student enrollment is in L. 

⇒ 23 – z > 18 + z 

⇒ 2z < 5 

⇒ z < 2.5 

Therefore, we can say that z can take three values = {0, 1, 2} 

Hence, the number of students enrolled in K = 18 + z = {18, 19, 20} 

It is given that after withdrawal the number of students enrolled in K went down by one. This one student must have left sports K. Hence we can say that the remaining 3 students must have left either G or L. 

Before withdraw there were a total of 24 students were enrolled in exactly 1 sports, 11 students were enrolled in exactly 2 courses and 4 students were enrolled in all three courses. 

The students which were enrolled in all three sports, withdrew from one of the sports. Hence, we can say that now the number of students who were enrolled in exactly 2 courses = 11 + 4 = 15. 

It is given that the number of students enrolled in G was six less than the number of students enrolled in L. Let ‘a’ be the number of students who were enrolled in G and K but not L. Then, the number of students who were enrolled in L and K but not G = a + 5 

Consequently, we can say that the number of students enrolled in G and L but not K = 15 – (2a + 5) = 10 – 2a 

Number of students enrolled in this case = a + a+5 + 9 = 14 + 2a. We can see that ’14+2a’ is an even number. It is given that the number of students enrolled in K went down by one. Therefore, we can say that the number of students 

enrolled in K earlier was an odd number. 

Hence, the number of students enrolled in K = 18 + z = {18, 19, 20} 

We can see that only ’19’ is an odd number. Hence, we can say that the number of students enrolled in K after withdrawal = 18 

⇒ 14 + 2a = 18 

⇒ a = 2 

From the diagram we can see that the number of students enrolled in both G and K = 2. 

Q. 50 Due to academic pressure, students who were enrolled in all three sports were asked to withdraw from one of the three sports. After the withdrawal, the number of students enrolled in G was six less than the number of students enrolled in L, while the number of students enrolled in K went down by one. After the withdrawal, how many students were enrolled in both G and L? 

A.

B.

C.

D.

Answer: A. 

Explanation: 

Let ‘x’ be the number of students enrolled in all three sports. Then the number of students enrolled only in L = 2x 

It is given that there are a total of 17 students enrolled in G. Also, ten students enrolled in G are also enrolled in at least one more sport. Hence, the number of students enrolled in only G = 17 – 10 = 7 

The number of students enrolled only in G is one less than the number of students enrolled only in L. Hence, the number of students enrolled only in L = 7+1 

⇒ 2x = 8 

⇒ x = 4 

Let us assume that ‘y’ students are enrolled in K and L but not G. Then, the number of students enrolled only in K = y + 4 

Let us assume that ‘z’ be the the number of students enrolled in G and K but not L. Then, the number of students enrolled G and L bot not K = 10 – 4 – z = 6 – z 

It is given that a total of 39 students in the sports. 

7 + z + 4 + 6 – z + 8 + y + y + 4 = 39 

⇒ y = 5

Number of students enrolled in G = 17 

Number of students enrolled in K = 9 + 4 + 5 + z = 18 + z 

Number of students enrolled in L = 6 – z + 4 + 5 + 8 = 23 – z 

It is given that the maximum student enrollment is in L. 

⇒ 23 – z > 18 + z 

⇒ 2z < 5 

⇒ z < 2.5 

Therefore, we can say that z can take three values = {0, 1, 2} 

Hence, the number of students enrolled in K = 18 + z = {18, 19, 20} 

It is given that after withdrawal the number of students enrolled in K went down by one. This one student must have left sports K. Hence we can say that the remaining 3 students must have left either G or L. 

Before withdraw there were a total of 24 students were enrolled in exactly 1 sports, 11 students were enrolled in exactly 2 courses and 4 students were enrolled in all three courses. 

The students which were enrolled in all three sports, withdrew from one of the sports. Hence, we can say that now the number of students who were enrolled in exactly 2 courses = 11 + 4 = 15. 

It is given that the number of students enrolled in G was six less than the number of students enrolled in L. Let ‘a’ be the number of students who were enrolled in G and K but not L. Then, the number of students who were enrolled in L and K but not G = a + 5 

Consequently, we can say that the number of students enrolled in G and L but not K = 15 – (2a + 5) = 10 – 2a

Number of students enrolled in this case = a + a+5 + 9 = 14 + 2a. We can see that ’14+2a’ is an even number. It is given that the number of students enrolled in K went down by one. Therefore, we can say that the number of students enrolled in K earlier was an odd number. 

Hence, the number of students enrolled in K = 18 + z = {18, 19, 20} 

We can see that only ’19’ is an odd number. Hence, we can say that the number of students enrolled in K after withdrawal = 18 

⇒ 14 + 2a = 18 

⇒ a = 2 

From the diagram we can see that the number of students enrolled in both G and L = 6. Hence, option A. is the correct answer. 

 

Instructions 

According to a coding scheme the sentence: 

“Peacock is designated as the national bird of India” is coded as 5688999 35 1135556678 56 458 13666689 1334 79 13366 

This coding scheme has the following rules: 

a: The scheme is case-insensitive (does not distinguish between upper case and lower case letters). 

b: Each letter has a unique code which is a single digit from among 1,2,3, …, 9. 

c: The digit 9 codes two letters, and every other digit codes three letters. 

d: The code for a word is constructed by arranging the digits corresponding to its letters in a non-decreasing sequence. Answer these questions on the basis of this information. 

Q. 51 What best can be concluded about the code for the letter L? 

A.

B.

C. 1 or 8 

D.

Answer: A. 

Explanation: 

We can see that India’s code is 13366 therefore we can say that I’s code is either 3 or 6. 

Also, we can see that code for word “is” is 35 therefore we can say that I’s code is 3. Consequently, we can say that S’s code is 5. 

Also, we can see that code of word ‘as’ is 56 therefore we can say that A’s code is 6. Consequently, we can say that S’s code is 5. 

There is only one letter ‘O’ common in words ‘of’ and ‘national’. In code word as well only digit ‘9’ is common in both. Hence, we can say that letter ‘O’ is assigned numerical ‘9’. Consequently, we can say that F is assigned number 7. 

It is given that ‘9’ is assigned to only two alphabets one of them is ‘O’. We can see that there are three 9’s in Peacock’s code. One of the digit ‘9’ is used for ‘O’.Remaining two 9’s must represent same letter. We can see that only letter ‘C’ has appeared twice in Peacock. Therefore, we can say that ‘C’ is assigned number ‘9’. 

In word national ‘N’ has appeared twice. In code only digit ‘6’ has appeared more than once. Hence, we can say that code of letter N is ‘6’. Consequently, we can say that code for letter ‘D’ is ‘1’ because in India rest of the numerals are already taken. 

In words, ‘the’ and ‘national’ only letter ‘t’ is common. In code as well only digit ‘8’ is common in two codes. Hence, we can say that letter code for letter ‘t’ is 8. 

In words, ‘the’ and ‘peacock’ only letter ‘e’ is common. In code as well only digit ‘5’ is common in two codes. Hence, we can say that letter code for letter ‘e’ is 5. Consequently, we can say that leftover letter, in word “the”, ‘H’s code is 4. 

We can see that code for word “NATIONAL” is 13666689. Hence, we can say that code for the letter L is ‘1’. Hence, option A. is the correct answer. 

Q. 52 What best can be concluded about the code for the letter B? 

A. 3 or 4 

B. 1 or 3 or 4 

C.

D.

Answer: A. 

Explanation: 

We can see that India’s code is 13366 therefore we can say that I’s code is either 3 or 6. 

Also, we can see that code for word “is” is 35 therefore we can say that I’s code is 3. Consequently, we can say that S’s code is 5. 

Also, we can see that code of word ‘as’ is 56 therefore we can say that A’s code is 6. Consequently, we can say that S’s code is 5. 

There is only one letter ‘O’ common in words ‘of’ and ‘national’. In code word as well only digit ‘9’ is common in both. Hence, we can say that letter ‘O’ is assigned numerical ‘9’. Consequently, we can say that F is assigned number 7. 

It is given that ‘9’ is assigned to only two alphabets one of them is ‘O’. We can see that there are three 9’s in Peacock’s code. One of the digit ‘9’ is used for ‘O’.Remaining two 9’s must represent same letter. We can see that only letter ‘C’ has appeared twice in Peacock. Therefore, we can say that ‘C’ is assigned number ‘9’. 

In word national ‘N’ has appeared twice. In code only digit ‘6’ has appeared more than once. Hence, we can say that 

code of letter N is ‘6’. Consequently, we can say that code for letter ‘D’ is ‘1’ because in India rest of the numerals are already taken. 

In words, ‘the’ and ‘national’ only letter ‘t’ is common. In code as well only digit ‘8’ is common in two codes. Hence, we can say that letter code for letter ‘t’ is 8. 

In words, ‘the’ and ‘peacock’ only letter ‘e’ is common. In code as well only digit ‘5’ is common in two codes. Hence, we can say that letter code for letter ‘e’ is 5. Consequently, we can say that leftover letter, in word “the”, ‘H’s code is 4. 

We can see that code for word “NATIONAL” is 13666689. Hence, we can say that code for the letter L is ‘1’. 

We can see that code for word “BIRD” is 1334. 1 corresponds to D. and one 3 corresponds to I. Hence, we can say that code for letters ‘R’ and ‘B’ are ‘3’ and ‘4’ in any order. 

Therefore, we can say that for letter ‘B’ there are two possible numbers: 3 or 4 

Hence, option A. is the correct answer. 

Q. 53 For how many digits can the complete list of letters associated with that digit be identified? 

A.

B.

C.

D.

Answer: B. 

Explanation: 

We can see that India’s code is 13366 therefore we can say that I’s code is either 3 or 6. 

Also, we can see that code for word “is” is 35 therefore we can say that I’s code is 3. Consequently, we can say that S’s code is 5. 

Also, we can see that code of word ‘as’ is 56 therefore we can say that A’s code is 6. Consequently, we can say that S’s code is 5. 

There is only one letter ‘O’ common in words ‘of’ and ‘national’. In code word as well only digit ‘9’ is common in both. Hence, we can say that letter ‘O’ is assigned numerical ‘9’. Consequently, we can say that F is assigned number 7. 

It is given that ‘9’ is assigned to only two alphabets one of them is ‘O’. We can see that there are three 9’s in Peacock’s code. One of the digit ‘9’ is used for ‘O’.Remaining two 9’s must represent same letter. We can see that only letter ‘C’ has appeared twice in Peacock. Therefore, we can say that ‘C’ is assigned number ‘9’. 

In word national ‘N’ has appeared twice. In code only digit ‘6’ has appeared more than once. Hence, we can say that code of letter N is ‘6’. Consequently, we can say that code for letter ‘D’ is ‘1’ because in India rest of the numerals are already taken. 

In words, ‘the’ and ‘national’ only letter ‘t’ is common. In code as well only digit ‘8’ is common in two codes. Hence, we can say that letter code for letter ‘t’ is 8. 

In words, ‘the’ and ‘peacock’ only letter ‘e’ is common. In code as well only digit ‘5’ is common in two codes. Hence, we can say that letter code for letter ‘e’ is 5. Consequently, we can say that leftover letter, in word “the”, ‘H’s code is 4. 

We can see that code for word “NATIONAL” is 13666689. Hence, we can say that code for the letter L is ‘1’. 

We can see that code for word “DESIGNATED” is 1135556678. Hence, we can say that code for the letter ‘G’ is ‘7’. 

We can see that code for word “PEACOCK” is 5688999. Hence, we can say that code for the letters ‘P’ and ‘K’ is ‘8’.

Digit ‘1’ is used for L and D. only. We can not figure out the third letter for which digit 1 is used. Digit ‘2’ is not used for any letter. Hence, we can not figure out all the letters for which digit 2 is correct code. Digit ‘3’ is used for letter ‘I’ only. Hence, we can not figure out all the letters for which digit 3 is correct code. 

Digit ‘4’ is used for letters ‘H’ and one of ‘B’ and ‘R’. Hence, we can not figure out all the letters for which digit 4 is correct code. 

Digit ‘5’ is used for letters ‘S’ and ‘E’. We can not figure out the third letter for which digit 5 is used. Digit ‘6’ is used for letters ‘A’ and ‘N’. We can not figure out the third letter for which digit 6 is used. Digit ‘7’ is used for letters ‘G’ and ‘F’. We can not figure out the third letter for which digit 7 is used. 

Digit ‘8’ is used for letters ‘T’, ‘P’ and K. Hence, we can say that this is one of the digit for which the complete list of letters associated is known. 

Digit ‘9’ is used for letters ‘C’ and ‘O’. Hence, we can say that this is one of the digit for which the complete list of letters associated is known. 

Therefore, we can say that for only two digits (8 and 9), the complete list of letters associated is known. Hence, option B. is the correct answer. 

Q. 54 Which set of letters CANNOT be coded with the same digit? 

A. S,E,Z 

B. I,B,M 

C. S,U,V 

D. X,Y,Z 

Answer: C. 

Explanation: 

We can see that India’s code is 13366 therefore we can say that I’s code is either 3 or 6. 

Also, we can see that code for word “is” is 35 therefore we can say that I’s code is 3. Consequently, we can say that S’s code is 5. 

Also, we can see that code of word ‘as’ is 56 therefore we can say that A’s code is 6. Consequently, we can say that S’s code is 5. 

There is only one letter ‘O’ common in words ‘of’ and ‘national’. In code word as well only digit ‘9’ is common in both. Hence, we can say that letter ‘O’ is assigned numerical ‘9’. Consequently, we can say that F is assigned number 7. 

It is given that ‘9’ is assigned to only two alphabets one of them is ‘O’. We can see that there are three 9’s in Peacock’s code. One of the digit ‘9’ is used for ‘O’.Remaining two 9’s must represent same letter. We can see that only letter ‘C’ has appeared twice in Peacock. Therefore, we can say that ‘C’ is assigned number ‘9’. 

In word national ‘N’ has appeared twice. In code only digit ‘6’ has appeared more than once. Hence, we can say that code of letter N is ‘6’. Consequently, we can say that code for letter ‘D’ is ‘1’ because in India rest of the numerals are already taken. 

In words, ‘the’ and ‘national’ only letter ‘t’ is common. In code as well only digit ‘8’ is common in two codes. Hence, we can say that letter code for letter ‘t’ is 8. 

In words, ‘the’ and ‘peacock’ only letter ‘e’ is common. In code as well only digit ‘5’ is common in two codes. Hence, we can say that letter code for letter ‘e’ is 5. Consequently, we can say that leftover letter, in word “the”, ‘H’s code is 4. 

We can see that code for word “NATIONAL” is 13666689. Hence, we can say that code for the letter L is ‘1’. 

We can see that code for word “DESIGNATED” is 1135556678. Hence, we can say that code for the letter ‘G’ is ‘7’. 

We can see that code for word “PEACOCK” is 5688999. Hence, we can say that code for the letters ‘P’ and ‘K’ is ‘8’.

Let us check this by options: 

(A) S,E,Z: If letter ‘Z’ is assigned code ‘5’ then this case is possible. 

(B) I,B,M: If letters ‘B’ and ‘M’ are assigned code ‘3’ then this case is possible. 

(C) S,U,V: If letters ‘U’ and ‘V’ are assigned code ‘5’ then this case is possible. But in that case digit 5 will have 4 letters associated with it which is not possible. Hence, this is the answer. 

(D) X,Y,Z: If letters ‘X’, ‘Y’ and ‘Z’ are assigned code ‘2’ then this case is possible. 

 

Instructions 

Each of the 23 boxes in the picture below represents a product manufactured by one of the following three companies: Alfa, Bravo and Charlie. The area of a box is proportional to the revenue from the corresponding product, while its centre represents the Product popularity and Market potential scores of the product (out of 20). The shadings of some of the boxes have got erased. 

The companies classified their products into four categories based on a combination of scores (out of 20) on the two parameters – Product popularity and Market potential as given below: 

The following facts are known: 

1. Alfa and Bravo had the same number of products in the Blockbuster category. 

2. Charlie had more products than Bravo but fewer products than Alfa in the No-hope category. 

3. Each company had an equal number of products in the Promising category. 

4. Charlie did not have any product in the Doubtful category, while Alfa had one product more than Bravo in this category 

5. Bravo had a higher revenue than Alfa from products in the Doubtful category. 

6. Charlie had a higher revenue than Bravo from products in the Blockbuster category. 

7. Bravo and Charlie had the same revenue from products in the No-hope category. 

8. Alfa and Charlie had the same total revenue considering all products. 

Q. 55 Considering all companies’ products, which product category had the highest revenue? 

A. No-hope 

B. Blockbuster 

C. Doubtful 

D. Promising 

Answer: B. 

Explanation: 

Let us divide the given figure in four quadrants (Q1, Q2, Q3, Q4). 

Let us solve this problem by considering only one category at a time. 

(A) Blockbuster category: We have two information regarding Blockbuster category. 

1. Alfa and Bravo had the same number of products in the Blockbuster category. 

There are a total of 7 products in Blockbuster category. Alfa already has two products in blockbuster category. If Alfa has 3 products in blockbuster category then Bravo will also have 3 products in Blockbuster category which is not possible as there are 2 products of Charlie. Hence, we can say that Alfa and Bravo have 2 products each in Blockbuster category whereas Charlie has 3 products in Blockbuster category. 

2. It is also given that Charlie had a higher revenue than Bravo from products in the Blockbuster category. 

We know that one of the product 1 and 2 is from Charlie and the other is from Bravo. 

If product 1 is from Charlie, then we can say that products 1, 7 and 5 are from Charlie. Therefore, revenue generated by products in Charlie category = 2 + 4 + 6 = 12 units. (Assuming area of a square to be 1 unit) 

In this case product 2 and product n are from Bravo.Therefore, revenue generated by products in Bravo category = 6 + 9 = 15 units. 

We can see that products from Charlie have a higher revenue than Bravo. Hence, this case in not possible. 

Therefore, we can say that Product 1 is from Bravo and Product 2 is from Charlie. We have determined each product’s company name in Blockbuster category. 

(B) No-hope category: We have two information regarding No-hope category. 

(1) Charlie had more products than Bravo but fewer products than Alfa in the No-hope category. Bravo and Charlie had the same revenue from products in the No-hope category.There are a total of 6 products in no-hope category. 

Therefore, we can say that Alfa, Charlie and Bravo have 3, 2 and 1 products in No-hope category in that order.

Bravo and Charlie had the same revenue from products in the No-hope category. 

Revenue generated for Bravo in the No-hope category = 4 units. We need same revenue for Charlie which ha s 2 products in this category. Hence, we can say that Product 10 and one of product 8 and 9 is from Charlie and other is from Alfa. Let’s assume that product 8 is from Charlie and product 9 is from Alfa. 

(C) Doubtful category: We have two information regarding Doubtful category. 

(1). Charlie did not have any product in the Doubtful category, while Alfa had one product more than Bravo in this category . 

(2). Bravo had a higher revenue than Alfa from products in the Doubtful category. 

We can see that there are a total of 7 products in this category. Hence, we can say that 4 products are from Alfa and 3 products are from Bravo. 

We can see that one of product 14, 15 and 16 is from Bravo company and others are from Alfa company. Bravo will have higher revenue than Alfa only when product no. 14 is from Bravo and others (15 and 16) are from Alfa category. 

In this case total revenue by Bravo = Product 14 + Product 19 + Product 20 = 9 + 6 + 2 = 17 Similarly, total revenue by Charlie = Product 15 + Product 16 + Product 17 + Product 18 = 6 + 1 + 1 + 4 = 12 

(D) Promising category: We have only 1 direct information regarding Promising category. 1. Each company had an equal number of products in the Promising category. 

There are a total of 3 products in promising category with different revenue. Therefore, we can say that each company had 1 product in promising category. We are given that Alfa and Charlie had the same total revenue considering all products. We can calculate the revenue generated by Alfa and Charlie from the products in categories. 

Revenue generated by Charlie from all categories except Promising = From Blockbuster + From No-hope + From Doubtful 

⇒ (9+6+2) + (3+1) + (0) = 21 units 

Revenue generated by Alfa from all categories except Promising = From Blockbuster + From No-hope + From Doubtful ⇒ (6+3) + (4+2+1) + (1+6+4+1) = 28 units 

We can see the difference between revenue generated by Charlie and Alfa from remaining categories is 7 units. Hence, we can say that Charlie’s product’s revenue should be 7 units more than Alfa’s product’s revenue in Promising category. That is possible only in one case where product 22 is from Charlie and product 21 is form Alfa. Consequently, we can say that product 23 is from Bravo. Now we have identified each product’s company name we can answer all the questions. 

Revenue generated by the products in Promising category = 2 + 9 + 3 = 14 units. 

Revenue generated by the products in Doubtful category = 1 + 9 + 4 + 6 + 2 + 1 + 6 = 29 units. Revenue generated by the products in No-hope category = 4 + 4 + 3 + 2 + 1 + 1 = 15 units. Revenue generated by the products in Blockbuster category = 6 + 3 + 6 + 2 + 4 + 6 + 9 = 36 units. We can see that the revenue generated is the highest for the Blockbuster category. Hence, option B. is the correct answer. 

Q. 56 Which of the following is the correct sequence of numbers of products Bravo had in No-hope, Doubtful, Promising and Blockbuster categories respectively? 

A. 1,3,1,2 

B. 1,3,1,3 

C. 3,3,1,2 

D. 2,3,1,2 

Answer: A. 

Explanation: 

Let us divide the given figure in four quadrants (Q1, Q2, Q3, Q4). 

Let us solve this problem by considering only one category at a time. 

(A) Blockbuster category: We have two information regarding Blockbuster category. 

1. Alfa and Bravo had the same number of products in the Blockbuster category. 

There are a total of 7 products in Blockbuster category. Alfa already has two products in blockbuster category. If Alfa has 3 products in blockbuster category then Bravo will also have 3 products in Blockbuster category which is not possible as there are 2 products of Charlie. Hence, we can say that Alfa and Bravo have 2 products each in Blockbuster category whereas Charlie has 3 products in Blockbuster category. 

2. It is also given that Charlie had a higher revenue than Bravo from products in the Blockbuster category. 

We know that one of the product 1 and 2 is from Charlie and the other is from Bravo. 

If product 1 is from Charlie, then we can say that products 1, 7 and 5 are from Charlie. Therefore, revenue generated by products in Charlie category = 2 + 4 + 6 = 12 units. (Assuming area of a square to be 1 unit) 

In this case product 2 and product n are from Bravo.Therefore, revenue generated by products in Bravo category = 6 + 9 = 15 units. 

We can see that products from Charlie have a higher revenue than Bravo. Hence, this case in not possible. 

Therefore, we can say that Product 1 is from Bravo and Product 2 is from Charlie. We have determined each product’s company name in Blockbuster category. 

(B) No-hope category: We have two information regarding No-hope category. 

(1) Charlie had more products than Bravo but fewer products than Alfa in the No-hope category. Bravo and Charlie had the same revenue from products in the No-hope category.There are a total of 6 products in no-hope category. 

Therefore, we can say that Alfa, Charlie and Bravo have 3, 2 and 1 products in No-hope category in that order.

Bravo and Charlie had the same revenue from products in the No-hope category. 

Revenue generated for Bravo in the No-hope category = 4 units. We need same revenue for Charlie which ha s 2 products in this category. Hence, we can say that Product 10 and one of product 8 and 9 is from Charlie and other is from Alfa. Let’s assume that product 8 is from Charlie and product 9 is from Alfa. 

(C) Doubtful category: We have two information regarding Doubtful category. 

(1). Charlie did not have any product in the Doubtful category, while Alfa had one product more than Bravo in this category . 

(2). Bravo had a higher revenue than Alfa from products in the Doubtful category. 

We can see that there are a total of 7 products in this category. Hence, we can say that 4 products are from Alfa and 3 products are from Bravo. 

We can see that one of product 14, 15 and 16 is from Bravo company and others are from Alfa company. Bravo will have higher revenue than Alfa only when product no. 14 is from Bravo and others (15 and 16) are from Alfa category. 

In this case total revenue by Bravo = Product 14 + Product 19 + Product 20 = 9 + 6 + 2 = 17 Similarly, total revenue by Charlie = Product 15 + Product 16 + Product 17 + Product 18 = 6 + 1 + 1 + 4 = 12. 

(D) Promising category: We have only 1 direct information regarding Promising category. 1. Each company had an equal number of products in the Promising category. 

There are a total of 3 products in promising category with different revenue. Therefore, we can say that each company had 1 product in promising category. We are given that Alfa and Charlie had the same total revenue considering all products. We can calculate the revenue generated by Alfa and Charlie from the products in categories.

Revenue generated by Charlie from all categories except Promising = From Blockbuster + From No-hope + From Doubtful 

⇒ (9+6+2) + (3+1) + (0) = 21 units 

Revenue generated by Alfa from all categories except Promising = From Blockbuster + From No-hope + From Doubtful 

⇒ (6+3) + (4+2+1) + (1+6+4+1) = 28 units 

We can see the difference between revenue generated by Charlie and Alfa from remaining categories is 7 units. Hence, we can say that Charlie’s product’s revenue should be 7 units more than Alfa’s product’s revenue in Promising category. That is possible only in one case where product 22 is from Charlie and product 21 is form Alfa. Consequently, we can say that product 23 is from Bravo. Now we have identified each product’s company name we can answer all the questions.

Bravo had in No-hope, Doubtful, Promising and Blockbuster categories respectively = 1, 3, 1, 2. Hence, option A. is the correct answer. 

Q. 57 Which of the following statements is NOT correct? 

A. Alfa’s revenue from Blockbuster products was the same as Charlie’s revenue from Promising products 

B. Bravo’s revenue from Blockbuster products was greater than Alfa’s revenue from Doubtful products 

C. Bravo and Charlie had the same revenues from No-hope products 

D. The total revenue from No-hope products was less than the total revenue from Doubtful products 

Answer: B. 

Explanation: 

Let us divide the given figure in four quadrants (Q1, Q2, Q3, Q4). 

Let us solve this problem by considering only one category at a time. 

(A) Blockbuster category: We have two information regarding Blockbuster category. 

1. Alfa and Bravo had the same number of products in the Blockbuster category. 

There are a total of 7 products in Blockbuster category. Alfa already has two products in blockbuster category. If Alfa has 3 products in blockbuster category then Bravo will also have 3 products in Blockbuster category which is not possible as there are 2 products of Charlie. Hence, we can say that Alfa and Bravo have 2 products each in Blockbuster category whereas Charlie has 3 products in Blockbuster category. 

2. It is also given that Charlie had a higher revenue than Bravo from products in the Blockbuster category.

We know that one of the product 1 and 2 is from Charlie and the other is from Bravo. 

If product 1 is from Charlie, then we can say that products 1, 7 and 5 are from Charlie. Therefore, revenue generated by products in Charlie category = 2 + 4 + 6 = 12 units. (Assuming area of a square to be 1 unit) 

In this case product 2 and product n are from Bravo.Therefore, revenue generated by products in Bravo category = 6 + 9 = 15 units. 

We can see that products from Charlie have a higher revenue than Bravo. Hence, this case in not possible. 

Therefore, we can say that Product 1 is from Bravo and Product 2 is from Charlie. We have determined each product’s company name in Blockbuster category. 

(B) No-hope category: We have two information regarding No-hope category. 

(1) Charlie had more products than Bravo but fewer products than Alfa in the No-hope category. Bravo and Charlie had the same revenue from products in the No-hope category.There are a total of 6 products in no-hope category. 

Therefore, we can say that Alfa, Charlie and Bravo have 3, 2 and 1 products in No-hope category in that order.

Bravo and Charlie had the same revenue from products in the No-hope category. 

Revenue generated for Bravo in the No-hope category = 4 units. We need same revenue for Charlie which ha s 2 products in this category. Hence, we can say that Product 10 and one of product 8 and 9 is from Charlie and other is from Alfa. Let’s assume that product 8 is from Charlie and product 9 is from Alfa. 

(C) Doubtful category: We have two information regarding Doubtful category. 

(1). Charlie did not have any product in the Doubtful category, while Alfa had one product more than Bravo in this category . 

(2). Bravo had a higher revenue than Alfa from products in the Doubtful category. 

We can see that there are a total of 7 products in this category. Hence, we can say that 4 products are from Alfa and 3 products are from Bravo. 

We can see that one of product 14, 15 and 16 is from Bravo company and others are from Alfa company. Bravo will have higher revenue than Alfa only when product no. 14 is from Bravo and others (15 and 16) are from Alfa category. 

In this case total revenue by Bravo = Product 14 + Product 19 + Product 20 = 9 + 6 + 2 = 17 Similarly, total revenue by Charlie = Product 15 + Product 16 + Product 17 + Product 18 = 6 + 1 + 1 + 4 = 12 

(D) Promising category: We have only 1 direct information regarding Promising category. 1. Each company had an equal number of products in the Promising category. 

There are a total of 3 products in promising category with different revenue. Therefore, we can say that each company had 1 product in promising category. We are given that Alfa and Charlie had the same total revenue considering all products. We can calculate the revenue generated by Alfa and Charlie from the products in categories. 

Revenue generated by Charlie from all categories except Promising = From Blockbuster + From No-hope + From Doubtful 

⇒ (9+6+2) + (3+1) + (0) = 21 units 

Revenue generated by Alfa from all categories except Promising = From Blockbuster + From No-hope + From Doubtful 

⇒ (6+3) + (4+2+1) + (1+6+4+1) = 28 units 

We can see the difference between revenue generated by Charlie and Alfa from remaining categories is 7 units. Hence, we can say that Charlie’s product’s revenue should be 7 units more than Alfa’s product’s revenue in Promising category. That is possible only in one case where product 22 is from Charlie and product 21 is form Alfa. Consequently, we can say that product 23 is from Bravo. Now we have identified each product’s company name we can answer all the questions. 

Let us check all options. 

Option (A): Alfa’s revenue from Blockbuster products was the same as Charlie’s revenue from Promising products Alfa’s revenue from Blockbuster products = 6 + 3 = 9 units. 

Charlie’s revenue from Promising products = 9 units. Hence, this statement is true

Option (B): Bravo’s revenue from Blockbuster products was greater than Alfa’s revenue from Doubtful products. Bravo’s revenue from Blockbuster products = 6 + 4 = 10 units. 

Alfa’s revenue from Doubtful products = 6 + 4 +1 + 1 = 12 units. Hence, this statement is false. Option (C): Bravo and Charlie had the same revenues from No-hope products. 

Bravo’s revenue from No-hope products = 4 units. 

Charlie’s revenue from No-hope products = 3 + 1 = 4 units. Hence, this statement is true. Option (D): The total revenue from No-hope products was less than the total revenue from Doubtful products Revenue generated by the products in Doubtful category = 1 + 9 + 4 + 6 + 2 + 1 + 6 = 29 units. 

Revenue generated by the products in No-hope category = 4 + 4 + 3 + 2 + 1 + 1 = 15 units. Hence, this statement is true

We can see that statement mentioned in option B. is false. Therefore, option B. is the correct answer. 

Q. 58 If the smallest box on the grid is equivalent to revenue of Rs.1 crore, then what approximately was the total revenue of Bravo in Rs. crore? 

A. 40 

B. 24 

C. 34 

D. 30 

Answer: C. 

Explanation: 

Let us divide the given figure in four quadrants (Q1, Q2, Q3, Q4). 

Let us solve this problem by considering only one category at a time. 

(A) Blockbuster category: We have two information regarding Blockbuster category. 

1. Alfa and Bravo had the same number of products in the Blockbuster category. 

There are a total of 7 products in Blockbuster category. Alfa already has two products in blockbuster category. If Alfa has 3 products in blockbuster category then Bravo will also have 3 products in Blockbuster category which is not possible as there are 2 products of Charlie. Hence, we can say that Alfa and Bravo have 2 products each in Blockbuster category whereas Charlie has 3 products in Blockbuster category. 

2. It is also given that Charlie had a higher revenue than Bravo from products in the Blockbuster category.

We know that one of the product 1 and 2 is from Charlie and the other is from Bravo. 

If product 1 is from Charlie, then we can say that products 1, 7 and 5 are from Charlie. Therefore, revenue generated by products in Charlie category = 2 + 4 + 6 = 12 units. (Assuming area of a square to be 1 unit) 

In this case product 2 and product n are from Bravo.Therefore, revenue generated by products in Bravo category = 6 + 9 = 15 units. 

We can see that products from Charlie have a higher revenue than Bravo. Hence, this case in not possible. 

Therefore, we can say that Product 1 is from Bravo and Product 2 is from Charlie. We have determined each product’s company name in Blockbuster category. 

(B) No-hope category: We have two information regarding No-hope category. 

(1) Charlie had more products than Bravo but fewer products than Alfa in the No-hope category. Bravo and Charlie had the same revenue from products in the No-hope category.There are a total of 6 products in no-hope category. 

Therefore, we can say that Alfa, Charlie and Bravo have 3, 2 and 1 products in No-hope category in that order.

Bravo and Charlie had the same revenue from products in the No-hope category. 

Revenue generated for Bravo in the No-hope category = 4 units. We need same revenue for Charlie which ha s 2 products in this category. Hence, we can say that Product 10 and one of product 8 and 9 is from Charlie and other is from Alfa. Let’s assume that product 8 is from Charlie and product 9 is from Alfa. 

(C) Doubtful category: We have two information regarding Doubtful category. 

(1). Charlie did not have any product in the Doubtful category, while Alfa had one product more than Bravo in this category . 

(2). Bravo had a higher revenue than Alfa from products in the Doubtful category. 

We can see that there are a total of 7 products in this category. Hence, we can say that 4 products are from Alfa and 3 products are from Bravo. 

We can see that one of product 14, 15 and 16 is from Bravo company and others are from Alfa company. Bravo will have higher revenue than Alfa only when product no. 14 is from Bravo and others (15 and 16) are from Alfa category. 

In this case total revenue by Bravo = Product 14 + Product 19 + Product 20 = 9 + 6 + 2 = 17 Similarly, total revenue by Charlie = Product 15 + Product 16 + Product 17 + Product 18 = 6 + 1 + 1 + 4 = 12 

(D) Promising category: We have only 1 direct information regarding Promising category. 1. Each company had an equal number of products in the Promising category. 

There are a total of 3 products in promising category with different revenue. Therefore, we can say that each company had 1 product in promising category. We are given that Alfa and Charlie had the same total revenue considering all products. We can calculate the revenue generated by Alfa and Charlie from the products in categories. 

Revenue generated by Charlie from all categories except Promising = From Blockbuster + From No-hope + From Doubtful 

⇒ (9+6+2) + (3+1) + (0) = 21 units 

Revenue generated by Alfa from all categories except Promising = From Blockbuster + From No-hope + From Doubtful ⇒ 

(6+3) + (4+2+1) + (1+6+4+1) = 28 units 

We can see the difference between revenue generated by Charlie and Alfa from remaining categories is 7 units. Hence, we can say that Charlie’s product’s revenue should be 7 units more than Alfa’s product’s revenue in Promising category. That is possible only in one case where product 22 is from Charlie and product 21 is form Alfa. Consequently, we can say that product 23 is from Bravo. Now we have identified each product’s company name we can answer all the questions. 

Total revenue generated by Bravo products alone = From Blockbuster + From No-hope + From Doubtful + From Promising 

⇒ (6+4) + (4) + (9+6+2) + (3) = 34 units 

One box is equivalent to Rs. 1 crore therefore, we can say that total revenue generated by Bravo = Rs. 34 crores. Hence, option C. is the correct answer. 

Instructions 

Seven candidates, Akil, Balaram, Chitra, Divya, Erina, Fatima, and Ganeshan, were invited to interview for a position. Candidates were required to reach the venue before 8 am. Immediately upon arrival, they were sent to one of three interview rooms: 101, 102, and 103. The following venue log shows the arrival times for these candidates. Some of the names have not been recorded in the log and have been marked as ‘?’. 

Additionally here are some statements from the candidates: 

Balaram: I was the third person to enter Room 101. 

Chitra: I was the last person to enter the room I was allotted to. 

Erina: I was the only person in the room I was allotted to. 

Fatima: Three people including Akhil were already in the room that I was allotted to when I entered it. Ganeshan : I was one among the two candidates allotted to Room 102. 

Q. 59 What best can be said about the room to which Divya was allotted? 

A. Definitely Room 101 

B. Definitely Room 103 

C. Definitely Room 102 

D. Either Room 101 or Room 102 

Answer: A. 

Explanation: 

It is given that there were a total of 3 rooms and seven candidates. Ganeshan said that he was one among the two candidates allotted to Room 102 whereas Erina said that she is the only person in the room she was allotted to. Therefore, we can say that there were 1 and 4 candidates in either 101 or 103 room. But it is given that Balaram was the third person to enter in the Room 101 therefore we can say that there were 4 candidates in Room 101 and only 1 candidate in room 103. 

Fatima said that three people including Akhil were already in the room that I was allotted to when I entered it. Hence, we can say that Fatima was the last person to enter in 101 and Akhil is the first person who entered in room 101. 

Chitra said that she was the last person to enter the room she was allotted to. Hence, we can say that Chitra was allotted room no 102 and she entered after Ganeshan. 

Erina was the only person in room no 103. 

Balaram said he was third to enter room no 101. Hence, we can say that Divya was second person who entered in room 101. 

Since Chitra and Fatima were already in by 7:40 AM we can say that the candidate who entered at 7:45 am is Erina.

From the table we can see that Divya was allotted room no 101. Hence, option A. is the correct answer. 

Q. 60 Who else was in Room 102 when Ganeshan entered? 

A. Akil 

B. Divya 

C. Chitra 

D. No one 

Answer: D. 

Explanation: 

It is given that there were a total of 3 rooms and seven candidates. Ganeshan said that he was one among the two candidates allotted to Room 102 whereas Erina said that she is the only person in the room she was allotted to. Therefore, we can say that there were 1 and 4 candidates in either 101 or 103 room. But it is given that Balaram was the third person to enter in the Room 101 therefore we can say that there were 4 candidates in Room 101 and only 1 candidate in room 103. 

Fatima said that three people including Akhil were already in the room that I was allotted to when I entered it. Hence, we can say that Fatima was the last person to enter in 101 and Akhil is the first person who entered in room 101. 

Chitra said that she was the last person to enter the room she was allotted to. Hence, we can say that Chitra was allotted room no 102 and she entered after Ganeshan. 

Erina was the only person in room no 103. 

Balaram said he was third to enter room no 101. Hence, we can say that Divya was second person who entered in room 101. 

Since Chitra and Fatima were already in by 7:40 AM we can say that the candidate who entered at 7:45 am is Erina.

From the table we can see that Ganeshan is the first person to enter in room 102.Hence, option D. is the correct answer. 

Q. 61 When did Erina reach the venue? 

A. 7:45 am 

B. 7:25 am 

C. 7:15 am 

D. 7:10 am 

Answer: A. 

Explanation: 

It is given that there were a total of 3 rooms and seven candidates. Ganeshan said that he was one among the two candidates allotted to Room 102 whereas Erina said that she is the only person in the room she was allotted to. Therefore, we can say that there were 1 and 4 candidates in either 101 or 103 room. But it is given that Balaram was the third person to enter in the Room 101 therefore we can say that there were 4 candidates in Room 101 and only 1 candidate in room 103. 

Fatima said that three people including Akhil were already in the room that I was allotted to when I entered it. Hence, we can say that Fatima was the last person to enter in 101 and Akhil is the first person who entered in room 101. 

Chitra said that she was the last person to enter the room she was allotted to. Hence, we can say that Chitra was allotted room no 102 and she entered after Ganeshan. 

Erina was the only person in room no 103. 

Balaram said he was third to enter room no 101. Hence, we can say that Divya was second person who entered in room 101. 

Since Chitra and Fatima were already in by 7:40 AM we can say that the candidate who entered at 7:45 am is Erina.

From the table we can see that Erina reached the venue at 7:45 am. Hence, option A. is the correct answer. 

Q. 62 If Ganeshan entered the venue before Divya, when did Balaram enter the venue? 

A. 7:25 am 

B. 7:10 am 

C. 7:15 am 

D. 7:45 am 

Answer: A. 

Explanation: 

It is given that there were a total of 3 rooms and seven candidates. Ganeshan said that he was one among the two candidates allotted to Room 102 whereas Erina said that she is the only person in the room she was allotted to. Therefore, we can say that there were 1 and 4 candidates in either 101 or 103 room. But it is given that Balaram was the third person to enter in the Room 101 therefore we can say that there were 4 candidates in Room 101 and only 1 candidate in room 103. 

Fatima said that three people including Akhil were already in the room that I was allotted to when I entered it. Hence, we can say that Fatima was the last person to enter in 101 and Akhil is the first person who entered in room 101. 

Chitra said that she was the last person to enter the room she was allotted to. Hence, we can say that Chitra was allotted room no 102 and she entered after Ganeshan. 

Erina was the only person in room no 103. 

Balaram said he was third to enter room no 101. Hence, we can say that Divya was second person who entered in room 101. 

Since Chitra and Fatima were already in by 7:40 AM we can say that the candidate who entered at 7:45 am is Erina. 

In the question it is given that Ganeshan entered the venue before Divya. Therefore, we can say that Ganesh must have entered with Akhil at 7:10 am. In that case, Divya and Balaram must have entered at 7:15 am and 7:25 am respectively. Hence, option A. is the correct answer. 

Instructions 

There are only four brands of entry level smartphones called Azra, Bysi, Cxqi, and Dipq in a country. Details about their market share, unit selling price, and profitability (defined as the profit as a percentage of the revenue) for the year 2016 are given in the table below: 

In 2017, sales volume of entry level smartphones grew by 40% as compared to that in 2016. Cxqi offered a 40% discount on its unit selling price in 2017, which resulted in a 15% increase in its market share. Each of the other three brands lost 5% market share. However, the profitability of Cxqi came down to half of its value in 2016. The unit selling prices of the other three brands and their profitability values remained the same in 2017 as they were in 2016. 

Q. 63 The brand that had the highest revenue in 2016 is: 

A. Cxqi 

B. Bysi 

C. Azra 

D. Dipq 

Answer: C. 

Explanation: 

Let ‘100x’ be the number of smartphones sold in year 2016. 

Total revenue generated by Azra = 40x*15000 = Rs. 600000x 

Total revenue generated by Bysi = 25x*20000 = Rs. 500000x 

Total revenue generated by Cxqi = 15x*30000 = Rs. 450000x 

Total revenue generated by Dipq = 20x*25000 = Rs. 500000x 

We can see that revenue generated by Azra is the highest among all four brands. Hence, option C. is the correct answer.

Q. 64 The brand that had the highest profit in 2016 is: 

A. Bysi 

B. Dipq 

C. Cxqi 

D. Azra 

Answer: C. 

Explanation: 

Let ‘100x’ be the number of smartphones sold in year 2016. 

Total revenue generated by Azra = 40x*15000 = Rs. 600000x

Profitability is defined as the profit as a percentage of the revenue. Therefore, profit generated by Azra = 10/100*600000x = Rs. 60000x 

Total revenue generated by Bysi = 25x*20000 = Rs. 500000x 

Profit generated by Bysi = 30/100*500000x = Rs. 150000x 

Total revenue generated by Cxqi = 15x*30000 = Rs. 450000x 

Profit generated by Cxqi =40/100*450000x = Rs. 180000x 

Total revenue generated by Dipq = 20x*25000 = Rs. 500000x 

Profit generated by Dipq =30/100*500000x = Rs. 150000x 

We can see that profit generated by Cxqi is the highest among all four brands. Hence, option C. is the correct answer. 

Q. 65 The brand that had the highest profit in 2017 is: 

A. Bysi 

B. Azra 

C. Cxqi 

D. Dipq 

Answer: A. 

Explanation: 

Let ‘100x’ be the number of smartphones sold in year 2016. Then the number of smartphones sold in 2017 = 1.4*100x = 140x 

It is given that Cxqi offered a 40% discount on its unit selling price in 2017 i.e. selling price in 2017 = 0.6*30000 = Rs. 18000 

Also Cxqi’s merket share increased by 15% whereas the other three brands lost 5% market share.

Amount of profit generated by Azra =10 /100 *15000*49x = 73500x 

Amount of profit generated by Bysi =30/100  *20000*28x = 168000x 

Amount of profit generated by Cxqi =20/100 *18000*18x = 64800x 

Amount of profit generated by Dipq =30/100 *25000*21x = 157500x 

We can see that brand Bysi generated maximum profit in 2017. Hence, option A. is the correct answer. 

Q. 66 The complete list of brands whose profits went up in 2017 from 2016 is: 

A. Azra, Bysi, Dipq 

B. Cxqi, Azra, Dipq 

C. Azra, Bysi, Cxqi 

D. Bysi, Cxqi, Dipq 

Answer: A. 

Explanation: 

Let ‘100x’ be the number of smartphones sold in year 2016. 

Total revenue generated by Azra = 40x*15000 = Rs. 600000x 

Profitability is defined as the profit as a percentage of the revenue. Therefore, profit generated by Azra =10/100 *600000x = Rs. 60000x 

Total revenue generated by Bysi = 25x*20000 = Rs. 500000x 

Profit generated by Bysi =30/100 *500000x = Rs. 150000x 

Total revenue generated by Cxqi = 15x*30000 = Rs. 450000x 

Profit generated by Cxqi =40/100 *450000x = Rs. 180000x 

Total revenue generated by Dipq = 20x*25000 = Rs. 500000x 

Profit generated by Dipq =30/100 *500000x = Rs. 150000x 

It is given that the market sales increased by 40%. Therefore, the number of smartphones sold in 2017 = 1.4*100x = 140x 

It is given that Cxqi offered a 40% discount on its unit selling price in 2017 i.e. selling price in 2017 = 0.6*30000 = Rs. 18000 

Also Cxqi’s merket share increased by 15% whereas the other three brands lost 5% market share.

Amount of profit generated by Azra =10/100 *15000*49x = 73500x 

Amount of profit generated by Bysi =30/100 *20000*28x = 168000x 

Amount of profit generated by Cxqi =20/100 *18000*18x = 64800x 

Amount of profit generated by Dipq =30/100 *25000*21x = 157500x 

We can see that profit of brands Azra, Bysi and Dipq increased in the year 2017 as compared to 2016. Hence, option A. is the correct answer. 

 

Quantitative Aptitude 

Instructions 

For the following questions answer them individually 

Q. 67 A water tank has inlets of two types A and B. All inlets of type A when open, bring in water at the same rate. All inlets of type B, when open, bring in water at the same rate. The empty tank is completely filled in 30 minutes if 10 inlets of type A and 45 inlets of type B are open, and in 1 hour if 8 inlets of type A and 18 inlets of type B are open. In how many minutes will the empty tank get completely filled if 7 inlets of type A and 27 inlets of type B are open? 

Answer:48 

Explanation: 

Let the efficiency of type A. pipe be ‘a’ and the efficiency of type B. be ‘b’. 

In the first case, 10 type A. and 45 type B. pipes fill the tank in 30 mins. 

So, the capacity of the tank = ½ (10a + 45b)……..(i) 

In the second case, 8 type A. and 18 type B. pipes fill the tank in 1 hour. 

So, the capacity of the tank = (8a + 18b)……….(ii) 

Equating (i) and (ii), we get 

10a + 45b = 16a + 36b 

=>6a = 9b 

From (ii), capacity of the tank = (8a + 18b) = (8a + 12a) = 20a 

In the third case, 7 type A. and 27 type B. pipes fill the tank. 

Net efficiency = (7a + 27b) = (7a + 18a) = 25a 

Time taken =20a/25a hour = 48 minutes. 

Hence, 48 is the correct answer. 

Q. 68 Let f(x)= max(5x, 52 − 2x )2, where x is any positive real number. Then the minimum possible value of f(x) 

Answer:20 

Explanation: 

The minimum value of the function will occur when the expressions inside the function are equal. 

So, 5x = 52 − 2x2

or, 2x2 + 5x − 52= 0 

x − 13/2 

On solving, we get x= 4 or 

But, it is given that is a positive number. 

So, x= 4 

And the minimum value = 5*4 = 20 

Hence, 20 is the correct answer. 

Q. 69 Points A, P, Q and B. lie on the same line such that P, Q and B. are, respectively, 100 km, 200 km and 300 km away from A. Cars 1 and 2 leave A at the same time and move towards B Simultaneously, car 3 leaves B. and moves towards A Car 3 meets car 1 at Q, and car 2 at P. If each car is moving in uniform speed then the ratio of the speed of car 2 to that of car 1 is 

A. 2 : 7 

B. 2 : 9 

C. 1 : 2 

D. 1 : 4 

Answer: D. 

Explanation: 

Car 3 meets car 1 at Q, which is 200 km from A. 

Therefore, at the time of their meeting car 1 must have travelled 200 km and car 3 must have travelled 100 km. As the time is same, ratio of speed of car 1 to speed of car 3 = 2 : 1. 

Car 3 meets car 2 at P, which is 100 km from A. 

Therefore, at the time of their meeting car 2 must have travelled 100 km and car 3 must have travelled 200 km. As the time is same, ratio of speed of car 2 to speed of car 3 = 1 : 2. 

Speed of car 1 : speed of car 3 = 2 : 1 

And speed of car 2 : speed of car 3 = 1 : 2 

So, speed of car 1 : speed of car 2 : speed of car 3 = 4 : 1 : 2 

Hence, option D. is the correct answer. 

Q. 70 The smallest integer n  such that n3 − 11n2 + 32n − 28 > 0 is

Answer:

Explanation: 

We can see that at n = 2, n3 − 11n2 + 32n − 28 = 0 i.e. (n-2) is a factor of n3 − 11n2 + 32n − 28

(n3 − 11n2 + 32n − 28)/(n − 2) = n2 − 9n + 14

We can further factorize n^2-9n+14 as (n-2)(n-7). 

n3 − 11n2 + 32n − 28 = (n − 2)2(n − 7) 

n3 − 11n2 + 32n − 28 > 0 

⇒ (n − 2)2(n − 7) > 0 

Therefore, we can say that n-7>0 

Hence, nmin = 8 

Q. 71 The scores of Amal and Bimal in an examination are in the ratio 11 : 14. After an appeal, their scores increase by the same amount and their new scores are in the ratio 47 : 56. The ratio of Bimal’s new score to that of his original score is 

A. 4 : 3 

B. 8 : 5 

C. 5 : 4 

D. 3 : 2 

Answer: A. 

Explanation: 

Let the score of Amal and Bimal be 11k and 14k 

Let the scores be increased by x 

So, after increment, Amal’s score = 11k + x and Bimal’s score = 14k + x 

According to the question, 

(11k + x )/(14k + x) = 47/56

On solving, we get x =42/9 k 

Ratio of Bimal’s new score to his original score 

= (14k + x)/14k

= [14k + 42/9k ] /14

=168k / 14*9k

= 4/3 

Hence, option A. is the correct answer. 

Q. 72 How many two-digit numbers, with a non-zero digit in the units place, are there which are more than thrice the number formed by interchanging the positions of its digits? 

Answer:

Explanation: 

Let ‘ab’ be the two digit number. Where b≠0. 

We will get number ‘ba’ after interchanging its digit. 

It is given that 10a+b > 3*(10b + a) 

7a > 29b 

If b = 1, then a = {5, 6, 7, 8, 9} 

If b = 2, then a = {9} 

If b = 3, then no value of ‘a’ is possible. Hence, we can say that there are a total of 6 such numbers. 

Q. 73 For two sets A. and B, let AΔB. denote the set of elements which belong to A. or B. but not both. If P = {1,2,3,4}, Q = {2,3,5,6,}, R = {1,3,7,8,9}, S = {2,4,9,10}, then the number of elements in (PΔQ)Δ(RΔS) is 

Answer:

Explanation: 

P = {1,2,3,4} and Q = {2,3,5,6,} 

PΔQ = {1, 4, 5, 6} 

R = {1,3,7,8,9} and S = {2,4,9,10} 

RΔS = {1, 2, 3, 4, 7, 8, 10} 

(PΔQ)Δ(RΔS) = {2, 3, 5, 6, 7, 8, 10} 

Thus, there are 7 elements in (PΔQ)Δ(RΔS) . 

hence, 7 is the correct answer. 

Q. 74 A parallelogram ABCD has area 48 sqcm. If the length of CD is 8 cm and that of AD. is s cm, then which one of the following is necessarily true? 

A. s ≠ 6

B. s ≥ 6

C. 5 ≤ s ≤ 7

D. s ≤ 6

Answer: B. 

Explanation: 

We can see that area of parallelogram ABCD = 2*Area of triangle ACD 

48 = 2*Area of triangle ACD 

Area of triangle ACD = 24 

(1/2) ∗ CD DA sinADC = 24 

AD sinADC = 6 

We know that sinθ ≤1, Hence, we can say that AD ≥ 6 

⇒s ≥ 6

Q. 75 A 20% ethanol solution is mixed with another ethanol solution, say, S of unknown concentration in the proportion 1:3 by volume. This mixture is then mixed with an equal volume of 20% ethanol solution. If the resultant mixture is a 31.25% ethanol solution, then the unknown concentration of S is 

Answer:50 

Explanation: 

Let the volume of the first and the second solution be 100 and 300. 

When they are mixed, quantity of ethanol in the mixture 

= (20 + 300S) 

Let this solution be mixed with equal volume i.e. 400 of third solution in which the strength of ethanol is 20%. So, the quantity of ethanol in the final solution 

= (20 + 300S + 80) = (300S + 100) 

It is given that, 31.25% of 800 = (300S + 100) 

or, 300S + 100 = 250 

or S =½  = 50% 

Hence, 50 is the correct answer. 

Q. 76 In a tournament, there are 43 junior level and 51 senior level participants. Each pair of juniors play one match. Each pair of seniors play one match. There is no junior versus senior match. The number of girl versus girl matches in junior level is 153, while the number of boy versus boy matches in senior level is 276. The number of matches a boy plays against a girl is 

Answer:1098 

Explanation: 

In a tournament, there are 43 junior level and 51 senior level participants. 

Let ‘n’ be the number of girls on junior level. It is given that the number of girl versus girl matches in junior level is 153. 

⇒ nC2 = 153 

⇒ n(n-1)/2 = 153 

⇒ n2(n-1) = 306 

=> n -n-306 = 0 

=> (n+17)(n-18)=0 

=> n=18 (rejecting n=-17) 

Therefore, number of boys on junior level = 43 – 18 = 25. 

Let ‘m’ be the number of boys on senior level. It is given that the number of boy versus boy matches in senior level is 276. 

⇒ mC2 = 276 

⇒ m = 24 

Therefore, number of girls on senior level = 51 – 24 = 27. 

Hence, the number of matches a boy plays against a girl = 18*25+24*27 = 1098 

Q. 77 A chord of length 5 cm subtends an angle of 60° at the centre of a circle. The length, in cm, of a chord that subtends an angle of 120° at the centre of the same circle is 

A. 5√3

B. 2π

C. 8

D. 6√2

Answer: A. 

Explanation: 

We are given that AB. = 5 cm and ∠AOB. = 60° 

Let us draw OM such that OM⊥AB. 

In right angle triangle AMO, 

sin30° = AM / AO 

⇒ AO = 2*AM = 2*2.5 = 5 cm. Therefore, we can say that the radius of the circle = 5 cm.

In right angle triangle PNO, 

sin60° = PN / PO 

⇒PN =√3/2 *PO = 5√3 /2

Therefore, PQ = 2*PN =5√3cm

Q. 78 Let a1, a2a52 be positive integers such that a1< a2< … <a52 . Suppose, their arithmetic mean is one less than arithmetic mean of a2, a3a52, , …. . If a52= 100, then the largest possible value of a1 is 

A. 48 

B. 20 

C. 23 

D. 45 

Answer: C. 

Explanation: 

Let ‘x’ be the average of all 52 positive integers a1, a2a52

a1 + a2 + a3 + … + a52 = 52x … (1) 

Therefore, average of a2, a3a52 = x+1 

a1 + a2 + a3 + … + a52 = = 51(x+1) … (2) 

From equation (1) and (2), we can say that 

a1 + 51(x + 1) = 52x

a1 = x – 51. 

We have to find out the largest possible value of a1. a1 will be maximum when ‘x’ is maximum. a2 a3 a52 a2 a3 a52 a52 

(x+1) is the average of terms a1, a2a52. We know that a2< a3< … <a52 and a52= 100. Therefore, (x+1) will be maximum when each term is maximum possible. If a52= 100, then a52= 99, a52= 98 ends so on.

a2 = 100 + (51-1)*(-1) = 50.

Hence, a2 + a3 + a4 + … + a52= 50+51+…+99+100 = 51(x+1) 

⇒(51 ∗ (50 + 100))/ 2 = 51(x + 1)

x = 74 

Therefore, the largest possible value of a1 = x – 51 = 74 – 51 = 23. 

Q. 79 The value of the sum 7 x 11 + 11 x 15 + 15 x 19 + …+ 95 x 99 is Answer:80707 

Explanation: 

S = 7 x 11 + 11 x 15 + 15 x 19 + …+ 95 x 99 

Nth term of the series can be written as Tn = (4n + 3) ∗ (4n + 7)

Last term, (4n+3) = 95 i.e. n = 23 

n=232n=0 (4n + 3) ∗ (4n + 7)

⇒ ∑n=232n=0 16n + 40n + 21

⇒ 16 ∗(23 ∗ 24 ∗ 47 )/ 6 + 40 ∗ (23 ∗ 24)/2 + 21 ∗ 23

⇒ 80707 

Q. 80 If N and x are positive integers such that NN = 2160 and N2 + 2N is an integral multiple of 2x, then the largest possible x is 

Answer:10 

Explanation: 

It is given that NN = 2160 

We can rewrite the equation as NN =(2 )5 160/5 = 3232 

⇒ N = 32 

N2 + 2N 322 + 232 = 210 + 232 = 210 ∗ (1 + 222

Hence, we can say that N2 + 2N can be divided by 210

Therefore, xmax = 10 

Q. 81 A tank is emptied everyday at a fixed time point. Immediately thereafter, either pump A or pump B or both start working until the tank is full. On Monday, A alone completed filling the tank at 8 pm. On Tuesday, B alone completed filling the tank at 6 pm. On Wednesday, A alone worked till 5 pm, and then B worked alone from 5 pm to 7 pm, to fill the tank. At what time was the tank filled on Thursday if both pumps were used simultaneously all along? 

A. 4:48 pm 

B. 4:12 pm 

C. 4:24 pm 

D. 4:36 pm 

Answer: C. 

Explanation: 

Let ‘t’ pm be the time when the tank is emptied everyday. Let ‘a’ and ‘b’ be the liters/hr filled by pump A. and pump B. respectively. 

On Monday, A. alone completed filling the tank at 8 pm. Therefore, we can say that pump A. worked for (8 – t) hours. Hence, the volume of the tank = a*(8 – t) liters. 

Similarly, on Tuesday, B. alone completed filling the tank at 6 pm. Therefore, we can say that pump B. worked for (6 – t) hours. Hence, the volume of the tank = b*(6 – t) liters. 

On Wednesday, A. alone worked till 5 pm, and then B. worked alone from 5 pm to 7 pm, to fill the tank. Therefore, we can say that pump A. worked for (5 – t) hours and pump B. worked for 2 hours. Hence, the volume of the tank = a*(5 – t)+2b liters. 

We can say that a*(8 – t) = b*(6 – t) = a*(5 – t) + 2b 

a*(8 – t) = a*(5 – t) + 2b 

⇒ 3a = 2b … (1) 

a*(8 – t) = b*(6 – t) 

Using equation (1), we can say that 

a ∗ (8 − t) = 3a / 2 ∗ (6 − t

t = 2 

Therefore, we can say that the tank gets emptied at 2 pm daily. We can see that A. takes 6 hours and pump B. takes 4 hours alone. 

Hence, working together both can fill the tank in = \dfrac{6*4}{6+4} = 2.4 hours or 2 hours and 24 minutes. The pumps started filling the tank at 2:00 pm. Hence, the tank will be filled by 4:24 pm. 

Q. 82 The arithmetic mean of x, y and z is 80, and that of x, y, z, u and v is 75, where u=(x+y)/2 and v=(y+z)/2. If x ≥ z, then the minimum possible value of x is 

Answer:105 

Explanation: 

Given that the arithmetic mean of x, y and z is 80. 

(x + y + z )/3=80

x + y + z = 240 … (1) 

Also, 

(x + y + z + v + u)/ 5 = 7

x + y + z + v + u = 375 

Substituting values from equation (1), 

v + u = 135 

It is given that u=(x+y)/2 and v=(y+z)/2. 

⇒ (x + y)/2 + (y + z)/2 = 135 

x + 2y + z = 270 

y = 30 x + y + z = 240 

(Since x + z = 240 − y = 210) 

Therefore, we can say that . We are also given that x ≥ z, 

Hence, xmin = 210/2 = 105. 

Q. 83 Points A and B are 150 km apart. Cars 1 and 2 travel from A to B, but car 2 starts from A when car 1 is already 20 km away from A Each car travels at a speed of 100 kmph for the first 50 km, at 50 kmph for the next 50 km, and at 25 kmph for the last 50 km. The distance, in km, between car 2 and B. when car 1 reaches B is 

Answer:

Explanation: 

Time taken to cover first 50 km at 100 km/hr = ½ hr. 

Time taken to cover second 50 km at 50 km/hr = 1 hr. 

Time taken to cover last 50 km at 25 km/hr = 2 hr. 

When car 2 starts, car 1 has already covered 20 km. 

So, time taken by car 1 to reach B. after car 1 starts = total time – time required to travel first 20 km = 3 hr 30 min – 12 min = 3 hr 18 min 

Distance travelled by car 1 = (50 + 50 + 45) = 145 km 

Distance from B. = (150 – 145) km = 5 km 

Hence, 5 is the correct answer. 

Q. 84 If the sum of squares of two numbers is 97, then which one of the following cannot be their product? 

A. -32 

B. 16 

C. 48 

D. 64 

Answer: D. 

Explanation: 

Let ‘a’ and ‘b’ are those two numbers. 

a2 + b2 = 97 

a2 + b2 − 2ab = 97 − 2ab 

⇒ (a b)2 = 97 − 2ab 

We know that (a b)2 ≥ 0 

⇒ 97-2ab ≥ 0 

⇒ ab≤48.5 

Hence, ab≠64. Therefore, option D. is the correct answer. 

Q. 85 The area of a rectangle and the square of its perimeter are in the ratio 1 ∶ 25. Then the lengths of the shorter and longer sides of the rectangle are in the ratio 

A. 1:4 

B. 2:9 

C. 1:3 

D. 3:8 

Answer: A. 

Explanation: 

Let ‘a’ and ‘b’ be the length of sides of the rectangle. (a > b) Area of the rectangle = a*b 

Perimeter of the rectangle = 2*(a+b) 

⇒(a b) / (2 ∗ (a + b))2= 1/25 

⇒ 25ab = 4(a + b)2 

⇒ 4a2 − 17ab + 4b2 = 0 

⇒ (4a b)(a − 4b) = 0 

a = 4b or b/4

We initially assumed that a > b, therefore a ≠ b/4 . 

Hence, a = 4b 

⇒ b : a = 1 : 4 

Q. 86 On a triangle ABC, a circle with diameter BC is drawn, intersecting AB and AC at points P and Q, respectively. If the lengths of AB, AC, and CP are 30 cm, 25 cm, and 20 cm respectively, then the length of BQ, in cm, is 

Answer:24 

Explanation: 

Let us draw the diagram according to the available information. 

We can see that triangle ∠BPC and ∠BQC are inscribed inside a semicircle. Hence, we can say that 

∠BPC = ∠BQC = 90° 

Therefore, we can say that BQ⊥AC and CP⊥AB 

In triangle ABC, 

Area of triangle = (1/2)*Base*Height = (1/2)*AB*CP = (1/2)*AC*BQ 

⇒ BQ =(AB CP)/AC =(30 ∗ 20)/25 = 24 cm.

Q. 87 A triangle ABC has area 32 sq units and its side BC, of length 8 units, lies on the line x = 4. Then the shortest possible distance between A. and the point (0,0) is 

A. 8 units 

B. 4 units 

C. 2√2units 

D. 4√2units 

Answer: B. 

Explanation: 

We know that area of the triangle = 32 sq. units, BC = 8 units 

Therefore, the height of the perpendicular drawn from point A. to BC. = 2*32/8 = 8 units. Let us draw a possible diagram of the given triangle. 

We can see that if A. coincide with (-4, 0) then the distance between A. and (0, 0) = 4 units. 

If we move the triangle up or down keeping the base BC. on x = 4, then point A. will move away from origin as vertical distance will come into factor whereas horizontal distance will remain as 4 units. 

Hence, we can say that minimum distance between A. and origin (0, 0) = 4 units. 

Q. 88 From a rectangle ABCD of area 768 sq cm, a semicircular part with diameter AB. and area 72π sq cm is removed. The perimeter of the leftover portion, in cm, is 

A. 80 + 16π 

B. 86 + 8π 

C. 88 + 12π 

D. 82 + 24π 

Answer: C. 

Explanation: 

Area of the semicircle with AB. as a diameter = ½ ∗ π ∗ (AB2 / 4 )

⇒ 72 ∗ π

AB = 24cm

Given that area of the rectangle ABCD = 768 sq.cm 

⇒ AB*BC = 768 

⇒ BC = 32 cm 

We can see that the perimeter of the remaining shape = AD + DC + BC + Arc(AB) 

⇒ 32+24+32+ π ∗ 24/2 

⇒ 88 + 12π 

Q. 89 If A ={62n − 35n − 1 }, where n= 1,2,3,… and B n= {35( -1)}, where n= 1,2,3,… then which of the following is true? 

A. Every member of A. is in B. and at least one member of B. is not in A. 

B. Neither every member of A. is in B. nor every member of B. is in A. 

C. Every member of B. is in A. 

D. At least one member of A. is not in B. 

Answer: A. 

Explanation: If we carefully observe set A, then we find that 62n − 35n − 1 is divisible by 35. So, set A. contains multiples of 35. 

However, not all the multiples of 35 are there in set A, for different values of .n 

For n = 1, the value is 0, for n = 2, the value is 1225 which is the 35th multiple of 3. 

If we observe set B, it consists of all the multiples of 35 including 0. 

So, we can say that every member of set A. will be in B. while every member of set B. will not necessarily be in set A. Hence, option A. is the correct answer. 

Q. 90 The smallest integer n for which 4n > 1719  holds, is closest to 

A. 37 

B. 35 

C. 33 

D. 39 

Answer: D. 

Explanation: 

4n > 1719 

⇒ 16n/2 > 1719 

Therefore, we can say that n/2 > 19 

n > 38 

Hence, option D. is the correct answer. 

Q. 91 A jar contains a mixture of 175 ml water and 700 ml alcohol. Gopal takes out 10% of the mixture and substitutes it by water of the same amount. The process is repeated once again. The percentage of water in the mixture is now 

A. 30.3 

B. 35.2 

C. 25.4 

D. 20.5 

Answer: B. 

Explanation: 

Final quantity of alcohol in the mixture =700 / (700 + 175)∗(90/100)2∗ [700 + 175]= 567 ml 

Therefore, final quantity of water in the mixture = 875 – 567 = 308 ml 

Hence, we can say that the percentage of water in the mixture =308 / 875 × 100 = 35.2 % 

Q. 92 If a and b are integers such that 2x2ax + 2 > 0 and x2bx + 8 ≥ 0 for all real numbers x, then the largest possible value of 2a−6b is 

Answer:36 

Explanation: 

Let f(x) = 2x2ax + 2. We can see that f(x) is a quadratic function. 

For, f(x) > 0, Discriminant (D) < 0 

⇒ (−a)2 − 4 ∗ 2 ∗ 2 < 0 

⇒ (a-4)(a+4)<0 

⇒ a ϵ (-4, 4) 

Therefore, integer values that ‘a’ can take = {-3, -2, -1, 0, 1, 2, 3} 

Let g(x) = x2bx + 8. We can see that g(x) is also a quadratic function. 

For, g(x)≥0, Discriminant (D)≤0 

⇒ (−b)2 − 4 ∗ 8 ∗ 1 < 0 

⇒ (b − √32)(b + √32) < 0 

⇒ b ϵ (-√32 ,√32 ) 

Therefore, integer values that ‘b’ can take = {-5, -4, -3, -2, -1, 0, 1, 2, 3, 4, 5} 

We have to find out the largest possible value of 2a−6b. The largest possible value will occur when ‘a’ is maximum and ‘b’ is minimum. 

amax = 3, bmin = -5

Therefore, the largest possible value of = 2*3 – 6*(-5) = 36.

Q. 93 1/(log2100) – 1/(log4100) + 1/(log5100) – 1/(log10100) + 1/(log20100) – 1/(log25100) + 1/(log50100) =?

A. ½ 

B. 10

C. 0

D. −4

Answer: A

Explanation:

We know that = 1 / logab = logx a / logx b

Therefore, we can say that =1 / log2b = log10 a / log10 b

=> 1/(log2100) – 1/(log4100) + 1/(log5100) – 1/(log10100) + 1/(log20100) – 1/(log25100) + 1/(log50100)

=> log102/(log10100) – log104/(log10100) + log105/(log10100) – log1010/(log10100) + log1020/(log10100) – log1025/(log10100) + log1050/(log10100)

We know that log10100 = 2

=> ½ * [log102 – log104 + log105 – log1010 + log1020 – log1025 + log1050]

=> ½ * [log10(2*5*20*50 / 4*10*25)]

=> ½ * [log1010]

=> ½

Q. 94 If p3 = q4 = r5= s6 , then the value of logs(pqr) is equal to

A. 47/10

B. 24/5

C. 16/5

D. 1

Answer: A

Explanation:

Given that, p3 = q4 = r5= s6

p3 =s6

p6/3 = s2 …(1)

Similarly, q6/4 = s3/2 = s …(2)

Similarly, r = s6/5 …(3)

=> logs(pqr)

By substituting value of p, q, and r from equation (1), (2) and (3)

=> logs(82 * 83/2 * 86/5)

=> logs(847/10)

=> 47/10

Hence, option A is the correct answer.

Q. 95There are two drums, each containing a mixture of paints A and B. In drum 1, A and B are in the ratio 18 :7. The mixtures from drums 1 and 2 are mixed in the ratio 3 : 4 and in this final mixture, A and B are in the ratio 13 : 7. In drum 2, then A and B were in the ratio

A.  251 : 163

B. 239 : 161

C. 220 : 149

D. 229 : 141

Answer: B

Explanation:

It is given that in drum 1, A and B are in the ratio 18 : 7.

Let us assume that in drum 2, A and B are in the ratio x : 1.

It is given that drums 1 and 2 are mixed in the ratio 3 : 4 and in this final mixture, A and B are in the ratio 13 : 7.

By equating concentration of A

=> {3* 18/(18+7) + 4 * x/(x+1)} / 3+4 = 13/ (13+4)

=> (54/25) + 4x /(x+1) = 91/20

=> 4x /(x+1) = 239/100

=> x = 239/161

Therefore, we can say that in drum 2, A and B are in the ratio 239 : 161.

Q. 96 Ramesh and Ganesh can together complete a work in 16 days. After seven days of working together, Ramesh got sick and his efficiency fell by 30%. As a result, they completed the work in 17 days instead of 16 days. If Ganesh had worked alone after Ramesh got sick, in how many days would he have completed the remaining work?

A.  14.5

B. 11

C. 13.5

D. 12

Answer: C

Explanation:

Let ‘R’ and ‘G’ be the amount of work that Ramesh and Ganesh can complete in a day.

It is given that they can together complete a work in 16 days. Hence, total amount of work = 16(R+G) … (1)

For first 7 days both of them worked together. From 8th day, Ramesh worked at 70% of his original efficiency whereas

Ganesh worked at his original efficiency. It took them 17 days to finish the same work. i.e. Ramesh worked at 70% of his

original efficiency for 10 days.

16(R+G) = 7(R+G)+10(0.7R+G)

16(R+G) = 14R+17G

R = 0.5G … (2)

Total amount of work left when Ramesh got sick = 16(R+G) – 7(R+G) = 9(R+G) = 9(0.5+G) = 13.5G

Therefore, time taken by Ganesh to complete the remaining work = 13.5G/ G= 13.5 days.

Q. 97 Gopal borrows Rs. X from Ankit at 8% annual interest. He then adds Rs. Y of his own money and lends Rs. X+Y to Ishan at 10% annual interest. At the end of the year, after returning Ankit’s dues, the net interest retained by Gopal is the same as that accrued to Ankit. On the other hand, had Gopal lent Rs. X+2Y to Ishan at 10%, then the net interest retained by him would have increased by Rs. 150. If all interests are compounded annually, then find the value of X + Y.

Answer: 4000

Explanation:

Amount of interest paid by Ishan to Gopal if the borrowed amount is Rs. (X+Y) =10/100 * (X+Y) = 0.1(X+Y)

Gopal also borrowed Rs. X from Ankit at 8% per annum. Therefore, he has to return Ankit Rs. 0.08X as the interest

amount on borrowed sum.

Hence, the interest retained by gopal = 0.1(X+Y) – 0.08X = 0.02X + 0.1Y … (1)

It is given that the net interest retained by Gopal is the same as that accrued to Ankit.

Therefore, 0.08X = 0.02X + 0.1Y

X = (5/3)Y … (2)

Amount of interest paid by Ishan to Gopal if the borrowed amount is Rs. (X+2Y) =10/100 * (X+2Y) = 0.1X+0.2Y

In this case the amount of interest retained by Gopal = 0.1X+0.2Y – 0.08X = 0.02X + 0.2Y … (3)

It is given that the interest retained by Gopal increased by Rs. 150 in the second case.

(0.02X + 0.2Y) – (0.02X + 0.1Y) = 150

Y = Rs. 1500

By substituting value of Y in equation (2), we can say that X = Rs. 2500

Therefore, (X+Y) = Rs. 4000.

Q. 98 The strength of a salt solution is p% if 100 ml of the solution contains p grams of salt. If three salt solutions A, B, C are mixed in the proportion 1 : 2 : 3, then the resulting solution has strength 20%. If instead the proportion is 3 : 2 : 1, then the resulting solution has strength 30%. A fourth solution, D, is produced by mixing B and C in the ratio 2 : 7. The ratio of the strength of D to that of A is

A. 3 : 10

B. 1 : 3

C. 1 : 4

D. 2 : 5

Answer: B

Explanation:

Let ‘a’, ‘b’ and ‘c’ be the concentration of salt in solutions A, B and C respectively.

It is given that three salt solutions A, B, C are mixed in the proportion 1 : 2 : 3, then the resulting solution has strength 20%.

=> (a + 2b + 3c)/(1 + 2 + 3)=20

=> (a + 2b + 3c) = 120 … (1)

If instead the proportion is 3 : 2 : 1, then the resulting solution has strength 30%.

=> (3a + 2b + c)/(1 + 2 + 3)=30

=> (3a + 2b + c) = 180 … (2)

From equation (1) and (2), we can say that

⇒ b + 2c = 45

⇒ b = 45 − 2c

Also, on subtracting (1) from (2), we get

⇒a − c = 30

⇒ a = 30 + c

In solution D, B and C are mixed in the ratio 2 : 7

So, the concentration of salt in D =(2b + 7c) / 9 = (90 − 4c + 7c)/9 = (90 + 3c)/9

Required ratio =(90 + 3c)/9a = (90 + 3c)/9(30 + c) = 1 : 3

Hence, option B is the correct answer.

Q. 99 On a long stretch of east-west road, A and B are two points such that B is 350 km west of A. One car starts from A and another from B at the same time. If they move towards each other, then they meet after 1 hour. If they both move towards east, then they meet in 7 hrs. The difference between their speeds, in km per hour, is

Answer:50

Explanation:

Let ‘a’ and ‘b’ be the speed (in km/hr) of cars starting from both A and B respectively.

If they both move in east direction, then B will catch A if and only if b > a.

Relative speed of both the cars when they move in east direction = (b – a) km/hr

It takes them 7 hours to meet. i.e. they travel 350 km in 7 hours with a relative speed of (b-a) km/hr.

Hence, (b – a) = 350/7= 50 km/hr.

Q. 100 Let t1, t2,… be real numbers such that t1 + t2 + … + tn = 2n2 + 9n + 13, for every positive integer n ≥ 2. If tk = 103, then k equals

Answer:24

Explanation:

It is given that t1 + t2 + … + tn = 2n2 + 9n + 13, for every positive integer n ≥ 2.

We can say that t1 + t2 + … + tn = 2n2 + 9n + 13 … (1)

Replacing k by (k-1) we can say that

t1 + t2 + … + tk−1 = 2(k − 1)2 + 9(k − 1) + 13… (2)

On subtracting equation (2) from equation (1)

⇒ tk = 2k2 + 9k + 13 − 2(k − 1)2 + 9(k − 1) + 13

⇒ 103 = 4k + 7

⇒ k = 24

CAT Previous Year Paper Session-I 2018

CAT 2018 Session-I

Verbal Ability 

Instructions 

Read the following passage and answer the questions that follow: 

Economists have spent most of the 20th century ignoring psychology, positive or otherwise. But today there is a great deal of emphasis on how happiness can shape global economies, or — on a smaller scale — successful business practice. This is driven, in part, by a trend in “measuring” positive emotions, mostly so they can be optimized. Neuroscientists, for example, claim to be able to locate specific emotions, such as happiness or disappointment, in particular areas of the brain. Wearable technologies, such as Spire, offer data-driven advice on how to reduce stress. 

We are no longer just dealing with “happiness” in a philosophical or romantic sense — it has become something that can be monitored and measured, including by our behavior, use of social media and bodily indicators such as pulse rate and facial expressions. 

There is nothing automatically sinister about this trend. But it is disquieting that the businesses and experts driving the quantification of happiness claim to have our best interests at heart, often concealing their own agendas in the process. In the workplace, happy workers are viewed as a “win-win.” Work becomes more pleasant, and employees, more productive. But this is now being pursued through the use of performance-evaluating wearable technology, such as Humanyze or Virgin Pulse, both of which monitor physical signs of stress and activity toward the goal of increasing productivity. 

Cities such as Dubai, which has pledged to become the “happiest city in the world,” dream up ever-more elaborate and intrusive ways of collecting data on well-being — to the point where there is now talk of using CCTV cameras to monitor facial expressions in public spaces. New ways of detecting emotions are hitting the market all the time: One company, Beyond Verbal, aims to calculate moods conveyed in a phone conversation, potentially without the knowledge of at least one of the participants. And Facebook [has] demonstrated . . . that it could influence our emotions through tweaking our news feeds — opening the door to ever-more targeted manipulation in advertising and influence. 

As the science grows more sophisticated and technologies become more intimate with our thoughts and bodies, a clear trend is emerging. Where happiness indicators were once used as a basis to reform society, challenging the obsession with money that G.D.P. measurement entrenches, they are increasingly used as a basis to transform or discipline individuals. 

Happiness becomes a personal project, that each of us must now work on, like going to the gym. Since the 1970s, depression has come to be viewed as a cognitive or neurological defect in the individual, and never a consequence of circumstances. All of this simply escalates the sense of responsibility each of us feels for our own feelings, and with it, the sense of failure when things go badly. A. society that deliberately removed certain sources of misery, such as precarious and exploitative employment, may well be a happier one. But we won’t get there by making this single, often fleeting emotion, the over-arching goal. 

Q. 1 From the passage we can infer that the author would like economists to: 

A. work closely with neuroscientists to understand human behaviour. 

B. incorporate psychological findings into their research cautiously. 

C. correlate measurements of happiness with economic indicators. 

D. measure the effectiveness of Facebook and social media advertising. 

Answer: B. 

Explanation: 

We can infer that the author adopts a cautionary tone in the passage. He warns that quantification of happiness might be useful in certain contexts but making measuring happiness the primary goal can lead to unwanted consequences. He warns that happiness will become a personal project if we take the metrics too seriously. Therefore, the author is likely to recommend economists to incorporate the research findings cautiously and hence, option B. is the right answer. 

Q. 2 According to the author, wearable technologies and social media are contributing most to: 

A. making individuals aware of stress in their lives. 

B. depression as a thing of the past. 

C. disciplining individuals to be happy. 

D. happiness as a “personal project”. 

Answer: C. 

Explanation: 

In the penultimate paragraph, the author mentions “Where happiness indicators were once used as a basis to reform society, challenging the obsession with money that G.D.P. measurement entrenches, they are increasingly used as a basis to transform or discipline individuals”. He states that wearable technologies shift the onus on the person for his depression. In the last paragraph, the author mentions how these technologies are helping in disciplining individuals to be happy rather than addressing the cause of depression. Therefore, option C. is the right answer. 

Q. 3 In the author’s opinion, the shift in thinking in the 1970s: 

A. put people in touch with their own feelings rather than depending on psychologists. 

B. was a welcome change from the earlier view that depression could be cured by changing circumstances. 

C. introduced greater stress into people’s lives as they were expected to be responsible for their own happiness. 

D. reflected the emergence of neuroscience as the authority on human emotions. 

Answer: C. 

Explanation: 

In the last paragraph, the author mentions that since 1970s, depression is viewed as the defect of the individual rather than as the effect of his circumstances. He feels that this approach puts the person under pressure since being depressed is being viewed as the fault of the individual. The author does not view the shift in a positive light. Only option C. captures the fact that the development was a detrimental step and hence, option C. is the right answer. 

Q. 4 The author’s view would be undermined by which of the following research findings? 

A. Stakeholders globally are moving away from collecting data on the well-being of individuals. 

B. There is a definitive move towards the adoption of wearable technology that taps into emotions. 

C. A. proliferation of gyms that are collecting data on customer well-being. 

D. Individuals worldwide are utilising technologies to monitor and increase their well-being. 

Answer: A. 

Explanation: 

The primary intention of the author is to warn about the trend of collecting data to monitor emotions and in turn promote happiness as an overarching goal. He says that such a practice will lead to adoption of intrusive methods and make happiness a personal project to be worked on. If it is proved that less data is being collected than earlier, it will weaken the very basis of the author’s arguments. 

Options B. and C. indicate a trend that the author is warning about. Therefore, we can eliminate these 2 options. Option D. states that individuals worldwide are using technologies to monitor their well-being. The author’s argument is not that such technologies should not be used. He just states that proliferation of such technologies, especially when used by external parties like nations and corporations, might put people under greater stress. Therefore, we can eliminate option D. as well. 

Option A. states that stakeholders are moving away from collecting data. This statement goes against the warning issued by the author. Therefore, option A. will undermine the author’s arguments the most and hence, option A. is the right answer. 

Q. 5 According to the author, Dubai: 

A. collaborates with Facebook to selectively influence its inhabitants’ moods. 

B. develops sophisticated technologies to monitor its inhabitants’ states of mind. 

C. is on its way to becoming one of the world’s happiest cities. 

D. incentivises companies that prioritise worker welfare. 

Answer: B. 

Explanation: 

The author does not consider happiness indicators to be the gold standard of happiness. Therefore, we cannot say that Dubai is on its way to becoming one of the happiest cities in the world just because it tries to discipline its citizens to be happy. 

Nowhere has it been mentioned that Dubai collaborates with Facebook or incentivises companies that promote worker welfare. 

‘Cities such as Dubai, which has pledged to become the “happiest city in the world,” dream up ever-more elaborate and intrusive ways of collecting data on well-being — to the point where there is now talk of using CCTV cameras to monitor facial expressions in public spaces’. 

We can infer that Dubai comes up with new intrusive ways of collecting data on the well-being of its citizens. Therefore, option B. is the right answer. 

 

Instructions 

Read the passage carefully and answer the questions given 

. . . “Everybody pretty much agrees that the relationship between elephants and people has dramatically changed,” [says psychologist Gay] Bradshaw. . . . “Where for centuries humans and elephants lived in relatively peaceful coexistence, there is now hostility and violence. Now, I use the term ‘violence’ because of the intentionality associated with it, both in the aggression of humans and, at times, the recently observed behavior of elephants.” . . . 

Typically, elephant researchers have cited, as a cause of aggression, the high levels of testosterone in newly matured male elephants or the competition for land and resources between elephants and humans. But. . . Bradshaw and several colleagues argue. . . that today’s elephant populations are suffering from a form of chronic stress, a kind of species-wide trauma. Decades of poaching and culling and habitat loss, they claim, have so disrupted the intricate web of familial and societal relations by which young elephants have traditionally been raised in the wild, and by which established elephant herds are governed, that what we are now witnessing is nothing less than a precipitous collapse of elephant culture. . . . 

Elephants, when left to their own devices, are profoundly social creatures. . . . Young elephants are raised within an extended, multitiered network of doting female caregivers that includes the birth mother, grandmothers, aunts and friends. These relations are maintained over a life span as long as 70 years. Studies of established herds have shown that young elephants stay within 15 feet of their mothers for nearly all of their first eight years of life, after which young females are socialized into the matriarchal network, while young males go off for a time into an all-male social group before coming back into the fold as mature adults. . . . 

This fabric of elephant society, Bradshaw and her colleagues [demonstrate], ha[s] effectively been frayed by years of habitat loss and poaching, along with systematic culling by government agencies to control elephant numbers and translocations of herds to different habitats. . . . As a result of such social upheaval, calves are now being born to and raised by ever younger and inexperienced mothers. Young orphaned elephants, meanwhile, that have witnessed the death of a parent at the hands of poachers are coming of age in the absence of the support system that defines traditional elephant life. “The loss of elephant elders,” [says] Bradshaw . . . “and the traumatic experience of witnessing 

the massacres of their family, impairs normal brain and behavior development in young elephants.” 

What Bradshaw and her colleagues describe would seem to be an extreme form of anthropocentric conjecture if the evidence that they’ve compiled from various elephant researchers. . . weren’t so compelling. The elephants of decimated herds, especially orphans who’ve watched the death of their parents and elders from poaching and culling, exhibit behavior typically associated with post-traumatic stress disorder and other trauma-related disorders in humans: abnormal startle response, unpredictable asocial behavior, inattentive mothering and hyperaggression. . . . 

[According to Bradshaw], “Elephants are suffering and behaving in the same ways that we recognize in ourselves as a result of violence. . . . Except perhaps for a few specific features, brain organization and early development of elephants and humans are extremely similar.” 

Q. 6 Which of the following statements best expresses the overall argument of this passage? 

A. The brain organisation and early development of elephants and humans are extremely similar. 

B. Recent elephant behaviour could be understood as a form of species-wide trauma related response. 

C. The relationship between elephants and humans has changed from one of coexistence to one of hostility. 

D. Elephants, like the humans they are in conflict with, are profoundly social creatures. 

Answer: B. 

Explanation: 

Through the passage, the author explains how the ways elephants behave is similar to the trauma related response evoked in individuals. He explains how the elephant society is affected by the human activity and the impact of the same on the brain development of young elephants. 

The primary purpose of the passage is not to draw an analogy between elephants and humans in any way. Therefore, we can eliminate options A. and D. Option C. states that the relationship between elephants and humans has changed from one of coexistence to one of hostility. Though this point is true, it is not the central theme of the passage. The author places much emphasis on how the elephant behaviour can be explained as a species-wide trauma response and hence, option B. is the right answer. 

Q. 7 In paragraph 4, the phrase, “The fabric of elephant society . . . has[s] effectively been frayed by . . .” is: 

A. an accurate description of the condition of elephant herds today. 

B. a metaphor for the effect of human activity on elephant communities. 

C. an exaggeration aimed at bolstering Bradshaw’s claims. 

D. an ode to the fragility of elephant society today. 

Answer: B. 

Explanation: 

The author uses strong comparison in the given line. The author has not mentioned that the elephant society, which is like a fabric, is frayed by human activities. He uses the term ‘the fabric of elephant society’ and this comparison is called a metaphor. 

We can eliminate option A. since it fails to capture the fact that a comparison has been used. 

Option D. states that the line is an ode to the fragility of elephant society today. Option D. fails to capture the fact that human activities are wrecking the social structure of elephants. 

Option C. states that the line is an exaggeration to bolster Bradshaw’s claims. The author is not exaggerating the facts to substantiate Bradshaw’s claims. He tries to capture the effects of human activities on the elephant society metaphorically. Therefore, option B. is the right answer. 

Q. 8 The passage makes all of the following claims EXCEPT: 

A. elephants establish extended and enduring familial relationships as do humans. 

B. human actions such as poaching and culling have created stressful conditions for elephant communities. 

C. the elephant response to deeply disturbing experiences is similar to that of humans. 

D. elephant mothers are evolving newer ways of rearing their calves to adapt to emerging threats. 

Answer: D. 

Explanation: 

The author explains how elephants are profoundly social creatures like humans and how human activities are putting elephants under stress. Then, he explains how the recent elephant behaviour is similar to post traumatic stress syndrome observed in humans. Options A, B, and C. can be inferred. 

The author expresses apprehension that young calves are raised by inexperienced elephant mothers and this, in turn, affects the brain development of the calves. Nowhere has it been mentioned that elephant mothers are developing newer ways of rearing their calves. Therefore, option D. is the right answer. 

Q. 9 In the first paragraph, Bradshaw uses the term “violence” to describe the recent change in the human elephant relationship because, according to him: 

A. both humans and elephants have killed members of each other’s species. 

B. there is a purposefulness in human and elephant aggression towards each other. 

C. human-elephant interactions have changed their character over time. 

D. elephant herds and their habitat have been systematically destroyed by humans. 

Answer: B. 

Explanation: 

In the first paragraph of the passage, the author uses the line “Now, I use the term ‘violence’ because of the intentionality associated with it”. Therefore, we can infer that the author specifically uses the term violence to emphasize that the actions of the elephants on humans are deliberate just like those of humans on elephants. Therefore, option B. is the right answer. 

Q. 10 Which of the following measures is Bradshaw most likely to support to address the problem of elephant aggression? 

A The development of treatment programmes for elephants drawing on insights gained from treating post-traumatic stress disorder in humans. 

B Increased funding for research into the similarity of humans and other animals drawing on insights gained from human-elephant similarities. 

C. Studying the impact of isolating elephant calves on their early brain development, behaviour and aggression. 

D. Funding of more studies to better understand the impact of testosterone on male elephant aggression. 

Answer: A. 

Explanation: 

The author tries to establish that the elephant behaviour is similar to stress related response induced in humans. From the tone of the passage, we can infer that the author is concerned about the elephants. He does not adopt a detached view point. The passage tries to evoke empathy from the audience and has not been written as a science research paper. 

Options B, C, and D. do not address the issue at hand. They are not steps towards addressing elephant aggression. Only option A. proposes a method to treat the elephants and hence, option A. is the right answer. 

 

Instructions 

Read the following passage and answer the questions that follow: 

The only thing worse than being lied to is not knowing you’re being lied to. It’s true that plastic pollution is a huge problem, of planetary proportions. And it’s true we could all do more to reduce our plastic footprint. The lie is that blame for the plastic problem is wasteful consumers and that changing our individual habits will fix it. 

Recycling plastic is to saving the Earth what hammering a nail is to halting a falling skyscraper. You struggle to find a place to do it and feel pleased when you succeed. But your effort is wholly inadequate and distracts from the real problem of why the building is collapsing in the first place. The real problem is that single-use plastic—the very idea of producing plastic items like grocery bags, which we use for an average of 12 minutes but can persist in the environment for half a millennium—is an incredibly reckless abuse of technology. Encouraging individuals to recycle more will never solve the problem of a massive production of single-use plastic that should have been avoided in the first place. 

As an ecologist and evolutionary biologist, I have had a disturbing window into the accumulating literature on the hazards of plastic pollution. Scientists have long recognized that plastics biodegrade slowly, if at all, and pose multiple threats to wildlife through entanglement and consumption. More recent reports highlight dangers posed by absorption of toxic chemicals in the water and by plastic odors that mimic some species’ natural food. Plastics also accumulate up the food chain, and studies now show that we are likely ingesting it ourselves in seafood. . . . 

Beginning in the 1950s, big beverage companies like Coca-Cola and Anheuser-Busch, along with Phillip Morris and others, formed a non-profit called Keep America Beautiful. Its mission is/was to educate and encourage environmental stewardship in the public. . . . At face value, these efforts seem benevolent, but they obscure the real problem, which is the role that corporate polluters play in the plastic problem. This clever misdirection has led journalist and author Heather Rogers to describe Keep America Beautiful as the first corporate greenwashing front, as it has helped shift the public focus to consumer recycling behavior and actively thwarted legislation that would increase extended producer responsibility for waste management. . . . [T]he greatest success of Keep America Beautiful has been to shift the onus of environmental responsibility onto the public while simultaneously becoming a trusted name in the environmental movement. . . . 

So what can we do to make responsible use of plastic a reality? First: reject the lie. Litterbugs are not responsible for the global ecological disaster of plastic. Humans can only function to the best of their abilities, given time, mental bandwidth and systemic constraints. Our huge problem with plastic is the result of a permissive legal framework that has allowed the uncontrolled rise of plastic pollution, despite clear evidence of the harm it causes to local communities and the world’s oceans. Recycling is also too hard in most parts of the U.S. and lacks the proper incentives to make it work well. 

Q. 11 It can be inferred that the author considers the Keep America Beautiful organisation: 

A. an innovative example of a collaborative corporate social responsibility initiative. 

B. a sham as it diverted attention away from the role of corporates in plastics pollution. 

C. an important step in sensitising producers to the need to tackle plastics pollution. 

D. a “greenwash” because it was a benevolent attempt to improve public recycling habits. 

Answer: B. 

Explanation: 

In the penultimate paragraph, the author uses the line “[T]he greatest success of Keep America Beautiful has been to shift the onus of environmental responsibility onto the public while simultaneously becoming a trusted name in the environmental movement”. From the tone of the line, we can infer that the author believes that the sole purpose of ‘Keep America Beautiful’ was to shift the blame on the consumers. Therefore, option B. is the right answer. 

Q. 12 Which of the following interventions would the author most strongly support: 

A. having all consumers change their plastic consumption habits. 

B. recycling all plastic debris in the seabed. 

C. passing regulations targeted at producers that generate plastic products. 

D. completely banning all single-use plastic bags. 

Answer: C. 

Explanation: 

The author believes that the corporates are responsible for the plastic menace. He states that recycling the plastics or targeting the consumers are ineffective to tackle the problem. 

‘Encouraging individuals to recycle more will never solve the problem of a massive production of single-use plastic that should have been avoided in the first place’. 

In the last paragraph, the author recommends responsible use of plastics. Therefore, he is unlikely to support a complete ban on single use plastics as well. 

The author holds the corporates squarely responsible for the plastic menace. Therefore, the author is most likely to suggest passing regulations targeted at the producers rather than at the consumers. Therefore, option C. is the right answer. 

Q. 13 The author lists all of the following as negative effects of the use of plastics EXCEPT the: 

A. air pollution caused during the process of recycling plastics. 

B. poisonous chemicals released into the water and food we consume. 

C. adverse impacts on the digestive systems of animals exposed to plastic. 

D. slow pace of degradation or non-degradation of plastics in the environment. 

Answer: A. 

Explanation: 

In the third paragraph, the author mentions that plastics get absorbed in the water and some animals mistake plastic for their natural food and consume them. Therefore, we can infer options B. and C. In the same paragraph, the author explains how plastics we use for a few minutes will stay on the planet for millions of years. Therefore, we can infer option D. as well. 

The author has not mentioned about air pollution caused while recycling the plastics anywhere in the passage. Therefore, option A. is the right answer. 

Q. 14 In the second paragraph, the phrase “what hammering a nail is to halting a falling skyscraper” means: 

A. focusing on single-use plastic bags to reduce the plastics footprint. 

B. encouraging the responsible production of plastics by firms. 

C. relying on emerging technologies to mitigate the ill-effects of plastic pollution. 

D focusing on consumer behaviour to tackle the problem of plastics pollution. 

Answer: D. 

Explanation: 

The author believes that plastic production should be restricted. He finds asking consumers to stop using plastics or recycling plastics to be inadequate measures. 

The author uses the analogy to drive home the point that focusing on consumer behaviour will be totally incommensurate to tackle plastic pollution. Therefore, option D. is the right answer. 

Q. 15 In the first paragraph, the author uses “lie” to refer to the: 

A. understatement of the effects of recycling plastics. 

B. understatement of the enormity of the plastics pollution problem. 

C. blame assigned to consumers for indiscriminate use of plastics. 

D. fact that people do not know they have been lied to. 

Answer: C. 

Explanation: 

The author uses the term ‘lie’ to emphasize that the fact that the consumers are made to believe that they are responsible for the plastic menace. Through out the passage, the author explains how the corporates tricked people into believing that the blame lies on them for using the plastics. Therefore, option C. is the right answer. 

 

Instructions 

Read the following passage and answer the questions that follow: 

When researchers at Emory University in Atlanta trained mice to fear the smell of almonds (by pairing it with electric shocks), they found, to their consternation, that both the children and grandchildren of these mice were spontaneously afraid of the same smell. That is not supposed to happen. Generations of schoolchildren have been taught that the inheritance of acquired characteristics is impossible. A. mouse should not be born with something its parents have learned during their lifetimes, any more than a mouse that loses its tail in an accident should give birth to tailless mice. . . . 

Modern evolutionary biology dates back to a synthesis that emerged around the 1940s-60s, which married Charles Darwin’s mechanism of natural selection with Gregor Mendel’s discoveries of how genes are inherited. The traditional, and still dominant, view is that adaptations – from the human brain to the peacock’s tail – are fully and satisfactorily explained by natural selection (and subsequent inheritance). Yet [new evidence] from genomics, epigenetics and developmental biology [indicates] that evolution is more complex than we once assumed. . . . 

In his book On Human Nature (1978), the evolutionary biologist Edward O Wilson claimed that human culture is held on a genetic leash. The metaphor [needs revision]. . . . Imagine a dog-walker (the genes) struggling to retain control of a brawny mastiff (human culture). The pair’s trajectory (the pathway of evolution) reflects the outcome of the struggle. Now imagine the same dog-walker struggling with multiple dogs, on leashes of varied lengths, with each dog tugging in different directions. All these tugs represent the influence of developmental factors, including epigenetics, antibodies and hormones passed on by parents, as well as the ecological legacies and culture they bequeath. . . . 

The received wisdom is that parental experiences can’t affect the characters of their offspring. Except they do. The way that genes are expressed to produce an organism’s phenotype – the actual characteristics it ends up with – is affected by chemicals that attach to them. Everything from diet to air pollution to parental behaviour can influence the addition or removal of these chemical marks, which switches genes on or off. Usually these socalled ‘epigenetic’ attachments are removed during the production of sperm and eggs cells, but it turns out that some escape the resetting process and are passed on to the next generation, along with the genes. This is known as ‘epigenetic inheritance’, and more and more studies are confirming that it really happens. Let’s return to the almond-fearing mice. The inheritance of an epigenetic mark transmitted in the sperm is what led the mice’s offspring to acquire an inherited fear. . . . 

Epigenetics is only part of the story. Through culture and society, [humans and other animals] inherit knowledge and skills acquired by [their] parents. . . . All this complexity . . . points to an evolutionary process in which genomes (over hundreds to thousands of generations), epigenetic modifications and inherited cultural factors (over several, perhaps tens or hundreds of generations), and parental effects (over single-generation timespans) collectively inform how organisms adapt. These extra-genetic kinds of inheritance give organisms the flexibility to make rapid adjustments to environmental challenges, dragging genetic change in their wake – much like a rowdy pack of dogs. 

 

Q. 16 The passage uses the metaphor of a dog walker to argue that evolutionary adaptation is most comprehensively understood as being determined by: 

A. ecological, hormonal, extra genetic and genetic legacies. 

B. genetic, epigenetic, developmental factors, and ecological legacies. 

C. extra genetic, genetic, epigenetic and genomic legacies. 

D. socio-cultural, genetic, epigenetic, and genomic legacies. 

Answer: B. 

Explanation: 

The author mentions “All these tugs represent the influence of developmental factors, including epigenetics, antibodies and hormones passed on by parents, as well as the ecological legacies and culture they bequeath”. 

Option A. misses ‘developmental factors’ and ‘antibodies’. 

Option C. misses ‘ecological legacies’. 

Option D. misses ‘developmental factors’. 

Option B. is the most comprehensive one among the given options and hence, it is the right answer. 

 

Q. 17 Which of the following options best describes the author’s argument? 

A. Wilson’s theory of evolution is scientifically superior to either Darwin’s or Mendel’s. 

B. Darwin’s theory of natural selection cannot fully explain evolution. 

C. Darwin’s and Mendel’s theories together best explain evolution. 

D. Mendel’s theory of inheritance is unfairly underestimated in explaining evolution. 

Answer: B. 

Explanation: 

The primary purpose of the passage is not to establish the scientific superiority of Wilson’s theory over that of Darwin’s and Mendel’s theories. The author begins the passage with an example that the theory of natural selection fails to explain. Then, he explains about ‘Epigenetic inheritance’ and elaborates on how epigenetic inheritance explains the transmission of acquired characteristics. Therefore, the author’s main argument is that Darwin’s theory cannot fully explain evolution and hence, option B. is the right answer. 

Q. 18 Which of the following, if found to be true, would negate the main message of the passage? 

A. A. study affirming the sole influence of natural selection and inheritance on evolution. 

B. A. study highlighting the criticality of epigenetic inheritance to evolution. 

C. A. study indicating the primacy of ecological impact on human adaptation. 

D. A. study affirming the influence of socio-cultural markers on evolutionary processes. 

Answer: A. 

Explanation: 

The main message of the passage is that natural selection cannot fully explain evolution. Therefore, any argument that attacks this message is most likely to weaken the author’s arguments. A. study indicating the sole influence of natural selection and inheritance on evolution will Q. the legitimacy of the theory of ‘epigenetic inheritance’ and hence, option A. is the right answer. 

Q. 19 The Emory University experiment with mice points to the inheritance of: 

A. acquired parental fears 

B. acquired characteristics 

C. psychological markers 

D. personality traits 

Answer: B. 

Explanation: 

The author uses the Emory University experiment to show that acquired characteristics can be passed on from one generation to another. In the second paragraph, the author explains how the acquired characteristics (fear in this case) should not have been passed according to the theory of natural selection. 

Option A. is too narrow in scope. The author uses fear as an example of acquired characteristic. Therefore, option B. is the right answer. 

 

Instructions 

Read the following passage and answer the questions that follow: 

[The] Indian government [has] announced an international competition to design a National War Memorial in New Delhi, to honour all of the Indian soldiers who served in the various wars and counter-insurgency campaigns from 1947 onwards. The terms of the competition also specified that the new structure would be built adjacent to the India Gate – a memorial to the Indian soldiers who died in the First World War. Between the old imperialist memorial and the proposed nationalist one, India’s contribution to the Second World War is airbrushed out of existence. 

The Indian government’s conception of the war memorial was not merely absent-minded. Rather, it accurately reflected the fact that both academic history and popular memory have yet to come to terms with India’s Second World War, which continues to be seen as little more than mood music in the drama of India’s advance towards independence and partition in 1947. Further, the political trajectory of the postwar subcontinent has militated against popular remembrance of the war. With partition and the onset of the India-Pakistan rivalry, both of the new nations needed fresh stories for self-legitimation rather than focusing on shared wartime experiences. 

However, the Second World War played a crucial role in both the independence and partition of India. . . . The Indian army recruited, trained and deployed some 2.5 million men, almost 90,000 of which were killed and many more injured. Even at the time, it was recognised as the largest volunteer force in the war. . . . 

India’s material and financial contribution to the war was equally significant. India emerged as a major military-industrial and logistical base for Allied operations in south-east Asia and the Middle East. This led the United States to take considerable interest in the country’s future, and ensured that this was no longer the preserve of the British government. 

Other wartime developments pointed in the direction of India’s independence. In a stunning reversal of its long-standing financial relationship with Britain, India finished the war as one of the largest creditors to the imperial power. 

Such extraordinary mobilization for war was achieved at great human cost, with the Bengal famine the most extreme manifestation of widespread wartime deprivation. The costs on India’s home front must be counted in millions of lives. 

Indians signed up to serve on the war and home fronts for a variety of reasons. . . . [M]any were convinced that their contribution would open the doors to India’s freedom. . . . The political and social churn triggered by the war was 

evident in the massive waves of popular protest and unrest that washed over rural and urban India in the aftermath of the conflict. This turmoil was crucial in persuading the Attlee government to rid itself of the incubus of ruling India. . . . Seventy years on, it is time that India engaged with the complex legacies of the Second World War. Bringing the war into the ambit of the new national memorial would be a fitting – if not overdue – recognition that this was India’s War. 

 

Q. 20 The author suggests that a major reason why India has not so far acknowledged its role in the Second World War is that it: 

A. wants to forget the human and financial toll of the War on the country. 

B. has been focused on building an independent, non-colonial political identity. 

C. views the War as a predominantly Allied effort, with India playing only a supporting role. 

D. blames the War for leading to the momentous partition of the country. 

Answer: B. 

Explanation: 

By the term “mood music”, the author intends to convey that the war set the stage for the Independence and partition of the country. He does not mean that the war was an allied effort and India’s contribution to the war was merely supportive. 

The author mentions that the political trajectory in both the countries has been against the popular remembrance of war. He states that the countries were focused on building a non-colonial identity and the war narrative did not fit in well in the picture. 

Q. 21 The phrase “mood music” is used in the second paragraph to indicate that the Second World War is viewed as: 

A. setting the stage for the emergence of the India-Pakistan rivalry in the subcontinent. 

B. a part of the narrative on the ill-effects of colonial rule on India. 

C. a tragic period in terms of loss of lives and national wealth. 

D. a backdrop to the subsequent independence and partition of the region. 

Answer: D. 

Explanation: 

The author uses the phrase “mood music” to indicate that (the contribution of Indians to) the Second World War is not given the importance it deserves. The author does not state that the war led to the rivalry. Though he mentions the ill effects of the war on India, he does not refer to them when he uses the term “mood music”. He feels that the war is largely seen as a warmer to the Independence and partition of the country. Therefore, option D. is the right answer. 

Q. 22 The author lists all of the following as outcomes of the Second World War EXCEPT: 

A. US recognition of India’s strategic location and role in the War. 

B. the large financial debt India owed to Britain after the War. 

C. large-scale deaths in Bengal as a result of deprivation and famine. 

D. independence of the subcontinent and its partition into two countries. 

Answer: B. 

Explanation: 

In the fourth paragraph, the author states “This led the United States to take considerable interest in the country’s future”. We can infer that India’s strategic location led to US’s interests towards India and hence, we can eliminate option A. 

In the first line of the second paragraph, the author mentions that the Second World War played a crucial role in the independence of India. 

In the sixth paragraph, the author mentions that the war was achieved at great human cost. He states that the Bengal famine was the most extreme manifestation of the human costs of the war. 

In the fifth paragraph, the author states “In a stunning reversal of its long-standing financial relationship with Britain, India finished the war as one of the largest creditors to the imperial power”. From this line, we can infer that India lent its resources to Britain, not the other way around. Therefore, option B. is an incorrect interpretation of the given sentence and hence, option B. is the right answer. 

Q. 23 The author claims that omitting mention of Indians who served in the Second World War from the new National War Memorial is: 

A. is something which can be rectified in future by constructing a separate memorial. 

B. a reflection of misplaced priorities of the post-independence Indian governments. 

C. appropriate as their names can always be included in the India Gate memorial. 

D. a reflection of the academic and popular view of India’s role in the War. 

Answer: D. 

Explanation: 

In the second paragraph, the author mentions “Rather, it accurately reflected the fact that both academic history and popular memory have yet to come to terms with India’s Second World War”. The author states that the act was not merely absent-minded. Therefore, the author considers the omission to be reflective of India’s academic and popular views and hence, option D. is the right answer. 

Q. 24 In the first paragraph, the author laments the fact that: 

A. the new war memorial will be built right next to India Gate. 

B. there is no recognition of the Indian soldiers who served in the Second World War. 

C. India lost thousands of human lives during the Second World War. 

D. funds will be wasted on another war memorial when we already have the India Gate memorial. 

Answer: B. 

Explanation: 

The author states that the new war memorial to commemorate various soldiers who lost their lives since independence will be built near India gate, a World War I memorial. The author regrets the fact that the contribution of Indian soldiers to World War II is being air brushed out of existence. The author laments the fact that the nation fails to recognize the sacrifice of the Indian soldiers who served in the World War II and hence, option B. is the right answer. 

 

Instructions 

For the following questions answer them individually 

Q. 25 The passage given below is followed by four summaries. Choose the option that best captures the author’s position. 

Production and legitimation of scientific knowledge can be approached from a number of perspectives. To study knowledge production from the sociology of professions perspective would mean a focus on the institutionalization of a body of knowledge. The professions-approach informed earlier research on managerial occupation, business schools and management knowledge. It however tends to reify institutional power structures in its understanding of the links between knowledge and authority. Knowledge production is restricted in the perspective to the selected members of the professional community, most notably to the university faculties and professional colleges. Power is understood as a negative mechanism, which prevents the non-professional actors from offering their ideas and information as legitimate knowledge. 

A. Professions-approach focuses on the creation of institutions of higher education and disciplines to promote knowledge production 

B. The study of knowledge production can be done through many perspectives. 

C. The professions-approach has been one of the most relied upon perspective in the study of management knowledge production. 

D. Professions-approach aims at the institutionalization of knowledge but restricts knowledge production as a function of a select few. 

Answer: D. 

Explanation: 

Let us note down the important points. 

Professions-approach structures and institutionalizes knowledge but knowledge production is restricted to the select members of the community. It prevents the non-professional actors from offering their ideas. 

Options A, B, and C. do not capture the negative aspects of the professions-approach at all. They just focus on the advantages offered by the approach but the given paragraph places a huge emphasis on the limitations of the approach as well. Only option D. captures both the advantage offered by the approach and its limitations. Therefore, option D. is the right answer. 

Q. 26 Together to form a meaningful and coherent short paragraph. Identify the odd one out. Choose its number as your answer and key it in. 

1) Translators are like bumblebees. 

2) Though long since scientifically disproved, this factoid is still routinely trotted out. 

3) Similar pronouncements about the impossibility of translation have dogged practitioners since Leonardo Bruni’s De interpretatione recta, published in 1424. 

4) Bees, unaware of these deliberations, have continued to flit from flower to flower, and translators continue to translate. 

5) In 1934, the French entomologist August Magnan pronounced the flight of the bumblebee to be aerodynamically impossible. 

Answer:

Explanation: 

On reading the sentences, we can infer that the author draws an analogy between translators and bumblebees in the 

paragraph. 

1 should be the opening sentence since it introduces the fact that the paragraph is going to be about the similarities of translators and bumblebees. After sentence 1, the author should have explained how they are analogous. 

5 states that the French entomologist August Magnan pronounced the flight of bumblebees to be aerodynamically impossible. Sentence 3 talks about similar statements made about translations. Sentence 4 should be the last sentence since it concludes by saying that both translators and bees have continued their work unaware of these deliberations. Sentences 1534 can be put together into a coherent paragraph. 

Sentence 2 does not add any valuable information to the topic of discussion. The author does not use the fact that the factoid (impossibility of the flight of the bumblebee) has been disproved to support his argument. Sentence 2 should be the one out of context and hence, 2 is the correct answer. 

Q. 27 Five sentences related to a topic are given below. Four of them can be put together to form a meaningful and coherent short paragraph. Identify the odd one out. 

1) Displacement in Bengal is thus not very significant in view of its magnitude. 

2) A. factor of displacement in Bengal is the shifting course of the Ganges leading to erosion of river banks. 3) The nature of displacement in Bengal makes it an interesting case study. 

4) Since displacement due to erosion is well spread over a long period of time, it remains invisible. 5) Rapid displacement would have helped sensitize the public to its human costs. 

Answer:

Explanation: 

On reading the sentences, we can infer that the paragraph revolves around the displacement of people in Bengal due to erosion. 3 should be the opening sentence since it introduces the topic of discussion – displacement in Bengal. 3 should be followed by sentence 2 since it elaborates that the displacement is due to the shifting of the course of the Ganges and the erosion. We have to decide the pair between 5, 4, and 1. 

Sentence 4 is definitely a part of the paragraph since it fits well with both the sentences. Also, it states an important detail – the displacement is spread out over a period of time and hence, remains invisible. Since the displacement is not rapid, its magnitude is not significant. Sentences 4 and 1 form a pair and hence, sentence 5 is the one out of context. 

Q. 28 The passage given below is followed by four summaries. Choose the option that best captures the author’s position. 

The conceptualization of landscape as a geometric object first occurred in Europe and is historically related to the European conceptualization of the organism, particularly the human body, as a geometric object with parts having a rational, three-dimensional organization and integration. The European idea of landscape appeared before the science of landscape emerged, and it is no coincidence that Renaissance artists such as Leonardo da Vinci, who studied the structure of the human body, also facilitated an understanding of the structure of landscape. Landscape which had been a subordinate background to religious or historical narratives, became an independent genre or subject of art by the end of sixteenth century or the beginning of the seventeenth century. 

A. The three-dimensional understanding of the organism in Europe led to a similar approach towards the understanding of landscape. 

B. Landscape became a major subject of art at the turn of the sixteenth century. 

C. The study of landscape as an independent genre was aided by the Renaissance artists. 

D. The Renaissance artists were responsible for the study of landscape as a subject of art. 

Answer: C. 

Explanation: 

Let us note down the main points of the given paragraph: 

The given paragraph describes how the study of landscape gained prominence and became an independent genre. Renaissance artists facilitated the development of the field as an independent genre. 

Let us evaluate the options one by one. 

Option A. states that understanding of the organism in Europe led to a similar approach towards the understanding of landscape. Though this option is true, it fails to capture the fact that the field evolved as an independent genre with the help of Renaissance artists. 

Option B. states that Landscape became a major subject of art at the turn of the sixteenth century. Again, option B. fails to capture the role played by the Renaissance artists. 

Option D. states that Renaissance artists were responsible for the study of landscape as a subject of art. The paragraph mentions that the artists facilitated in the transformation of the field into an independent genre. Option D. establishes a strong relationship and holds Renaissance artists ‘responsible’ for the study of landscape ‘as a subject of art’. The parts within the quotes disregard the fact that the artists just aided the process. They were not solely responsible for the 

development. Therefore, we can eliminate option D. 

Only option C. captures the fact that the renaissance artists ‘aided’ in the development of the study of landscape as an independent genre and hence, option C. is the right answer. 

Q. 29 The four sentences (labelled 1,2,3,4) given in this question, when properly sequenced, form a coherent paragraph. Each sentence is labelled with a number. Decide on the proper sequence of order of the sentences and key in this sequence of four numbers as your answer. 

1. Impartiality and objectivity are fiendishly difficult concepts that can cause all sorts of injustices even if transparently implemented. 

2. It encourages us into bubbles of people we know and like, while blinding us to different perspectives, but the deeper problem of ‘transparency’ lies in the words “…and much more”. 

3. Twitter’s website says that “tweets you are likely to care about most will show up first in your timeline…based on accounts you interact with most, tweets you engage with, and much more.” 

4. We are only told some of the basic principles, and we can’t see the algorithm itself, making it hard for citizens to analyze the system sensibly or fairly or be convinced of its impartiality and objectivity. 

Answer:1324 

Explanation: 

On reading the sentences, we can infer that the paragraph is about the difficulty in implementing impartiality and objectivity. 

Sentence 3 states that Twitter’s website says that the algorithm shows tweets that are likely to suit the taste of the user and much more. Sentence 2 continues sentence 3 by stating that the catch lies in the term ‘much more’. Also, it criticizes how catering to the taste of the user forces him into a bubble. Sentence 2 should be followed by sentence 4 since it states the implications of the term ‘much more’ and how it makes believing in the impartiality and objectivity of twitter hard. 

Sentences 324 form a group. The entire group has been provided as an illustration to explain how hard it is to implement impartiality and objectively. Therefore, sentence 1 should be the opening sentence. 

Sentences 1324 form a coherent paragraph. Hence, 1324 is the correct answer. 

Q. 30 Five sentences related to a topic are given below. Four of them can be put together to form a meaningful and coherent short paragraph. Identify the odd one out. 

1) In many cases time inconsistency is what prevents our going from intention to action. 

2) For people to continuously postpone getting their children immunized, they would need to be constantly fooled by themselves. 

3) In the specific case of immunization, however, it is hard to believe that time inconsistency by itself would be sufficient to make people permanently postpone the decision if they were fully cognizant of its benefits. 

4) In most cases, even a small cost of immunization was large enough to discourage most people. 

5) Not only do they have to think that they prefer to spend time going to the camp next month rather than today, they also have to believe that they will indeed go next month. 

Answer:

Explanation: 

All sentences except sentence 4 talk about how time inconsistency prevents people from immunizing their children. Sentence 4 states that the cost of immunization acts as a deterrent which is not in line with the other 4 sentences. 

1 should be the opening sentence since it is a general statement introducing time inconsistency. Sentence 1 should be followed by sentence 3 since explains how time inconsistency, in itself, acts as a deterrent in the specific case of immunization. Sentences 2 and 5 form a pair. Sentence 2 states how people should be fooling themselves to postpone their child’s immunization. Sentence 5 explains how they should be fooling themselves to not get their child immunized. 

Sentences 1325 can be put together into a coherent paragraph. Therefore, 4 is the correct answer. 

Q. 31 The passage given below is followed by four summaries. Choose the option that best captures the author’s position. 

Artificial embryo twinning is a relatively low-tech way to make clones. As the name suggests, this technique mimics the natural process that creates identical twins. In nature, twins form very early in development when the embryo splits in two. Twinning happens in the first days after egg and sperm join, while the embryo is made of just a small number of unspecialized cells. Each half of the embryo continues dividing on its own, ultimately developing into separate, complete individuals. Since they developed from the same fertilized egg, the resulting individuals are genetically identical. 

A. Artificial embryo twinning is low-tech and mimetic of the natural development of genetically identical twins from the embryo after fertilization. 

B. Artificial embryo twinning is low-tech and is close to the natural development of twins where the embryo splits into two identical twins. 

C. Artificial embryo twinning is low-tech unlike the natural development of identical twins from the embryo after fertilization. 

D. Artificial embryo twinning is just like the natural development of twins, where during fertilization twins are formed. 

Answer: A. 

Explanation: 

The author mentions that artificial embryo twinning is ‘low tech’ to introduce the topic. Then, he explains how the process is exactly similar to the process of development of twins. He states that the process mimics the natural development of twins. He has not highlighted any of the differences between the 2 processes. 

Let us evaluate the options. 

Option C. states that artificial embryo twinning is ‘low tech’ unlike the natural development of twins. The author makes no such comparison in the paragraph and hence, option C. can be eliminated. 

Option D. states that the twins are formed during fertilization but the paragraph mentions that the twins are formed after 

the process of fertilization (i.e, after the sperm and the egg join). 

Option B. fails to capture the fact that the twins are ‘genetically’ identical. Also, it states that the artificial twinning process is ‘close to’ the natural development of twins. Though this option is not incorrect, option A. is worded in a better way. Option A. states that the process is mimetic of the natural development of the twins (emphasizing that no difference has been highlighted), the twins are genetically identical and the process is similar to the process of development of twins after fertilization. Therefore, option A. is the right answer. 

Q. 32 The four sentences (labelled 1, 2, 3, and 4) given in this question, when properly sequenced, form a coherent paragraph. Decide on the proper order for the sentences and key in this sequence of four numbers as your answer. 

1. The eventual diagnosis was skin cancer and after treatment all seemed well. 

2. The viola player didn’t know what it was; nor did her GP. 

3. Then a routine scan showed it had come back and spread to her lungs. 

4. It started with a lump on Cathy Perkins’ index finger. 

Answer:4213 

Explanation: 

Sentence 4 should be the opening sentence since it sets the scene and introduces the person suffering from cancer. Sentence 2 states that the viola player and her physician did not know what it was. ‘It’ refers to the lump on the finger. Sentence 1 should follow sentence 2 since it states what the eventual diagnosis was. The GP did not know what the lump was and later it was identified to be skin cancer. Sentence 1 states that all seemed well after the treatment, implying it was not. 

Sentence 3 should be the last sentence since it states that the cancer had spread to her lungs. Sentences 4213 form a coherent paragraph and hence, 4213 is the right answer. 

Q. 33 The four sentences (labelled 1,2,3,4) given in this question, when properly sequenced, form a coherent paragraph. Each sentence is labelled with a number. Decide on the proper sequence of order of the sentences and key in this sequence of four numbers as your Answer: 

1) But now we have another group: the unwitting enablers. 

2) Democracy and high levels of inequality of the kind that have come to characterize the United States are simply incompatible. 

3) Believing these people are working for a better world, they are, actually, at most, chipping away at the margins, making slight course corrections, ensuring the system goes on as it is, uninterrupted. 4) Very rich people will always use money to maintain their political and economic power. 

Answer:2413 

Explanation: 

2 should be the opening sentence since it sets the context for the discussion. It states that democracy and high levels of inequality are not compatible. Sentence 2 should be followed by sentence 4 since it states that very rich people will always try to buy power. The author is not surprised by this fact. Sentence 1 should follow sentence 4 since it states what the really unexpected thing is. The author states that we have a new group of people, ‘the unwitting enablers’. Sentence 3 should follow sentence 4 since it elaborates on the nature of the unwitting enablers. 

Sentences 2413 form a coherent paragraph and hence, 2413 is the right answer. 

Q. 34 The four sentences (labelled 1, 2, 3, and 4) given in this question, when properly sequenced, form a coherent paragraph. Decide on the proper order for the sentences and key in this sequence of four numbers as your answer. 

1) The woodland’s canopy receives most of the sunlight that falls on the trees. 

2) Swifts do not confine themselves to woodlands, but hunt wherever there are insects in the air. 3) With their streamlined bodies, swifts are agile flyers, ideally adapted to twisting and turning through the air as they chase flying insects – the creatures that form their staple diet. 

4) Hundreds of thousands of insects fly in the sunshine up above the canopy, some falling prey to swifts and swallows 

Answer:1432 

Explanation: 

1 should be the opening sentence since it sets the context. Sentence 1 states that the woodland’s canopy receives most of the sunshine that falls on the trees. Sentence 4 continues sentence 1 by stating that thousands of insects fly above the canopy in the sunlight. The insects fall prey to the swifts and swallows. 

We have to decide whether the order of the remaining 2 sentences is 32 or 23. 

Sentence 3 states that swifts are agile flyers, adapted to chasing flying insects. Sentence 3 states that flying insects form the staple diet of the swifts. Sentence 2 states that swifts do not confine themselves to woodlands and hunt wherever they can find insects. Therefore, sentence 3 should precede sentence 2 (We cannot introduce that insects form the staple diet after stating that swifts hunt wherever they can find flying insects). 

Sentences 1432 form a coherent paragraph. 

Therefore, 1432 is the correct answer. 

LRDI 

Instructions 

Adriana, Bandita, Chitra, and Daisy are four female students, and Amit, Barun, Chetan, and Deb are four male students. Each of them studies in one of three institutes – X, Y, and Z. Each student majors in one subject among Marketing, Operations, and Finance, and minors in a different one among these three subjects. The following facts are known about the eight students: 

1. Three students are from X, three are from Y, and the remaining two students, both female, are from Z. 2. Both the male students from Y minor in Finance, while the female student from Y majors in Operations. 3. Only one male student majors in Operations, while three female students minor in Marketing. 4. One female and two male students major in Finance. 

5. Adriana and Deb are from the same institute. Daisy and Amit are from the same institute. 6. Barun is from Y and majors in Operations. Chetan is from X and majors in Finance. 

7. Daisy minors in Operations. 

Q. 35 Who are the students from the institute Z? 

A. Chitra and Daisy 

B. Adriana and Bandita 

C. Bandita and Chitra 

D. Adriana and Daisy 

Answer: C. 

Explanation: 

There are 8 students in total – 4 male and 4 female. There are 3 institutes X, Y, and Z. 

3 students are from institute X, 3 students are from institute Y, and 2 students are from institute Z. No student majors and minors in the same subject. 

It has been given that both the students from institute Z are female. Also, it has been given that both the male students from institute Y minor in Finance. Therefore, the third student from institute Y should be female. Institute X should also have 2 male and 1 female student. 

Both the male students from Y minor in Finance, while the female student from Y majors in Operations. Barun is from Y and majors in Operations. Chetan is from X and majors in Finance. 

 

It has been given that one female student and 2 male students major in finance. We know that the male student from Y minors in finance. Therefore, he cannot major in finance. Therefore, both the male students from X should major in finance. 

 

Daisy and Amit are from the same institute. Therefore, Daisy cannot be from institute Z (since Amit is a male student and both the students from Z are female). Daisy minors in operations. The girl from institute Y majors in Operations. Therefore, Daisy cannot be from institute Y as well. Daisy and Amit should be from institute X. 3 female students minor in marketing. Therefore, all girls except Daisy should minor in marketing. 

Adriana and Deb are from the same institute. Therefore, both of them should be from institute Y. Bandita and Chitra should be from institute Z. Only one male student majors in Operations. We know that Barun is the student. Two male students major in Finance. We know that Amit and Chetan major in finance. Therefore, Deb should major in Marketing. 

 

 

Bandita and Chitra are from institute Z. Therefore, option C. is the right answer. 

Q. 36 Which subject does Deb minor in? 

A. Operations 

B. Finance 

C. Marketing 

D. Cannot be determined uniquely from the given information 

Answer: B. 

Explanation: 

There are 8 students in total – 4 male and 4 female. There are 3 institutes X, Y, and Z. 

3 students are from institute X, 3 students are from institute Y, and 2 students are from institute Z. No student majors and minors in the same subject. 

It has been given that both the students from institute Z are female. Also, it has been given that both the male students from institute Y minor in Finance. Therefore, the third student from institute Y should be female. Institute X should also have 2 male and 1 female student. 

Both the male students from Y minor in Finance, while the female student from Y majors in Operations. Barun is from Y and majors in Operations. Chetan is from X and majors in Finance. 

 

It has been given that one female student and 2 male students major in finance. We know that the male student from Y minors in finance. Therefore, he cannot major in finance. Therefore, both the male students from X should major in finance. 

 

Daisy and Amit are from the same institute. Therefore, Daisy cannot be from institute Z (since Amit is a male student and both the students from Z are female). Daisy minors in operations. The girl from institute Y majors in Operations. Therefore, Daisy cannot be from institute Y as well. Daisy and Amit should be from institute X. 3 female students minor in marketing. Therefore, all girls except Daisy should minor in marketing. 

Adriana and Deb are from the same institute. Therefore, both of them should be from institute Y. Bandita and Chitra should be from institute Z. 

 

Only one male student majors in Operations. We know that Barun is the student. Two male students major in Finance. We know that Amit and Chetan major in finance. Therefore, Deb should major in Marketing. 

Deb minors in Finance. Therefore, option B. is the right answer. 

Q. 37 Which subject does Amit major in? 

A. Marketing 

B. Operations 

C. Cannot be determined uniquely from the given information 

D. Finance 

Answer: D. 

Explanation: 

There are 8 students in total – 4 male and 4 female. There are 3 institutes X, Y, and Z. 

3 students are from institute X, 3 students are from institute Y, and 2 students are from institute Z. No student majors and minors in the same subject. 

It has been given that both the students from institute Z are female. Also, it has been given that both the male students from institute Y minor in Finance. Therefore, the third student from institute Y should be female. Institute X should also have 2 male and 1 female student. 

Both the male students from Y minor in Finance, while the female student from Y majors in Operations. Barun is from Y and majors in Operations. Chetan is from X and majors in Finance. 

It has been given that one female student and 2 male students major in finance. We know that the male student from Y minors in finance. Therefore, he cannot major in finance. Therefore, both the male students from X should major in finance. 

 

Daisy and Amit are from the same institute. Therefore, Daisy cannot be from institute Z (since Amit is a male student and both the students from Z are female). Daisy minors in operations. The girl from institute Y majors in Operations. Therefore, Daisy cannot be from institute Y as well. Daisy and Amit should be from institute X. 3 female students minor in marketing. Therefore, all girls except Daisy should minor in marketing. 

Adriana and Deb are from the same institute. Therefore, both of them should be from institute Y. Bandita and Chitra should be from institute Z. 

Only one male student majors in Operations. We know that Barun is the student. Two male students major in Finance. We know that Amit and Chetan major in finance. Therefore, Deb should major in Marketing. 

Amit majors in finance. Therefore, option D. is the right answer. 

Q. 38 If Chitra majors in Finance, which subject does Bandita major in? 

A. Finance 

B. Cannot be determined uniquely from the given information 

C. Operations 

D. Marketing 

Answer: C. 

Explanation: 

There are 8 students in total – 4 male and 4 female. There are 3 institutes X, Y, and Z. 

3 students are from institute X, 3 students are from institute Y, and 2 students are from institute Z. No student majors and minors in the same subject. 

It has been given that both the students from institute Z are female. Also, it has been given that both the male students from institute Y minor in Finance. Therefore, the third student from institute Y should be female. Institute X should also have 2 male and 1 female student. 

Both the male students from Y minor in Finance, while the female student from Y majors in Operations. Barun is from Y and majors in Operations. Chetan is from X and majors in Finance. 

It has been given that one female student and 2 male students major in finance. We know that the male student from Y minors in finance. Therefore, he cannot major in finance. Therefore, both the male students from X should major in finance. 

 

Daisy and Amit are from the same institute. Therefore, Daisy cannot be from institute Z (since Amit is a male student and both the students from Z are female). Daisy minors in operations. The girl from institute Y majors in Operations. Therefore, Daisy cannot be from institute Y as well. Daisy and Amit should be from institute X. 3 female students minor in marketing. Therefore, all girls except Daisy should minor in marketing. 

Adriana and Deb are from the same institute. Therefore, both of them should be from institute Y. Bandita and Chitra should be from institute Z. 

Only one male student majors in Operations. We know that Barun is the student. Two male students major in Finance. We know that Amit and Chetan major in finance. Therefore, Deb should major in Marketing. 

If Chitra majors in finance, Bandita cannot major in finance (only one female student majors in finance). She cannot major in marketing as well (since she has a minor degree in marketing). Therefore, Bandita should major in operations and hence, option C. is the right answer. 

Instructions 

An ATM dispenses exactly Rs. 5000 per withdrawal using 100, 200 and 500 rupee notes. The ATM requires every customer to give her preference for one of the three denominations of notes. It then dispenses notes such that the number of notes of the customer’s preferred denomination exceeds the total number of notes of other denominations dispensed to her. 

Q. 39 In how many different ways can the ATM serve a customer who gives 500 rupee notes as her preference? 

Answer:

Explanation: 

It has been given that the customer gives 500 rupee notes as her preferred denomination. 

Therefore, the number of 500 rupee notes dispensed must be greater than the number of the notes of other denominations dispensed. 

If Rs.3500 is dispensed as 500 rupee notes (7 notes), the remaining 1500 rupees should be dispensed using Rs.100 and Rs.200 notes. The minimum number of notes of other denomination required in this case will be 8 (7*200 + 1*100). Therefore, at least Rs.4000 should be dispensed as 500 rupee notes. 

Case (1): 

Rs.4000 is dispensed using 500 rupee notes, 8 five hundred rupee notes will be dispensed. The remaining 1000 rupees cannot be fully dispensed as 100 rupee notes (since 10 notes will be required). If 800 rupees is dispensed as 100 rupee notes, then 9 notes will be required to dispense 1000 rupees (8*100+200). Therefore, we can eliminate these 2 cases. 

If 600 rupees is dispensed using 100 rupee notes, then a minimum of 8 notes will be required to dispense 1000 rupees (6*100 + 2*200). Therefore, we can eliminate this case as well. 

If 400 rupees is dispensed using 100 rupee notes, then 7 notes will be required (4*100+3*200). This is a valid case. If 200 rupees is dispensed using 100 rupee notes, then 6 notes will be required (2*100+4*200). This is a valid case. 1000 rupees can be dispensed using 5 notes of Rs.200. 

Therefore, there are 3 valid cases. 

Case (2): 

Rs.4500 is dispensed using 500 rupee notes. 9 five hundred rupee notes will be dispensed in this case. The remaining 500 rupees can be dispensed as 100 rupee notes ( 5 notes) or a combination of 100 rupee and 200 rupee notes. 

200*a + 100*b = 500 

‘a’ can take 0, 1, and 2. 

Therefore, there are 3 valid cases. 

Case (3): 

5000 rupees is dispensed using 10 five hundred rupee notes. 

There is only 1 valid case. 

Total number of valid cases = 3+3+1 = 7. 

Therefore, 7 is the right answer. 

Q. 40 If the ATM could serve only 10 customers with a stock of fifty 500 rupee notes and a sufficient number of notes of other denominations, what is the maximum number of customers among these 10 who could have given 500 rupee notes as their preferences? 

Answer:

Explanation: 

If a customer gives 500 rupee notes as her preferred denomination, the number of 500 rupee notes dispensed must be greater than the number of the notes of other denominations dispensed. 

If Rs.3500 is dispensed as 500 rupee notes (7 notes), the remaining 1500 rupees should be dispensed using Rs.100 and Rs.200 notes. The minimum number of notes of other denomination required in this case will be 8 (7*200 + 1*100). Therefore, at least Rs.4000 should be dispensed as 500 rupee notes. 

Case (1): 

Rs.4000 is dispensed using 500 rupee notes, 8 five hundred rupee notes will be dispensed. 

The remaining 1000 rupees cannot be fully dispensed as 100 rupee notes (since 10 notes will be required). If 800 rupees is dispensed as 100 rupee notes, then 9 notes will be required to dispense 1000 rupees (8*100+200). Therefore, we can eliminate these 2 cases. 

If 600 rupees is dispensed using 100 rupee notes, then a minimum of 8 notes will be required to dispense 1000 rupees (6*100 + 2*200). Therefore, we can eliminate this case as well. 

If 400 rupees is dispensed using 100 rupee notes, then 7 notes will be required (4*100+3*200). This is a valid case. If 200 rupees is dispensed using 100 rupee notes, then 6 notes will be required (2*100+4*200). This is a valid case. 1000 rupees can be dispensed using 5 notes of Rs.200. 

Therefore, there are 3 valid cases. 

Case (2): 

Rs.4500 is dispensed using 500 rupee notes. 9 five hundred rupee notes will be dispensed in this case. The remaining 500 rupees can be dispensed as 100 rupee notes ( 5 notes) or a combination of 100 rupee and 200 rupee notes. 

200*a + 100*b = 500 

‘a’ can take 0, 1, and 2. 

Therefore, there are 3 valid cases. 

Case (3): 

5000 rupees is dispensed using 10 five hundred rupee notes. 

There is only 1 valid case. 

It has been given that the ATM could serve only 10 customers with a stock of fifty 500 rupee notes. We have to find the maximum number of customers who could have given Rs.500 as their preference. 

The least number of 500 rupee notes required to serve a customer who has given Rs.500 as the preference is 8. Using 50 five hundred rupee notes, we can serve [500/8] = 6 customers. Therefore, 6 is the correct answer. 

Q. 41 What is the maximum number of customers that the ATM can serve with a stock of fifty 500 rupee notes and a sufficient number of notes of other denominations, if all the customers are to be served with at most 20 notes per withdrawal? 

A. 12 

B. 10 

C. 13 

D. 16 

Answer: A. 

Explanation: 

It has been given that the customer has to receive 20 notes at the maximum. Also, we have restrictions on the number of 500 rupee notes (fifty) but we do not have any restriction on the number of notes of other denominations. Therefore, in order to serve the maximum number of customers, we have to minimize the number of 500 rupee notes dispensed as much as possible. 

If no 500 rupee note is dispensed, then a minimum of 25 notes will be required (25 200 rupee notes). If one 500 rupee note is dispensed, then a minimum of one 100 rupee note and twenty two 200 rupee notes will be required. The total number of notes required = 1 + 1 + 22 = 24. Therefore, we can eliminate this case. 

If two 500 rupee notes are dispensed, then a minimum of 20 two hundred rupee notes will be required. We can eliminate this case as well since the number of notes required is greater than 20. 

If three 500 rupee notes are dispensed, then a minimum of 1 hundred rupee note and 17 two hundred rupee notes will be required. The number of notes required in this case is 3+1+17 = 21. Therefore, we can eliminate this case as well. 

If four 500 rupee notes are dispensed, then a minimum of 15 two hundred rupee notes will be required. Total number of notes required in this case is 4+15 = 19 < 20. Therefore, this is a valid case. 

The least number of 500 rupee notes with which we can serve a customer such that the total number of notes dispensed does not exceed 20 is 4. Therefore, a maximum of [50/4] = 12 customers can be served with 50 five hundred rupee notes and hence, option A. is the right answer. 

Q. 42 What is the number of 500 rupee notes required to serve 50 customers with 500 rupee notes as their preferences and another 50 customers with 100 rupee notes as their preferences, if the total number of notes to be dispensed is the smallest possible? 

A. 900 

B. 800 

C. 750 

D. 1400 

Answer: A. 

Explanation: 

It has been given that the total number of notes dispensed is the smallest possible. Therefore, we have to minimize the number of notes dispensed in each of the 2 cases given. 

The least number of notes required to serve a customer who has given 500 rupees as his preference is 10. 50 customers who have given 500 rupee notes as their preference have to be served. We will require 50*10 = 500 notes for this purpose. 

Let us consider the case when a customer has given Rs.100 as his preference. 

As we have seen, a minimum number of notes will be required when we maximize the number of five hundred rupee notes as much as possible. 

If Rs.4000 is dispensed using 500 rupee notes, the remaining 1000 rupees can be dispensed using ten 100 rupee notes. In this case, the number of 100 rupee notes (10) is greater than the number of 500 rupee notes (8). This is a valid case. 

We have to find if we can reduce the number of notes required any further. 

We cannot increase the number of 500 rupee notes to 9 since only 5 hundred rupee notes can be dispensed, violating the condition that the customer has given 100 as his preferred denomination. 

If we replace two 100 rupee notes with one 200 rupee note, then the number of 100 rupee notes will become 6. The number of 500 rupee notes (8) exceeds the number of 100 rupee note (6). Therefore, dispensing 4000 rupees using 500 rupee notes and the rest using 100 rupee notes represents the optimum condition. 

The minimum number of notes required to serve 1 customer = 8 (five hundred notes) + 10 (hundred notes) = 18 Number of five hundred notes required to serve 50 customers = 8*50 = 400 

Therefore, the total number of notes required = 400 + 500 = 900. 

Therefore, option A. is the right answer. 

Instructions 

You are given an n×n square matrix to be filled with numerals so that no two adjacent cells have the same numeral. Two cells are called adjacent if they touch each other horizontally, vertically or diagonally. So a cell in one of the four corners has three cells adjacent to it, and a cell in the first or last row or column which is not in the corner has five cells adjacent to it. Any other cell has eight cells adjacent to it. 

Q. 43 What is the minimum number of different numerals needed to fill a 3×3 square matrix? Answer:

Explanation: 

Let us use 1 to denote the first number that we fill. We have to fill as many squares with 1 as possible. If we start with the top-left square, we can fill 4 squares with the number 1. 

Now, we can fill number 2 only in 2 of the 5 squares available. 

The 3 squares available now are adjacent to each other. Therefore, we will require at least 2 numbers to fill these squares. 

We need a minimum of 4 numbers to fill a 3×3 square matrix such that no 2 adjacent cells contain the same number. Therefore, 4 is the correct answer. 

Q. 44 What is the minimum number of different numerals needed to fill a 5×5 square matrix? 

Answer:

Explanation: 

Let us consider a 5×5 matrix. Let us start with the top left square and fill number 1 in as many squares as possible.

We have to use a second number, 2 to fill the gap between two 1s. 

All the cells in row 2 and row 4 are adjacent to the cells containing numbers 1 and 2. Therefore, rows 2 and 4 should be filled with a new set of numbers. We need at least 2 numbers to fill a row such that the adjacent cells do not contain the same number (by alternating the numbers in the consecutive cells). Rows 2 and 4 are completely isolated from each other and hence, the same set of numbers can be used to fill both the rows. 

As we can see, a minimum of 4 numbers are required to fill a 5×5 matrix. Therefore, 4 is the correct answer. 

Q. 45 Suppose you are allowed to make one mistake, that is, one pair of adjacent cells can have the same numeral. What is the minimum number of different numerals required to fill a 5×5 matrix? 

A.

B. 16 

C.

D. 25 

Answer: A. 

Explanation: 

Let us consider a 5×5 matrix. Let us start with the top left square and fill number 1 in as many squares as possible.

We have to use a second number, 2 to fill the gap between two 1s. 

All the cells in row 2 and row 4 are adjacent to the cells containing numbers 1 and 2. Therefore, rows 2 and 4 should be filled with a new set of numbers. We need at least 2 numbers to fill a row such that the adjacent cells do not contain the same number (by alternating the numbers in the consecutive cells). Rows 2 and 4 are completely isolated from each other and hence, the same set of numbers can be used to fill both the rows. 

4 numbers are required to fill a 5×5 matrix. 

It has been given that we are allowed to make 1 mistake – One pair of adjacent cells can contain the same number. In the arrangement given above, we can alter any value along the edge to satisfy this condition. For example, the 2 in the bottom-most row can be changed to 4. Still, the number of numbers required to fill the matrix will be 4. 

Another way to approach this problem is as follows: 

We know that a minimum of 4 numbers are required to fill a 5×5 matrix. If we are allowed to make a mistake, then the number of numbers required should either remain the same or go down. 4 is the smallest value among the given options. Therefore, we can be sure that even if we are allowed to make a mistake, 4 numbers will be required to fill the matrix and hence, option A. is the right answer. 

Q. 46 Suppose that all the cells adjacent to any particular cell must have different numerals. What is the minimum number of different numerals needed to fill a 5×5 square matrix? 

A. 25 

B.

C. 16 

D.

Answer: D. 

Explanation: 

It has been given that all the cells adjacent to a cell must have different numerals. Let us start filling the matrix from the central square since the central square has the maximum number of squares adjacent to it (8) and it will be easier to work around the central 9 squares. A. minimum of 9 numbers will be required to fill the central 9 squares. 

Now we have to fill the remaining squares. Let us start with the top left square. We have to check whether the 9 numbers will be sufficient to fill all the squares such that no 2 squares adjacent to a square have the same number. We can use any of the 3 numbers 4, 5, and 6 to fill the top left square since none of the numbers in the second column are adjacent to these numbers. 

Let us assume that we use 4 to fill the top left square. Now, one of the cells with the number 4 has become adjacent to the cell with number 2 and no other cell adjacent to cell with number 2 (in the second row and second column) can have 4 as its neighbour. Similarly, we can fill the first row with numbers 8 and 7. 

In essence, we are trying to create a gird around each of the numbers in the corners of the inner 3×3 matrix such that no 2 cells adjacent to a cell have the same number. Filling the other cells similarly, we get the following matrix as one of the possible cases. 

We need a minimum of 9 numbers to fill a 5×5 matrix such that for any cell, no 2 cells adjacent to it contain the same value. Therefore, option D. is the right answer. 

Instructions 

Fuel contamination levels at each of 20 petrol pumps P1, P2, …, P20 were recorded as either high, medium, or low. 

1. Contamination levels at three pumps among P1 – P5 were recorded as high. 

2. P6 was the only pump among P1 – P10 where the contamination level was recorded as low. 3. P7 and P8 were the only two consecutively numbered pumps where the same levels of contamination were recorded. 4. High contamination levels were not recorded at any of the pumps P16 – P20. 

5. The number of pumps where high contamination levels were recorded was twice the number of pumps where low contamination levels were recorded. 

Q. 47 Which of the following MUST be true? 

A. The contamination level at P20 was recorded as medium. 

B. The contamination level at P13 was recorded as low. 

C. The contamination level at P12 was recorded as high. 

D. The contamination level at P10 was recorded as high. 

Answer: D. 

Explanation: 

Let us draw the table and fill all absolute information present. 

In statement 3, it is given that P7 and P8 were the only two consecutively numbered pumps where the same levels of contamination were recorded. 

In statement 1, it is given that contamination levels at three pumps among P1 – P5 were recorded as high. This is only possible when pumps 1, 3 and 5 have high level of contamination. Also, P6 was the only pump among P1 – P10 where the contamination level was recorded as low. Therefore, we can say that pumps 2 and 4 have medium level of contamination. 

It is given that High contamination levels were not recorded at any of the pumps P16 – P20. Therefore, we can say that High contamination was recorded in only first 15 pumps. Therefore, we can say that the maximum number of pumps that can have high contamination level is ‘8’. (Consecutive pumps don’t have same contamination level except one case) 

Also, it is given that the number of pumps where high contamination levels were recorded was twice the number of pumps where low contamination levels were recorded. Hence, we can say that the number of pumps that have high contamination level is an even number less than or equal to ‘8’. 

If the number of high contamination level pumps is ‘6’, then there will be only ‘3’ pumps with low contamination level. Consequently, we will need 11 (20 – 6 – 3) pumps with medium contamination level which is not possible since the number of pumps of a single type can’t exceed 10.(Consecutive pumps don’t have same contamination level except one case) 

Therefore, we can say that the number of pumps that have high contamination level = 8 

The number of pumps that have low contamination level = 8/2 = 4 

Also, the number of pumps that have medium contamination level = 20 – 8 – 4 = 12 

It is given that P7 and P8 were the only two consecutively numbered pumps where the same levels of contamination were recorded. If P7 and P8 recorded medium contamination level then there can be at max 7 pumps (P1, P3, P5, P9, P11, P13, P15) with high contamination level. Hence, we can say that pumps P7 and P8 recorded High contamination level. Therefore, we can uniquely determine the contamination level till P10. 

It is given that High contamination levels were not recorded at any of the pumps P16 – P20. Therefore, we can say that these 5 pumps recorded low and medium contamination level. There are two cases possible. 

Case 1: When there were 3 Low and 2 Medium contaminated level recorded in pumps P16 – P20. 3 Low contamination level must have recorded in P16, P18 and P20. We can fill the table as follows.

Case 2: When there were 2 Low and 3 Medium contaminated level recorded in pumps P16 – P20. 

3 Medium contamination level must be recorded in P16, P18 and P20. We can fill the table as follows. Let us check the options one by one. 

(Option:A) The contamination level at P20 was recorded as medium. This need not be true as we can see that in Case 1 at P20 low contamination level is recorded. 

(Option:B) The contamination level at P13 was recorded as low. This need not be true as we can see that in Case 2(a), at P13 high contamination level is recorded. 

(Option:C) The contamination level at P12 was recorded as high. This need not be true as we can see that in Case 2(a), at P12 medium contamination level is recorded. 

(Option:D) The contamination level at P10 was recorded as high.. This is true for all cases. Hence, we can say that option D. is the correct answer. 

Q. 48 What best can be said about the number of pumps at which the contamination levels were recorded as medium? 

A. At least 8 

B. More than 4 

C. Exactly 8 

D. At most 9 

Answer: C. 

Explanation: 

Let us draw the table and fill all absolute information present. 

In statement 3, it is given that P7 and P8 were the only two consecutively numbered pumps where the same levels of contamination were recorded. 

In statement 1, it is given that contamination levels at three pumps among P1 – P5 were recorded as high. This is only possible when pumps 1, 3 and 5 have high level of contamination. Also, P6 was the only pump among P1 – P10 where the contamination level was recorded as low. Therefore, we can say that pumps 2 and 4 have medium level of contamination. 

It is given that High contamination levels were not recorded at any of the pumps P16 – P20. Therefore, we can say that High contamination was recorded in only first 15 pumps. Therefore, we can say that the maximum number of pumps that can have high contamination level is ‘8’. (Consecutive pumps don’t have same contamination level except one case) 

Also, it is given that the number of pumps where high contamination levels were recorded was twice the number of pumps where low contamination levels were recorded. Hence, we can say that the number of pumps that have high contamination level is an even number less than or equal to ‘8’. 

If the number of high contamination level pumps is ‘6’, then there will be only ‘3’ pumps with low contamination level. Consequently, we will need 11 (20 – 6 – 3) pumps with medium contamination level which is not possible since the number of pumps of a single type can’t exceed 10.(Consecutive pumps don’t have same contamination level except one case) 

Therefore, we can say that the number of pumps that have high contamination level = 8 

The number of pumps that have low contamination level = 8/2 = 4 

Also, the number of pumps that have medium contamination level = 20 – 8 – 4 = 12 

It is given that P7 and P8 were the only two consecutively numbered pumps where the same levels of contamination were recorded. If P7 and P8 recorded medium contamination level then there can be at max 7 pumps (P1, P3, P5, P9, P11, P13, P15) with high contamination level. Hence, we can say that pumps P7 and P8 recorded High contamination level. Therefore, we can uniquely determine the contamination level till P10. 

It is given that High contamination levels were not recorded at any of the pumps P16 – P20. Therefore, we can say that these 5 pumps recorded low and medium contamination level. There are two cases possible. 

Case 1: When there were 3 Low and 2 Medium contaminated level recorded in pumps P16 – P20. 3 Low contamination level must have recorded in P16, P18 and P20. We can fill the table as follows.

Case 2: When there were 2 Low and 3 Medium contaminated level recorded in pumps P16 – P20. 3 Medium contamination level must have recorded in P16, P18 and P20. We can fill the table as follows. 

We know that medium contamination level was recorded at exactly 8 pumps. Hence, option C. is the correct answer. 

Q. 49 If the contamination level at P11 was recorded as low, then which of the following MUST be true? 

A. The contamination level at P12 was recorded as high. 

B. The contamination level at P15 was recorded as medium. 

C. The contamination level at P18 was recorded as low. 

D. The contamination level at P14 was recorded as medium. 

Answer: D. 

Explanation: 

Let us draw the table and fill all absolute information present. 

In statement 3, it is given that P7 and P8 were the only two consecutively numbered pumps where the same levels of contamination were recorded. 

In statement 1, it is given that contamination levels at three pumps among P1 – P5 were recorded as high. This is only possible when pumps 1, 3 and 5 have high level of contamination. Also, P6 was the only pump among P1 – P10 where the contamination level was recorded as low. Therefore, we can say that pumps 2 and 4 have medium level of contamination. 

It is given that High contamination levels were not recorded at any of the pumps P16 – P20. Therefore, we can say that High contamination was recorded in only first 15 pumps. Therefore, we can say that the maximum number of pumps that can have high contamination level is ‘8’. (Consecutive pumps don’t have same contamination level except one case) 

Also, it is given that the number of pumps where high contamination levels were recorded was twice the number of pumps where low contamination levels were recorded. Hence, we can say that the number of pumps that have high contamination level is an even number less than or equal to ‘8’. 

If the number of high contamination level pumps is ‘6’, then there will be only ‘3’ pumps with low contamination level. Consequently, we will need 11 (20 – 6 – 3) pumps with medium contamination level which is not possible since the number of pumps of a single type can’t exceed 10.(Consecutive pumps don’t have same contamination level except one case) 

Therefore, we can say that the number of pumps that have high contamination level = 8 

The number of pumps that have low contamination level = 8/2 = 4 

Also, the number of pumps that have medium contamination level = 20 – 8 – 4 = 12 

It is given that P7 and P8 were the only two consecutively numbered pumps where the same levels of contamination were recorded. If P7 and P8 recorded medium contamination level then there can be at max 7 pumps (P1, P3, P5, P9, P11, P13, P15) with high contamination level. Hence, we can say that pumps P7 and P8 recorded High contamination level. Therefore, we can uniquely determine the contamination level till P10. 

It is given that High contamination levels were not recorded at any of the pumps P16 – P20. Therefore, we can say that these 5 pumps recorded low and medium contamination level. There are two cases possible. 

Case 1: When there were 3 Low and 2 Medium contaminated level recorded in pumps P16 – P20. 3 Low contamination level must have recorded in P16, P18 and P20. We can fill the table as follows. 

Case 2: When there were 2 Low and 3 Medium contaminated level recorded in pumps P16 – P20. 3 Medium contamination level must have recorded in P16, P18 and P20. We can fill the table as follows. 

We can see that in case 2(a) the contamination level at P11 was recorded as low. Let us check all the option one by 

one. 

(Option : A) The contamination level at P12 was recorded as high. This statement is incorrect as we can see in the table, the contamination level at P12 was recorded as medium. 

(Option : B) The contamination level at P15 was recorded as medium. This statement is incorrect as we can see in the table, the contamination level at P15 was recorded as High. 

(Option : C) The contamination level at P18 was recorded as low. This statement is incorrect as we can see in the table, the contamination level at P18 was recorded as Medium. 

(Option : D) The contamination level at P14 was recorded as medium. This statement is correct as we can see in the table, the contamination level at P14 was recorded as Medium. Hence, we can say that option D. is the correct answer. 

Q. 50 If contamination level at P15 was recorded as medium, then which of the following MUST be FALSE? 

A. Contamination levels at P13 and P17 were recorded as the same. 

B. Contamination levels at P11 and P16 were recorded as the same. 

C. Contamination level at P14 was recorded to be higher than that at P15. 

D. Contamination levels at P10 and P14 were recorded as the same. 

Answer: B. 

Explanation: 

Let us draw the table and fill all absolute information present. 

In statement 3, it is given that P7 and P8 were the only two consecutively numbered pumps where the same levels of contamination were recorded. 

In statement 1, it is given that contamination levels at three pumps among P1 – P5 were recorded as high. This is only possible when pumps 1, 3 and 5 have high level of contamination. Also, P6 was the only pump among P1 – P10 where the contamination level was recorded as low. Therefore, we can say that pumps 2 and 4 have medium level of contamination. 

It is given that High contamination levels were not recorded at any of the pumps P16 – P20. Therefore, we can say that High contamination was recorded in only first 15 pumps. Therefore, we can say that the maximum number of pumps that can have high contamination level is ‘8’. (Consecutive pumps don’t have same contamination level except one case) 

Also, it is given that the number of pumps where high contamination levels were recorded was twice the number of pumps where low contamination levels were recorded. Hence, we can say that the number of pumps that have high contamination level is an even number less than or equal to ‘8’. 

If the number of high contamination level pumps is ‘6’, then there will be only ‘3’ pumps with low contamination level. Consequently, we will need 11 (20 – 6 – 3) pumps with medium contamination level which is not possible since the number of pumps of a single type can’t exceed 10.(Consecutive pumps don’t have same contamination level except one case) 

Therefore, we can say that the number of pumps that have high contamination level = 8 

The number of pumps that have low contamination level = 8/2 = 4 

Also, the number of pumps that have medium contamination level = 20 – 8 – 4 = 12 

It is given that P7 and P8 were the only two consecutively numbered pumps where the same levels of contamination were recorded. If P7 and P8 recorded medium contamination level then there can be at max 7 pumps (P1, P3, P5, P9, P11, P13, P15) with high contamination level. Hence, we can say that pumps P7 and P8 recorded High contamination level. Therefore, we can uniquely determine the contamination level till P10. 

It is given that High contamination levels were not recorded at any of the pumps P16 – P20. Therefore, we can say that these 5 pumps recorded low and medium contamination level. There are two cases possible. 

Case 1: When there were 3 Low and 2 Medium contaminated level recorded in pumps P16 – P20. 3 Low contamination level must have recorded in P16, P18 and P20. We can fill the table as follows.

Case 2: When there were 2 Low and 3 Medium contaminated level recorded in pumps P16 – P20. 3 Medium contamination level must have recorded in P16, P18 and P20. We can fill the table as follows. 

We can see that in case 1 the contamination level at P15 was recorded as medium. Let us check all the option one by one. 

 

(Option :A) Contamination levels at P13 and P17 were recorded as the same. From the table, we can see that the contamination levels at P13 and P17 were recorded as medium. Hence, we can say that this statement is correct. 

(Option :B) Contamination levels at P11 and P16 were recorded as the same. From the table, we can see that the contamination levels at P11 was recorded as Medium whereas at P16 it was recorded as Low. Hence, we can say that this statement is incorrect. Thus, option B. is the correct answer. 

Instructions 

The multi-layered pie-chart below shows the sales of LED. television sets for a big retail electronics outlet during 2016 and 2017. The outer layer shows the monthly sales during this period, with each label showing the month followed by sales figure of that month. For some months, the sales figures are not given in the chart. The middle-layer shows quarterwise aggregate sales figures (in some cases, aggregate quarter-wise sales numbers are not given next to the quarter). The innermost layer shows annual sales. It is known that the sales figures during the three months of the second quarter (April, May, June) of 2016 form an arithmetic progression, as do the three monthly sales figures in the fourth quarter (October, November, December) of that year. 

Q. 51 What is the percentage increase in sales in December 2017 as compared to the sales in December 2016? 

A. 38.46 

B. 22.22 

C. 28.57 

D. 50.00 

Answer: C. 

Explanation: 

We have been given details about the quarterly sales figures. Also, we have been given details about the sales figures every month. Some of the data are missing and some additional conditions have been given in the question. Let us try to complete the pie chart as much as possible with the data available to us. 

It is known that the sales figures during the three months of the second quarter (April, May, June) of 2016 form an arithmetic progression. 

We know that the sales in April is 40. 

Let the sales in May be 40+x and the sales in June be 40+2x. 

We know that the total sales in Q2 is 150. 

=> 40 + 40 + x + 40 + 2x = 150 

3x = 30 

x = 10 

Therefore, sales in May 2016 = 40 + 10 = 50 

Sales in June 2016 = 40 + 20 = 60 

Similarly, it has been given that the sales in October, November, and December 2016 form an arithmetic progression. Sales in October = 100 

Sales in Q4 = 360 

Let the sales in November be 100+y and the sales in December be 100+2y. 

100 + 100 + y + 100 + 2y = 360 

300 + 3y = 360 

=> y = 20 

Sales in November 2016 = 120 and Sales in December 2016 = 140 

Sales in Q1 of 2016 = Sum of the sales in the months of January, February, and March 2016 = 80 + 60 + 100 

= 240 

Sales in Q3 of 2016 = Sum of the sales in the months of July, August, and September 2016 

= 75 + 120 + 55 

= 250 

Sales in Q1 of 2017 = 120 + 100 + 160 = 380 

Sales in Q2 of 2017 = 65 + 75 + 60 = 200 

We know that sales in Q3 of 2017 = 220 

Let the sales in August of 2017 be ‘a’. 

60 + 70 + a = 220 

=> a = 90 

Sales in August 2017 = 90 

We know that sales in Q4 of 2017 = 500 

Let the sales in December of 2017 be ‘d’. 

150 + 170 + d = 500 

=> d = 180 

Sales in December 2017 = 180 

Sales in December 2016 = 140 

Sales in December 2017 = 180 

Percentage change = (180-140)/140 = 40/140 = 28.57% 

Therefore, option C. is the right answer. 

Q. 52 In which quarter of 2017 was the percentage increase in sales from the same quarter of 2016 the highest? 

A. Q2 

B. Q1 

C. Q4 

D. Q3 

Answer: B. 

Explanation: 

We have been given details about the quarterly sales figures. Also, we have been given details about the sales figures every month. Some of the data are missing and some additional conditions have been given in the question. Let us try to complete the pie chart as much as possible with the data available to us. 

It is known that the sales figures during the three months of the second quarter (April, May, June) of 2016 form an arithmetic progression. 

We know that the sales in April is 40. 

Let the sales in May be 40+x and the sales in June be 40+2x. 

We know that the total sales in Q2 is 150. 

=> 40 + 40 + x + 40 + 2x = 150 

3x = 30 

x = 10 

Therefore, sales in May 2016 = 40 + 10 = 50 

Sales in June 2016 = 40 + 20 = 60 

Similarly, it has been given that the sales in October, November, and December 2016 form an arithmetic progression. Sales in October = 100 

Sales in Q4 = 360 

Let the sales in November be 100+y and the sales in December be 100+2y. 

100 + 100 + y + 100 + 2y = 360 

300 + 3y = 360 

=> y = 20 

Sales in November 2016 = 120 and Sales in December 2016 = 140 

Sales in Q1 of 2016 = Sum of the sales in the months of January, February, and March 2016 = 80 + 60 + 100 = 240 

 

Sales in Q3 of 2016 = Sum of the sales in the months of July, August, and September 2016 

= 75 + 120 + 55  = 250 

 

Sales in Q1 of 2017 = 120 + 100 + 160 = 380 

Sales in Q2 of 2017 = 65 + 75 + 60 = 200 

We know that sales in Q3 of 2017 = 220 

Let the sales in August of 2017 be ‘a’. 

60 + 70 + a = 220 => a = 90 

 

Sales in August 2017 = 90 

We know that sales in Q4 of 2017 = 500 

Let the sales in December of 2017 be ‘d’. 

150 + 170 + d = 500 => d = 180 

 

Sales in December 2017 = 180 

Among the given 4 options, we have to find the quarter in which the increase in sale from the previous quarter was the highest. 

Q2: 

Sales in 2017 = 200 

Sales in 2016 = 150 

Q1: 

Sales in 2017 = 380 

Sales in 2016 = 240 

Q3: 

Sales in 2017 = 220 

Sales in 2016 = 250 

Q4: 

Sales in 2017 = 500 

Sales in 2016 = 360 

We can eliminate Q3 since the sales has decreased. 

Growth in Q2 sales = 50/150 = 1/3 = 33.33% 

Growth in Q1 sales = (380-240)/240 = 140/240 = 58.33% 

Growth in Q4 sales = (500-360)/360 = 140/360 

140/240 > 140/360 

Therefore, Q1 has recorded the highest growth in sales and hence, option B. is the right answer. 

Q. 53 During which quarter was the percentage decrease in sales from the previous quarter’s sales the highest? 

A. Q2 of 2017 

B. Q4 of 2017 

C. Q2 of 2016 

D. Q1 of 2017 

Answer: A. 

Explanation: 

We have been given details about the quarterly sales figures. Also, we have been given details about the sales figures every month. Some of the data are missing and some additional conditions have been given in the question. Let us try to complete the pie chart as much as possible with the data available to us. 

It is known that the sales figures during the three months of the second quarter (April, May, June) of 2016 form an arithmetic progression. 

We know that the sales in April is 40. 

Let the sales in May be 40+x and the sales in June be 40+2x. 

We know that the total sales in Q2 is 150. 

=> 40 + 40 + x + 40 + 2x = 150 

3x = 30 

x = 10 

Therefore, sales in May 2016 = 40 + 10 = 50 

Sales in June 2016 = 40 + 20 = 60 

Similarly, it has been given that the sales in October, November, and December 2016 form an arithmetic progression. Sales in October = 100 

Sales in Q4 = 360 

Let the sales in November be 100+y and the sales in December be 100+2y. 

100 + 100 + y + 100 + 2y = 360 

300 + 3y = 360 

=> y = 20 

Sales in November 2016 = 120 and Sales in December 2016 = 140 

Sales in Q1 of 2016 = Sum of the sales in the months of January, February, and March 2016 = 80 + 60 + 100 

= 240 

Sales in Q3 of 2016 = Sum of the sales in the months of July, August, and September 2016 

= 75 + 120 + 55 

= 250 

Sales in Q1 of 2017 = 120 + 100 + 160 = 380 

Sales in Q2 of 2017 = 65 + 75 + 60 = 200 

We know that sales in Q3 of 2017 = 220 

Let the sales in August of 2017 be ‘a’. 

60 + 70 + a = 220 

=> a = 90 

Sales in August 2017 = 90 

We know that sales in Q4 of 2017 = 500 

Let the sales in December of 2017 be ‘d’. 

150 + 170 + d = 500 

=> d = 180 

Sales in December 2017 = 180 

Q2 of 2017: 

Sales in Q2 of 2017 = 200 

Sales in Q1 of 2017 = 380 

% decrease = 180/380 

Q4 of 2017: 

We can eliminate this option since the sales has increased in Q4 of 2017 as compared to the previous quarter. 

Q2 of 2016: 

Sales in Q2 of 2016 = 150 

Sales in Q1 of 2016 = 240 

% decrease = 90/240 

Q1 of 2017: 

Sales in Q1 of 2017 has increased as compared to sales in the previous quarter. We can eliminate this option as well. 180/380 is very close to 50%. 90/240 is closer to 33.33%. Therefore, option A. is the right answer. 

Q. 54 During which month was the percentage increase in sales from the previous month’s sales the highest? 

A. March of 2017 

B. October of 2017 

C. March of 2016 

D. October of 2016 

Answer: B. 

Explanation: 

We have been given details about the quarterly sales figures. Also, we have been given details about the sales figures every month. Some of the data are missing and some additional conditions have been given in the question. Let us try to complete the pie chart as much as possible with the data available to us. 

It is known that the sales figures during the three months of the second quarter (April, May, June) of 2016 form an arithmetic progression. 

We know that the sales in April is 40. 

Let the sales in May be 40+x and the sales in June be 40+2x. 

We know that the total sales in Q2 is 150. 

=> 40 + 40 + x + 40 + 2x = 150 

3x = 30 

x = 10 

Therefore, sales in May 2016 = 40 + 10 = 50 

Sales in June 2016 = 40 + 20 = 60 

Similarly, it has been given that the sales in October, November, and December 2016 form an arithmetic progression. Sales in October = 100 

Sales in Q4 = 360 

Let the sales in November be 100+y and the sales in December be 100+2y. 

100 + 100 + y + 100 + 2y = 360 

300 + 3y = 360 

=> y = 20 

Sales in November 2016 = 120 and Sales in December 2016 = 140 

Sales in Q1 of 2016 = Sum of the sales in the months of January, February, and March 2016 = 80 + 60 + 100 

= 240 

Sales in Q3 of 2016 = Sum of the sales in the months of July, August, and September 2016 

= 75 + 120 + 55 

= 250 

Sales in Q1 of 2017 = 120 + 100 + 160 = 380 

Sales in Q2 of 2017 = 65 + 75 + 60 = 200 

We know that sales in Q3 of 2017 = 220 

Let the sales in August of 2017 be ‘a’. 

60 + 70 + a = 220 

=> a = 90 

Sales in August 2017 = 90 

We know that sales in Q4 of 2017 = 500 

Let the sales in December of 2017 be ‘d’. 

150 + 170 + d = 500 

=> d = 180 

Sales in December 2017 = 180 

March of 2017: 

Sales in March of 2017 = 160 

Sales in February of 2017 = 100 

% increase = 60/100 = 60% 

October of 2017: 

Sales in October of 2017 = 150 

Sales in September of 2017 = 70 

As we can see, the sales has increased by more than 100%. 

March of 2016: 

Sales in March of 2016 = 100 

Sales in February of 2016 = 60 

% increase in sales is less than 100%. 

October of 2016: 

Sales in October of 2016 = 100 

Sales in September of 2016 = 55 

% increase is less than 100% 

As we can see, the percentage increase in sale as compared to the previous month was highest in October of 2017 among the given options. Therefore, option B. is the right answer. 

Instructions 

Twenty four people are part of three committees which are to look at research, teaching, and administration respectively. No two committees have any member in common. No two committees are of the same size. Each committee has three types of people: bureaucrats, educationalists, and politicians, with at least one from each of the three types in each committee. The following facts are also known about the committees: 

1. The numbers of bureaucrats in the research and teaching committees are equal, while the number of bureaucrats in the research committee is 75% of the number of bureaucrats in the administration committee. 2. The number of educationalists in the teaching committee is less than the number of educationalists in the research committee. The number of educationalists in the research committee is the average of the numbers of educationalists in the other two committees. 

3. 60% of the politicians are in the administration committee, and 20% are in the teaching committee. 

Q. 55 Based on the given information, which of the following statements MUST be FALSE? 

A. In the teaching committee the number of educationalists is equal to the number of politicians 

B. In the administration committee the number of bureaucrats is equal to the number of educationalists 

C. The size of the research committee is less than the size of the teaching committee 

D. The size of the research committee is less than the size of the administration committee 

Answer: C. 

Explanation: 

Let us draw a table according to the information given. 

It is given that the numbers of bureaucrats in the research and teaching committees are equal, while the number of bureaucrats in the research committee is 75% of the number of bureaucrats in the administration committee. Let ‘4x’ be the number of bureaucrats in Administration committee. 

The number of educationalists in the teaching committee is less than the number of educationalists in the research committee. The number of educationalists in the research committee is the average of the numbers of educationalists in the other two committees. Let us assume that ‘y’ is the number of educationalists in the research committee and ‘d’ be the difference in the number of educationalists in Research and teaching committees. 

60% of the politicians are in the administration committee, and 20% are in the teaching committee. Let ‘5z’ be the number of total number of politicians. 

We can say that 

⇒ 10x+3y+5z = 24 

We can see that each of x, y and z has to a natural number integer. If x > 1, then both y and z can’t take any natural number. 

Hence, we can say that x = 1. 

At x = 1, 3y+5z = 24. If y = 1 or 2, Z is not an integer. 

At x = 1 and y = 3, z = 1 which is the only possible solution. 

We can see that ‘d’ can assume two possible values. d = 1 or 2. 

Let us check the option one by one. 

Option A: In the teaching committee the number of educationalists is equal to the number of politicians. We can see that in the teaching committee the number of educationalists can be equal to the number of politicians when both the numbers are ‘1’. Hence, this statement can be correct. 

Option B: In the administration committee the number of bureaucrats is equal to the number of educationalists. We can see that in the administration committee the number of bureaucrats can be equal to the number of educationalists when both the numbers are ‘4’. Hence, this statement can be correct. 

Option C: The size of the research committee is less than the size of the teaching committee. We can see the maximum size of teaching committee can be ‘6’ which is less than the size of the research committee. Hence, the sentence is incorrect. 

Option D: The size of the research committee is less than the size of the administration committee. We can see the minimum size of Administration committee can be ‘9’ which is more than the size of the research committee. Hence, this statement is correct. 

Therefore, we can say that option C. is the correct answer. 

Q. 56 What is the number of bureaucrats in the administration committee? 

Answer:

Explanation: 

Let us draw a table according to the information given. 

It is given that the numbers of bureaucrats in the research and teaching committees are equal, while the number of bureaucrats in the research committee is 75% of the number of bureaucrats in the administration committee. Let ‘4x’ be the number of bureaucrats in Administration committee. 

The number of educationalists in the teaching committee is less than the number of educationalists in the research committee. The number of educationalists in the research committee is the average of the numbers of educationalists in the other two committees. Let us assume that ‘y’ is the number of educationalists in the research committee and ‘d’ be the difference in the number of educationalists in Research and teaching committees. 

60% of the politicians are in the administration committee, and 20% are in the teaching committee. Let ‘5z’ be the number of total number of politicians. 

We can say that 

⇒ 10x+3y+5z = 24 

We can see that each of x, y and z has to a natural number integer. If x > 1, then both y and z can’t take any natural number. 

Hence, we can say that x = 1. 

At x = 1, 3y+5z = 24. If y = 1 or 2, Z is not an integer. 

At x = 1 and y = 3, z = 1 which is the only possible solution. 

We can see that ‘d’ can assume two possible values. d = 1 or 2. 

From the table, we can see that the number of bureaucrats in the administration committee = 4. 

Q. 57 What is the number of educationalists in the research committee? 

Answer:

Explanation: 

Let us draw a table according to the information given. 

It is given that the numbers of bureaucrats in the research and teaching committees are equal, while the number of bureaucrats in the research committee is 75% of the number of bureaucrats in the administration committee. Let ‘4x’ be the number of bureaucrats in Administration committee. 

The number of educationalists in the teaching committee is less than the number of educationalists in the research committee. The number of educationalists in the research committee is the average of the numbers of educationalists in the other two committees. Let us assume that ‘y’ is the number of educationalists in the research committee and ‘d’ be the difference in the number of educationalists in Research and teaching committees. 

60% of the politicians are in the administration committee, and 20% are in the teaching committee. Let ‘5z’ be the number of total number of politicians. 

We can say that 

⇒ 10x+3y+5z = 24 

 

We can see that each of x, y and z has to a natural number integer. If x > 1, then both y and z can’t take any natural number. 

Hence, we can say that x = 1. 

At x = 1, 3y+5z = 24. If y = 1 or 2, Z is not an integer. 

At x = 1 and y = 3, z = 1 which is the only possible solution. 

We can see that ‘d’ can assume two possible values. d = 1 or 2. 

From the table, we can see that the number of educationalists in the research committee = 3. 

Q. 58 Which of the following CANNOT be determined uniquely based on the given information? 

A. The size of the teaching committee 

B. The size of the research committee 

C. The total number of bureaucrats in the three committees 

D. The total number of educationalists in the three committees 

Answer: A. 

Explanation: 

Let us draw a table according to the information given. 

It is given that the numbers of bureaucrats in the research and teaching committees are equal, while the number of bureaucrats in the research committee is 75% of the number of bureaucrats in the administration committee. Let ‘4x’ be the number of bureaucrats in Administration committee. 

The number of educationalists in the teaching committee is less than the number of educationalists in the research committee. The number of educationalists in the research committee is the average of the numbers of educationalists in the other two committees. Let us assume that ‘y’ is the number of educationalists in the research committee and ‘d’ be the difference in the number of educationalists in Research and teaching committees. 

 

60% of the politicians are in the administration committee, and 20% are in the teaching committee. Let ‘5z’ be the number of total number of politicians. 

We can say that 

⇒ 10x+3y+5z = 24 

We can see that each of x, y and z has to a natural number integer. If x > 1, then both y and z can’t take any natural number. 

Hence, we can say that x = 1. 

At x = 1, 3y+5z = 24. If y = 1 or 2, Z is not an integer. 

At x = 1 and y = 3, z = 1 which is the only possible solution. 

We can see that ‘d’ can assume two possible values. d = 1 or 2. 

From the table, we can not uniquely determine the size of the teaching committee. Hence, option A. is the correct 

answer. 

Instructions 

1600 satellites were sent up by a country for several purposes. The purposes are classified as broadcasting (B), communication (C), surveillance (S), and others (O). A. satellite can serve multiple purposes; however a satellite serving either B, or C, or S does not serve O. The following facts are known about the satellites: 

1. The numbers of satellites serving B, C, and S (though may be not exclusively) are in the ratio 2:1:1. 2. The number of satellites serving all three of B, C, and S is 100. 

3. The number of satellites exclusively serving C. is the same as the number of satellites exclusively serving S. This number is 30% of the number of satellites exclusively serving B. 

4. The number of satellites serving O is the same as the number of satellites serving both C. and S but not B. 

Q. 59 What best can be said about the number of satellites serving C? 

A. Must be at least 100 

B. Cannot be more than 800 

C. Must be between 450 and 725 

D. Must be between 400 and 800 

Answer: C. 

Explanation: 

It is given that a satellite serving either B, or C, or S does not serve O. So we can say that it’s basically 3 satellites broadcasting (B), communication (C), surveillance (S) which can have intersections. Those satellites which are not part of any category are placed in others. We can draw the Venn diagram as follows. 

1. The numbers of satellites serving B, C, and S (though may be not exclusively) are in the ratio 2:1:1. 2. The number of satellites serving all three of B, C, and S is 100. 

3. The number of satellites exclusively serving C. is the same as the number of satellites exclusively serving S. This number is 30% of the number of satellites exclusively serving B. 

4. The number of satellites serving O is the same as the number of satellites serving both C. and S but not B. 

Let ’10x’ be the number of satellites exclusively serving B. Then, the number of satellites exclusively serving C. and S = 0.30*10x = 3x

Let ‘y’ be the number of satellites serving others(O). 

Let ‘z’ be the number of satellites serving B, C. but not S. Since the numbers of satellites serving B, C, and S (though may be not exclusively) are in the ratio 2:1:1. Therefore, we can can say that number of satellites serving B, S but not C. = z. 

It is given that 

⇒ 10x+2z+2y+6x = 1600 

⇒ 8x+z+y = 750 … (1) 

The numbers of satellites serving B, C, and S (though may be not exclusively) are in the ratio 2:1:1. 

10x + 2z + 100 

⇒ z + 100 + 3x + y = 2/1

⇒ 10x + 2z + 100 = 2(z + 100 + 3x + y) ⇒ 4x = 100 + 2y 

⇒ 2x = 50 + y 

⇒ y = 2x − 50 …(2)

We can substitute this in equation (1) 

⇒ 8x+z+2x – 50 = 750 

⇒ z = 800 – 10x … (3) 

Let us define boundary condition for x, 

⇒ 2x – 50≥0 

⇒ x≥25 

Also, 800 – 10x ≥ 0 

⇒ x ≤80 

Therefore, we can say that xϵ[25, 80]. 

The number of satellites serving C. = 800 – 10x + 100 + 3x + 2x – 50 = 850 – 5x 

At x = 25, The number of satellites serving C. = 850 – 5x = 850 – 5*25 = 725 

At x = 80, The number of satellites serving C. = 850 – 5x = 850 – 5*80 = 450 

Hence, we can say that the number of satellites serving C. must be between 450 and 725. Hence, option C. is the correct answer. 

Q. 60 What is the minimum possible number of satellites serving B. exclusively? 

A. 250 

B. 100 

C. 500 

D. 200 

Answer: A. 

Explanation: 

It is given that a satellite serving either B, or C, or S does not serve O. So we can say that it’s basically 3 satellites broadcasting (B), communication (C), surveillance (S) which can have intersections. Those satellites which are not part of any category are placed in others. We can draw the Venn diagram as follows. 

1. The numbers of satellites serving B, C, and S (though may be not exclusively) are in the ratio 2:1:1. 2. The number of satellites serving all three of B, C, and S is 100. 3. The number of satellites exclusively serving C. is the same as the number of satellites exclusively serving S. This number is 30% of the number of satellites exclusively serving B. 4. The number of satellites serving O is the same as the number of satellites serving both C. and S but not B. 

Let ’10x’ be the number of satellites exclusively serving B. Then, the number of satellites exclusively serving C. and S = 0.30*10x = 3x 

Let ‘y’ be the number of satellites serving others(O). 

Let ‘z’ be the number of satellites serving B, C. but not S. Since the numbers of satellites serving B, C, and S (though may be not exclusively) are in the ratio 2:1:1. Therefore, we can can say that number of satellites serving B, S but not C. = z. 

It is given that 

⇒ 10x+2z+2y+6x = 1600 

⇒ 8x+z+y = 750 … (1) 

The numbers of satellites serving B, C, and S (though may be not exclusively) are in the ratio 2:1:1. 

10x + 2z + 100 

⇒ z + 100 + 3x + y =2/1 

⇒ 10x + 2z + 100 = 2(z + 100 + 3x + y) ⇒ 4x = 100 + 2y 

⇒ 2x = 50 + y 

⇒ y = 2x − 50 … (2) 

We can substitute this in equation (1) 

⇒ 8x+z+2x – 50 = 750 

⇒ z = 800 – 10x … (3) 

Let us define boundary condition for x, 

⇒ 2x – 50≥0 

⇒ x≥25 

Also, 800 – 10x≥0 

⇒ x≤80 

Therefore, we can say that x ϵ [25, 80]. 

The number of satellites serving B. exclusively = 10x. This will be minimum when ‘x’ is minimum. 

At x = 25, The number of satellites serving B. exclusively = 10*25 =250. Hence, option A. is the correct answer. min 

Q. 61 If at least 100 of the 1600 satellites were serving O, what can be said about the number of satellites serving S? 

A. At most 475 

B. Exactly 475 

C. No conclusion is possible based on the given information 

D. At least 475 

Answer: A. 

Explanation: 

It is given that a satellite serving either B, or C, or S does not serve O. So we can say that it’s basically 3 satellites broadcasting (B), communication (C), surveillance (S) which can have intersections. Those satellites which are not part of any category are placed in others. We can draw the Venn diagram as follows. 

1. The numbers of satellites serving B, C, and S (though may be not exclusively) are in the ratio 2:1:1. 2. The number of satellites serving all three of B, C, and S is 100. 3. The number of satellites exclusively serving C. is the same as the number of satellites exclusively serving S. This number is 30% of the number of satellites exclusively serving B. 4. The number of satellites serving O is the same as the number of satellites serving both C. and S but not B. 

Let ’10x’ be the number of satellites exclusively serving B. Then, the number of satellites exclusively serving C. and S = 0.30*10x = 3x 

Let ‘y’ be the number of satellites serving others(O). 

Let ‘z’ be the number of satellites serving B, C. but not S. Since the numbers of satellites serving B, C, and S (though may be not exclusively) are in the ratio 2:1:1. Therefore, we can can say that number of satellites serving B, S but not C. = z. 

It is given that 

⇒ 10x+2z+2y+6x = 1600 

⇒ 8x+z+y = 750 … (1) 

The numbers of satellites serving B, C, and S (though may be not exclusively) are in the ratio 2:1:1. 

10x + 2z + 100

⇒ z + 100 + 3x + y = 2/1

⇒ 10x + 2z + 100 = 2(z + 100 + 3x + y) ⇒ 4x = 100 + 2y 

⇒ 2x = 50 + y 

⇒ y = 2x − 50 … (2) 

We can substitute this in equation (1) 

⇒ 8x+z+2x – 50 = 750 

⇒ z = 800 – 10x … (3) 

Let us define boundary condition for x, 

⇒ 2x – 50 ≥0 

⇒ x ≥25 

Also, 800 – 10x ≥0 

⇒ x≤80 

Therefore, we can say that xϵ[25, 80]. 

It is given that at least 100 of the 1600 satellites were serving O. 

⇒ 2x – 50≥100 

⇒ x≥75 

The number of satellites serving S = 100 + 800 – 10x + 2x – 50 + 3x = 850 – 5x 

At x = 75, the number of satellites serving S = 850 – 5*75 = 475 

min 

At x = 80, the number of satellites serving S = 850 – 5*80 = 450 

max 

Hence, we can say that the number of satellites serving S must be from 425 to 475. Therefore, we can say that option A. is the correct answer. 

Q. 62 If the number of satellites serving at least two among B, C, and S is 1200, which of the following MUST be FALSE? 

A. The number of satellites serving B. exclusively is exactly 250 

B. The number of satellites serving B. is more than 1000 

C. The number of satellites serving C. cannot be uniquely determined 

D. All 1600 satellites serve B. or C. or S 

Answer: C. 

Explanation: 

It is given that a satellite serving either B, or C, or S does not serve O. So we can say that it’s basically 3 satellites broadcasting (B), communication (C), surveillance (S) which can have intersections. Those satellites which are not part of any category are placed in others. We can draw the Venn diagram as follows. 

1. The numbers of satellites serving B, C, and S (though may be not exclusively) are in the ratio 2:1:1. 2. The number of satellites serving all three of B, C, and S is 100. 3. The number of satellites exclusively serving C. is the same as the number of satellites exclusively serving S. This number is 30% of the number of satellites exclusively serving B. 4. The number of satellites serving O is the same as the number of satellites serving both C. and S but not B. 

Let ’10x’ be the number of satellites exclusively serving B. Then, the number of satellites exclusively serving C. and S = 0.30*10x = 3x 

Let ‘y’ be the number of satellites serving others(O). 

Let ‘z’ be the number of satellites serving B, C. but not S. Since the numbers of satellites serving B, C, and S (though may be not exclusively) are in the ratio 2:1:1. Therefore, we can can say that number of satellites serving B, S but not C. =z

It is given that 

⇒ 10x+2z+2y+6x = 1600 

⇒ 8x+z+y = 750 … (1) 

The numbers of satellites serving B, C, and S (though may be not exclusively) are in the ratio 2:1:1. 

10x + 2z + 100 

⇒ z + 100 + 3x + y =2/1 

⇒ 10x + 2z + 100 = 2(z + 100 + 3x + y) ⇒ 4x = 100 + 2y 

⇒ 2x = 50 + y 

⇒ y = 2x − 50 … (2) 

We can substitute this in equation (1)

⇒ 8x+z+2x – 50 = 750 

⇒ z = 800 – 10x … (3)

Let us define boundary condition for x, 

⇒ 2x – 50≥0 

⇒ x≥ 25 

Also, 800 – 10x≥0 

⇒ x≤80 

Therefore, we can say that xϵ[25, 80]. 

It is given that the number of satellites serving at least two among B, C, and S is 1200. 

⇒ 800 – 10x + 800 – 10x + 2x -50 + 100 = 1200 

⇒ 18x = 450 

⇒ x = 25 

We can determine number of satellites in each of the following category. Hence, option C. is definitely false. Therefore, we can say that option C. is incorrect. 

Instructions 

A. company administers a written test comprising of three sections of 20 marks each – Data Interpretation (DI), Written English (WE) and General Awareness (GA), for recruitment. A. composite score for a candidate (out of 80) is calculated by 

doubling her marks in DI and adding it to the sum of her marks in the other two sections. Candidates who score less than 70% marks in two or more sections are disqualified. From among the rest, the four with the highest composite scores are recruited. If four or less candidates qualify, all who qualify are recruited. 

Ten candidates appeared for the written test. Their marks in the test are given in the table below. Some marks in the table are missing, but the following facts are known: 

1. No two candidates had the same composite score. 

2. Ajay was the unique highest scorer in WE. 

3. Among the four recruited, Geeta had the lowest composite score. 

4. Indu was recruited. 

5. Danish, Harini, and Indu had scored the same marks the in GA. 

6. Indu and Jatin both scored 100% in exactly one section and Jatin’s composite score was 10 more than Indu’s.

Q. 63 Which of the following statements MUST be true? 

1. Jatin’s composite score was more than that of Danish. 

2. Indu scored less than Chetna in DI. 

3. Jatin scored more than Indu in GA. 

A. Both 2 and 3 

B. Only 1 

C. Only 2 

D. Both 1 and 2 

Answer: D. 

Explanation: 

It is given that Indu and Jatin both scored 100% in exactly one section. We can say that Jatin scored 100% marks in DI. Therefore, Jatin’s composite score = 2*20+16+14 = 70 

It is given that Jatin’s composite score was 10 more than Indu’s. Therefore, we can say that Indu’s composite score = 70 – 10 = 60. 

Indu also scored 100% in exactly one section. 

Case 1: Indu scored 100% marks in DI. 

If Indu scored 100% marks in DI, then Indu’s score in GA. = 60 – 2*20 – 8 = 12 which is less than 70% of maximum possible marks. Indu already has less than 70% in WE, therefore we Indu can’t be recruited . Hence, we can reject this case. 

Consequently, we can say that Indu scored 100% marks in WE. Therefore, Indu’s score in DI =(60 − 8 − 20)/2  = 16 It is also given that Danish, Harini, and Indu had scored the same marks the in GA. 

We are given that, among the four recruited, Geeta had the lowest composite score. 

Maximum composite score that Geeta can get = 2*14 + 6 + 20 = 54 {Assuming 100% marks in WE}. Since, Geeta was recruited at a composite score of 54 or less we can say that Ester was definitely recruited. 

It is given that no two candidates had the same composite score. We can see that Chetna’s composite score is 54. Hence, Geeta can’t have a composite score of 54. Therefore, we can say that Geeta’s composite score is 53 or less. 

We already know the four people(Jatin, Indu, Geeta, Ester) which were recruited. Hence, we cab say that Danish was rejected at a composite score of 51. Hence, we can say that Geeta’s composite score in 52 or more. 

Consequently, we can say that Geeta’s composite score if either 52 or 53. Therefore we can say that Geeta scored either 18 {52-(2*14+6)} or 19 {53-(2*14+6)} marks in WE. 

Ajay was the unique highest scorer in WE. 

Case 1: Geeta scored 19 marks in WE. 

We can say that if Geeta scored 19 marks in WE, then Ajay scored 20 marks in DI. In that case Ajay’s composite score = 2*8 + 20 + 16 = 52. Which is a possible case. 

Case 1: Geeta scored 18 marks in WE. 

We can say that if Geeta scored 18 marks in WE, then Ajay can score either 19 or 20 marks in DI. 

If Ajay scored 20 marks in DI then in that case Ajay’s composite score = 2*8 + 20 + 16 = 52 which will be same as Geeta’s composite score. Hence, we can say that in this case Ajay can’t score 20 marks. 

If Ajay scored 19 marks in DI then in that case Ajay’s composite score = 2*8 + 19 + 16 = 51 which will be same as Danish’s composite score. Hence, we can say that in this case Ajay can’t score 19 marks. 

Therefore, we can say that case 2 is not possible at all. 

Let us check all the statement one by one. 

Statement 1: Jatin’s composite score was more than that of Danish. We can see that this statement is correct. Statement 2: Indu scored less than Chetna in DI. We can see that Indu scored 16 marks in DI whereas Chetna scored 19 marks in DI. Hence, we can say that this statement is also correct. 

Statement 3: Jatin scored more than Indu in GA. We can see that Jatin scored 14 marks in GA. whereas Indu scored 20 marks in GA. Hence, we can say that this statement is incorrect. 

Hence, we can say that option D. is the correct answer. 

Q. 64 Which of the following statements MUST be FALSE? 

A. Bala scored same as Jatin in DI 

B. Harini’s composite score was less than that of Falak 

C. Bala’s composite score was less than that of Ester 

D. Chetna scored more than Bala in DI 

Answer: A. 

Explanation: 

It is given that Indu and Jatin both scored 100% in exactly one section. We can say that Jatin scored 100% marks in DI. Therefore, Jatin’s composite score = 2*20+16+14 = 70 

It is given that Jatin’s composite score was 10 more than Indu’s. Therefore, we can say that Indu’s composite score = 70 – 10 = 60. 

Indu also scored 100% in exactly one section. 

Case 1: Indu scored 100% marks in DI. 

If Indu scored 100% marks in DI, then Indu’s score in GA. = 60 – 2*20 – 8 = 12 which is less than 70% of maximum possible marks. Indu already has less than 70% in WE, therefore we Indu can’t be recruited . Hence, we can reject this case. 

Consequently, we can say that Indu scored 100% marks in WE. Therefore, Indu’s score in DI =(60 − 8 − 20)/2 = 16 It is also given that Danish, Harini, and Indu had scored the same marks the in GA. 

We are given that, among the four recruited, Geeta had the lowest composite score. 

Maximum composite score that Geeta can get = 2*14 + 6 + 20 = 54 {Assuming 100% marks in WE}. Since, Geeta was recruited at a composite score of 54 or less we can say that Ester was definitely recruited. 

It is given that no two candidates had the same composite score. We can see that Chetna’s composite score is 54. Hence, Geeta can’t have a composite score of 54. Therefore, we can say that Geeta’s composite score is 53 or less. 

We already know the four people(Jatin, Indu, Geeta, Ester) which were recruited. Hence, we cab say that Danish was rejected at a composite score of 51. Hence, we can say that Geeta’s composite score in 52 or more. 

Consequently, we can say that Geeta’s composite score if either 52 or 53. Therefore we can say that Geeta scored either 18 {52-(2*14+6)} or 19 {53-(2*14+6)} marks in WE. 

Ajay was the unique highest scorer in WE. 

Case 1: Geeta scored 19 marks in WE. 

We can say that if Geeta scored 19 marks in WE, then Ajay scored 20 marks in DI. In that case Ajay’s composite score = 2*8 + 20 + 16 = 52. Which is a possible case. 

Case 1: Geeta scored 18 marks in WE. 

We can say that if Geeta scored 18 marks in WE, then Ajay can score either 19 or 20 marks in DI. 

If Ajay scored 20 marks in DI then in that case Ajay’s composite score = 2*8 + 20 + 16 = 52 which will be same as Geeta’s composite score. Hence, we can say that in this case Ajay can’t score 20 marks. 

If Ajay scored 19 marks in DI then in that case Ajay’s composite score = 2*8 + 19 + 16 = 51 which will be same as Danish’s composite score. Hence, we can say that in this case Ajay can’t score 19 marks. 

Therefore, we can say that case 2 is not possible at all. 

Let us check all the statement one by one. 

Option A: Bala scored same as Jatin in DI. We can say that Bala scored 20 marks in DI. In that Bala’s composite score = 2*20 + 9 + 11 = 60 which is sam as Indu’s composite score. Therefore, we can say that this is a false statement. Hence, option A. is the correct answer. 

Q. 65 If all the candidates except Ajay and Danish had different marks in DI, and Bala’s composite score was less than Chetna’s composite score, then what is the maximum marks that Bala could have scored in DI? 

Answer:13 

Explanation: 

It is given that Indu and Jatin both scored 100% in exactly one section. We can say that Jatin scored 100% marks in DI. Therefore, Jatin’s composite score = 2*20+16+14 = 70 

It is given that Jatin’s composite score was 10 more than Indu’s. Therefore, we can say that Indu’s composite score = 70 – 10 = 60. 

Indu also scored 100% in exactly one section. 

Case 1: Indu scored 100% marks in DI. 

If Indu scored 100% marks in DI, then Indu’s score in GA. = 60 – 2*20 – 8 = 12 which is less than 70% of maximum possible marks. Indu already has less than 70% in WE, therefore we Indu can’t be recruited . Hence, we can reject this case. 

Consequently, we can say that Indu scored 100% marks in WE. Therefore, Indu’s score in DI =(60 − 8 − 20)/2 = 16 

It is also given that Danish, Harini, and Indu had scored the same marks the in GA. 

We are given that, among the four recruited, Geeta had the lowest composite score. 

Maximum composite score that Geeta can get = 2*14 + 6 + 20 = 54 {Assuming 100% marks in WE}. Since, Geeta was recruitedat a composite score of 54 or less we can say that Ester was definitely recruited. 

It is given that no two candidates had the same composite score. We can see that Chetna’s composite score is 54. 

Hence, Geeta can’t have a composite score of 54. Therefore, we can say that Geeta’s composite score is 53 or less. 

We already know the four people(Jatin, Indu, Geeta, Ester) which were recruited. Hence, we cab say that Danish was rejected at a composite score of 51. Hence, we can say that Geeta’s composite score in 52 or more. 

Consequently, we can say that Geeta’s composite score if either 52 or 53. Therefore we can say that Geeta scored either 18 {52-(2*14+6)} or 19 {53-(2*14+6)} marks in WE. 

Ajay was the unique highest scorer in WE. 

Case 1: Geeta scored 19 marks in WE. 

We can say that if Geeta scored 19 marks in WE, then Ajay scored 20 marks in DI. In that case Ajay’s composite score = 2*8 + 20 + 16 = 52. Which is a possible case. 

Case 1: Geeta scored 18 marks in WE. 

We can say that if Geeta scored 18 marks in WE, then Ajay can score either 19 or 20 marks in DI. 

If Ajay scored 20 marks in DI then in that case Ajay’s composite score = 2*8 + 20 + 16 = 52 which will be same as Geeta’s composite score. Hence, we can say that in this case Ajay can’t score 20 marks. 

If Ajay scored 19 marks in DI then in that case Ajay’s composite score = 2*8 + 19 + 16 = 51 which will be same as Danish’s composite score. Hence, we can say that in this case Ajay can’t score 19 marks. 

Therefore, we can say that case 2 is not possible at all. 

It is given that all the candidates except Ajay and Danish had different marks in DI and Bala’s composite score was less than Chetna’s composite score. 

Let us assume that Bala scored ‘x’ marks in DI. 

⇒ 2*x + 9 + 11 < 54 

⇒ x < 17 

We can see that Bala’s score will be less than 17. Bala’s maximum score in DI will be the largest possibel number less than 17 which is not same as any other candidate’s score in DI. From, the table we can see that 16, 15 and 14 are already taken by Indu, Falak and Geeta respectively. 

Therefore, we can say that Bala can score a maximum of 13 marks in DI. 

Q. 66 If all the candidates scored different marks in WE then what is the maximum marks that Harini could have scored in WE? 

Answer:14 

Explanation: 

It is given that Indu and Jatin both scored 100% in exactly one section. We can say that Jatin scored 100% marks in DI. Therefore, Jatin’s composite score = 2*20+16+14 = 70 

It is given that Jatin’s composite score was 10 more than Indu’s. Therefore, we can say that Indu’s composite score = 70 – 10 = 60. 

Indu also scored 100% in exactly one section. 

Case 1: Indu scored 100% marks in DI. 

If Indu scored 100% marks in DI, then Indu’s score in GA. = 60 – 2*20 – 8 = 12 which is less than 70% of maximum possible marks. Indu already has less than 70% in WE, therefore we Indu can’t be recruited . Hence, we can reject this case.  

Consequently, we can say that Indu scored 100% marks in WE. Therefore, Indu’s score in DI =(60 − 8 − 20)/2 = 16 

It is also given that Danish, Harini, and Indu had scored the same marks the in GA. 

We are given that, among the four recruited, Geeta had the lowest composite score. 

Maximum composite score that Geeta can get = 2*14 + 6 + 20 = 54 {Assuming 100% marks in WE}. Since, Geeta was recruitedat a composite score of 54 or less we can say that Ester was definitely recruited. 

It is given that no two candidates had the same composite score. We can see that Chetna’s composite score is 54. Hence, Geeta can’t have a composite score of 54. Therefore, we can say that Geeta’s composite score is 53 or less. 

We already know the four people(Jatin, Indu, Geeta, Ester) which were recruited. Hence, we cab say that Danish was rejected at a composite score of 51. Hence, we can say that Geeta’s composite score in 52 or more. 

Consequently, we can say that Geeta’s composite score if either 52 or 53. Therefore we can say that Geeta scored either 18 {52-(2*14+6)} or 19 {53-(2*14+6)} marks in WE. 

Ajay was the unique highest scorer in WE. 

Case 1: Geeta scored 19 marks in WE. 

We can say that if Geeta scored 19 marks in WE, then Ajay scored 20 marks in DI. In that case Ajay’s composite score = 2*8 + 20 + 16 = 52. Which is a possible case. 

Case 1: Geeta scored 18 marks in WE. 

We can say that if Geeta scored 18 marks in WE, then Ajay can score either 19 or 20 marks in DI. 

If Ajay scored 20 marks in DI then in that case Ajay’s composite score = 2*8 + 20 + 16 = 52 which will be same as Geeta’s composite score. Hence, we can say that in this case Ajay can’t score 20 marks. 

If Ajay scored 19 marks in DI then in that case Ajay’s composite score = 2*8 + 19 + 16 = 51 which will be same as Danish’s composite score. Hence, we can say that in this case Ajay can’t score 19 marks. 

Therefore, we can say that case 2 is not possible at all. 

It is given that all the candidates scored different marks in WE. 

We can see that Ajay, Geeta and Ester has already scored 20, 19 and 18 marks in WE. Therefore, Harini can score a maximum of 17 marks in WE. If Harini’s score in WE is 17, then Harini’s composite score = 2*5+17+20 = 47 which is same as Falak’s composite score. Hence, we can say that Harini can’t score 17 marks in WE. Jatin and Danish have already scored 16 and 15 marks respectively. 

Therefore, we can say that the maximum marks that Harini could have scored in WE = 14. 

Quantitative Aptitude 

Instructions 

For the following questions answer them individually 

Q. 67 If x is a positive quantity such that 2x  = 3log5 2. then x is equal to 

A. log5 8

B. 1 + log3( 5/3 )

C. log5 9

D. 1 + log5( 3/5 )

Answer: D.

Explanation: 

Given that: 

2x = 3log5 2 

⇒ 2x = 2log5 3 

⇒ x = log5

⇒ x = log5 (3 ∗ 5)/5 

⇒ x = log5 5 + log5 3/5 

⇒ x = 1 + log5 3/5 

. Hence, option D. is the correct answer. 

Q. 68 In a circle, two parallel chords on the same side of a diameter have lengths 4 cm and 6 cm. If the distance between these chords is 1 cm, then the radius of the circle, in cm, is

A. √13

B. √14

C. √11

D. √12

Answer: A. 

Explanation: 

Given that two parallel chords on the same side of a diameter have lengths 4 cm and 6 cm.

In the diagram we can see that AB. = 6 cm, CD. = 4 cm and MN = 1 cm. We can see that M and N are the mid points of AB. and CD. respectively. AM = 3 cm and CD. = 2 cm. Let ‘OM’ be x cm. 

In right angle triangle AMO, AO2 = AM2 + OM2 

⇒ AO2 = 32 + x2 … (1) 

In right angle triangle CNO, CO2 = CN2 + ON2 

⇒ CO2 = 22 + (OM + MN)2 

⇒ CO2 = 22 + (x + 1)2 … (2) 

We know that both AO and CO are the radius of the circle. Hence AO = CO Therefore, we can equate equation (1) and (2) 

32 + x2 =  22 + (x + 1)2 

⇒ x = 2 cm 

Therefore, the radius of the circle 

AO = (AM2 + OM2 )

⇒ AO =( 32 + 22 )= 13 

Hence, option A. is the correct answer.

 

Q. 69 Humans and robots can both perform a job but at different efficiencies. Fifteen humans and five robots working together take thirty days to finish the job, whereas five humans and fifteen robots working together take sixty days to finish it. How many days will fifteen humans working together (without any robot) take to finish it? 

A. 45 

B. 36 

C. 32 

D. 40 

Answer: C. 

Explanation: 

Let the efficiency of humans be ‘h’ and the efficiency of robots be ‘r’. 

In the first case, 

Total work = (15h + 5r) * 30……(i) 

In the second case, 

Total work = (5h + 15r) * 60……(ii) 

On equating (i) and (ii), we get 

(15h + 5r) * 30 = (5h + 15r) * 60 

Or, 15h + 5r = 10h + 30r 

Or, 5h = 25r 

Or, h = 5r 

Total work = (15h + 5r) * 30 = (15h + h) * 30 = 480h 

Time taken by 15 humans = 480h / 15hdays= 32 days. 

Hence, option C. is the correct answer. 

Q. 70 Let ABCD. be a rectangle inscribed in a circle of radius 13 cm. Which one of the following pairs can represent, in cm, the possible length and breadth of ABCD? 

A. 24, 10 

B. 25, 9 

C. 25, 10 

D. 24, 12 

Answer: A. 

Explanation: 

Let ABCD. be a rectangle inscribed in a circle of radius 13 cm. Which one of the following pairs can represent, in cm, the possible length and breadth of ABCD? 

We know that AC. is the diameter and ∠ABC. = 90°. AC. = 2*13 = 26 cm 

In right angle triangle ABC, 

AC2 = AB2 + BC2 

⇒ AB2 + BC2 = 262 

⇒ AB2 + BC2 = 676 

Let us check with the options. 

242 + 102 = 676 

Option (A): . Hence, this is a possible answer. 

252 + 92 = 706 ≠ 676 

Option (B): . Hence, this is an incorrect pair. 

252 + 102 = 725 ≠ 676 

Option (C): . Hence, this is an incorrect pair. 

242 + 122 = 720 ≠ 676 

Option (D): . Hence, this is an incorrect pair. 

Therefore, we can say that option A. is the correct answer. 

Q. 71 Points E, F, G, H lie on the sides AB, BC, CD, and DA, respectively, of a square ABCD. If EFGH is also a square whose area is 62.5% of that of ABCD. and CG is longer than EB, then the ratio of length of EB. to that of CG is 

A. 3 : 8 

B. 2 : 5 

C. 4 : 9 

D. 1 : 3 

Answer: D. 

Explanation: 

It is given that EFGH is also a square whose area is 62.5% of that of ABCD. Let us assume that E divides AB in x : 1. Because of symmetry we can see that points F, G and H divide BC, CD and DA in x : 1.

Let us assume that ‘x+1’ is the length of side of square ABCD.

Area of square ABCD =(x+1)2 sq. units.

Therefore, area of square EFGH =62.5 / 100 * (x+1)2  = 5(x+1)2/8… (1)

In right angle triangle EBF,

EF2 = EB2 + BF2

EF = √(12+x2)

Therefore, the area of square EFGH = EF2= x2+1… (2)

By equating (1) and (2),

⇒ x2+1= 5(x+1)2/8

⇒ 8×2 + 8 = 5×2 + 10x + 5

⇒ 3×2 − 10x + 3 = 0

⇒ (x − 3)(3x − 1) = 0

⇒ x= 3 or ⅓

The ratio of length of EB to that of CG = 1 : x

EB : CG = 1 : 3 or 3 : 1. Hence, option D is the correct answer.

Q. 72 Point P lies between points A and B such that the length of BP is thrice that of AP. Car 1 starts from A and moves towards B. Simultaneously, car 2 starts from B and moves towards A. Car 2 reaches P one hour after car 1 reaches P. If the speed of car 2 is half that of car 1, then the time, in minutes, taken by car 1 in reaching P from A is

Answer:12

Explanation:

Let the distance between A and B be 4x.

Length of BP is thrice the length of AP.

=> AP = x and BP = 3x

Let the speed of car 1 be s and the speed of car 2 be 0.5s.

Car 2 reaches P one hour (60 minutes) after Car 1 reaches P.

=> x/s + 60 = 3x/0.5s

=> x/s + 60 = 6x/s

=> 5x/s = 60

=> x/s = 12

Time taken by car 1 in reaching P from A = x/s = 12 minutes.

Therefore, 12 is the correct answer.

Q. 73 Let f(x) = min (2x2, 52-5x ) where x is any positive real number. Then the maximum possible value of f(x) is

Answer:32

Explanation:

f(x) = min (2×2, 52-5x )

The maximum possible value of this function will be attained at the point in which 2×2 is equal to 52-5x.

2×2 = 52 − 5x

2×2 + 5x − 52 = 0

(2x + 13)(x − 4) = 0

=> x = -13/2 or x = 4

It has been given that is a positive real number. Therefore, we can eliminate the case x = -13/2.

x = 4 is the point at which the function attains the maximum value. 4 is not the maximum value of the function.

Substituting x = 4 in the original function, we get, 2×2 = 2*42=32.

f(x) = 32.

Therefore, 32 is the right answer.

Q. 74 While multiplying three real numbers, Ashok took one of the numbers as 73 instead of 37. As a result, the product went up by 720. Then the minimum possible value of the sum of squares of the other two numbers is

Answer:40

Explanation:

We know that one of the 3 numbers is 37.

Let the product of the other 2 numbers be x.

It has been given that 73x-37x = 720

36x = 720

x = 20

Product of 2 real numbers is 20.

We have to find the minimum possible value of the sum of the squares of the 2 numbers.

Let x=a*b

It has been given that a*b=20

The least possible sum for a given product is obtained when the numbers are as close to each other as possible.

Therefore, when a=b, the value of a and b will be √20.

Sum of the squares of the 2 numbers = 20 + 20 = 40.

Therefore, 40 is the correct answer.

Q. 75 Train T leaves station X for station Y at 3 pm. Train S, traveling at three quarters of the speed of T, leaves Y for X at 4 pm. The two trains pass each other at a station Z, where the distance between X and Z is three-fifths of that between X and Y. How many hours does train T take for its journey from X to Y?

Answer:15

Explanation:

Train T starts at 3 PM and train S starts at 4 PM.

Let the speed of train T be t.

=> Speed of train S = 0.75t.

When the trains meet, train t would have traveled for one more hour than train S.

Let us assume that the 2 trains meet x hours after 3 PM. Trains S would have traveled for x-1 hours.

Distance traveled by train T = xt

Distance traveled by train S = (x-1)*0.75t = 0.75xt-0.75t

We know that train T has traveled three fifths of the distance. Therefore, train S should have traveled two-fifths the distance between the 2 cities.

=> (xt)/(0.75xt-0.75t) = 3/2

2xt = 2.25xt-2.25t

0.25x = 2.25

x = 9 hours.

Train T takes 9 hours to cover three-fifths the distance. Therefore, to cover the entire distance, train T will take 9*(5/3)= 15 hours.

Therefore, 15 is the correct answer.

Q. 76 Two types of tea, A and B, are mixed and then sold at Rs. 40 per kg. The profit is 10% if A and B are mixed in the ratio 3 : 2, and 5% if this ratio is 2 : 3. The cost prices, per kg, of A and B are in the ratio

A. 17 : 25

B. 18 : 25

C. 19 : 24

D. 21 : 25

Answer: C

Explanation:

The selling price of the mixture is Rs.40/kg.

Let a be the price of 1 kg of tea A in the mixture and b be the price per kg of tea B.

It has been given that the profit is 10% if the 2 varieties are mixed in the ratio 3:2

Let the cost price of the mixture be x.

It has been given that 1.1x = 40

x = 40/1.1

Price per kg of the mixture in ratio 3:2 = (3a+2b)/5

(3a+2b)/5 = 40/1.1

3.3a + 2.2b = 200 …….(1)

The profit is 5% if the 2 varieties are mixed in the ratio 2:3.

Price per kg of the mixture in ratio 2:3 =(2a+3b)/5

(2a+3b)/5 = 40/1.05

2.1a + 3.15b = 200 …….(2)

Equating (1) and (2), we get,

3.3a + 2.2b = 2.1a + 3.15b

1.2a = 0.95b

a/b = 0.95/1.2

a/b = 19/24

Therefore, option C is the right answer.

Q. 77 Given an equilateral triangle T1 with side 24 cm, a second triangle T2 is formed by joining the midpoints of the sides of T1. Then a third triangle T3 is formed by joining the midpoints of the sides of T2. If this process of forming triangles is continued, the sum of the areas, in sq cm, of infinitely many such triangles T1, T2, T3,… will be

A. 188√3

B. 248√3

C. 164√3

D. 192√3

Answer: D

Explanation:

We can see that T is formed by using the mid points of T . Hence, we can say that area of triangle of T will be (1/4)th of the area of triangle T .

Area of triangle T1 =√3/4 * (24)2 =144√3 sq. cm

Area of triangle T2 =144√3/4 =36√3 sq. cm

Sum of the area of all triangles = T1 + T2 + T3 + …

=> (T1 + T2 /4 + T3 /42 + …)/ T1

=> 1 − 0.25

=> 4/3 * T1

=>4/3 * 144√3

=> 192√3

Hence, option D is the correct answer.

Q. 78 If log12 81 = p, then 3* (4-p)/(4+p) is equal to

A. log4 16

B. log6 16

C. log2 8

D. log6 8

Answer: D

Explanation:

Given that: log12 81 = p

⇒ log81 12 = 1/p

⇒ 4 log3 3 ∗ 4 = 1/p

⇒ 1 + log3 4 = p

Using Componendo and Dividendo,

⇒ (1 + log3 4 − 1)/(1 + log3 4 + 1) = (4 − p)/(4 + p)

⇒ (log3 4) / 2 + log3 4 = (4 − p)/(4 + p)

⇒(log3 4) /log3 9 + log3 4 = (4 − p)/(4 + p)

⇒(log3 4) / log3 36 = (4 − p)/(4 + p)

⇒ 3 ∗ (4 − p)/(4 + p) =3 * (log3 4) / log3 36

⇒ 3 ∗ (4 − p)/(4 + p) = (log3 64) / log3 36

⇒ 3 ∗ (4 − p)/(4 + p) = (log36 64) 

⇒ 3 ∗ (4 − p)/(4 + p) =(log62 82)  = log6 8 Hence, option D is the correct answer.

Q. 79John borrowed Rs. 2,10,000 from a bank at an interest rate of 10% per annum, compounded annually. The loan was repaid in two equal instalments, the first after one year and the second after another year. The first instalment was interest of one year plus part of the principal amount, while the second was the rest of the principal amount plus due interest thereon. Then each instalment, in Rs., is

Answer:121000

Explanation:

We have to equate the installments and the amount due either at the time of borrowing or at the time when the entire loan is repaid. Let us bring all values to the time frame in which all the dues get settled, i.e, by the end of 2 years.

John borrowed Rs. 2,10,000 from the bank at 10% per annum. This loan will amount to 2,10,000*1.1*1.1 = Rs.2,54,100 by the end of 2 years.

Let the amount paid as installment every year be Rs.x.

John would pay the first installment by the end of the first year. Therefore, we have to calculate the interest on this

amount from the end of the first year to the end of the second year. The loan will get settled the moment the second

installment is paid.

=> 1.1x + x = 2,54,100

2.1x = 2,54,100

=> x = Rs. 1,21,000.

Therefore, 121000 is the correct answer.

Q. 80 When they work alone, B needs 25% more time to finish a job than A does. They two finish the job in 13 days in the following manner: A works alone till half the job is done, then A and B work together for 4 days, and finally B works alone to complete the remaining 5% of the job. In how many days can B alone finish the entire job?

A 20

B 22

C 16

D 18

Answer: A

Explanation:

Let us assume that A can complete ‘a’ units of work in a day and B can complete ‘b’ units of work in a day.

A works alone till half the work is completed.

A and B work together for 4 days and B works alone to complete the last 5% of the work.

=> A and B in 4 days can complete 45% of the work.

Let us assume the total amount of work to be done to be 100 units.

4a + 4b = 45 ———(1)

B needs 25% more time than A to finish a job.

=> 1.25*b = a ———-(2)

Substituting (2) in (1), we get,

5b+4b = 45

9b = 45

b = 5 units/day

B alone can finish the job in 100/5 = 20 days.

Therefore, option A is the right answer.

Q. 81 How many numbers with two or more digits can be formed with the digits 1,2,3,4,5,6,7,8,9, so that in every such number, each digit is used at most once and the digits appear in the ascending order?

Answer:502

Explanation:

It has been given that the digits in the number should appear in the ascending order. Therefore, there is only 1 possible

arrangement of the digits once they are selected to form a number.

There are 9 numbers (1,2,3,4,5,6,7,8,9) in total.

2 digit numbers can be formed in 9C2 ways.

3 digit numbers can be formed in 9C3 ways.

…………………………………………..

9 digit number can be formed in 9C9 ways.

We know that nC0 + nC1 + nC2 + ……… + nCn = 2n

=> 9C0 + 9C1 + 9C2 + ……… + 9C9 = 29

=> 9C0 + 9C1 + 9C2 + ……… + 9C9 = 512

We have to subtract and from both the sides of the equations since we cannot form single digit numbers.

=> 1 + 9 + 9C2 +9C3 + ……… + 9C9 = 512

=> 9C2 +9C3 + ……… + 9C9 = 512 -1-9

=> 9C2 +9C3 + ……… + 9C9 = 502

Therefore, 502 is the right answer.

Q. 82 A right circular cone, of height 12 ft, stands on its base which has diameter 8 ft. The tip of the cone is cut off with a plane which is parallel to the base and 9 ft from the base. With π= 22/7, the volume, in cubic ft, of the remaining part of the cone is

Answer:198

Explanation:

We are given that diameter of base = 8 ft. Therefore, the radius of circular base = 8/2 = 4 ft

In triangle OAB and OCD

OA/AB = OC/CD

AB = 3 ∗ 4 / 12= 1 ft.

Therefore, the volume of remaining part = Volume of entire cone – Volume of smaller cone

⇒ ⅓ ∗ π ∗ 42 ∗12 –  ⅓ ∗ π ∗ 12 ∗3

⇒ ⅓ ∗ π ∗189

⇒ ⅓ ∗ 22/7 ∗189

⇒ 198 cubic ft

Q. 83 Raju and Lalitha originally had marbles in the ratio 4:9. Then Lalitha gave some of her marbles to Raju. As a result, the ratio of the number of marbles with Raju to that with Lalitha became 5:6. What fraction of her original number of marbles was given by Lalitha to Raju?

A.

B. ⁶/₁₉

C. ¼

D. ⁷/₃₃

Answer: D

Explanation:

Let the number of marbles with Raju and Lalitha initially be 4x and 9x.

Let the number of marbles that Lalitha gave to Raju be a.

It has been given that (4x+a)/(9x-a) = 5/6

24x + 6a = 45x – 5a

11a = 21x

a/x = 21/11

Fraction of original marbles given to Raju by Lalitha = a/9x (Since Lalitha had 9x marbles initially).

a/9x = 21/99

= 7/33.

Therefore, option D is the right answer.

Q. 84 Each of 74 students in a class studies at least one of the three subjects H, E and P. Ten students study all three subjects, while twenty study H and E, but not P. Every student who studies P also studies H or E or both. If the number of students studying H equals that studying E, then the number of students studying H is

Answer:52

Explanation:

Let us draw a Venn diagram using the information present in the question.

It is given that the number of students studying H equals that studying E.

Let ‘x’ be the total number of students who studied H, and H and P but not E.We can also say that the same will be the number of students who studied E, and E and P but not H.Therefore,

 

x + 20 + 10 + x = 74

x = 22

Hence, the number of students studying H = 22 + 10+ 20 = 52

Q. 85 A wholesaler bought walnuts and peanuts, the price of walnut per kg being thrice that of peanut per kg. He then sold 8 kg of peanuts at a profit of 10% and 16 kg of walnuts at a profit of 20% to a shopkeeper. However, the shopkeeper lost 5 kg of walnuts and 3 kg of peanuts in transit. He then mixed the remaining nuts and sold the mixture at Rs. 166 per kg, thus making an overall profit of 25%. At what price, in Rs. per kg, did the wholesaler buy the walnuts?

A.  96

B.  98

C.  86

D.  84

Answer: A

Explanation:

Let the price of peanuts be Rs. 100x per kg

Then, the price of walnuts = Rs. 300x per kg

Cost price of peanuts for the shopkeeper = Rs. 110x per kg

Cost price of walnuts for the shopkeeper = Rs. 360x per kg

Total cost incurred to the shopkeeper while buying = Rs.(8 * 110x + 16 * 360x) = Rs. 6640x

Total selling price that the shopkeeper got = Rs. (166 * 16) = Rs. 2656

Profit = 25%

So, cost price = Rs. 2124.80

Therefore, 6640x = 2124.80

On solving, we get x = 0.32

Therefore, price of walnuts = Rs. (300 * 0.32) = Rs. 96 per kg.

Hence, option A is the correct answer

Q. 86 If among 200 students, 105 like pizza and 134 like burger, then the number of students who like only burger can possibly be

A.  23

B.  26

C.  96

D.  93

Answer: D

Explanation:

It has been given that among 200 students, 105 like pizza and 134 like burger.

The question asks us to find out the number of students who can be liking only burgers among the given values.

The least number of students who like only burger will be obtained when everyone who likes pizza likes burger too.

In this case, 105 students will like pizza and burger and 134-105 = 29 students will like only burger. Therefore, the number of students who like only burger cannot be less than 29.

The maximum number of students who like only burger will be obtained when we try to separate the 2 sets as much as possible.

There are 200 students in total. 105 of them like pizza. Therefore, the remaining 95 students can like only burger and 134-95 = 39 students can like both pizza and burger. As we can see, the number of students who like burger cannot exceed 95.

The number of students who like only burger should lie between 29 and 95 (both the values are included).

93 is the only value among the given options that satisfies this condition and hence, option D is the right answer.

Q. 87 If f(x+2) = f(x)+f(x+1) for all positive integers x, f(11) = 91 and f(15) = 617, then f(10) equals

Answer:54

Explanation:

f(x+2) = f(x)+f(x+1)

As we can see, the value of a term is the sum of the 2 terms preceding it.

It has been given that f(11) = 91 and f(15) = 617.

We have to find the value of f(10).

Let f(10)= b

f(12)= b + 91

f(13)= 91 + b + 91 = 182 + b

f(14)= 182+b+91+b = 273+2b

f(15)= 273+2b+182+b = 455+3b

It has been given that 455+3b = 617

3b = 162

=> b = 54

Therefore, 54 is the correct answer.

Q. 88 In an apartment complex, the number of people aged 51 years and above is 30 and there are at most 39 people whose ages are below 51 years. The average age of all the people in the apartment complex is 38 years. What is the largest possible average age, in years, of the people whose ages are below 51 years?

A.  27

B.  25

C.  26

D.  28

Answer: D

Explanation:

In an apartment complex, the number of people aged 51 years and above is 30 and there are at most 39 people whose ages are below 51 years. The average age of all the people in the apartment complex is 38 years. What is the largest possible average age, in years, of the people whose ages are below 51 years?

The possible average age of people whose ages are below 51 years will be maximum if the average age of the number of people aged 51 years and above is minimum. Hence, we can say that there are 30 people of the same age 51years.

Let ‘x’ be the maximum average age of people whose ages are below 51.

Then we can say that,

=> 51 ∗ 30 + 39 ∗ x

30 + 39 = 38

⇒ 1530 + 39x = 2622

⇒ x = 1092/39 = 28

Then we can say that, Hence, we can say that option D is the correct answer.

Q. 89 A trader sells 10 litres of a mixture of paints A and B, where the amount of B in the mixture does not exceed that of A. The cost of paint A per litre is Rs. 8 more than that of paint B. If the trader sells the entire mixture for Rs. 264 and makes a profit of 10%, then the highest possible cost of paint B, in Rs. per litre, is

A.  16

B.  26

C.  20

D.  22

Answer: C

Explanation:

Let the price of paint B be x.

Price of paint A = x+8

We know that the amount of paint B in the mixture does not exceed the amount of paint A. Therefore, paint B can at the maximum compose 50% of the mixture.

The seller sells 10 litres of paint at Rs.264 earning a profit of 10%.

=> The cost price of 10 litres of the paint mixture = Rs. 240

Therefore, the cost of 1 litre of the mixture = Rs.24

We have to find the highest possible cost of paint B.

When we increase the cost of paint B, the cost of paint A will increase too. If the cost price of the mixture is closer to the cost of paint B, then the amount of paint B present in the mixture should be greater than the amount of paint A present in the mixture.

The highest possible cost of paint B will be obtained when the volumes of paint A and paint B in the mixture are equal.

=> (x+x+8)/2 = 24

2x = 40

x = Rs. 20

Therefore, option C is the right answer.

Q. 90 In a parallelogram ABCD of area 72 sq cm, the sides CD and AD have lengths 9 cm and 16 cm, respectively. Let P be a point on CD such that AP is perpendicular to CD. Then the area, in sq cm, of triangle APD is

A. 32√3

B. 18√3

C. 24√3

D. 12√3

Answer: A

Explanation:

In a parallelogram ABCD of area 72 sq cm, the sides CD and AD have lengths 9 cm and 16 cm, respectively. Let P be a point on CD such that AP is perpendicular to CD. Then the area, in sq cm, of triangle APD is

Given that area of parallelogram = 72 sq cm

Area of triangle ABC = ½*area of parallelogram

(1/2)*AB*BC*sinABC = ½ *72

sinABC = ½

∠ABC = 30°

Let us draw a perpendicular CQ from C to AB.

By symmetry we can say that CQ = AP, CP = AQ and QB = PQ,

Therefore, we can say that area of triangle APD = area of triangle CQB

In right angle triangle CQB,

QB = CB cos30° = 16 * √3/2= 8√3cm

CQ = CB sin30° = 16 * 1/2= 8 cm

Therefore, area of triangle CQB = 1/2*CQ*QB = ½ *8*8√3 = 32√3

Hence, we can say that area of triangle APD = 32√3.

Q. 91 If U2 + (U – 2V-1)= −4V(U+V) , then what is the value of U+3V?

A. 0

B. ½ 

C. -¼ 

D. ¼ 

Answer: C

Explanation:

Given that =U2 + (U – 2V-1)= −4V(U+V) 

=> U2 + (U2 − 2UV − U − 2UV + 4V2 + 2V − U + 2V + 1) = −4V (U + V )

=>U2 + (U2 − 4UV − 2U + 4V2 + 4V + 1)= −4V (U + V )

=>2U2 − 4UV − 2U + 4V2 + 4V + 1 = −4UV − 4V2

=>2U2 − 2U + 8V2 + 4V + 1 = 0

=>2[U2 − U + ¼] +8[V2 + V/2 +1/16] =0

=>2(U − ½ )2 +8(V + ¼ )2 = 0

Sum of two square terms is zero i.e. the individual square term is equal to zero.

U − ½ = 0 and V + ¼ = 0

U =½  and V = -¼ 

Therefore, U + 3V = ½ +(-1*3)/4 = -¼ . Hence, option C is the correct answer.

Q. 92 In a circle with center O and radius 1 cm, an arc AB makes an angle 60 degrees at O. Let R be the region bounded by the radii OA, OB and the arc AB. If C and D are two points on OA and OB, respectively, such that OC = OD and the area of triangle OCD is half that of R, then the length of OC, in cm, is

A. (π/ 4√3 )1/2

B.  (π/ 6 )1/2

C. (π/ 3√3 )1/2

D. (π/ 4 )1/2

Answer: C

Explanation:

It is given that radius of the circle = 1 cm

Chord AB subtends an angle of 60° on the centre of the given circle. R be the region bounded by the radii OA, OB and the arc AB.

Therefore, R = 60°/ 360° *Area of the circle =⅙ * π * (1)2  = π/6 sq. cm

It is given that OC = OD and area of triangle OCD is half that of R. Let OC = OD = x.

Area of triangle COD =1/2 *OC ∗ OD ∗ sin 60°

=>  π / 6*2 = 1/2 * x * x * √3/2

=> x2 =π /3√3

=> x = (π/ 3√3 )1/2 cm. Hence, option C is the correct answer.

Q. 93 The distance from A to B is 60 km. Partha and Narayan start from A at the same time and move towards B. Partha takes four hours more than Narayan to reach B. Moreover, Partha reaches the mid-point of A and B two hours before Narayan reaches B. The speed of Partha, in km per hour, is

A.  6

B.  4

C.  3

D.  5

Answer: D

Explanation:

Let the time taken by Partha to cover 60 km be x hours.

Narayan will cover 60 km in x-4 hours.

Speed of Partha = 60/x

Speed of Narayan =60/x-4

Partha reaches the mid-point of A and B two hours before Narayan reaches B.

=> 30 /[60/x]+2 = 60 /[60/x-4]

=> x/2 +2 = x-4

=> (x+4) /2 = x-4

=>x + 4 = 2x − 8

=>x = 12

Partha will take 12 hours to cross 60 km.

=> Speed of Partha = 60/12 = 5 Kmph.

Therefore, option D is the right answer.

Q. 94 Let x, y, z be three positive real numbers in a geometric progression such that x < y < z. If 5x, 16y, and 12z are in an arithmetic progression then the common ratio of the geometric progression is

A. 3/6 

B. 1/6

C. 5/2

D. 3/2

Answer: C

Explanation:

Let x =a , y =ar and z =ar2

It is given that, 5x, 16y and 12z are in AP.

so, 5x + 12z = 32y

On replacing the values of x, y and z, we get

5a + 12ar2 = 32ar 

Or, 12r2 – 32r + 5 = 0

On solving, r= 5/2 or 1/6

For r = 1/6 , x < y < z is not satisfied.

So, r = 5/2

Hence, option C is the correct answer.

Q. 95 Given that x2018 y2017 = ½ and x2016 y2019 = 8,then value of x2+y3 is

A. 31/4

B. 35/4

C. 37/4

D. 33/4

Answer: D

Explanation:

Given that x2018 y2017 = ½ … (1)

x2016 y2019 = 8 … (2)

Equation (2)/ Equation (1)

y2/ x2 = 8/ ½ 

y/ x = 4 or -4

Case 1: When y/ x = 4

x2018 (4x)2017 = ½ 

x2018+2017 (2)4034 = ½ 

x4035 = (½)4035

x = ½

Since, y/ x = 4 , => y = 2

Therefore,x2+y3=¼ +8 = 33/4

Case 2: When  y/ x = – 4

x2018 (-4x)2017 = ½ 

x2018+2017 (-2)4034 = ½ 

x4035 = (-½)4035

x = -½

Since, y/ x = -4 , => y = 2

Therefore, x2+y3=¼ +8 = 33/4 . Hence, option D is the correct answer.

Q. 96 A tank is fitted with pipes, some filling it and the rest draining it. All filling pipes fill at the same rate, and all draining pipes drain at the same rate. The empty tank gets completely filled in 6 hours when 6 filling and 5 draining pipes are on, but this time becomes 60 hours when 5 filling and 6 draining pipes are on. In how many hours will the empty tank get completely filled when one draining and two filling pipes are on?

Answer:10

Explanation:

Let the efficiency of filling pipes be ‘x’ and the efficiency of draining pipes be ‘-y’.

In the first case,

Capacity of tank = (6x – 5y) * 6……….(i)

In the second case,

Capacity of tank = (5x – 6y) * 60…..(ii)

On equating (i) and (ii), we get

(6x – 5y) * 6 = (5x – 6y) * 60

or, 6x – 5y = 50x – 60y

or, 44x = 55y

or, 4x = 5y

or, x = 1.25y

Capacity of the tank = (6x – 5y) * 6 = (7.5y – 5y) * 6 = 15y

Net efficiency of 2 filling and 1 draining pipes = (2x – y) = (2.5y – y) = 1.5y

Time required =15y/1.5y hours = 10 hours.

Hence, 10 is the correct answer.

Q. 97 If log2(5+log3 a) = 3 and log2(4a+12+log2 b) = 3, then a + b is equal to

A. 59

B. 40

C. 32

D. 67

Answer: A

Explanation:

log2(5+log3 a) = 3

=>5+log3 a= 8

=> log3 a= 3

or a = 27

log2(4a+12+log2 b) = 3

=> 4a+12+log2 b= 125

Putting a= 27, we get

log2 b= 5

or,  b= 32

So, a + b= 27 + 32 = 59

Hence, option A is the correct answer.

Q. 98 A CAT aspirant appears for a certain number of tests. His average score increases by 1 if the first 10 tests are not considered, and decreases by 1 if the last 10 tests are not considered. If his average scores for the first 10 and the last 10 tests are 20 and 30, respectively, then the total number of tests taken by him is

Answer:60

Explanation:

Let the total number of tests be ‘n’ and the average by ‘A’

Total score = n*A

When 1st 10 tests are excluded, decrease in total value of scores = (nA – 20 * 10) = (nA – 200)

Also, (n – 10)(A + 1) = (nA – 200)

On solving, we get 10A – n = 190……….(i)

When last 10 tests are excluded, decrease in total value of scores = (nA – 30 * 10) = (nA – 300)

Also, (n – 10)(A – 1) = (nA – 300)

On solving, we get 10A + n = 310……….(ii)

From (i) and (ii), we get n = 60

Hence, 60 is the correct answer.

Q. 99 The number of integers x such that 0.25 ≤ 2x ≤ 200 and 2x +2 is perfectly divisible by either 3 or 4, is

Answer:5

Explanation:

At x = 0, 2x  = 1 which is in the given range [0.25, 200]

2x +2 = 1 + 2 = 3 Which is divisible by 3. Hence, x = 0 is one possible solution.

At x = 1, 2x  = 2  which is in the given range [0.25, 200]

2x +2= 2 + 2 = 3 Which is divisible by 4. Hence, x = 1 is one possible solution.

At x = 2, 2x  = 3 which is in the given range [0.25, 200]

2x +2= 4 + 2 = 6 Which is divisible by 3. Hence, x = 2 is one possible solution.

At x = 3, 2x  = 4  which is in the given range [0.25, 200]

2x +2= 8 + 2 = 3 Which is not divisible by 3 or 4. Hence, x = 3 can’t be a solution.

At x = 4, 2x  = 5 which is in the given range [0.25, 200]

2x +2= 16 + 2 = 18 Which is divisible by 3. Hence, x = 4 is one possible solution.

At x = 5, 2x  = 6  which is in the given range [0.25, 200]

2x +2= 32 + 2 = 34 Which is not divisible by 3 or 4. Hence, x = 5 can’t be a solution.

At x = 6, 2x  = 7 which is in the given range [0.25, 200]

2x +2= 64 + 2 = 66 Which is divisible by 3. Hence, x = 6 is one possible solution.

At x = 7, 2x  = 8  which is in the given range [0.25, 200]

2x +2 = 128 + 2 = 130 Which is not divisible by 3 or 4. Hence, x = 7 can’t be a solution.

At x = 8, 2x  = 256  which is not in the given range [0.25, 200]. Hence, x can’t take any value greater than 7.

Therefore, all possible values of x = {0,1,2,4,6}. Hence, we can say that ‘x’ can take 5 different integer values.

Q. 100 In an examination, the maximum possible score is N while the pass mark is 45% of N. A candidate obtains 36 marks, but falls short of the pass mark by 68%. Which one of the following is then correct?

A. N ≤ 200

B. 243 ≤ N ≤ 252

C. 201 ≤ N ≤ 242

D. N ≥ 353

Answer: B

Explanation:

Total marks = N

Pass marks = 45% of N = 0.45N

Marks obtained = 36

It is given that, obtained marks is 68% less than that pass marks

=>the obtained marks is 32% of the pass marks.

So, 0.32 * 0.45N = 36

On solving, we get N = 250

Hence, option B is the correct answer.

CAT Previous Year Paper Session-II 2017

CAT 2017

Shift-2 

VARC 

Instructions 

The passage below is accompanied by a set of six questions. Choose the best answer to each question. 

Creativity is at once our most precious resource and our most inexhaustible one. As anyone who has ever spent any time with children knows, every single human being is born creative; every human being is innately endowed with the ability to combine and recombine data, perceptions, materials and ideas, and devise new ways of thinking and doing. What fosters creativity? More than anything else: the presence of other creative people. The big myth is that creativity is the province of great individual geniuses. In fact creativity is a social process. Our biggest creative breakthroughs come when people learn from, compete with, and collaborate with other people. 

Cities are the true fonts of creativity… With their diverse populations, dense social networks, and public spaces where people can meet spontaneously and serendipitously, they spark and catalyze new ideas. With their infrastructure for finance, organization and trade, they allow those ideas to be swiftly actualized. 

As for what staunches creativity, that’s easy, if ironic. It’s the very institutions that we build to manage, exploit and perpetuate the fruits of creativity — our big bureaucracies, and sad to say, too many of our schools. Creativity is disruptive; schools and organizations are regimented, standardized and stultifying. 

The education expert Sir Ken Robinson points to a 1968 study reporting on a group of 1,600 children who were tested over time for their ability to think in out-of-the-box ways. When the children were between 3 and 5 years old, 98 percent achieved positive scores. When they were 8 to 10, only 32 percent passed the same test, and only 10 percent at 13 to 15. When 280,000 25-year-olds took the test, just 2 percent passed. By the time we are adults, our creativity has been wrung out of us. 

I once asked the great urbanist Jane Jacobs what makes some places more creative than others. She said, essentially, that the question was an easy one. All cities, she said, were filled with creative people; that’s our default state as people. But some cities had more than their shares of leaders, people and institutions that blocked out that creativity. She called them “squelchers.” 

Creativity (or the lack of it) follows the same general contours of the great socio-economic divide — our rising inequality — that plagues us. According to my own estimates, roughly a third of us across the United States, and perhaps as much as half of us in our most creative cities — are able to do work which engages our creative faculties to some extent, whether as artists, musicians, writers, techies, innovators, entrepreneurs, doctors, lawyers, journalists or educators — those of us who work with our minds. That leaves a group that I term “the other 66 percent,” who toil in low-wage rote and rotten jobs — if they have jobs at all — in which their creativity is subjugated, ignored or wasted. 

Creativity itself is not in danger. It’s flourishing is all around us — in science and technology, arts and culture, in our rapidly revitalizing cities. But we still have a long way to go if we want to build a truly creative society that supports and rewards the creativity of each and every one of us. 

Q. 1 In the author’s view, cities promote human creativity for all the following reasons EXCEPT that they 

A. contain spaces that enable people to meet and share new ideas. 

B. expose people to different and novel ideas, because they are home to varied groups of people. 

C. provide the financial and institutional networks that enable ideas to become reality. 

D. provide access to cultural activities that promote new and creative ways of thinking. 

Answer: D. 

Explanation: 

In the paragraph starting with ‘cities are true fronts of creativity’, the author mentions that cities have diverse populations. The author also mentions that cities provide the space where people can meet and share ideas. Then, the author discusses the financial and organizational infrastructure that cities provide for ideas to flourish. 

No where has it been mentioned that cities provide access to cultural activities. We cannot infer option D from the passage. 

Therefore, option D is the right answer. 

Q. 2 The author uses ‘ironic’ in the third paragraph to point out that 

A. people need social contact rather than isolation to nurture their creativity. 

B. institutions created to promote creativity eventually stifle it. 

C. the larger the creative population in a city, the more likely it is to be stifled. 

D. large bureaucracies and institutions are the inevitable outcome of successful cities. 

Answer: B. 

Explanation: 

‘Irony’ is a term used to define an activity defeating its very purpose. Therefore, the answer must be along similar lines – a method or activity that stifles its purpose. 

In the passage (1968 survey), the author describes how schools and colleges, the institutions that were supposed to foster creativity, stifle it. Also, in the paragraph preceding the paragraph about survey, the author mentions explicitly that the institutes created to promote creativity stifle it. Therefore, option B. is the right answer. 

Q. 3 The central idea of this passage is that 

A. social interaction is necessary to nurture creativity. 

B. creativity and ideas are gradually declining in all societies. 

C. The creativity divide is widening in societies in line with socio-economic trends. 

D. more people should work in jobs that engage their creative faculties. 

Answer: A. 

Explanation: 

The entire passage revolves around how cities provide grounds for creativity to flourish and how our education system stifles it. 

Option B states that creativity and ideas are gradually declining. But, in the last paragraph, the author mentions that ‘Creativity itself is not in danger’. Therefore, we can rule out option B 

Option D states that more people must engage in creative jobs. But it cannot be said to be the central idea of the passage. As we have discussed, the passage revolves around social interaction and creativity divide. Therefore, we can eliminate option D too. 

Options A and C are close. But, the author describes creativity divide more as an effect than the problem itself. Barring the last 2 paragraphs, the author describes the importance of social interaction and how the lack of it kills creativity. Since the question is about the central idea, option A can be deemed a better fit than option C 

Therefore, option A is the right answer. 

Q. 4 Jane Jacobs believed that cities that are more creative 

A. have to struggle to retain their creativity. 

B. have to ‘squelch’ unproductive people and promote creative ones. 

C. have leaders and institutions that do not block creativity. 

D. typically do not start off as creative hubs. 

Answer: C. 

Explanation: 

In the passage, the author clearly describes that Jane Jacobs attributes creativity to the type of leaders. From the paragraph about ‘squelchers’, we can infer that Jane Jacobs holds leaders responsible for the creativity of the people. Therefore, option C. is the right answer. 

Q. 5 The 1968 study is used here to show that 

A. as they get older, children usually learn to be more creative. 

B. schooling today does not encourage creative thinking in children. 

C. the more children learn, the less creative they become. 

D. technology today prevents children from being creative. 

Answer: B. 

Explanation: 

There has been no talk about technology in the entire passage. Therefore, we can eliminate option B straight away. Also, option A states that children become more creative as they get older. However, the exact opposite has been discussed in the passage. Therefore, we can eliminate option A too. 

Among options B and C, option C attributes reduction in creativity to learning more. But, in the paragraph about ‘what staunches creativity’, the author mentions that institutions that were created to promote creativity stifle it. He then produces the 1968 study as a validation of the argument. Therefore, the author implies that schools and colleges stifle creativity. 

Hence, option B is the right answer. 

Q. 6 The author’s conclusions about the most ‘creative cities’ in the US (paragraph 6) are based on his assumption that 

A. people who work with their hands are not doing creative work. 

B. more than half the population works in non-creative jobs. 

C. only artists, musicians, writers, and so on should be valued in a society. 

D. most cities ignore or waste the creativity of low-wage workers. 

Answer: A. 

Explanation: 

In the paragraph regarding creative cities, the author makes a remark that the creativity of only those people who work with their mind are utilized. Therefore, we can infer that the author thinks that the creativity of people who do not work with their minds (who work with their hands) is not utilized. Therefore, option A is the right answer. 

 

Instructions 

The passage below is accompanied by a set of six questions. Choose the best answer to each question. 

During the frigid season… it’s often necessary to nestle under a blanket to try to stay warm. The temperature difference between the blanket and the air outside is so palpable that we often have trouble leaving our warm refuge. Many plants and animals similarly hunker down, relying on snow cover for safety from winter’s harsh conditions. The small area between the snowpack and the ground, called the subnivium… might be the most important ecosystem that you have never heard of. 

The subnivium is so well-insulated and stable that its temperature holds steady at around 32 degree Fahrenheit (0 degree Celsius). Although that might still sound cold, a constant temperature of 32 degree Fahrenheit can often be 30 to 40 degrees warmer than the air temperature during the peak of winter. Because of this large temperature difference, a wide variety of species…depend on the subnivium for winter protection. 

For many organisms living in temperate and Arctic regions, the difference between being under the snow or outside it is a matter of life and death. Consequently, disruptions to the subnivium brought about by climate change will affect everything from population dynamics to nutrient cycling through the ecosystem. 

The formation and stability of the subnivium requires more than a few flurries. Winter ecologists have suggested that eight inches of snow is necessary to develop a stable layer of insulation. Depth is not the only factor, however. More accurately, the stability of the subnivium depends on the interaction between snow depth and snow density. Imagine being under a stack of blankets that are all flattened and pressed together. When compressed, the blankets essentially form one compacted layer. In contrast, when they are lightly placed on top of one another, their insulative capacity increases because the air pockets between them trap heat. Greater depths of low-density snow are therefore better at insulating the ground. 

Both depth and density of snow are sensitive to temperature. Scientists are now beginning to explore how climate change will affect the subnivium, as well as the species that depend on it. At first glance, warmer winters seem beneficial for species that have difficulty surviving subzero temperatures; however, as with most ecological phenomena, the consequences are not so straightforward. Research has shown that the snow season (the period when snow is more likely than rain) has become shorter since l970. When rain falls on snow, it increases the density of the snow and reduces its insulative capacity. Therefore, even though winters are expected to become warmer overall from future climate change, the subnivium will tend to become colder and more variable with less protection from the above-ground temperatures. 

The effects of a colder subnivium are complex… For example, shrubs such as crowberry and alpine azalea that grow along the forest floor tend to block the wind and so retain higher depths of snow around them. This captured snow helps to keep soils insulated and in turn increases plant decomposition and nutrient release. In field experiments, researchers removed a portion. of the snow cover to investigate the importance of the subnivium’s insulation. They found that soil frost in the snow-free area resulted in damage to plant roots and sometimes even the death of the plant. 

Q. 7 The purpose of this passage is to 

A. introduces readers to a relatively unknown ecosystem: the subnivium. 

B. explain how the subnivium works to provide shelter and food to several species. 

C. outline the effects of climate change on the subnivium. 

D. draw an analogy between the effect of blankets on humans and of snow cover on species living in the subnivium. 

Answer: C. 

Explanation: 

The entire passage revolves around the effects of climate change on subnivium. 

We can eliminate option D directly as it talks about a small illustration. It cannot be said to be the purpose of the passage. Options A and B emphasize subnivium as the subject. However, the passage is about the effects of climate change on subnivium rather than subnivium itself. Throughout the passage, the author discusses the effects of various climatic changes and how it affects the subnivium. 

Therefore, option C is the right answer. 

Q. 8 All of the following statements are true EXCEPT 

A. Snow depth and Snow density both influence the stability of the subnivium. 

B. Climate change has some positive effects on the subnivium. 

C. The subnivium maintains a steady temperature that can be 30 to 40 degrees warmer than the winter air temperature. 

D. Researchers have established the adverse effects of dwindling snow cover on the subnivium. 

Answer: B. 

Explanation: 

The author mentions that ‘Both depth and density of snow are sensitive to temperature.’ Therefore, we can easily eliminate option A 

Option C talks about the insulating properties of subnivium which has been explicitly mentioned in the passage – ‘Although that might still sound cold, a constant temperature of 32°F can often be 30 to 40 degrees warmer than the air temperature during the peak of winter.’ Therefore, we can eliminate option C too. 

Option D states that researchers have established the adverse effects of the dwindling snow cover in subnivium. From the line starting with ‘research has shown that…’, we can infer that the effects of the dwindling snow cover on subnivium has been established. 

The entire passage does not discuss any positive effect of climate change on the subnivium. Therefore, we can say that option B is the right answer. 

Q. 9 Based on this extract, the author would support which one of the following actions? 

A. The use of snow machines in winter to ensure snow cover of at least eight inches. 

B. Government action to curb climate change. 

C. Adding nutrients to the soil in winter. 

D. Planting more shrubs in areas of short snow season. 

Answer: B. 

Explanation: 

The author mentions in the passage that the quality of snow also plays a vital role. Therefore, maintaining 8 inches of snow with a machine will not fix the problem. Moreover, the option feels too shallow and unsustainable. Therefore, we can eliminate option A 

Option C also feels shallow and unrealistic. Moreover, it has not been mentioned that adding nutrients will fix the issue. 

Option D suggests planting shrubs. But, in the last paragraph the author mentions that the effects are multilayered and complex. Options A, C, and D try to address the symptom than attacking the cause. Option B offers a more viable solution and addresses the cause of the issue rather than its manifestation. Therefore, the author is most likely to agree with option B. and hence, option B. is the right answer. 

Q. 10 In paragraph 6, the author provides the examples of crowberry and alpine azalea to demonstrate that 

A. Despite frigid temperatures, several species survive in temperate and Arctic regions. 

B. Due to frigid temperatures in the temperate and Arctic regions, plant species that survive tend to be shrubs rather than trees. 

C. The crowberry and alpine azalea are abundant in temperate and Arctic regions. 

D. The stability of the subnivium depends on several interrelated factors, including shrubs on the forest floor. 

Answer: D. 

Explanation: 

The reason for the inclusion of the shrubs must be in line with the central idea of the passage. Options A and C are too general and hence, can be ruled out easily. Option B states that plants that tend to survive turn out to be shrubs. But, it has not been mentioned anywhere in the passage. 

The last paragraph clearly mentions that the effects of colder subnivium are multilayered and interrelated. The shrubs tend to prove the point. The paragraph discusses the effect on the shrubs in detail, adding substance to the statement. 

Therefore, option D is the right answer. 

Q. 11 Which one of the following statements can be inferred from the passage? 

A. In an ecosystem, altering any one element has a ripple effect on all others. 

B. Climate change affects temperate and Artie regions more than equatorial or arid ones.

C. A compact layer of wool is warmer than a similarly compact layer of goose down. 

D. The loss of the subnivium, while tragic, will affect only temperate and Artic regions. 

Answer: A. 

Explanation: 

Options B and D mention that it will be the arctic and the temperate regions that will be affected. Though we do not know the effect of climate change on the tropical regions, we cannot claim that there will be no effects. The passage does not give us sufficient information to make that claim. Therefore, we can rule out options B and D 

Option C states that a compact layer of wool is warmer than a similarly compact layer of goose down. Again, the passage does not provide us with sufficient information to substantiate this claim. We do not have sufficient details to compare 2 different materials. Therefore, option C can be ruled out as well. 

Option A talks about ripple effect. The entire passage is about how the effects of climate change are interrelated. Ripple effect also discusses the same. Therefore, option A is the right answer. 

Q. 12 In paragraph 1, the author uses blankets as a device to 

A. evoke the bitter cold of winter in the minds of readers. 

B. explain how blankets work to keep us warm. 

C. draw an analogy between blankets and the snow pack. 

D. alert readers to the fatal effects of excessive exposure to the cold. 

Answer: C. 

Explanation: 

In the passage, author uses the example to explain how having some spaces between layers increases the insulating property. He then uses the same logic to explain the effects of increase in density of snow on subnivium. Therefore, the author uses the example to draw an analogy. Therefore, option C is the right answer. 

 

Instructions 

The passage below is accompanied by a set of six questions. Choose the best answer to each question. 

The end of the age of the internal combustion engine is in sight. There are small signs everywhere: the shift to hybrid vehicles is already under way among manufacturers. Volvo has announced it will make no purely petrol-engined cars after 2019…and Tesla has just started selling its first electric car aimed squarely at the middle classes: the Tesla 3 sells for $35,000 in the US, and 400,000 people have put down a small, refundable deposit towards one. Several thousand have already taken delivery, and the company hopes to sell half a million more next year. This is a remarkable figure for a machine with a fairly short range and a very limited number of specialised charging stations. 

Some of it reflects the remarkable abilities of Elon Musk, the company’s founder, as a salesman, engineer, and a man able to get the most out his factory workers and the governments he deals with…Mr Musk is selling a dream that the world wants to believe in. This last may be the most important factor in the story. The private car is…a device of immense practical help and economic significance, but at the same time a theatre for myths of unattainable self fulfilment. The one thing you will never see in a car advertisement is traffic, even though that is the element in which drivers spend their lives. Every single driver in a traffic jam is trying to escape from it, yet it is the inevitable consequence of mass car ownership. 

The sleek and swift electric car is at one level merely the most contemporary fantasy of autonomy and power. But it might also disrupt our exterior landscapes nearly as much as the fossil fuel-engined car did in the last century. Electrical cars would of course pollute far less than fossil fuel-driven ones; instead of oil reserves, the rarest materials for batteries would make undeserving despots and their dynasties fantastically rich. Petrol stations would disappear. The air in cities would once more be breathable and their streets as quiet as those of Venice. This isn’t an unmixed good. Cars that were as silent as bicycles would still be as dangerous as they are now to anyone they hit without audible warning. 

The dream goes further than that. The electric cars of the future will be so thoroughly equipped with sensors and reaction mechanisms that they will never hit anyone. Just as brakes don’t let you skid today, the steering wheel of tomorrow will swerve you away from danger before you have even noticed it… 

This is where the fantasy of autonomy comes full circle. The logical outcome of cars which need no driver is that they will become cars which need no owner either. Instead, they will work as taxis do, summoned at will but only for the journeys we actually need. This the future towards which Uber…is working. The ultimate development of the private car will be to reinvent public transport. Traffic jams will be abolished only when the private car becomes a public utility. What then will happen to our fantasies of independence? We’ ll all have to take to electrically powered bicycles. 

Q. 13 Which of the following statements best reflects the author’s argument? 

A. Hybrid and electric vehicles signal the end of the age of internal combustion engines. 

B. Elon Musk is a remarkably gifted salesman. 

C. The private car represents an unattainable myth of independence. 

D. The future Uber car will be environmentally friendlier than even the Tesla. 

Answer: C. 

Explanation: 

The main argument around which the passage revolves can be referred from following statemenst made by the author “…The private car is…a device of immense practical … but at the same time a theatre for myths of unattainable self fulfilment… “. In the later part, the author substanitiates this argument. Hence, option C, which states the same, must be the right answer. 

Q. 14 The author points out all of the following about electric cars EXCEPT 

A. Their reliance on rare materials for batteries will support despotic rule. 

B. They will reduce air and noise pollution. 

C. They will not decrease the number of traffic jams. 

D. They will ultimately undermine rather than further driver autonomy. 

Answer: D. 

Explanation: 

Refer to the following lines, ” …the rarest materials for batteries would make undeserving despots and their dynasties fantastically rich…” Thus, the author states that relying on rare materials will support despotic rule. Hence, option A is correct and can be eliminated. 

Refer to the following lines, “…The air in cities would once more be breathable and their streets as quiet as .. Cars that were as silent as bicycles …”. Hence, option B is correct and can be eliminated. 

Refer to the following lines, “…traffic jam is trying to escape from it, yet it is the inevitable consequence of mass car ownership…” Hence, the author points out that the problem of traffic jam which persists with electric car as it is a consequence of mass car ownership. Hence, option C is correct and can be eliminated. 

Option D, which states that electric cars will undermine the driver autonomy, cannot be inferred from the passage. Hence, option D is the right choice. 

Q. 15 According to the author, the main reason for Tesla’s remarkable sales is that 

A. in the long run, the Tesla is more cost effective than fossil fuel-driven cars. 

B. the US government has announced a tax subsidy for Tesla buyers. 

C. III the company is rapidly upscaling the number of specialised charging stations for customer convenience. 

D. people believe in the autonomy represented by private cars. 

Answer: D. 

Explanation: 

Refer to following lines, “…Mr Musk is selling a dream that the world …This last may be the most important factor in the story. The private car..”. Since people believe in the autonomy represented by private cars, Tesla had remarkable sales. 

Option A is nowhere stated as the reason for high sales for Tesla. Hence, it can be eliminated. 

Nowhere in the passage has the author mentioned about tax subsidy to Tesla buyers. Hence, option B can be eliminated. 

The author states that there are limited number of specialised charging stations. But the author doesn’t state that there will be significant increase in charging stations rapidly which would increase the sales. Hence, option C can be eliminated. 

Option D correctly highlights the main reason for Tesla’s remarkable sales. Hence, option D is the right choice. 

Q. 16 The author comes to the conclusion that 

A. car drivers will no longer own cars but will have to use public transport. 

B. cars will be controlled by technology that is more efficient than car drivers. 

C. car drivers dream of autonomy but the future may be public transport. 

D. electrically powered bicycles are the only way to achieve autonomy in transportation. 

Answer: C. 

Explanation: 

Refer to the following lines, “…Traffic jams will be abolished only when the private car becomes a public utility…”. According to the author the fantasy of autonomy comes full circle. Thus the author states that car drivers want 

autonomy but public transport will be the future as only then the traffic problem will be solved. Option A, which states that car drivers will no longer own cars, is too extreme and can be ruled out. Option B specifies something that is out of the scope of the paragraph. Hence, option B is incorrect. Option C completely reflects what the author says in the end. Hence, it is the right choice. 

Option D is again too extreme and cannot be found in the passage. Also, the author mentions electric bicycles just to provide an illustration. Hence,it can be eliminated. 

Hence, option C is the right answer. 

Q. 17 In paragraphs 5 and 6, the author provides the example of Uber to argue that 

A. in the future, electric cars will be equipped with mechanisms that prevent collisions. 

B. in the future, traffic jams will not exist. 

C. in the future, the private car will be transformed into a form of public transport. 

D. in the future, Uber rides will outstrip Tesla sales. 

Answer: C. 

Explanation: 

In paragraph 5 and 6 the author states that instead of cars having owners they’ll work as taxis do, call the taxis at will and use only for journey which we actually need. According to the author this is the future towards which Uber is working. 

Option A, which states that electric cars will have mechanisms to prevent collisions, is out of context. Hence, it is incorrect and can be eliminated. 

Option B, which states that future will definitely have no trafiic jams, is incorrect. Hence, option B can be eliminated. Option C captures the points which we discussed. Hence, option C is the right answer. 

The passage doesn’t give any comparison about Uber rides and Tesla sales. Hence, option D. is incorrect and can be eliminated. 

Q. 18 In paragraph 6, the author mentions electrically powered bicycles to argue that 

A. if Elon Musk were a true visionary, he would invest funds in developing electric bicycles. 

B. our fantasies of autonomy might unexpectedly require us to consider electric bicycles. C. in terms of environmental friendliness and safety, electric bicycles rather than electric cars are the future. 

D. electric buses are the best form of public transport. 

Answer: B. 

Explanation: 

According to the author we’ll have no traffic jams only when cars become public utility. And, for us(people) to have independence, we’ll have to start using electrically powered bicycles. Hence, option B, which states this, is the right answer. 

Option A. is nowhere mentioned in the passage. Hence, can be eliminated. 

Author doesn’t give any comparison, as mentioned in option C, between electric powered bicycle and electric cars. Hence, option C can be eliminated. 

The author nowhere mentions that electric buses are the best form of public transport. Hence, option D can be eliminated. 

Hence, option B is the right choice. 

 

Instructions 

The passage below is accompanied by a set of three questions. Choose the best answer to each question. 

Typewriters are the epitome of a technology that has been comprehensively rendered obsolete by the digital age. The ink comes off the ribbon, they weigh a ton, and second thoughts are a disaster. But they are also personal, portable and, above all, private. Type a document and lock it away and more or less the only way anyone else can get it is if you give it to them. That is why the Russians have decided to go back to typewriters in some government offices, and why in the US, some departments have never abandoned them. Yet it is not just their resistance to algorithms and secret surveillance that keeps typewriter production lines — well one, at least — in business (the last British one closed a year ago). Nor is it only the nostalgic appeal of the metal body and the stout well-defined keys that make them popular on eBay. A typewriter demands something particular: attentiveness. By the time the paper is loaded, the ribbon tightened, the carriage returned, the spacing and the margins set, there’s a big premium on hitting the right key. That means sorting out ideas, pulling together a kind of order and organising details before actually striking off. There can be no thinking on screen with a typewriter. Nor are there any easy distractions. No online shopping. No urgent emails. No Twitter. No need even for electricity — perfect for writing in a remote hideaway. The thinking process is accompanied by the encouraging clack of keys, and the ratchet of the carriage return. Ping! 

Q. 19 Which one of the following best describes what the passage is trying to do? 

A. It describes why people continue to use typewriters even in the digital age. 

B. It argues that typewriters will continue to be used even though they are an obsolete technology. 

C. It highlights the personal benefits of using typewriters. 

D. It shows that computers offer fewer options than typewriters. 

Answer: A. 

Explanation: 

The passage starts by introducing typewriters. The author later states that some government offices in Russia are going back to typewriters and that in the US some offices still use them. The author then goes on to give reasons for the same by highlighting positive aspects of the typewriter. 

Option A correctly describes what the passage is trying to do. Hence, it is the right choice. 

The author doesn’t state that use of typewriters will be perennial. Hence, it can be eliminated. The main aim of the passage is not to highlight personal benefit. Hence, it can be eliminated. 

Option D is out of scope as the author nowhere states or highlights that computers offer less options than typewriters. Hence, it can be eliminated. 

Hence, option A is the right answer. 

Q. 20 According to the passage, some governments still use typewriters because: 

A. they do not want to abandon old technologies that may be useful in the future. 

B. they want to ensure that typewriter production lines remain in business. 

C. they like the nostalgic appeal of typewriter. 

D. they can control who reads the document. 

Answer: D. 

Explanation: 

Refer to the sentence, ” … the only way anyone else can get it is if you give it to them. That is why the Russians have decided to go back to typewriters in some government offices…” Hence, the government uses typewriters to control who views the document as the only way someone can read the document is by physically accessing it to them. Option D, which highlights this, is the right answer. 

Q. 21 The writer praises typewriters for all the following reasons EXCEPT 

A. Unlike computers, they can only be used for typing. 

B. You cannot revise what you have typed on a typewriter. 

C. Typewriters are noisier than computers. 

D. Typewriters are messier to use than computers. 

Answer: D. 

Explanation: 

The author states that, “..Nor are there any easy distractions…”. Hence, the only thing one can do using typewriter is write, unlike computers. Hence, option A is correct. 

Refer to following lines “… there’s a big premium on hitting the right key…”. Thus, there’s premium attached on hitting right keys because as you cannot revise what you’ve typed on the typewriter. Hence, option B is correct. 

Refer to following lines “…thinking process is accompanied by the encouraging clack of keys…”. Hence, typewriters are noisier than computers. So, option C. is correct. 

Nowhere in the passage does the author state or highlight that typewriters are messiar than the computers. Hence, option D is not the reason why the author praises typewriters. 

Thus, option D. is the right choice. 

 

Instructions 

The passage below is accompanied by a set of three questions. Choose the best answer to each question. 

Despite their fierce reputation, Vikings may not have always been the plunderers and pillagers popular culture imagines them to be. In fact, they got their start trading in northern European markets, researchers suggest. 

Combs carved from animal antlers, as well as comb manufacturing waste and raw antler material has turned up at three archaeological sites in Denmark, including a medieval marketplace in the city of Ribe. A. team of researchers from Denmark and the U.K. hoped to identify the species of animal to which the antlers once belonged by analyzing collagen proteins in the samples and comparing them across the animal kingdom, Laura Geggel reports for LiveScience. Somewhat surprisingly, molecular analysis of the artifacts revealed that some combs and other material had been carved from reindeer antlers…. Given that reindeer (Rangifer tarandus) don’t live in Denmark, the researchers posit that it arrived on Viking ships from Norway. Antler craftsmanship, in the form of decorative combs, was part of Viking culture. Such combs served as symbols of good health, Geggel writes. The fact that the animals shed their antlers also made them easy to collect from the large herds that inhabited Norway. 

Since the artifacts were found in marketplace areas at each site it’s more likely that the Norsemen came to trade rather than pillage. Most of the artifacts also date to the 780s, but some are as old as 725. That predates the beginning of Viking raids on Great Britain by about 70 years. (Traditionally, the so-called “Viking Age” began with these raids in 793 and ended with the Norman conquest of Great Britain in l066.) Archaeologists had suspected that the Vikings had experience with long maritime voyages [that] might have preceded their raiding days. Beyond Norway, these combs would have been a popular industry in Scandinavia as wela: It’ s possible that the antler combs represent a larger trade network, where the Norsemen supplied raw material to craftsmen in Denmark and elsewhere. 

Q. 22 The primary purpose of the passage is: 

A. to explain the presence of reindeer antler combs in Denmark. 

B. to contradict the widely-accepted beginning date for the Viking Age in Britain, and propose an alternate one. 

C. to challenge the popular perception of Vikings as raiders by using evidence that suggests their early trade relations with Europe. 

D. to argue that besides being violent pillagers„Vikings were also skilled craftsmen and efficient traders. 

Answer: C. 

Explanation: 

The passage revolves around how vikings did not start out as pillagers but as traders. The intention of the author seems to dispel the notion that the Vikings were pillagers. The combs have been used just as an illustration to prove the author’s hypothesis. Therefore, option A can be ruled out. 

Option B states that the purpose was to change the period of Viking age. However, the passage does not hint any such intention. The author cites that the combs had made their way to Britain before the Viking age to substantiate the fact that Vikings were traders before they became pillagers. 

Therefore, we can rule out option B too. 

Option D states that despite being pillagers, Vikings were efficient traders and craftsmen. However, the passage talks about a period prior to which Vikings turned pillagers. Therefore, we can eliminate option D too. 

Option C states that the purpose of the passage is to dispel the notion that Vikings were pillagers. This seems the most appropriate option as the passage tries to establish the fact that Vikings started out as traders. Therefore, option C is the right answer. 

Q. 23 The evidence – “Most of the artifacts also date to the 780s, but some are as old as 725” — has been used in the passage to argue that: 

A. the beginning date of the Viking Age should be changed from 793 to 725. 

B. the Viking raids started as early as 725. 

C. some of the antler artifacts found in Denmark and Great Britain could have come from Scandinavia. 

D. the Vikings’ trade relations with Europe pre-dates the Viking raids. 

Answer: D. 

Explanation: 

The author mentions the statement to imply that the Vikings had trade relations with the British before the Viking age. The Viking age started in 793, whereas the artifacts predate this period. Therefore, the intention of the line “Most of the artifacts also date to the 780s, but some are as old as 725” is to emphasize that Vikings had trade relations. Therefore, option D is the right answer. 

Q. 24 All of the following hold true for Vikings EXCEPT 

A. Vikings brought reindeer from Norway to Denmark for trade purposes. 

B. Before becoming the raiders of northern Europe, Vikings had trade relations with European nations. 

C. Antler combs, regarded by the Vikings as a symbol of good health, were part of the Viking culture. 

D. Vikings, once upon a time, had trade relations with Denmark and Scandinavia. 

Answer: A. 

Explanation: 

In the passage, it has been mentioned that “Such combs served as symbols of good health, Geggel writes” . Therefore, we can infer option C. and hence, it can be eliminated. 

Option D states that “Vikings, once upon a time, had trade relations with Denmark and Scandinavia”. The last paragraph mentions that “Beyond Norway, these combs would have been a popular industry in Scandinavia as well. It’s possible that the antler combs represent a larger trade network, where the Norsemen supplied raw material to craftsmen in Denmark and elsewhere”. Therefore, we can rule out option D too. 

Option B states that the Vikings had trade relations with Northern Europe. This is the very theme of the passage. Hence, we can eliminate option B too. 

Option A states that “Vikings brought reindeer from Norway to Denmark for trade purposes”. However, the passage only mentions that they brought the combs – not the Reindeers itself. Therefore, option A is the right answer. 

 

Instructions 

For the following questions answer them individually 

Q. 25 The passage given below is followed by four summaries. Choose the option that best captures the author’ s position. 

North American walnut sphinx moth caterpillars (Amorpha juglandis) look like easy meals for birds, but they have a trick up their sleeves — they produce whistles that sound like bird alarm calls, scaring potential predators away. At first, scientists suspected birds were simply startled by the loud noise. But a new study suggests a more sophisticated mechanism: the caterpillar’s whistle appears to mimic a bird alarm call, sending avian predators scrambling for cover. When pecked by a bird, the caterpillars whistle by compressing their bodies like an accordion and forcing air out through specialized holes in their sides. The whistles are impressively loud — they have been measured at over 80 dB. from 5 cm away from the caterpillar — considering they are made by a two-inch long insect. 

A. North American walnut sphinx moth caterpillars will whistle periodically to ward off predator birds – they have a specialized vocal tract that helps them whistle. 

B. North American walnut sphinx moth caterpillars can whistle very loudly; the loudness of their whistles is shocking as they are very small insects. 

C. The North American walnut sphinx moth caterpillars, in a case of acoustic deception, produce whistles that mimic bird alarm calls to defend themselves. 

D. North American. walnut sphinx moth caterpillars, in. a case of deception and camouflage, produce whistles that mimic bird alarm calls to defend themselves. 

Answer: C. 

Explanation: 

According to the paragraph, the North American walnut sphinx moth caterpillars produce whistles which are extremely loud considering their size. These whistles appear to mimic bird(predator) alarm calls which scares them to look for cover. Thus, these sounds act as acoustic deception and help the insect to defend themselves against predators. 

Option A mentions about vocal tracts which is out of scope. Hence, it can be eliminated. 

Option B though correct, fails to mention the use of sound to defend against the predators. Hence, it can be eliminated. Option C captures all the main points and hence is right choice. 

Option D mentions ‘camouflage’ which is also out of context. Hence, it can be eliminated. 

Hence, option C is the right answer. 

Q. 26 The passage given below is followed by four summaries. Choose the option that best captures the author’s position. 

Both Socrates and Bacon were very good at asking useful questions. In fact, Socrates is largely credited with coming up with a way of asking questions, ‘the Socratic method,’ which itself is at the core of the ‘scientific method,’ popularised by Bacon. The Socratic method disproves arguments by finding exceptions to them, and can therefore lead your opponent to a point where they admit something that contradicts their original position. In common with Socrates, Bacon stressed it was as important to disprove a theory as it was to prove one — and real-world observation and experimentation were key to achieving both aims. Bacon also saw science as a collaborative affair, with scientists working together, challenging each other. 

A. Both Socrates and Bacon advocated clever questioning of the opponents to disprove their arguments and theories. 

B. Both Socrates and Bacon advocated challenging arguments and theories by observation and experimentation. 

C. Both Socrates and Bacon advocated confirming arguments and theories by finding exceptions. 

D. Both Socrates and Bacon advocated examining arguments and theories from both sides to prove them. 

Answer: D. 

Explanation: 

According to the paragraph, Socrates and Bacon were good at asking questions. The Socratic method works in a way by finding exceptions to the arguments of the opponent, which makes the opponent to agree on something that contradicts their original position. In a similar way, Bacon stressed that it was important to disprove theory as it is to prove it. Thus both Socrates and Bacon stressed on examining arguments from both ends – to prove as well as disprove. 

Option A, which speaks only about disproving of arguments, can be eliminated. 

Option B talks only about examining and observation. Hence, it can be eliminated. 

Option C talks only about confirming of arguments and not the other way. Hence, it can be eliminated. Option D captures the main points which we discussed earlier. 

Hence, option D is the right answer. 

Q. 27 The passage given below is followed by four sumrmries. Choose the option that best captures the author’ s position. 

A. fundamental property of language is that it is slippery and messy and more liquid than solid, a gelatinous mass that changes shape to fit. As Wittgenstein would remind us, “usage has no sharp boundary.” Oftentimes, the only way to determine the meaning of a word is to examine how it is used. This insight is often described as the “meaning is use” doctrine. There are differences between the “meaning is use” doctrine and a dictionary-first theory of meaning. “The dictionary’s careful fixing of words to definitions, like butterflies pinned under glass, can suggest that this is how language works. The definitions can seem to ensure and fix the meaning of words, just as the gold standard can back a country’s currency.” What Wittgenstein found in the circulation of ordinary language, however, was a free floating currency of meaning. The value of each word arises out of the exchange. The lexicographer abstracts a meaning from that exchange, which is then set within the conventions of the dictionary definition. 

A. Dictionary definitions are like ‘gold standards’ — artificial, theoretical and dogmatic. Actual meaning of words is their free-exchange value. 

B. Language is already slippery; given this, accounting for ‘meaning in use’ will only exasperate the problem. That is why lexicographers ‘fix’ meanings. 

C. Meaning is dynamic; definitions are static. The ‘meaning in use’ theory helps us understand that definitions of words are culled from their meaning in exchange and use and not vice versa. 

D. The meaning of words in dictionaries is clear, fixed and less dangerous and ambiguous than the meaning that arises when words are exchanged between people. 

Answer: C. 

Explanation: 

According to the paragraph, language is like a gelatinous mass that changes shape to fit. Also, many times the only way to find meaning of word is to examine how it is used. It is stated that definitions are fixed for the word by dictionary.Wittgenstein found that circulation of ordinary language was a free-floating currency of meaning. So the meanings are dynamic. Thus, the value of word arises from the exchange and then the lexicographer abstracts meaning from that exchange. Thus, definitions are picked up from the meaning in use. 

Option A, which states that definitions are like dogmatic, cannot be found in the paragraph. Hence, it can be eliminated. The paragraph doesn’t talk about why lexicographers fix meanings. Hence, option B can be eliminated. Option C covers all the main points. Hence, it is the right choice. 

The purpose of the passage is not to compare meaning of words in dictionaries with meaning which arises from exchange. Hence, option D can be eliminated. 

Hence, option C is the right choice. 

 

Q. 28 The five sentences labelled (1, 2, 3, 4, 5) given in this question, when properly sequenced, form a coherent paragraph. Each sentence is labelled with a number. Decide on the proper order for the sentences and key in this sequence of five numbers as your answer. 

1: The implications of retelling of Indian stories, hence, takes on new meaning in a modern India. 

2. The stories we tell reflect the world around us. 

3. We cannot help but retell the stories that we value — after all, they are never quite right for us — in our time. 

4. And even if we manage to get them quite right, they are only right for us — other people living around us will have different reasons for telling similar stories. 

5. As soon as we capture a story, the world we were trying to capture has changed. Answer:25341 

Explanation: 

Sentence 2, which introduces the topic of what stories tell, must be the starting sentence. Sentence 5 elaborates on sentence 2. Hence sentence 5 logically follows sentence 2. According to the sentence 3, the stories we retell are never quite right for us. Sentence 4 elaborates on what if we get the stories quite right. Hence, 3-4 forms a pair which must come after sentence 5. Sentence 1 which concludes the topics of discussion must be the ending sentence. Hence, 25341 is the right answer. 

 

Q. 29 The five sentences labelled (1, 2, 3, 4, 5) given in this question, when properly sequenced, form a coherent paragraph. Each sentence is labelled with a number. Decide on the proper order for the sentences and key in this sequence of five numbers as your answer. 

1. Before plants can take life from atmosphere, nitrogen must undergo transformations similar to ones that food undergoes in our digestive machinery. 

2. In its aerial form nitrogen is insoluble, unusable and is in need of transformation. 3. Lightning starts the series of chemical reactions that need to happen to nitrogen, ultimately helping it nourish our earth. 

4. Nitrogen — an essential food for plants — is an abundant resource, with about 22 million tons of it floating over each square mile of earth. 

5. One of the most dramatic examples in nature of ill wind that blows goodness is lightning. Answer:53421 

Explanation: 

On closely reading the sentences, we see that the topic of discussion is how chemical reactions started by lightning affects nitrogen in the air which ultimately help nourish the earth. Hence, sentence 5 which introduces lightning to us must be the starting sentence. Sentence 3 further elaborates on chemical reactions, started by lightning, which affects nitrogen. Hence, sentence 3 logically follows sentence 5. Sentence 4 states how nitrogen helps in nourishing the earth. Hence, sentence 4 must follow sentence 3. Sentence 2 states why in its natural form nitrogen is unusable and sentence 1 further elaborates on why nitrogen must undergo transformation for plants to use it. Hence sentences 2 and 1 form a pair which must follow sentence 4. Hence, the correct sequence 53421. 

Q. 30 The five sentences (labelled 1, 2, 3, 4, 5) given in this question, when properly sequenced, form a coherent paragraph. Each sentence is labelled with a number. Decide on the proper order for the sentences and key in this sequence of five numbers as your answer. 

1. This has huge implications for the health care system as it operates today, where depleted resources and time lead to patients rotating in and out of doctor’s offices, oftentimes receiving minimal care or concern (what is commonly referred to as “bed side manner”) from doctors. 

2. The placebo effect is when an individual’s medical condition or pain shows signs of improvement based on a fake intervention that has been presented to them as a real one and used to be regularly dismissed by researchers as a psychological effect. 

3. The placebo effect is not solely based on believing in treatment, however, as the clinical setting in which treatments are administered is also paramount. 

4. That the mind has the power to trigger biochemical changes because the individual believes that a given drug or intervention will be effective could empower chronic patients through the notion of our bodies’ capacity for self-healing. 

5. Placebo effects are now studied not just as foils for “real” interventions but as a potential portal into the self-healing powers of the body. 

Answer:25431 

Explanation: 

On closely reading the sentences, we can see that the passage is about placebo effect. Sentence 2, which introduces the placebo effect, must be the starting sentence. Sentence 5 states that placebo effect are now not just studied for real interventions, as stated in sentence 2, but as potential portal into self healing power. Hence, sentence 5 logically follows sentence 2. Sentence 4 which elaborates on self healing must follow sentence 5. Sentence 3 makes a point that apart from the belief in the treatment, the clinical setting also has a role to play. Sentence 1 elaborates on this. Hence, 31 is a pair which must follow sentence 4. Thus, the correct order is 25431. 

Q. 31 The five sentences (labelled 1, 2, 3, 4, 5) given in this question, when properly sequenced, form a coherent paragraph. Each sentence is labelled with a number. Decide on the proper order for the sentences and key in this sequence of five numbers as your answer. 

1. Johnson treated English very practically, as a living language, with many different shades of meaning and adopted his definitions on the principle of English common law — according to precedent. 

2. Masking a profound inner torment, Johnson found solace in compiling the words of a language that was, in its coarse complexity and comprehensive genius, the precise analogue of his character. 

3. Samuel Johnson was a pioneer who raised common sense to heights of genius, and a man of robust popular instincts whose watchwords were clarity, precision and simplicity. 

4. The 18th century English reader, in the new world of global trade and global warfare, needed a dictionary with authoritative acts of definition of words of a language that was becoming seeded throughout the first British empire by a vigorous and practical champion. 

5. The Johnson who challenged Bishop Berkeley’s solipsist theory of the nonexistence of matter by kicking a large stone (“I refute it thus”) is the same Johnson for whom language must have a daily practical use. 

Answer:43512 

Explanation: 

Sentence 4 should be the opening sentence since it talks about the need for a dictionary in the 18th century. The other 4 statements talk about Samuel Johnson. 

3 must follow 4 since it introduces the subject, Samuel Johnson. Only sentence 3 contains the full name of Samuel Johnson. 

3 should be followed by 5 since it describes Johnson’s character. 5 plays the role of a general introduction and hence, it should be placed before any specific detail regarding Johnson’s contribution to the dictionary is introduced. Out of sentences 1 and 2, 1 should precede 2 since it establishes that Johnson worked on English and sentence 2 explains the innate connection between Johnson and the language (English). 

43512 is the correct order. 

Q. 32 Five sentences related to a topic are given below. Four of them can be put together to form a meaningful and coherent short paragraph. Identify the odd one out. 

1. Although we are born with the gift of language, research shows that we are surprisingly unskilled when it comes to communicating with others. 

2. We must carefully orchestrate our speech if we want to achieve our goals and bring our dreams to fruition. 

3. We often choose our words without thought, oblivious of the emotional effects they can have on others. 

4. We talk more than we need to, ignoring the effect we are having on those listening to us. 

5. We listen poorly, without realizing it, and we often fail to pay attention to the subtle meanings conveyed by facial expressions, body gestures, and the tone and cadence of our voice. 

Answer:

Explanation: 

The paragraph is about how inefficient we are when it comes to usage of language. Sentence 2 stands out as an imperative or instructive statement whereas the other statements simply elaborate on the point that we are unskilled at language usage. Therefore, option 2 is the odd sentence. 

Q. 33 Five sentences related to a topic are given below. Four of them can be put together to form a meaningful and coherent short paragraph. Identify the odd one out. 

1: Over the past fortnight, one of its finest champions managed to pull off a similar impression. 

2. Wimbledon’s greatest illusion is the sense of timelessness it evokes. 

3. At 35 years and 342 days, Roger Federer became the oldest man to win the singles title in the Open Era — a full 14 years after he first claimed the title as a scruffy, pony-tailed upstart. 

4. Once he had survived the opening week, the second week witnessed the range of a rested Federer’s genius. 

5. Given that his method isn’t reliant on explosive athleticism or muscular ball-striking, both vulnerable to decay, there is cause to believe that Federer will continue to enchant for a while longer. 

Answer:

Explanation: 

The paragraph is about the timelessness of the Wimbledon and Roger Federer. Sentences 1, 2 and 3 are clearly the part of the paragraph. 

Wimbledon – one of its finest champion – Roger Federer 

Now, sentences 4 and 5 are close. Sentence 4 talks about a specific event. Whereas, sentence 5 is inline with the idea of timelessness and hence, can be used as the concluding sentence of the paragraph. Moreover, sentence 4 appears hanging (Missing some previous statement). Therefore, sentence 4 is the odd one out. 

Q. 34 Five sentences related to a topic are given below. Four of them can be put together to form a meaningful and coherent short paragraph. Identify the odd one out. 

1. Those geometric symbols and aerodynamic swooshes are more than just skin deep. 

2. The Commonwealth Bank logo — a yellow diamond, with a black chunk sliced out in one corner — is so recognisable that the bank doesn’t even use its full name in its advertising. 

3. It’s not just logos with hidden shapes; sometimes brands will have meanings or stories within them that are deliberately vague or lost in time, urging you to delve deeper to solve the riddle. 

4. Graphic designers embed cryptic references because it adds a story to the brand; they want people to spend more time with a brand and have that idea that they are an insider if they can understand the hidden message. 

5. But the CommBank logo has more to it than meets the eye, as squirrelled away in that diamond is the Southern Cross constellation. 

Answer:

Explanation: 

After reading all the sentences, we know that the paragraph is talking about the logos and brands and the purpose behind them. Statement 2 is the opening sentence which tells us about the Commonwealth Bank’s logo. Statement 5 provides more information about the logo, mentioned in statement 2, that there is more to what is visible to eyes about the Bank’s logo. Statement 3 gives another insight that it’s not only logos, but brand names are also created with hidden meanings. Statement 4 provides the reasoning behind passing subtle messages through brands and logos. Thus, 2534 forms a meaningful paragraph. 

Statement 1 describes something as geometric symbols and aerodynamic swooshes which does not connect well with the other four sentences. Thus, statement 1 does not fit in the paragraph. 

Hence, option 1 is the correct answer. 

DILR 

Instructions 

Funky Pizzeria was required to supply pizzas to three different parties. The total number of pizzas it had to deliver was 800, 70% of which were to be delivered to Party 3 and the rest equally divided between Party 1 and Party 2. 

Pizzas could be of the Thin Crust (T) or Deep Dish (D) variety and come in either Normal Cheese (NC) or Extra Cheese (EC) versions. Hence, there are four types of pizzas: T-NC, T-EC, D-NC. and D-EC. Partial information about proportions of T and NC. pizzas ordered by the three parties is given below: 

Q. 35 How many Thin Crust pizzas were to be delivered to Party 3? 

A. 398 

B. 162 

C. 96 

D. 364 

Answer: B. 

Explanation: 

We are given that Party 3 received 70% of total pizzas,therefore, number of pizzas received by Party 3 =70/100 × 800 = 560 

Remaining 240 pizzas are equally divided among party 1 and party 2 hence we can say that each of Party 1 and Party 2 received 120 pizzas. 

We know that all of the pizza can be classified into a total of 4 types. Hence, on drawing a table which can accommodate all of the cases: 

Total number of Thin Crust pizzas = 0.375*800 = 300. Therefore, total number of Deep Dish pizzas = 800 – 300 = 500. 

Out of 120 pizzas that Party 1 received, 60% were of Thin Crust type hence, total number of Thin Crust pizza received by Party 1 = 0.6*120 = 72. Consequently Party 1, must have received 42 Deep Dish type pizzas. 

Out of 120 pizzas that Party 2 received, 55% were of Thin Crust type hence, total number of Thin Crust pizza received by Party 2 = 0.55*120 = 66. Consequently Party 1, must have received 54 Deep Dish type pizzas. 

Therefore, total number of Thin Crust pizzas ordered by Party 3 = Total Thin Crust pizzas ordered – Thin Crust pizzas ordered by Party 1 – Thin Crust pizzas ordered by Party 2 

⇒ 300 – 72 – 66 = 162 

Hence, option B. is the correct answer. 

Q. 36 How many Normal Cheese pizzas were required to be delivered to Party 1? 

A. 104 

B. 84 

C. 16 

D. 196 

Answer: C. 

Explanation: 

We are given that Party 3 received 70% of total pizzas,therefore, number of pizzas received by Party 3 =70/100 × 800 = 560 

Remaining 240 pizzas are equally divided among party 1 and party 2 hence we can say that each of Party 1 and Party 2 received 120 pizzas. 

We know that all of the pizza can be classified into a total of 4 types. Hence, on drawing a table which can accommodate all of the cases: 

Total number of Thin Crust pizzas = 0.375*800 = 300. Therefore, total number of Deep Dish pizzas = 800 – 300 = 500. 

Out of 120 pizzas that Party 1 received, 60% were of Thin Crust type hence, total number of Thin Crust pizza received by Party 1 = 0.6*120 = 72. Consequently Party 1, must have received 42 Deep Dish type pizzas. 

Out of 120 pizzas that Party 2 received, 55% were of Thin Crust type hence, total number of Thin Crust pizza received by Party 2 = 0.55*120 = 66. Consequently Party 1, must have received 54 Deep Dish type pizzas. 

Therefore, total number of Thin Crust pizzas ordered by Party 3 = Total Thin Crust pizzas ordered – Thin Crust pizzas ordered by Party 1 – Thin Crust pizzas ordered by Party 2 

⇒ 300 – 72 – 66 = 162 

Hence number of Deep Dish type of pizzas order by Party 3 = 560 – 162 = 398 

Total number of Normal Cheese pizzas require to be delivered = 0.52*800 = 416 

Number of Normal Cheese pizzas require to be delivered to Party 2 = 0.3*120 = 36 

Number of Normal Cheese pizzas require to be delivered to Party 3 = 0.65*560 = 364 

Therefore, total number of Normal Cheese pizzas require to be delivered to Party 1 = Total Normal Cheese pizzas to be delivered – Normal Cheese pizzas require to be delivered to Party 2 – Normal Cheese pizzas require to be delivered to Party 3 

⇒ 416 – 36 – 364 = 16 

Hence, option C is the correct answer. 

Q. 37 For Party 2, if 50% of the Normal Cheese pizzas were of Thin Crust variety, what was the difference between the numbers of T-EC. and D-EC. pizzas to be delivered to Party 2? 

A. 18 

B. 12 

C. 30 

D. 24 

Answer: B. 

Explanation: 

We are given that Party 3 received 70% of total pizzas,therefore, number of pizzas received by Party 3 =70/100 × 800 = 560 

Remaining 240 pizzas are equally divided among party 1 and party 2 hence we can say that each of Party 1 and Party 2 received 120 pizzas. 

We know that all of the pizza can be classified into a total of 4 types. Hence, on drawing a table which can accommodate all of the cases: 

Total number of Thin Crust pizzas = 0.375*800 = 300. Therefore, total number of Deep Dish pizzas = 800 – 300 = 500. 

Out of 120 pizzas that Party 1 received, 60% were of Thin Crust type hence, total number of Thin Crust pizza received by Party 1 = 0.6*120 = 72. Consequently Party 1, must have received 42 Deep Dish type pizzas. 

Out of 120 pizzas that Party 2 received, 55% were of Thin Crust type hence, total number of Thin Crust pizza received by Party 2 = 0.55*120 = 66. Consequently Party 1, must have received 54 Deep Dish type pizzas. 

Therefore, total number of Thin Crust pizzas ordered by Party 3 = Total Thin Crust pizzas ordered – Thin Crust pizzas ordered by Party 1 – Thin Crust pizzas ordered by Party 2 

⇒ 300 – 72 – 66 = 162 

Hence number of Deep Dish type of pizzas order by Party 3 = 560 – 162 = 398 

Number of Normal Cheese pizzas require to be delivered to Party 2 = 0.3*120 = 36 

It is given that 50% of these Normal Cheese pizzas were of Thin Crust variety, then We can say that remaining 50% were of Deep Dish variety. We can find out each of 4 types of pizzas require to be delivered to Party 2. 

Hence, the difference between the numbers of T-EC. and D-EC. pizzas to be delivered to Party 2 = 48 – 36 = 12 Therefore, option B. is the correct answer. 

Q. 38 Suppose that a T-NC pizza cost as much as a D-NC pizza, but 3/5th of the price of a D-EC pizza A D-EC pizza costs Rs. 50 more than a T-EC pizza, and the latter costs Rs. 500. 

If 25% of the Normal Cheese pizzas delivered to Party 1 were of Deep Dish variety, what was the total bill for Party 1? 

A. Rs. 59480 

B. Rs. 59840 

C. Rs. 42520 

D. Rs. 45240 

Answer: A. 

Explanation: 

We are given that Party 3 received 70% of total pizzas,therefore, number of pizzas received by Party 3 =70/100 × 800  = 560 

Remaining 240 pizzas are equally divided among party 1 and party 2 hence we can say that each of Party 1 and Party 2 received 120 pizzas. 

We know that all of the pizza can be classified into a total of 4 types. Hence, on drawing a table which can accommodate all of the cases: 

Total number of Thin Crust pizzas = 0.375*800 = 300. Therefore, total number of Deep Dish pizzas = 800 – 300 = 500. 

Out of 120 pizzas that Party 1 received, 60% were of Thin Crust type hence, total number of Thin Crust pizza received by Party 1 = 0.6*120 = 72. Consequently Party 1, must have received 42 Deep Dish type pizzas. 

Out of 120 pizzas that Party 2 received, 55% were of Thin Crust type hence, total number of Thin Crust pizza received by Party 2 = 0.55*120 = 66. Consequently Party 1, must have received 54 Deep Dish type pizzas. 

Therefore, total number of Thin Crust pizzas ordered by Party 3 = Total Thin Crust pizzas ordered – Thin Crust pizzas 

ordered by Party 1 – Thin Crust pizzas ordered by Party 2 

⇒ 300 – 72 – 66 = 162 

Hence number of Deep Dish type of pizzas order by Party 3 = 560 – 162 = 398 

Total number of Normal Cheese pizzas require to be delivered = 0.52*800 = 416 

Number of Normal Cheese pizzas require to be delivered to Party 2 = 0.3*120 = 36 

Number of Normal Cheese pizzas require to be delivered to Party 3 = 0.65*560 = 364 

Therefore, total number of Normal Cheese pizzas require to be delivered to Party 1 = Total Normal Cheese pizzas to be delivered – Normal Cheese pizzas require to be delivered to Party 2 – Normal Cheese pizzas require to be delivered to Party 3 

⇒ 416 – 36 – 364 = 16 

It is given that 25% of these 16 Normal Cheese pizzas were of Deep Dish type, hence the number of D- NC. type pizza require to be delivered to Party 1 = 0.25*16 = 4 

Consequently, the number of T- NC. type pizza require to be delivered to Party 1 = 16 – 4 = 12 We can find out each type of pizza that is required to be delivered to Party 1. 

Cost Price of a T-EC. pizza = Rs. 500 

Cost Price of a D-EC. pizza = Rs. 550 

Cost Price of a T-NC. pizza =3/5 ×550= Rs. 330 

Cost Price of a D-NC. pizza =3/5 ×550= Rs. 330 

Therefore the total bill amount for Party 1 = 12*330 + 60*500 + 4*330 + 44*550 = Rs. 59480 Therefore, option A is the correct answer. 

 

Instructions 

There were seven elective courses – E1 to E7 – running in a specific term in a college. Each of the 300 students enrolled had chosen just one elective from among these seven. However, before the start of the term, E7 was withdrawn as the instructor concerned had left the college. The students who had opted for E7 were allowed to join any of the remaining 

electives. Also, the students who had chosen other electives were given one chance to change their choice. The table below captures the movement of the students from one elective to another during this process. Movement from one elective to the same elective simply means no movement. Some numbers in the table got accidentally erased; however, it is known that these were either 0 or 1. 

Further, the following are known: 

1. Before the change process there were 6 more students in E1 than in E4, but after the reshuffle, the number of students in E4 was 3 more than that in E1. 

2. The number of students in E2 increased by 30 after the change process. 

3. Before the change process, E4 had 2 more students than E6, while E2 had 10 more students than E3. 

Q. 39 How many elective courses among E1 to E6 had a decrease in their enrollments after the change process? 

A.

B.

C.

D.

Answer: C. 

Explanation: 

From the table we can say that number of students who opted for E2 after reshuffle = 5 + 34 + 6 + 3 + 5 + 7 + 16 = 76. 

It is given us that the number of students in E2 increased by 30 after the change process. Hence, we can say that the number of students who were enrolled in E2 before reshuffle = 76 – 30 = 46. 

It is given that before the change process there were 10 more students in E2 than in E3. Therefore, the number of students who were enrolled in E3 before reshuffle = 46 – 10 = 36. 

Number of students who moved from E1 to all other electives are known. Therefore, the number of students who were enrolled in E1 before reshuffle = 9 + 5 + 10 + 1 + 4 + 2 = 31. 

It is given that before the change process there were 6 more students in E1 than in E4. Therefore, the number of students who were enrolled in E4 before reshuffle = 31 – 6 = 25. 

Also, it is given that E4 had 2 more students than E6 before reshuffle. Therefore, the number of students who were enrolled in E6 before reshuffle = 25 – 2 = 23. 

All the students from E7 moved to one of electives among E1 to E6. 

Therefore, the number of students who were enrolled in E7 before reshuffle = 4 + 16 + 30 + 5 + 5 + 41 = 101. 

Except E5 we know the number of students who were enrolled in all electives. We also know that there were a total 300 students who opted for exactly 1 elective. 

Hence, the the number of students who were enrolled in E7 before reshuffle = 300 – (46+36+31+25+23+101) = 38. 

For each elective, the number of students who were enrolled before reshuffle will be same as sum of the number of students who moved from that elective to another elective including no movement cases. 

For elective E2, 

Number of students who moved to E1 + 34 + 8 + Number of students who moved to E4 + 2 + 2 = 46 i.e. Number of students who moved from to E1 = Number of students who moved from to E4 = 0 

For elective E4, 

Number of students who moved to E1 + 3 + 2 + 14 + Number of students who moved to E5 + 4 = 25 

i.e. Number of students who moved from to E1 = Number of students who moved from to E5 = 1 {As the remaining blanks can be filled by either 0 or 1} 

For elective E6, 

Number of students who moved to E1 + 7 + 3 + Number of students who moved to E4 + 2 + 9 = 23 

i.e. Number of students who moved from to E1 = Number of students who moved from to E4 = 1 {As the remaining blanks can be filled by either 0 or 1} 

It is given that after the reshuffle, the number of students in E4 was 3 more than that in E1. 

As of now the number of students enrolled in E4 after reshuffle = 1 + 0 + E3 to E4 + 14 + E5 to E4 + 1 + 5 = 21 + {E3 to E4} + {E5 to E4} 

Also, the number of students enrolled in E1 after reshuffle = 9 + 0 + 2 + 1 + E5 to E1 + 1 + 4 = 17 + E5 to E1. Hence, it is possible only when E5 to E1 = 1 and E3 to E4 = E5 to E4 = 0. 

Remaining blank places can be filled easily as we know the total sum of each row. 

Therefore, the number of students who moved from E3 to E5 = the number of students who moved from E5 to E3 = the number of students who moved from E5 to E6 = 1. 

Form the table we can see that the number of students who enrolled for E1 and E4 decreased from 31 and 25 to 18 and 21 respectively. 

Therefore, option C. is the correct answer. 

Q. 40 After the change process, which of the following is the correct sequence of number of students in the six electives E 1 to E6? 

A. 19, 76, 79, 21, 45, 60 

B. 19, 76, 78, 22, 45, 60 

C. 18, 76, 79, 23, 43, 61 

D. 18, 76, 79, 21, 45, 61 

Answer: D. 

Explanation: 

From the table we can say that the number of students who opted for E2 after reshuffle = 5 + 34 + 6 + 3 + 5 + 7 + 16 = 76. 

It is given to us that the number of students in E2 increased by 30 after the change process. Hence, we can say that the number of students who were enrolled in E2 before reshuffle = 76 – 30 = 46. 

It is given that before the change process there were 10 more students in E2 than in E3. Therefore, the number of students who were enrolled in E3 before reshuffle = 46 – 10 = 36. 

Number of students who moved from E1 to all other electives are known. Therefore, the number of students who were enrolled in E1 before reshuffle = 9 + 5 + 10 + 1 + 4 + 2 = 31. 

It is given that before the change process there were 6 more students in E1 than in E4. Therefore, the number of students who were enrolled in E4 before reshuffle = 31 – 6 = 25. 

Also, it is given that E4 had 2 more students than E6 before reshuffle. Therefore, the number of students who were enrolled in E6 before reshuffle = 25 – 2 = 23. 

All the students from E7 moved to one of electives among E1 to E6. Therefore, the number of students who were enrolled in E7 before reshuffle = 4 + 16 + 30 + 5 + 5 + 41 = 101. 

Except E5 we know the number of students who were enrolled in all electives. We also know that there were total 300 students who opted for exactly 1 elective. 

Hence, the the number of students who were enrolled in E7 before reshuffle = 300 – (46+36+31+25+23+101) = 38. 

For each elective, the number of students who were enrolled before reshuffle will be same as sum of the number of students who moved from that elective to another elective including no movement cases. 

For elective E2, 

Number of students who moved to E1 + 34 + 8 + Number of students who moved to E4 + 2 + 2 = 46 i.e. Number of students who moved from to E1 = Number of students who moved from to E4 = 0

For elective E4, 

Number of students who moved to E1 + 3 + 2 + 14 + Number of students who moved to E5 + 4 = 25 

i.e. Number of students who moved from to E1 = Number of students who moved from to E5 = 1 {As the remaining blanks can be filled by either 0 or 1} 

For elective E6, 

Number of students who moved to E1 + 7 + 3 + Number of students who moved to E4 + 2 + 9 = 23 

i.e. Number of students who moved from to E1 = Number of students who moved from to E4 = 1 {As the remaining blanks can be filled by either 0 or 1} 

It is given that after the reshuffle, the number of students in E4 was 3 more than that in E1. As of now the number of 

students enrolled in E4 after reshuffle = 1 + 0 + E3 to E4 + 14 + E5 to E4 + 1 + 5 = 21 + {E3 to E4} + {E5 to E4} Also, the number of students enrolled in E1 after reshuffle = 9 + 0 + 2 + 1 + E5 to E1 + 1 + 4 = 17 + E5 to E1. Hence, it is possible only when E5 to E1 = 1 and E3 to E4 = E5 to E4 = 0. 

Remaining blank places can be filled easily as we know the total sum of each row. 

Therefore, the number of students who moved from E3 to E5 = the number of students who moved from E5 to E3 = the number of students who moved from E5 to E6 = 1. 

Form the table, we can see that after the reshuffle the number of students in electives E1 to E6 are 18, 76, 79, 21, 45 and 61 in that order. 

Therefore, option D. is the correct answer. 

Q. 41 After the change process, which course among E1 to E6 had the largest change in its enrollment as a percentage of its original enrollment? 

A. E1 

B. E2 

C. E3 

D. E6 

Answer: D. 

Explanation: 

From the table we can say that number of students who opted for E2 after reshuffle = 5 + 34 + 6 + 3 + 5 + 7 + 16 = 76. 

It is given us that the number of students in E2 increased by 30 after the change process. Hence, we can say that the number of students who were enrolled in E2 before reshuffle = 76 – 30 = 46. 

It is given that before the change process there were 10 more students in E2 than in E3. Therefore, the number of students who were enrolled in E3 before reshuffle = 46 – 10 = 36. 

Number of students who moved from E1 to all other electives are known. Therefore, the number of students who were enrolled in E1 before reshuffle = 9 + 5 + 10 + 1 + 4 + 2 = 31. 

It is given that before the change process there were 6 more students in E1 than in E4. Therefore, the number of students who were enrolled in E4 before reshuffle = 31 – 6 = 25. 

Also, it is given that E4 had 2 more students than E6 before reshuffle. Therefore, the number of students who were enrolled in E6 before reshuffle = 25 – 2 = 23. 

All the students from E7 moved to one of electives among E1 to E6. Therefore, the number of students who were enrolled in E7 before reshuffle = 4 + 16 + 30 + 5 + 5 + 41 = 101. 

Except E5 we know the number of students who were enrolled in all electives. We also know that there were total 300 students who opted for exactly 1 elective. 

Hence, the the number of students who were enrolled in E7 before reshuffle = 300 – (46+36+31+25+23+101) = 38. 

For each elective, the number of students who were enrolled before reshuffle will be same as sum of the number of students who moved from that elective to another elective including no movement cases. 

For elective E2, 

Number of students who moved to E1 + 34 + 8 + Number of students who moved to E4 + 2 + 2 = 46 i.e. Number of students who moved from to E1 = Number of students who moved from to E4 = 0

For elective E4, 

Number of students who moved to E1 + 3 + 2 + 14 + Number of students who moved to E5 + 4 = 25 

i.e. Number of students who moved from to E1 = Number of students who moved from to E5 = 1 {As the remaining blanks can be filled by either 0 or 1} 

For elective E6, 

Number of students who moved to E1 + 7 + 3 + Number of students who moved to E4 + 2 + 9 = 23 

i.e. Number of students who moved from to E1 = Number of students who moved from to E4 = 1 {As the remaining blanks can be filled by either 0 or 1} 

It is given that after the reshuffle, the number of students in E4 was 3 more than that in E1. As of now the number of 

students enrolled in E4 after reshuffle = 1 + 0 + E3 to E4 + 14 + E5 to E4 + 1 + 5 = 21 + {E3 to E4} + {E5 to E4} Also, the number of students enrolled in E1 after reshuffle = 9 + 0 + 2 + 1 + E5 to E1 + 1 + 4 = 17 + E5 to E1. Hence, it is possible only when E5 to E1 = 1 and E3 to E4 = E5 to E4 = 0. 

Remaining blank places can be filled easily as we know the total sum of each row. 

Therefore, the number of students who moved from E3 to E5 = the number of students who moved from E5 to E3 = the number of students who moved from E5 to E6 = 1. 

We are asked the largest change in its enrollment as a percentage of its original enrollment for all 6 electives but as we can see there are only 4 electives. Hence, we will check only for E1, E2, E3 and E6. 

The percentage change in the number of students for E1 =(18 − 31)/31  × 100 ≈ 42 % 

The percentage change in the number of students for E2 =(76 − 46)/46  × 100 ≈ 65 % 

The percentage change in the number of students for E3 =(79 − 36)/36  × 100 ≈ 119 % 

The percentage change in the number of students for E6 =(61 − 23)/23  × 100 ≈ 165 % 

We can see that the percent change in the number of student for E6 is the largest. Therefore, option D. is the correct 

answer. 

Q. 42 Later, the college imposed a condition that if after the change of electives, the enrollment in any elective (other than E7) dropped to less than 20 students, all the students who had left that course will be required to re-enroll for that elective. 

Which of the following is a correct sequence of electives in decreasing order of their final enrollments? 

A. E2, E3, E6, E5, E1, E4 

B. E3, E2, E6, E5, E4, E1 

C. E2, E5, E3, E1, E4, E6 

D. E2, E3, E5, E6, E1, E3 

Answer: A. 

Explanation: 

From the table we can say that number of students who opted for E2 after reshuffle = 5 + 34 + 6 + 3 + 5 + 7 + 16 = 76. 

It is given us that the number of students in E2 increased by 30 after the change process. Hence, we can say that the number of students who were enrolled in E2 before reshuffle = 76 – 30 = 46. 

It is given that before the change process there were 10 more students in E2 than in E3. Therefore, the number of students who were enrolled in E3 before reshuffle = 46 – 10 = 36. 

Number of students who moved from E1 to all other electives are known. Therefore, the number of students who were enrolled in E1 before reshuffle = 9 + 5 + 10 + 1 + 4 + 2 = 31. 

It is given that before the change process there were 6 more students in E1 than in E4. Therefore, the number of students who were enrolled in E4 before reshuffle = 31 – 6 = 25. 

Also, it is given that E4 had 2 more students than E6 before reshuffle. Therefore, the number of students who were enrolled in E6 before reshuffle = 25 – 2 = 23. 

All the students from E7 moved to one of electives among E1 to E6. Therefore, the number of students who were enrolled in E7 before reshuffle = 4 + 16 + 30 + 5 + 5 + 41 = 101. 

Except E5 we know the number of students who were enrolled in all electives. We also know that there were total 300 students who opted for exactly 1 elective. 

Hence, the the number of students who were enrolled in E7 before reshuffle = 300 – (46+36+31+25+23+101) = 38. 

For each elective, the number of students who were enrolled before reshuffle will be same as sum of the number of students who moved from that elective to another elective including no movement cases. 

For elective E2, 

Number of students who moved to E1 + 34 + 8 + Number of students who moved to E4 + 2 + 2 = 46 i.e. Number of students who moved from to E1 = Number of students who moved from to E4 = 0 

For elective E4, 

Number of students who moved to E1 + 3 + 2 + 14 + Number of students who moved to E5 + 4 = 25 

i.e. Number of students who moved from to E1 = Number of students who moved from to E5 = 1 {As the remaining blanks can be filled by either 0 or 1} 

For elective E6, 

Number of students who moved to E1 + 7 + 3 + Number of students who moved to E4 + 2 + 9 = 23 

i.e. Number of students who moved from to E1 = Number of students who moved from to E4 = 1 {As the remaining blanks can be filled by either 0 or 1} 

It is given that after the reshuffle, the number of students in E4 was 3 more than that in E1. As of now the number of 

students enrolled in E4 after reshuffle = 1 + 0 + E3 to E4 + 14 + E5 to E4 + 1 + 5 = 21 + {E3 to E4} + {E5 to E4} Also, the number of students enrolled in E1 after reshuffle = 9 + 0 + 2 + 1 + E5 to E1 + 1 + 4 = 17 + E5 to E1. Hence, it is possible only when E5 to E1 = 1 and E3 to E4 = E5 to E4 = 0. 

Remaining blank places can be filled easily as we know the total sum of each row. 

Therefore, the number of students who moved from E3 to E5 = the number of students who moved from E5 to E3 = the number of students who moved from E5 to E6 = 1. 

We can see from the table that number of students enrolled in E1 dropped to 18. Hence, all the students who moved 

from E1 to any other elective will have to re-enroll in E1. 

We can see that the number of students who enrolled for E1 prior to reshuffle = 31. Out of these 31 students, 9 students didn’t move to any other elective whereas remaining 22 students moved to other electives. Hence, all these 22 students have to re-enroll in E1. 

Therefore, the total number of students in E1 post re-enrollment = 18 + 22 = 40 which is shown in the table.

Therefore, the sequence of electives in decreasing order of their final enrollments = E2, E3, E6, E5, E1, E4. 

Hence, option A. is the correct answer. 

 

Instructions 

An old woman had the following assets: 

(a) Rs. 70 lakh in bank deposits 

(b) 1 house worth Rs. 50 lakh 

(c) 3 flats, each worth Rs. 30 lakh 

(d) Certain number of gold coins, each worth Rs. 1 lakh 

She wanted to distribute her assets among her three children; Neeta, Seeta and Geeta. 

The house, any of the flats or any of the coins were not to be split. That is, the house went entirely to one child; a flat 

went to one child and similarly, a gold coin went to one child. 

Q. 43 Among the three, Neeta received the least amount in bank deposits, while Geeta received the highest. The value of the assets was distributed equally among the children, as were the gold coins. How much did Seeta receive in bank deposits (in lakhs of rupees)? 

A. 30 

B. 40 

C. 20 

D. 10 

Answer: C. 

Explanation: 

Neeta received the least amount in bank deposits implies she received the highest amount in property and the vice-versa for Geeta. The assets are 3 flats worth 90 lakh,a house worth 50 lakh, and a deposit worth 70 lakh. The total value of assets is 210 lakhs. They are divided equally, so each will receive assets worth 70 lakh. 

No one daughter can get 3 flats as the total value of asset will be 90 lakhs which is greater than actual share. 

All three daughters can’t get 1-1 flat each as well. In that case, the daughter who owns 1 flat and the house will have assets worth 30+50 = 80 lakhs which is more than the actual share. Hence, we can conclude the one of the three daughter gets 2 flats and 10 lakhs bank deposit. 

Out of the remaining two daughters, one will get the house and bank deposit worth 20 lakhs and the other one must have 1 flat and 40 lakhs in bank deposit. On the basis of bank distribution we can easily determine that property and bank deposits for each Neera, Seeta and Geeta. 

From the table, we can see that Seeta must have received Rs. 20 lakh in bank deposits. Hence, option C. is the correct answer. 

Q. 44 Among the three, Neeta received the least amount in bank deposits, while Geeta received the highest. The value of the assets was distributed equally among the children, as were the gold coins. 

How many flats did Neeta receive? 

Answer:

Explanation: 

Neeta received least amount in bank deposits implies she received highest amount in property and the vice-versa for Geeta. The assets are 3 flats worth 90 lakh,a house worth 50 lakh, and a deposit worth 70 lakh. The total value of assets is 210 lakhs. They are divided equally, so each will receive assets worth 70 lakh. 

No one daughter can get 3 flats as the total value of asset will be 90 lakhs which is greater than actual share. 

All three daughters can’t get 1-1 flat each as well. In that case, the daughter who owns 1 flat and the house will have assets worth 30+50 = 80 lakhs which is more than the actual share. Hence, we can conclude the one of the three daughter gets 2 flats and 10 lakhs bank deposit. 

Out of the remaining two daughters, one will get the house and bank deposit worth 20 lakhs and the other one must have 1 flat and 40 lakhs in bank deposit. On the basis of bank distribution we can easily determine that property and bank deposits for each Neera, Seeta and Geeta. 

From the table, we can see that Neeta received 2 flats. 

Q. 45 The value of the assets distributed among Neeta, Seeta and Geeta was in the ratio of 1:2:3, while the gold coins were distributed among them in the ratio of 2:3:4. One child got all three flats and she did not get the house. One child, other than Geeta, got Rs. 30 lakh in bank deposits. 

How many gold coins did the old woman have? 

A. 72 

B. 90 

C. 180 

D. 216 

Answer: B. 

Explanation: 

Let the total number of gold coins with the old woman be ‘9n’. 

Total value of the assets with the old woman = 50 + 3*30 + 70 + 9n = 210+9n. 

We know that the assets have been distributed in the ratio 1:2:3. 

Therefore, Neeta must have received 35+1.5n (by value), Seeta must have received 70+3n and Geeta must have received 105+4.5n. 

Further, it has been given that the gold coins distributed were in the ratio 2:3:4. 

Therefore, the number of gold coins with Neeta must be ‘2n’, Seeta must be ‘3n’ and Geeta must be ‘4n’. Seeta has ‘3n’ gold coins. Therefore, the total value of the assets with her must be 70. Seeta could not have inherited all the flats. Therefore, Seeta must have received the house ( worth 50 lakh) and 20 lakh from bank deposits. 

We know that Geeta did not receive Rs. 30 lakh from the bank deposits. Therefore, Neeta must have received Rs. 30 lakh. 

The remaining 5 lakh must be contributed by the gold coins (Since there is no other asset worth 5 lakh). => 5 + 1.5n = 2n 

=> 0.5n = 5 

=> n = 10 

The old-woman must have had 10*9 = 90 gold coins. Therefore, option B. is the right answer. 

Q. 46 The value of the assets distributed among Neeta, Seeta and Geeta was in the ratio of 1:2:3, while the gold coins were distributed among them in the ratio of 2:3:4. One child got all three flats and she did not get the house. One child, other than Geeta, got Rs. 30 lakh in. bank deposits. 

How much did Geeta get in bank deposits (in lakhs of rupees)? 

Answer:20 

Explanation: 

Let the total number of gold coins with the old woman be ‘9n’. 

Total value of the assets with the old woman = 50 + 3*30 + 70 + 9n = 210+9n. 

We know that the assets have been distributed in the ratio 1:2:3. 

Therefore, Neeta must have received 35+1.5n (by value), Seeta must have received 70+3n and Geeta must have received 105+4.5n. 

Further, it has been given that the gold coins distributed were in the ratio 2:3:4. 

Therefore, the number of gold coins with Neeta must be ‘2n’, Seeta must be ‘3n’ and Geeta must be ‘4n’. Seeta has 3n gold coins. Therefore, the total value of the assets with her must be 70. Seeta could not have inherited all the flats. Therefore, Seeta must have received the house ( worth 50 lakh) and 20 lakh from bank deposits. 

We know that Geeta did not receive Rs. 30 lakh from the bank deposits. Therefore, Neeta must have received Rs. 30 lakh. 

The remaining 5 lakh must be contributed by the gold coins (Since there is no other asset worth 5 lakh). 

=> 5 + 1.5n = 2n 

=> 0.5n = 5 

=> n = 10 

The oldwoman must have had 10*9 = 90 gold coins. 

Total assets = 210 + 90*1 = 300 lakh 

 

Neeta has received 50 lakh in total, Seeta has received 100 lakh and Geeta has received 150 lakh. Geeta must have received 90 lakh from 3 flats. Out of the remaining 60 lakh, 4*10 = 40 lakh has been contributed by the gold coins. Geeta must have received 150 – 90 – 40 = 20 lakh from bank deposits. Therefore, 20 is the right answer. 

 

Instructions 

At a management school, the oldest M dorms, numbered 1 to 10, need to be repaired urgently. This following diagram represents the estimated repair costs (in Rs. Crores for, the 10 dorms. For any dorm, the estimated repair cost (in Rs. Crores ) is an integer. Repairs with estimated cost Rs. 1 or 2 Crores are considered light repairs, repairs with estimated cost Rs. 3 or 4 are considered moderate repairs and repairs with estimated cost Rs. 5 or 6 Crores are considered extensive repairs. 

Further, the following information is known. 

1. Odd-numbered dorms do not need light repair; even-numbered dorms do not need moderate repair and dorms, whose numbers are divisible by 3, do not need extensive repair. 

2. Dorms 4 to 9 all need different repair costs, with Dorm 7 needing the maximum and Dorm 8 needing the minimum. 

Q. 47 Which of the following is NOT necessarily true? 

A. Dorm 1 needs a moderate repair 

B. Dorm 5 repair will cost no more than Rs. 4 Crores 

C. Dorm 7 needs an extensive repair 

D. Dorm 10 repair will cost no more than Rs. 4 Crores 

Answer: D. 

Explanation: 

Odd numbered dorms need either moderate or extensive repair. 

Even numbered dorms need either light or extensive repair. 

It has been given that dorms 4 to 9 all require different repairing costs. The dorms 3 and 9 should require moderate repair (going by the table). Dorm 7 costs the highest. Therefore, dorm 7 should require 6 crores to repair. Dorm 8 requires the least cost to repair. Therefore, dorm 8 should cost 1 crore to repair. We can eliminate these dorm numbers from other 2 lists. 

Dorms 4 to 9 cost different costs to repair. => Both dorms 5 and 9 cannot require the same cost of repair. Dorms 1 and 3 should require 3 crores to repair. 

Dorm 6 should require light repair (2 crores) since dorm 8 requires 1 crore to repair. 

=> Dorm 4 requires 5 crore to repair. 

We can see that all options except option D. are definitely true. Option D. cannot be ascertained to be true. Dorm 10 can 

cost Rs. 1 crore or Rs. 6 crores to repair. Therefore, option D. is the right answer. 

Q. 48 What is the total cost of repairing the odd-numbered dorms (in Rs. Crores)? 

Answer:19 

Explanation: 

Odd numbered dorms need either moderate or extensive repair. 

Even numbered dorms need either light or extensive repair. 

It has been given that dorms 4 to 9 all require different repairing costs. The dorms 3 and 9 should require moderate repair (going by the table). Dorm 7 costs the highest. Therefore, dorm 7 should require 6 crores to repair. Dorm 8 requires the least cost to repair. Therefore, dorm 8 should cost 1 crore to repair. We can eliminate these dorm numbers from other 2 lists. 

Dorms 4 to 9 cost different costs to repair. => Both dorms 5 and 9 cannot require the same cost of repair. Dorms 1 and 3 should require 3 crores to repair. 

Dorm 6 should require light repair (2 crores) since dorm 8 requires 1 crore to repair. 

=> Dorm 4 requires 5 crore to repair. 

Cost = 3 + 3 + 6 + 3 + 4 = Rs.19 crores. Therefore, 19 is the correct answer. 

Q. 49 Suppose further that: 

1. 4 of the 10 dorms needing repair are women’s dorms and need a total of Rs. 20 Crores for repair. 

2. Only one of Dorms 1 to 5 is a women’s dorm. 

What is the cost for repairing Dorm 9 (in Rs. Crores)? 

Answer:

Explanation: 

Odd numbered dorms need either moderate or extensive repair. 

Even numbered dorms need either light or extensive repair. 

It has been given that dorms 4 to 9 all require different repairing costs. The dorms 3 and 9 should require moderate repair (going by the table). Dorm 7 costs the highest. Therefore, dorm 7 should require 6 crores to repair. Dorm 8 requires the least cost to repair. Therefore, dorm 8 should cost 1 crore to repair. We can eliminate these dorm numbers from the other 2 lists. 

Dorms 4 to 9 cost different costs to repair. => Both dorms 5 and 9 cannot require the same cost of repair. Dorms 1 and 3 should require 3 crores to repair. 

Dorm 6 should require light repair (2 crores) since dorm 8 requires 1 crore to repair. 

=> Dorm 4 requires 5 crore to repair. 

There are 3 dorms from 6 to 10 which are women’s dorms. 

It has been given that the cost of repairing the woman dorms add up to 20. Therefore, the distribution of the costs should be 6+6+5+3. 

Dorm 4 is the dorm whose number is below 5 but is a woman’s dorm. Therefore, dorm 9 should cost Rs.3 crores to repair. Dorm 8 cannot be a woman’s dorm. Therefore, dorm 10 should be a woman’s dorm and should cost Rs. 6 crore to repair. 

Dorm 9 will cost Rs.9 crore to repair and hence, 9 is the correct answer. 

Q. 50 Suppose further that: 

1. 4 of the 10 dorms needing repair are women’s dorms and need a total of Rs. 20 Crores for repair. 

2. Only one of Dorms 1 to 5 is a women’s dorm. 

Which of the following is a women’s dorm? 

A. Dorm 2 

B. Dorm 5 

C. Dorm 8 

D. Dorm 10 

Answer: D. 

Explanation: 

Odd numbered dorms need either moderate or extensive repair. 

Even numbered dorms need either light or extensive repair. 

It has been given that dorms 4 to 9 all require different repairing costs. The dorms 3 and 9 should require moderate repair (going by the table). Dorm 7 costs the highest. Therefore, dorm 7 should require 6 crores to repair. Dorm 8 requires the least cost to repair. Therefore, dorm 8 should cost 1 crore to repair. We can eliminate these dorm numbers from the other 2 lists. 

Dorms 4 to 9 cost different costs to repair. => Both dorms 5 and 9 cannot require the same cost of repair. Dorms 1 and 3 should require 3 crores to repair. 

Dorm 6 should require light repair (2 crores) since dorm 8 requires 1 crore to repair. 

=> Dorm 4 requires 5 crore to repair. 

It has been given that the cost of repairing the woman dorms adds up to 20. Therefore, the distribution of the costs should be 6+6+5+3. 

Dorm 4 is the dorm whose number is below 5 but is a woman’s dorm. Therefore, dorm 9 should cost Rs.3 crores to repair. Dorm 8 cannot be a woman’s dorm. Therefore, dorm 10 should be a woman’s dorm and should cost Rs. 6 crore to repair. 

Hence, Option D. is the right answer. 

 

Instructions 

A. tea taster was assigned to rate teas from six different locations — Munnar, Wayanad, Ooty, Darjeeling, Assam and Himachal: These teas were placed in six cups, numbered 1 to 6, not necessarily in the same order. The tea taster was asked to rate these tears on the strength of their flavour on a scale of 1 to 10. He gave a unique integer rating to each tea. Some other information is given below: 

1. Cup 6 contained tea from Himachal. 

2. Tea from Ooty got the highest rating, but it was not in Cup 3. 

3. The rating of tea in Cup 3 was double the rating of the tea in Cup 5. 

4. Only two cups got ratings in even numbers. 

5. Cup 2 got the minimum rating and this rating was an even number. 

6. Tea in Cup 3 got a higher rating than that in Cup 1. 

7. The rating of tea from Wayanad was more than the rating of tea from Munnar, but less than that from Assam. 

Q. 51 What was the second highest rating given? 

Answer:

Explanation: 

Now we are given that the lowest rating is an even number and only 2 cups got an even number rating. Let’s take cases:- 

1. The lowest rating is 4 

If the lowest rating is 4 then the other ratings will be in the range 5-10. 

From this we need 4 odd and 1 even numbers. 

This is not possible as there are only 3 odd numbers from 5-10. 

Thus, the lowest rating is not 4. 

2. The lowest rating is 2. 

If the lowest rating in 4 then the other ratings will be in the range 5-10. 

From this we need 4 odd and 1 even numbers. 

This is possible when the odd ratings are 3,5,7 and 9. 

We are given that the highest rating is not even. Thus, 10 rating is not possible. 

We are also given that the rating of tea in Cup 3 was double the rating of the tea in Cup 5. Thus, the rating of the tea in cup 3 is an even number. 

Thus, the rating of the tea in cup 5 must be an odd number. 

Only 1 such pair is possible of 3 and 6. 

Thus, the tea in cup 2 got the rating of 2. 

The tea in cup 3 got a rating of 6 and the tea in cup 5 got a rating of 3. 

We are given that:- 

Tea in Cup 3 got a higher rating than that in Cup 1. 

Thus, the tea in cup 1 got a rating of 5. 

Cup 6 contained tea from Himachal and the Tea from Ooty got the highest rating. 

Thus, cup 6 got a rating of 7 and cup 4 got a rating of 9. 

The table is as shown below:- 

Hence, 7 is the 2nd highest rating given. 

Q. 52 What was the number of the cup that contained tea from Ooty? 

Answer:

Explanation: 

Now we are given that the lowest rating is an even number and only 2 cups got an even number rating. Let’s take cases:- 

1. The lowest rating is 4 

If the lowest rating in 4 then the other ratings will be in the range 5-10. 

From this we need 4 odd and 1 even numbers. 

This is not possible as there are only 3 odd numbers from 5-10. 

Thus, the lowest rating is not 4. 

2. The lowest rating is 2. 

If the lowest rating in 4 then the other ratings will be in the range 5-10. 

From this we need 4 odd and 1 even numbers. 

This is possible when the odd ratings are 3,5,7 and 9. 

We are given that the highest rating is not even. Thus, 10 rating is not possible. 

We are also given that the rating of tea in Cup 3 was double the rating of the tea in Cup 5. Thus, the rating of the tea in cup 3 is an even number. 

Thus, the rating of the tea in cup 5 must be an odd number. 

Only 1 such pair is possible of 3 and 6. 

Thus, the tea in cup 2 got the rating of 2. 

The tea in cup 3 got a rating of 6 and the tea in cup 5 got a rating of 3. 

We are given that:- 

Tea in Cup 3 got a higher rating than that in Cup 1. 

Thus, the tea in cup 1 got a rating of 5. 

Cup 6 contained tea from Himachal and the Tea from Ooty got the highest rating. 

Thus, cup 6 got a rating of 7 and cup 4 got a rating of 9. 

The table is as shown below:- 

Thus, 4 was the number of the cup that contained tea from Ooty. 

Q. 53 If the tea from Munnar did not get the minimum rating, what was the rating of the tea from Wayanad? 

A.

B.

C.

D.

Answer: B. 

Explanation: 

Now we are given that the lowest rating is an even number and only 2 cups got an even number rating. Let’s take cases:- 

1. The lowest rating is 4 

If the lowest rating in 4 then the other ratings will be in the range 5-10. 

From this we need 4 odd and 1 even numbers. 

This is not possible as there are only 3 odd numbers from 5-10. 

Thus, the lowest rating is not 4. 

2. The lowest rating is 2. 

If the lowest rating in 4 then the other ratings will be in the range 5-10. 

From this we need 4 odd and 1 even numbers. 

This is possible when the odd ratings are 3,5,7 and 9. 

We are given that the highest rating is not even. Thus, 10 rating is not possible. 

We are also given that the rating of tea in Cup 3 was double the rating of the tea in Cup 5. Thus, the rating of the tea in cup 3 is an even number. 

Thus, the rating of the tea in cup 5 must be an odd number. 

Only 1 such pair is possible of 3 and 6. 

Thus, the tea in cup 2 got the rating of 2. 

The tea in cup 3 got a rating of 6 and the tea in cup 5 got a rating of 3. 

We are given that:- 

Tea in Cup 3 got a higher rating than that in Cup 1. 

Thus, the tea in cup 1 got a rating of 5. 

Cup 6 contained tea from Himachal and the Tea from Ooty got the highest rating. 

Thus, cup 6 got a rating of 7 and cup 4 got a rating of 9. 

The table is as shown below:- 

If the tea from Munnar did not get the minimum rating then it must have got the 2nd lowest rating as we know, 

Assam>Wyanand>Munnar. 

Thus, Wyanand must have got a rating of 5. 

Q. 54 If cups containing teas from Wayanad and Ooty had consecutive numbers, which of the following statements may be true? 

A. Cup 5 contains tea from Assam 

B. Cup 1 contains tea from Darjeeling 

C. Tea from Wayanad has got a rating of 6 

D. Tea from Darjeeling got the minimum rating 

Answer: B. 

Explanation: 

Now we are given that the lowest rating is an even number and only 2 cups got an even number rating. Let’s take cases:- 

1. The lowest rating is 4 

If the lowest rating in 4 then the other ratings will be in the range 5-10. 

From this we need 4 odd and 1 even numbers. 

This is not possible as there are only 3 odd numbers from 5-10. 

Thus, the lowest rating is not 4. 

2. The lowest rating is 2. 

If the lowest rating in 4 then the other ratings will be in the range 5-10. 

From this we need 4 odd and 1 even numbers. 

This is possible when the odd ratings are 3,5,7 and 9. 

We are given that the highest rating is not even. Thus, 10 rating is not possible. 

We are also given that the rating of tea in Cup 3 was double the rating of the tea in Cup 5. 

Thus, the rating of the tea in cup 3 is an even number. 

Thus, the rating of the tea in cup 5 must be an odd number. 

Only 1 such pair is possible of 3 and 6. 

Thus, the tea in cup 2 got the rating of 2. 

The tea in cup 3 got a rating of 6 and the tea in cup 5 got a rating of 3. 

We are given that:- 

Tea in Cup 3 got a higher rating than that in Cup 1. 

Thus, the tea in cup 1 got a rating of 5. 

Cup 6 contained tea from Himachal and the Tea from Ooty got the highest rating. 

Thus, cup 6 got a rating of 7 and cup 4 got a rating of 9. 

The table is as shown below:- 

It is given that the rating of Assam>Wayanad>Munnar 

Hence, since Wayanad and Ooty are in consecutive cups, Wayanad can be either in cup number 3 or 5. 

So Wayanad can only be in cup number 5, then Munnar will be in cup number 2. So Darjeeling and Assam can be in cup 1 and 3 in any order. 

Hence B. is a possibility. 

 

Instructions 

In an 8 X 8 chess board a queen placed any where can attack another piece if the piece is present in the same row, or in the same column or in any diagonal position in any possible 4 directions, provided there is no other piece in between in the path from the queen to that piece. 

The columns are labelled a to h (left to right) and the rows are numbered 1 to 8 (bottom to top). The position of a piece  is given by the combination of column and row labels. For example, position c5 means that the piece is in column cth And 5th row. 

Q. 55 If the queen is at c5, and the other pieces at positions c2, g1, g3, g5 and a3, how many are under attack by the queen? There are no other pieces on the board. 

A.

B.

C.

D.

Answer: C. 

Explanation: 

Let us draw the diagram and mark position of various pieces as given in the question. 

Attack line is shown by the yellow color. All the pieces on this line will be under attack. 

From the diagram we can see that a3, g1, c2 and g5 are under attack. Hence, option C. is the correct answer. 

Q. 56 If the other pieces are only at positions a1, a3, b4, d7, h7 and h8, then which of the following positions of the queen results in the maximum number of pieces being under attack? 

A. f8 

B. a7 

C. c1 

D. d3 

Answer: D. 

Explanation: 

Option (a): When queen is at f8. In this case h8 and b4 will be under attack.

Option (b): When queen is at a7. In this case a3 and d7 will be under attack.

Option (c): When queen is at c1. In this case a1 and a3 will be under attack.

Option (d): When queen is at d3. In this case a3, d7 and h3 will be under attack. 

Therefore, we can say that option D. is the correct answer. 

Q. 57 If the other pieces are only at positions a1, a3, b4, d7, h7 and h8, then from how many positions the queen cannot attack any of the pieces? 

A.

B.

C.

D.

Answer: C. 

Explanation: 

From the diagram we can see that except positions e2, f2, g2 and g5 queen can attack at least one among the given pieces. 

Hence, we can say that there are exactly for position from where queen can’t attack any of the given pieces. Therefore, option C. is the correct answer. 

Q. 58 Suppose the queen is the only piece on the board and it is at position d5. In how many positions can another piece be placed on the board such that it is safe from attack from the queen? 

A. 32 

B. 35 

C. 36 

D. 37 

Answer: C. 

Explanation: 

From the diagram we can see that the number of positions those are safe from queen’s attack = 6 + 5 + 5 + 5 + 5 + 5 + 5 = 36. Therefore, option C is the correct answer. 

 

Instructions 

Eight friends: Ajit, Byomkesh, Gargi, Jayanta, Kikira, Manik, Prodosh and Tapesh are going to Delhi from Kolkata by a flight operated by Cheap Air. In the flight, sitting is arranged in 30 rows, numbered 1 to 30, each consisting of 6 seats, marked by letters A. to F from left to right, respectively. Seats A. to C. are to the left of the aisle (the passage running from the front of the aircraft to the back), and seats D. to F are to the right of the aisle. Seats A. and F are by the windows and referred to as Window seats, C. and D. are by the aisle and are referred to as Aisle seats while B. and E are referred to as Middle seats. Seats marked by consecutive letters are called consecutive seats (or seats next to each other). A. seat number is a combination of the row number, followed by the letter indicating the position in the row; e.g., 1A. is the left window seat in the first row, while 12E is the right middle seat in the 12th row. 

Cheap Air charges Rs. 1000 extra for any seats in Rows 1, 12 and 13 as those have extra legroom. For Rows 2- 10, it charges Rs. 300 extra for Window seats and Rs. 500 extra for Aisle seats. For Rows 11 and 14 to 20, it charges Rs. 200 extra for Window seats and Rs. 400 extra for Aisle seats. All other seats are available at no extra charge. 

The following are known: 

1. The eight friends were seated in six different rows. 

2. They occupied 3 Window seats, 4 Aisle seats and 1 Middle seat. 

3. Seven of them had to pay extra amounts, totaling to Rs. 4600, for their choices of seat. One of them did not pay any additional amount for his/her choice of seat. 

4. Jayanta, Ajit and Byomkesh were sitting in seats marked by the same letter, in consecutive rows in increasing order of row numbers; but all of them paid different amounts for their choices of seat. One of these amounts may be zero. 5. Gargi was sitting next to Kikira, and Manik was sitting next to Jayanta. 

6. Prodosh and Tapesh were sitting in seats marked by the same letter, in consecutive rows in increasing order of row numbers; but they paid different amounts for their choices of seat. One of these amounts may be zero. 

Q. 59 In which row was Manik sitting? 

A. 10 

B. 11 

C. 12 

D. 13 

Answer: A. 

Explanation: 

We are given that Jayanta, Ajit and Byomkesh were sitting in seats marked by the same letter, in consecutive rows in increasing order of row numbers; but all of them paid different amounts for their choices of seat. Let us see how the friends are supposed to pay for the seats they choose:- 

In row 1-1000 

In row 2-10 – 300 for window and 500 for aisle 

In row 11 – 200 for window and 400 for aisle 

In row 12,13 – 1000 

In row 14-20 – 200 for window and 400 for aisle 

In row 21-30 – 0 

Thus, As we can see 10, 11 and 12 are the only consecutive seats in which the amounts are different. Thus, Jayanth, Ajit and Byomkesh sat in row 10, row 11 and row 12. 

Manik sat beside Jayantha and thus Manik is also sitting in row 10. 

Now we are given that 7 of the 8 friends paid a total of 4600 Rs. 

Let’s start with the cases:- 

It is obvious that 5 friends cannot pay 1000 Rs for their seat because the amount will exceed 4600 Case 1:- 4 friends pay 1000 Rs each. Thus, the remaining friends will pay 600 Rs. 

This is possible only when each of them pay 200 Rs. 

So the case is- 1000*4 , 200*3 

Case 2 :- 3 friends pay 1000 Rs each. Thus, the remaining friends will pay 1600 Rs. 

There are 2 cases where this is possible:- 

1000*3, 500*2, 400, 200 

1000*3, 400*4 

Case 3:- 2 friends pay 1000 Rs each. Thus, the remaining 5 friends will pay 2600 Rs. 

This is not possible as each friend can pay a maximum of 500 Rs. 

Thus, the possible cases are 

1000*4 , 200*3 

1000*3, 500*2, 400, 200 

1000*3, 400*4 

As there is no case in which a friend has to pay 300 Rs thus, Jayantha must be sitting in row 10 aisle seat. Thus, Jayantha paid 500 Rs. 

Thus, the case is:- 1000*3, 500*2, 400, 200 

Thus, Manik must have also paid 500 sitting in row 10 aisle seat 

Ajit must be sitting in row 11 aisle seat paying 400 Rs. 

Byomyesh must be sitting in a row 12 aisle seat paying 1000 Rs. 

Thus, among Gargi, Kikira, Pradosh and Tapesh 2 must have paid 1000, 1 must have paid 200 and the remaining person must have paid nothing. 

Now we know Gargi and Kikira are sitting adjacent to each other and thus, either both or none of them must have paid 1000 Rs. 

Among Pradosh and Tapesh a maximum of 1 person could have paid 1000 Rs. 

Thus, the only possible case here is :- 

Gargi and Kikira paid 1000 each. 

Pradosh is sitting ahead of Tapesh and one of them paid 200 Rs. 

Since, both of them were sitting in seats marked by the same letter, in consecutive rows thus, the only possibility is Pradosh sitting in row 20 window seat and paying 200 and Tapesh sitting in row 21 paying nothing. Thus, the amount paid by each friend is as shown below: 

Manik is sitting in row 10. 

Q. 60 How much extra did Jayanta pay for his choice of seat? 

A. Rs. 300 

B. Rs. 400 

C. Rs. 500 

D. Rs. 1000 

Answer: C. 

Explanation: 

We are given that Jayanta, Ajit and Byomkesh were sitting in seats marked by the same letter, in consecutive rows in increasing order of row numbers; but all of them paid different amounts for their choices of seat. Let us see how the friends are supposed to pay for the seats they choose:- 

In row 1-1000 

In row 2-10 – 300 for window and 500 for aisle 

In row 11 – 200 for window and 400 for aisle 

In row 12,13 – 1000 

In row 14-20 – 200 for window and 400 for aisle 

In row 21-30 – 0 

Thus, As we can see 10, 11 and 12 are the only consecutive seats in which the amounts is different. Thus, Jayanth, Ajit and Byomkesh sat in row 10, row 11 and row 12. 

Manik sat beside Jayantha and thus Manik is also sitting in row 10. 

Now we are given that 7 of the 8 friends paid a total of 4600 Rs. 

Let’s start with the cases:- 

It is obvious that 5 friends cannot pay 1000 Rs for their seat because the amount will exceed 4600 Case 1:- 4 friends pay 1000 Rs each. Thus, the remaining friends will pay 600 Rs. 

This is possible only when each of them pay 200 Rs. 

So the case is- 1000*4 , 200*3 

Case 2 :- 3 friends pay 1000 Rs each. Thus, the remaining friends will pay 1600 Rs. 

There are 2 cases where this is possible:- 

1000*3, 500*2, 400, 200 

1000*3, 400*4 

Case 3:- 2 friends pay 1000 Rs each. Thus, the remaining 5 friends will pay 2600 Rs. 

This is not possible as each friend can pay a maximum of 500 Rs. 

Thus, the possible cases are 

1000*4 , 200*3 

1000*3, 500*2, 400, 200 

1000*3, 400*4 

As there is no case in which a friend has to pay 300 Rs thus, Jayantha must be sitting in row 10 aisle seat. Thus, Jayantha paid 500 Rs. 

Thus, the case is:- 

1000*3, 500*2, 400, 200 

Thus, Manik must have also paid 500 sitting in row 10 aisle seat 

Ajit must be sitting in row 11 aisle seat paying 400 Rs. 

Byomyesh must be sitting row 12 aisle seat paying 1000 Rs. 

Thus, among Gargi, Kikira, Pradosh and Tapesh 2 must have paid 1000, 1 must have paid 200 and the remaining person must have paid nothing. 

Now we know Gargi and Kikira are sitting adjacent to each other and thus, either both or none of them must have paid 1000 Rs. 

Among Pradosh and Tapesh a maximum of 1 person could have paid 1000 Rs. 

Thus, the only possible case here is :- 

Gargi and Kikira paid 1000 each. 

Pradosh is sitting ahead of Tapesh and one of them paid 200 Rs. 

Since, both of them were sitting in seats marked by the same letter, in consecutive rows thus, the only possibility is Pradosh sitting in row 20 window seat and paying 200 and Tapesh sitting in row 21 paying nothing. Thus, the amount paid by each friend is as shown below:- 

Jayanta paid 500 for her choice of seat. 

Q. 61 How much extra did Gargi pay for her choice of seat? 

A.

B. Rs. 300 

C. Rs. 400 

D. Rs. 1000 

Answer: D. 

Explanation: 

We are given that Jayanta, Ajit and Byomkesh were sitting in seats marked by the same letter, in consecutive rows in increasing order of row numbers; but all of them paid different amounts for their choices of seat. Let us see how the friends are supposed to pay for the seats they choose:- 

In row 1-1000 

In row 2-10 – 300 for window and 500 for aisle 

In row 11 – 200 for window and 400 for aisle 

In row 12,13 – 1000 

In row 14-20 – 200 for window and 400 for aisle 

In row 21-30 – 0 

Thus, As we can see 10, 11 and 12 are the only consecutive seats in which the amounts is different. Thus, Jayanth, Ajit and Byomkesh sat in row 10, row 11 and row 12. 

Manik sat beside Jayantha and thus Manik is also sitting in row 10. 

Now we are given that 7 of the 8 friends paid a total of 4600 Rs. 

Let’s start with the cases:- 

It is obvious that 5 friends cannot pay 1000 Rs for their seat because the amount will exceed 4600 Case 1:- 4 friends pay 1000 Rs each. Thus, the remaining friends will pay 600 Rs. 

This is possible only when each of them pay 200 Rs. 

So the case is- 1000*4 , 200*3 

Case 2 :- 3 friends pay 1000 Rs each. Thus, the remaining friends will pay 1600 Rs. 

There are 2 cases where this is possible:- 

1000*3, 500*2, 400, 200 

1000*3, 400*4 

Case 3:- 2 friends pay 1000 Rs each. Thus, the remaining 5 friends will pay 2600 Rs. 

This is not possible as each friend can pay a maximum of 500 Rs. 

Thus, the possible cases are 

1000*4 , 200*3 

1000*3, 500*2, 400, 200 

1000*3, 400*4 

As there is no case in which a friend has to pay 300 Rs thus, Jayantha must be sitting in row 10 aisle seat. Thus, Jayantha paid 500 Rs. 

Thus, the case is:- 

1000*3, 500*2, 400, 200 

Thus, Manik must have also paid 500 sitting in row 10 aisle seat 

Ajit must be sitting in row 11 aisle seat paying 400 Rs. 

Byomyesh must be sitting row 12 aisle seat paying 1000 Rs. 

Thus, among Gargi, Kikira, Pradosh and Tapesh 2 must have paid 1000, 1 must have paid 200 and the remaining person must have paid nothing. 

Now we know Gargi and Kikira are sitting adjacent to each other and thus, either both or none of them must have paid 1000 Rs. 

Among Pradosh and Tapesh a maximum of 1 person could have paid 1000 Rs. 

Thus, the only possible case here is :- 

Gargi and Kikira paid 1000 each. 

Pradosh is sitting ahead of Tapesh and one of them paid 200 Rs. 

Since, both of them were sitting in seats marked by the same letter, in consecutive rows thus, the only possibility is Pradosh sitting in row 20 window seat and paying 200 and Tapesh sitting in row 21 paying nothing. Thus, the amount paid by each friend is as shown below:- 

Gargi paid 1000 rs for her choice of seat 

Q. 62 Who among the following did not pay any extra amount for his his/her choice of seat? 

A. Kikira 

B. Manik 

C. Gargi 

D. Tapesh 

Answer: D. 

Explanation: 

We are given that Jayanta, Ajit and Byomkesh were sitting in seats marked by the same letter, in consecutive rows in increasing order of row numbers; but all of them paid different amounts for their choices of seat. 

Let us see how the friends are supposed to pay for the seats they choose:- 

In row 1-1000 

In row 2-10 – 300 for window and 500 for aisle 

In row 11 – 200 for window and 400 for aisle 

In row 12,13 – 1000 

In row 14-20 – 200 for window and 400 for aisle 

In row 21-30 – 0 

Thus, As we can see 10, 11 and 12 are the only consecutive seats in which the amounts is different. Thus, Jayanth, Ajit and Byomkesh sat in row 10, row 11 and row 12. 

Manik sat beside Jayantha and thus Manik is also sitting in row 10. 

Now we are given that 7 of the 8 friends paid a total of 4600 Rs. 

Let’s start with the cases:- 

It is obvious that 5 friends cannot pay 1000 Rs for their seat because the amount will exceed 4600 Case 1:- 4 friends pay 1000 Rs each. Thus, the remaining friends will pay 600 Rs. 

This is possible only when each of them pay 200 Rs. 

So the case is- 1000*4 , 200*3 

Case 2 :- 3 friends pay 1000 Rs each. Thus, the remaining friends will pay 1600 Rs. 

There are 2 cases where this is possible:- 

1000*3, 500*2, 400, 200 

1000*3, 400*4 

Case 3:- 2 friends pay 1000 Rs each. Thus, the remaining 5 friends will pay 2600 Rs. 

This is not possible as each friend can pay a maximum of 500 Rs. 

Thus, the possible cases are 

1000*4 , 200*3 

1000*3, 500*2, 400, 200 

1000*3, 400*4 

As there is no case in which a friend has to pay 300 Rs thus, Jayantha must be sitting in row 10 aisle seat. Thus, Jayantha paid 500 Rs. 

Thus, the case is:- 

1000*3, 500*2, 400, 200 

Thus, Manik must have also paid 500 sitting in row 10 aisle seat 

Ajit must be sitting in row 11 aisle seat paying 400 Rs. 

Byomyesh must be sitting row 12 aisle seat paying 1000 Rs. 

Thus, among Gargi, Kikira, Pradosh and Tapesh 2 must have paid 1000, 1 must have paid 200 and the remaining person must have paid nothing. 

Now we know Gargi and Kikira are sitting adjacent to each other and thus, either both or none of them must have paid 1000 Rs. 

Among Pradosh and Tapesh a maximum of 1 person could have paid 1000 Rs. 

Thus, the only possible case here is :- 

Gargi and Kikira paid 1000 each. 

Pradosh is sitting ahead of Tapesh and one of them paid 200 Rs. 

Since, both of them were sitting in seats marked by the same letter, in consecutive rows thus, the only possibility is Pradosh sitting in row 20 window seat and paying 200 and Tapesh sitting in row 21 paying nothing. Thus, the amount paid by each friend is as shown below:- 

Tapesh did not pay any amount. 

Instructions 

A. high security research lab requires the researchers to set a pass key sequence based on the scan of the five fingers of their left hands. When an employee first joins the lab, her fingers are scanned in an order of her choice, and then when she wants to re-enter the facility, she has to scan the five fingers in the same sequence. 

The lab authorities are considering some relaxation of the scan order requirements, since it is observed that some employees often get locked-out because they forget the sequence. 

Q. 63 The lab has decided to allow a variation in the sequence of scans of the five fingers so that at most two scans (out of five) are out of place. For example, if the original sequence is Thumb (T), index finger (I), middle finger (M), ring finger (R) and little finger (L) then TLMRI is also allowed, but TMRLI is not. How many different sequences of scans are allowed for any given person’s original scan? 

Answer:11 

Explanation: 

Let the original sequence be TIMRL 

Two fingers can be out of place. This can be done if and only if two fingers interchange their position. These two can be 5C2 = 10 

selected in ways. In addition to these, the original sequence will also be accepted. Hence the total number of acceptable sequences = 10 + 1 = 11 

Q. 64 The lab has decided to allow variations of the original sequence so that input of the scanned sequence of five fingers is allowed to vary from the original sequence by one place for any of the fingers. Thus, for example, if TIMRL is the original sequence, then ITRML is also allowed, but LIMRT is not. How many different sequences are allowed for any given person’s original scan? 

A.

B.

C.

D. 13 

Answer: C. 

Explanation: 

Input of the scanned sequence of five fingers is allowed to vary from the original sequence by one place for any of the fingers. This can be achieved only when two consecutive fingers are interchanged. Let the original sequence be TIMRL Case 1: Only a set of two consecutive numbers are interchanged. 

They can be selected in 5-1 = 4 ways 

Case 2: Two sets of two consecutive numbers are interchanged. 

(i) TI are interchanged, => (MR, RL) => 2 ways 

(ii) IM are interchanged => (RL) => 1 way 

Total no of ways possible = 4 + 2 + 1 = 7 

Including the original sequence, we get the total number of allowed combinations as 8 

Q. 65 The lab has now decided to require six scans in the pass key sequence, where exactly one finger is scanned twice, and the other fingers are scanned exactly once, which can be done in any order. For example, a possible sequence is TIMTRL. 

Suppose the lab allows a variation of the original sequence (of six inputs) where at most two scans (out of six) are out of place, as long as the finger originally scanned twice is scanned twice and other fingers are scanned once. 

How many different sequences of scans are allowed for any given person’s original scan? 

Answer:15 

Explanation: 

There can be two scans out of place. 

TIMTRL is the original sequence. 

If T is interchanged: There will be four ways: ITMTRL, MITTRL, RIMTTL, LIMTRT 

If I is interchanged: There will be four ways 

If M is interchanged: There will be three ways 

If T is interchanged: There will be two ways 

If R is interchanged: There will be one way 

Total 14. 

Another sequence allowed is original, So total 15 ways. 

Q. 66 The lab has now decided to require six scans in the pass key sequen.ce, where exactly one finger is scanned twice, and the other fingers are scanned exactly once, which can be done in any order. For example, a possible sequence is TIMTRL. 

Suppose the lab allows a variation of the original sequence (of six inputs) so that input in the form of scanned sequence of six fingers is allowed to vary from the original sequence by one place for any of the fingers, as long as the finger originally scanned twice is scanned twice and other fingers are scanned once. How many different sequences of scans are allowed if the original scan sequence is LRLTIM? 

A.

B. 11 

C. 13 

D. 14 

Answer: C. 

Explanation: 

1. If original sequence is given. 

2. If either of LR, RL, LT, TI, IM is interchanged => 5 ways. 

3. If LR and LT and IM interchanged. The sequence will look like: RLTLMI 

4. If LR and LT are interchanged. 

5. If LR and TI are interchanged. 

6. If LR and IM are interchanged. 

7. If RL and TI are interchanged. 

8. If RL and IM are interchanged. 

9. If LT and IM are interchanged. 

Total 13 ways possible. 

Quant 

Instructions 

For the following questions answer them individually 

Q. 67 The numbers 1, 2, …, 9 are arranged in a 3 X 3 square grid in such a way that each number occurs once and the entries along each column, each row, and each of the two diagonals add up to the same value. 

If the top left and the top right entries of the grid are 6 and 2, respectively, then the bottom middle entry is 

Answer:

Explanation: 

According to the question each column, each row, and each of the two diagonals of the 3X3 matrix add up to the same value. This value must be 15. 

Let us consider the matrix as shown below: 

Now we’ll try substituting values from 1 to 9 in the exact middle grid shown as ‘x’. 

If x = 1 or 3, then the value in the left bottom grid will be more than 9 which is not possible. x cannot be equal to 2. 

If x = 4, value in the left bottom grid will be 9. But then addition of first column will come out to be more than 15. Hence, not possible. 

If x=5, we get the grid as shown below: 

Hence, for x = 5 all conditions are satisfied. We see that the bottom middle entry is 3. 

Hence, 3 is the correct answer. 

Q. 68 In a 10 km race, A, B, and C, each running at uniform speed, get the gold, silver, and bronze medals, respectively. I f A beats B by 1 km and B beats C by 1 km, then by how many metres does A beat C? 

Answer:1900 

Explanation: 

By the time A traveled 10 KM, B traveled 9 KM 

SpeedA : SpeedB = 10 : 9 

Hence 

SpeedB : SpeedC = 10 : 9 

Similarly 

SpeedA : SpeedB : SpeedC = 100 : 90 : 81 

Hence 

Hence by the time A traveled 10 KMs , C should have traveled 8.1 KMs 

So A beat C by 1.9 KMs = 1900 Mts 

Q. 69 Bottle 1 contains a mixture of milk and water in 7: 2 ratio and Bottle 2 contains a mixture of milk and water in 9: 4 ratio. In what ratio of volumes should the liquids in Bottle 1 and Bottle 2 be combined to obtain a mixture of milk and water in 3:1 ratio? 

A. 27:14 

B. 27:13 

C. 27:16 

D. 27:18 

Answer: B. 

Explanation: 

The ratio of milk and water in Bottle 1 is 7:2 and the ratio of milk and water in Bottle 2 is 9:4 9713

Therefore, the proportion of milk in Bottle 1 is and the proportion of milk in Bottle 2 is Let the ratio in which they should be mixed be equal to X:1. 

Hence, the total volume of milk is  7X / 9 + 9/13 

The total volume of water is 2X / 9 + 4/13 

They are in the ratio 3:1 

Hence, 7X / 9 + 9/13 = 3*(2X / 9 + 4/13)

Therefore, 91X + 81 = 78X + 108 

Therefore X = 27/13 

Q. 70 Arun drove from home to his hostel at 60 miles per hour. While returning home he drove half way along the same route at a speed of 25 miles per hour and then took a bypass road which increased his driving distance by 5 miles, but allowed him to drive at 50 miles per hour along this bypass road. If his return journey took 30 minutes more than his onward journey, then the total distance traveled by him is 

A. 55 miles 

B. 60 miles 

C. 65 miles 

D. 70 miles 

Answer: C. 

Explanation: 

Let the distance between the home and office be miles 2x

Time taken for going in the morning = 2x/60hrs 

Time taken for going back in the evening = x/25+(x+5)/50. hrs 

It is given that he took 30 minutes (0.5 hrs) more in the evening

Hence 2x/60 hrs + 0.5 = x/25+(x+5)/50. hrs

Solving for x, we get x = 15 miles. 

Total distance traveled = 2x + x + x + 5 = 4x + 5 = 65 Miles 

Q. 71 Out of the shirts produced in a factory, 15% are defective, while 20% of the rest are sold in the domestic market. If the remaining 8840 shirts are left for export, then the number of shirts produced in the factory is 

A. 13600 

B. 13000 

C. 13400 

D. 14000 

Answer: B. 

Explanation: 

Let the total number of shirts be x. Hence number of non defective shirts = x – 15% of x = 0.85x 

Number of shirts left for export = No of non defective shirts – number of shirts sold in domestic market 

= No of non defective shirts – 20% of No of non defective shirts 

= 80% of No of non defective shirts 

Hence 8840 = 0.8 * (0.85x) . Solving for x we get, x = 13000 

Q. 72 The average height of 22 toddlers increases by 2 inches when two of them leave this group. If the average height of these two toddlers is one-third the average height of the original 22, then the average height, in inches, of the remaining 20 toddlers is 

A. 30 

B. 28 

C. 32 

D. 26 

Answer: C. 

Explanation: 

Let the average height of 22 toddlers be 3x. 

Sum of the height of 22 toddlers = 66x 

Hence average height of the two toddlers who left the group = x 

Sum of the height of the remaining 20 toddlers = 66x – 2x = 64x 

Average height of the remaining 20 toddlers = 64x/20 = 3.2x 

Difference = 0.2x = 2 inches => x = 10 inches 

Hence average height of the remaining 20 toddlers = 3.2x = 32 inches 

Q. 73 The manufacturer of a table sells it to a wholesale dealer at a profit of 10%. The wholesale dealer sells the table to a retailer at a profit of 30% Finally, the retailer sells it to a customer at a profit of 50%. If the customer pays Rs 4290 for the table, then its manufacturing cost (in Rs) is 

A. 1500 

B. 2000 

C. 2500 

D. 3000 

Answer: B. 

Explanation: 

Let the manufacturing price of the table = x

Hence the price at which the wholesaler bought from the manufacturer = 1.1 × x

The price at which the retailer bought from the wholesaler = 1.3 × 1.1 × x

The price at which the customer bought from the retailer = 1.5 × 1.3 × 1.1 × x

1.5 × 1.3 × 1.1 × x = 4290 

=> x = 2000

Q. 74 A tank has an inlet pipe and an outlet pipe. If the outlet pipe is closed then the inlet pipe fills the empty tank in 8 hours. If the outlet pipe is open then the inlet pipe fills the empty tank in 10 hours. If only the outlet pipe is open then in how many hours the full tank becomes half-full? 

A. 20 

B. 30 

C. 40 

D. 45 

Answer: A. 

Explanation: 

Let the time taken by the outlet pipe to empty = x hours 

Then, 1/81/x = 1/10

=>x = 40 

Hence time taken by the outlet pipe to make the tank half-full = 40/2 = 20 hour 

Q. 75 Mayank buys some candies for Rs 15 a dozen and an equal number of different candies for Rs 12 a dozen. He sells all for Rs 16.50 a dozen and makes a profit of Rs 150. How many dozens of candies did he buy altogether? 

A. 50 

B. 30 

C. 25 

D. 45 

Answer: A. 

Explanation: 

Let the number of dozens of candies he bought of each variety be x 

Hence total cost = 12x + 15x = 27x 

Total selling price = 16.50*2x = 33x 

Profit = 33x – 27x = 6x 

Given 6x = 150 => x = 25 

Hence he bought 50 dozens of candies in total 

Q. 76 In a village, the production of food grains increased by 40% and the per capita production of food grains increased by 27% during a certain period. The percentage by which the population of the village increased during the same period is nearest to 

A. 16 

B. 13 

C. 10 

D.

Answer: C. 

Explanation: 

Let initial population and production be x,y and final population be z 

Final production = 1.4y, final percapita = 1.27 times initial percapita 

=>1.4y/z = 1.27 × y/x 

=> z/x = 1.4/1.27 ≈ 1.10

Hence the percentage increase in population = 10% 

Q. 77 If a, b, c are three positive integers such that a and b are in the ratio 3 : 4 while b and c are in the ratio 2:1, then which one of the following is a possible value of (a + b + c)? 

A. 201 

B. 205 

C. 207 

D. 210 

Answer: C. 

Explanation: 

a : b = 3:4 and b : c = 2:1 => a:b:c = 3:4:2 

=> a = 3x, b = 4x , c = 2x 

=> a + b + c = 9x 

=> a + b + c is a multiple of 9. 

From the given options only, option C. is a multiple of 9 

Q. 78 A motorbike leaves point A at 1 pm and moves towards point B at a uniform speed. A car leaves point B at 2 pm and moves towards point A at a uniform speed which is double that of the motorbike. They meet at 3:40 pm at a point which is 168 km away from A What is the distance, in km, between A and B7 

A. 364 

B. 378 

C. 380 

D. 388 

Answer: B. 

Explanation: 

Let the distance traveled by the car be x KMs 

Distance traveled by the bike = 168 KMs 

Speed of car is double the speed of bike

=> x/ (3:40−2:00) = 2 × 168/(3:40−1:00) 

=> x/ 100 = 2 × 168/160 

=> x = 210

Hence the distance between A. and B. is x + 168 = 378 KMs 

Q. 79 Amal can complete a job in 10 days and Bimal can complete it in 8 days. Amal, Bimal and Kamal together complete the job in 4 days and are paid a total amount of Rs 1000 as remuneration. If this amount is shared by them in proportion to their work, then Kamal’s share, in rupees, is 

A. 100 

B. 200 

C. 300 

D. 400 

Answer: A. 

Explanation: 

Let the time take by kamal to complete the task be x days. 

Hence we have 1/10 + 1/8 + 1/x = ¼ 

=> x = 40 days. 

Ratio of the work done by them =1/10 : 1/8 : 1/40 = 4 : 5 : 1 

Hence the wage earned by Kamal = 1/10 * 1000 = 100 

Q. 80 Consider three mixtures — the first having water and liquid A in the ratio 1:2, the second having water and liquid B. in the ratio 1:3, and the third having water and liquid C in the ratio 1:4. These three mixtures of A, B, and C, respectively, are further mixed in the proportion 4: 3: 2. Then the resulting mixture has 

A. The same amount of water and liquid B 

B. The same amount of liquids B and C

C. More water than liquid B 

D. More water than liquid A 

Answer: C. 

Explanation: 

The proportion of water in the first mixture is ⅓  

The proportion of Liquid A in the first mixture is ⅔ 

The proportion of water in the second mixture is ¼ 

The proportion of Liquid B in the second mixture is ¾  

The proportion of water in the third mixture is ⅕ 

The proportion of Liquid C in the third mixture is ⅘ 

As they are mixed in the ratio 4:3:2, the final amount of water is 

4 × ⅓   + 3 × ¼ + 2 × ⅕ = 149/160

The final amount of Liquid A. in the mixture is 4 × ⅔ = 8/3

The final amount of Liquid B. in the mixture is 3 × ¾ = 9/4

The final amount of Liquid C. in the mixture is 2 × ⅘ = 8/5

Hence, the ratio of Water : A. : B. : C. in the final mixture is 149/160 : 8/3 : 9/4 : 8/5 = 

From the given choices, only option C. is correct. 

Q. 81 Let ABCDEF be a regular hexagon with each side of length 1 cm. The area (in sq cm) of a square with AC. as one side is 

A. 3√2

B. 4

C. 3

D. √3

Answer: B. 

Explanation: 

The length of the diagonals of a regular hexagon with side s are √3s. 

Here length of AC =√3s =√3 cms 

Hence area of the square =(√3)2  = 3 sq cm 

Q. 82 The base of a vertical pillar with uniform cross section is a trapezium whose parallel sides are of lengths 10 cm and 20 cm while the other two sides are of equal length. The perpendicular distance between the parallel sides of the trapezium is 12 cm. If the height of the pillar is 20 cm, then the total area, in sq cm, of all six surfaces of the pillar is 

A. 1300 

B. 1340 

C. 1480 

D. 1520 

Answer: C. 

Explanation: 

See the diagram below 

Length of side AD = √(122 + 52 )= 13

Area of the trapezium = 12 * (10 + 20)/2 = 180 

Perimeter of the trapezium = 10 + 20 + 13 + 13 = 56 

Area of the sides of the pillar = 56 * height = 56 * 20 = 1120. 

Total are of the pillar = 1120 + area of base + area of the top = 1120 + 180 + 180 = 1480 

Q. 83 The points (2, 5) and (6, 3) are two end points of a diagonal of a rectangle. If the other diagonal has the equation y =3x+c,then c is 

A. -5 

B. -6 

C. -7 

D. -8 

Answer: D. 

Explanation: 

The midpoint of one diagonal lies on the other diagonal. 

Midpoint is ((2+6)/2, (5+3)/2) = (4,4) 

Hence 4 = 3 * 4 + c => c = -8 

Q. 84 ABCD is a quadrilateral inscribed in a circle with centre O. If COD = 120 degrees and BAC = 30 degrees, BCD then the value of (in degrees) is 

Answer:90 

Explanation: 

COD = 120 => ∠CAD = 120/2 = 60 (The angle subtended by the chord DC. at the major arc is half the angle subtended at the centre of the circle.) 

BAC = 30 

BAD = ∠BAC + ∠CAD= 30 + 60 = 90. 

BCD = 180 − ∠BAD = 180 – 90 = 90 

Q. 85 If three sides of a rectangular park have a total length 400 ft, then the area of the park is maximum when the length (in ft) of its longer side is 

Answer:200 

Explanation: 

Let the length and breadth of the park be l,b, l > b 

Case 1: 2l + b = 400 

Area = lb. Area is maximum when 2l * b is maximum, which is maximum when 2l = b (using AM≥GM inequality) => l = 100, b = 200. Which can’t happen since l > b 

Case 2: l + 2b = 400 

Area = lb. Area is maximum when l *2 b is 

maximum, which is maximum when l = 2b (using AM≥GM inequality) => l = 200, b = 100. 

Hence length of the longer side is 200 ft 

Q. 86 Let P be an interior point of a right-angled isosceles triangle ABC. with hypotenuse AB. If the perpendicular 4(√2 − 1) distance of P from each of AB,BC,and CA. is cm,then the area, in sq cm, of the triangle ABC. is Answer:16 

Explanation: 

Let the length of non-hypotenuse sides of the right angled triangle be a. Then the hypotenuse h =√2a. 

P is equidistant from all the side of the triangle. Hence P is the incenter and the perpendicular distance is the inradius. 

In a right angled triangle, inradius =(a+bh)/ 2

=> (a+a− √2a)/2 = 4(√2 − 1) 

=> √2 a(√2 − 1) = 8(√2 − 1)

=> a = 4√2 

Area of the triangle =½ a2 = 16 sq cm 

Q. 87 If the product of three consecutive positive integers is 15600 then the sum of the squares of these integers is 

A. 1777 

B. 1785 

C. 1875 

D. 1877 

Answer: D. 

Explanation: 

(x − 1)x(x + 1) = 15600 

=> x3 x = 15600 

The nearest cube to 15600 is 15625 = 253

We can verify that x = 25 satisfies the equation above. 

Hence the three numbers are 24, 25, 26. Sum of their squares = 1877 

Q. 88 If x is a real number such that log3 5 = log5(2 + x), then which of the following is true? 

A. 0 < x < 3 

B. 23 < x < 30 

C. x > 30 

D. 3 < x < 23 

Answer: D. 

Explanation: 

=> 1 < log3 5 < 2 

=> 1 < log5(2 + x) < 2 

=> 5 < 2 + x < 25 

=> 3 < x < 23 

Q. 89 Let f(x) = x2 and g(x) = 2x , for all real x. Then the value of f[f(g(x)) + g(f(x))] at x = 1 is 

A. 16 

B. 18 

C. 36 

D. 40 

Answer: C. 

Explanation: 

f[f(g(1)) + g(f(1))] 

= f[f(21) + g(1 )]2 

=f[f(2) + g(1)] 

= f[22 + 2 ]1 

= f(6) 

=62 = 36 

Q. 90 The minimum possible value of the sum of the squares of the roots of the equation x2 + (a + 3)x − (a + 5) = 0 is

A.

B.

C.

D.

Answer: C. 

Explanation: 

Let the roots of the equation x2 + (a + 3)x − (a + 5) = 0 be equal to p, q

Hence ,p + q = −(a + 3) and p × q = −(a + 5) 

Therefore, p2 + q2 = a2 + 6a + 9 + 2a + 10 = a2 + 8a + 19 = (a + 4)2 + 3 

As (a + 4)2 is always non negative, the least value of the sum of squares is 3 

Q. 91  If 9x−1/2 − 22x−2= 4x − 32x−3, then x is 

A. 3/2 

B. 2/5 

C. 3/4 

D. 4/9 

Answer: A. 

Explanation: It is given that 9x−1/2 − 22x−2= 4x − 32x−3 

Let us try to reduce them to powers of 3 and 2

The given equation can be reduced to 32x−1 + 32x−3 = 22x + 22x−2 

Hence, 32x−3 × 10 = 22x−2 × 5 

Therefore, 32x−3 = 22x−3

This is possible only if 2x − 3 = 0 or x = 3/2

Q. 92 If log(2a × 3b × 5c )is the arithmetic mean of log(22 × 33 × 5), log(26 × 3 × 57 ), and log(2 × 32 × 54 ), then a equals 

Answer:

Explanation: 

log(2a × 3b × 5c ) = [log(22 × 33 × 5)+log(26 × 3 × 57 )+log(2 × 32 × 54 )] /3

log(2a × 3b × 5c ) = [log(22+6+1 ×33+1+2 ×51+7+4 ) ] /3

log(2a × 3b × 5c ) = [log(29 ×36 ×512 ) ] /3

3 * log(2a × 3b × 5c ) = [log(29 ×36 ×512 )]

Hence, 3a = 9 or a = 3 

Q. 93 Let a1, a2, a3, a4, a5 be a sequence of five consecutive odd numbers. Consider a new sequence of five consecutive even numbers ending with 2a3

If the sum of the numbers in the new sequence is 450, then a5 is 

Answer:51 

Explanation: 

Sum of the sequence of even numbers is 2a3 + (2a3 − 2) + (2a3 − 4) +(2a3 − 6) + (2a3 − 8)

=> 10a3 − 20 = 450

=> a3 = 47

Hence a5 = 47 + 4 = 51

Q. 94 How many different pairs(a,b) of positive integers are there such that a b and 1/a+1/b=1/9? 

Answer:

Explanation: 

1/a+1/b=1/9 

=>ab = 9(a + b) 

=>ab − 9(a + b) = 0 

=>ab − 9(a + b) + 81 = 81 

=>(a − 9)(b − 9) = 81, a > b 

Hence we have the following cases, 

=> a − 9 = 81, b − 9 = 1 (a, b) = (90, 10) 

=> a − 9 = 27, b − 9 = 3 (a, b) = (36, 12) 

=>a − 9 = 9, b − 9 = 9 (a, b) = (18, 18) 

Hence there are three possible positive integral values of (a,b) 

Q. 95 In how many ways can 8 identical pens be distributed among Amal, Bimal, and Kamal so that Amal gets at least 1 pen, Bimal gets at least 2 pens, and Kamal gets at least 3 pens? 

Answer:

Explanation: 

After Amal, Bimal and Kamal are given their minimum required pens, the pens left are 8 – (1 + 2 + 3) = 2 pens

Now these two pens have to be divided between three persons so that each person can get zero pens = 2+3−1C3−1 =

4C2  = 6 

Q. 96 How many four digit numbers, which are divisible by 6, can be formed using the digits 0, 2, 3, 4, 6, such that no digit is used more than once and 0 does not occur in the left-most position? 

Answer:50 

Explanation: 

For the number to be divisible by 6, the sum of the digits should be divisible by 3 and the units digit should be even. Hence we have the digits as 

Case I: 2, 3, 4, 6 

Now the units place can be filled in three ways (2,4,6), and the remaining three places can be filled in 3! = 6 ways. Hence total number of ways = 3*6 = 18 

Case II: 0, 2, 3, 4 

case II a: 0 is in the units place => 3! = 6 ways 

case II b: 0 is not in the units place => units place can be filled in 2 ways( 2,4) , thousands place can be filled in 2 ways ( remaining 3 – 0) and remaining can be filled in 2! = 2 ways. Hence total number of ways = 2 * 2 * 2 = 8 Total number of ways in this case = 6 + 8 = 14 ways. 

Case III: 0, 2, 4, 6 

case III a: 0 is in the units place => 3! = 6 ways 

case II b: 0 is not in the units place => units place can be filled in 3 ways( 2,4,6) , thousands place can be filled in 2 ways (remaining 3 – 0) and remaining can be filled in 2! = 2 ways. Hence total number of ways = 3 * 2 * 2 = 12 Total number of ways in this case = 6 + 12= 18 ways. 

Hence the total number of ways = 18 + 14 + 18 = 50 ways 

Q. 97 If f(ab) = f(a)f(b) for all positive integers a and b, then the largest possible value of f(1) is Answer:

Explanation: 

f(1 * 1) = f(1)f(1) 

=> f(1) = f(1)f(1) 

=> f(1) = 0 or f(1) = 1 

Hence maximum value of f(1) is 1 

Q. 98 Let f(x) = 2x − 5  and g(x) = 7 − 2x . Then |f(x)+ g(x)| = |f(x)|+ |g(x)| if and only if 

A. 5/2 <  x < 7/2

B.  x ≤ 5/2 or  x ≥ 7/2

C.  x < 5/2 or  x ≥ 7/2 

D. 5/2 ≤  x ≤ 7/2

Answer: D. 

Explanation: 

f(x) + g(x)∣ = ∣f(x)∣ + ∣g(x)∣ if and only if 

case 1: f(x) ≥ 0 and g(x) ≥ 0

=>2x − 5 ≥ 0 and 7 − 2x ≥ 0

=> x ≥ 5/2 and 7/2 ≥ x

=> 5/2 ≤  x ≤ 7/2

case 2: f(x) ≤ 0and g(x) ≤ 0

=> 2x − 5 ≤ 0 and 7 − 2x ≤ 0

=> x ≤ 5/2 and 7/ 2 ≤ x

So x<=5/2 and x>=7/2 which is not possible. 

Hence, answer is 5/2 ≤  x ≤ 7/2

Q. 99 An infinite geometric progression a1, a2, … has the property that an = 3(an+1 + an+2 + …) for every n 1. If the sum a1 + a2 + a3…+ = 32, then a5 is 

A. 1/32 

B. 2/32 

C. 3/32 

D. 4/32 

Answer: C. 

Explanation: 

Let the common ratio of the G.P. be r. 

an = 3(an+1 + an+2 + …) 

Hence we have 

The sum up to infinity of GP is given by a/1−r where a here is an+1

=> an = 3(an+1 / 1−r )

=> an = 3(an×r / 1−r )

=> r = ¼

Now, a1 + a2 + a3…+ = 32 

=> (a1 / 1−r ) =32

=> (a1 / ¾  ) =32

=> a1 = 24 

=> a5 = a1 × r4 

=> a5 = 24 × (1/4)4 = 3/32 

Q. 100 If a1=1/(2×5), a2=1/(5×8), a3=1/(8×11), then a1 + a2 + a3…+a100 is 

A. 25/151

B. ½ 

C. ¼ 

D. 111/55

Answer: A. 

Explanation: 

a100 =1/ [(3×100−1)×(3×100+2)] = 1/(299×302)

1/(2×5) = 1/3 × (½ – ⅕ )

1/(5×8) = 1/3 ×(⅕ -⅛ )

1/(8×11) = 1/3 × (⅛ -1/11)

…..

…..

…..

1/(299×302) = 1/3 × (1/299 -1/302)

Hence = a1 + a2 + a3 + … +a100 = 1/3 × (½ – ⅕ ) + 1/3 ×(⅕ -⅛ ) + 1/3 × (⅛ -1/11)+….. + 1/3 × (1/299 -1/302)

=> 1/3 × (1/2 -1/302)

=>25/151 

CAT Previous Year Paper Session-I 2017

CAT 2017 Session 1

 

Questions: 1 – 6

Understanding where you are in the world is a basic survival skill, which is why we, like most species come hard-wired with specialized brain areas to create cognitive maps of our surroundings. Where humans are unique, though, with the possible exceptions of honeybees, is that we try to communicate this understanding of the world with others. We have a long history of doing this by drawing maps – the earliest versions yet discovered were scrawled on cave walls 14,000 years ago. human cultures have been drawing them on stone tablets, papyrus, paper and now computer screens ever since. Given such a long history of human map-making, it is perhaps surprising that it is only within the last few hundred years that north has been consistenetly considered to be at the top. In fact, for much of human history, north almost never appreared at the top, according to Jerry Brotton, a map historian…” North was rarely put at the top for the simple fact that north is where darkness comes from,”he says. “West is also very unlikely to be put at the top because west is where the sun disappears.” Confusingly, early Chinese maps seem to buck this trend. But, Broton, says even though they did have compasses at the time, that isn’t the reason that they placed north at the top. Early Chinese compasses were actually oriented to point south, which was considered to be more desirable than deepest darkest north. But in Chinese maps, the emperor who lived in the north of the country was always put at the top of the map, with everyone else, his loyal subjects, looking up towards him. ” In Chinese culture the emperor looks south because it’s where the wind comes from, it’s a good direction. North is not very good but you are in a position of subjection to the emperor, so you look up to him.” says Brotton. Given that each culture has a very different idea of who, or what, they should look up to it’s perhaps not surprising that there is very little consistency in which way early maps pointed. In ancient Egyptian times the top of the world was east the position of Sunrise. Early Islamic maps favored south at the top because of the early Muslim cultures were north of the Mecca. So they imagined looking up (south) towards it. Christian maps from the same era ( called Mappa Mundi) put east at the top, towards the Garden of Eden and with Jerusalem in the center. So when did everyone get together and decide that north was the top? It’s tempting to put it down to European explorers like Christopher Columbus and Ferianand Megellan, who where navigating by the North Star. ” Columbus says he going to towards paradise, so his mentality is form medieval mappa mundi.”We’ve got to remember, adds Erotton, that at the time,”no one knows what they are doing and where they are going.” 

 

Q. 1 Which one of the following best describes what the passage is trying to do?

A. It questions an explanation about how maps are designed

B. It corrects a misconception about the way maps are designed

C. It critiques a methodology used to create maps

D. It explores some myths about maps

 

Q. 2 Early maps did NOT put north at the top for all the following reasons EXCEPT

A. North was the source of darkness

B. South was favored by some emperors

C. East and South were more important for all religious reasons for some civilizations.

D. East was considered by some civilizations to be a more positive direction.

 

Q. 3 According to the passage, early Chinese maps placed north at the top because

A. the Chinese invented the compass and were aware of magnetic north.

B. they wanted to show respect to the emperor.

C. the Chinese emperor appreciated the winds from the South

D. north was considered the most desirable direction.

 

Q. 4 It can be inferred from the passage that European explorers like Columbus and Megellan

A. set the precedent for north-up maps

B. navigated by the compass

C. used an eastward orientation for religious reasons

D. navigated with the help of early maps

 

Q. 5 Which one of the following about the northern orientation of modern maps is asserted in the passage?

A. The biggest contributory factor was the understanding of magnetic north

B. The biggest contributory factor was the role of European explorers

C. The biggest contributory factor was the influence of Chinese maps

D. The biggest contributory factor is not stated in the passage

 

Q. 6 The role of natural phenomena is influencing map-making conventions is seen most clearly in

A. early Egyptian maps

B. early Islamic maps

C. early Chinese maps

D. early Christian maps

 

Questions: 7 – 12

I used a smartphone GPS to find my way through the cobble stoned maze of Geneva’s Old Town, in search of a handmade machine that changed the world more than any other invention. Near a 13th-century cathedral in this Swiss city on the shores of a lovely lake. I found what I was looking for : a Gutenberg printing press was the Internet of its day – at least as influential as the iPhone,” said Gabriel de Montmollin, the director of the Museum of the Reformation, toying with the replica of Johann Gutenberg’s great invention. [Before the invention of the printing press] it used to take four monks… up to a year to produce a single book. With the advance in movable type in 15th century Europe one press could crank out 3,000 pages a day. Before long, average people could travel to places that used to be unknown to them – with maps! Medical information passed more freely and quickly, diminishing the sway of quacks… The printing press offered prospects that tyrants would never be able to kill a book or suppress an idea. Gutenberg’s brainchild broke the monopoly that clerics had on scripture. And later stirred by pamphlets from a version of that same press, the American colonies rose up against a king and a gave a birth to a nation. So, a question in the summer of this 10th anniversary of the iPhone has the device that is perhaps the most revolutionary of all time given us a single magnificent idea? Nearly every advancement of the written word through new technology has also advanced humankind. The iPhone has made us more narcissistic – here’s more of me doing cool stuff! – and it unleashed an army of awful trolls. We no longer have the patience to sit through a baseball game without that reach to the pocket. And one more casualty of Apple selling more than a billion phones in a decade’s time, daydreaming has become a lost art. For all of that, I’m still waiting to see if the iPhone can do what the printing press did for religion and democracy… the Geneva museum makes a strong case that the printing press opened more minds than anything else… it’s hard to imagine the French or American revolutions without those enlightened voices in print…. Not long after Steve jobs introduced his iPhone, he said the bound book was probably headed for history’s attic. Not so fast. After a period of rapid growth in e-books, something closer to the medium for Chaucer’s volumes has made a great comeback. The hope of the iPhone , and the internet in general, was that it would free people in closed societies. But the failure of the Arab Spring, and the continued Suppression of ideas in North Korea, China and Iran, has not borne that out.. The iPhone is still Young. It has certainly been ” one of the most important, world-changing and successful products in history,” as Apple C.E.O. Tim Cook said. But I’m not sure if he would changed for the better with the iPhone – as it did with the printing press – or merely changed.

 

Q. 7 The printing press has been likened to the Internet for which one of the following reasons?

A. It enabled rapid access to new information and the sharing of new ideas

B. It represented new and revolutionary technology compared to the past

C. It encouraged reading among people by giving them access to thousands of books

D. It gave people access to pamphlets and literature in several languages

 

Q. 8 According to the passage, the invention of the printing press did all of the following EXCEPT

A. promoted the spread of enlightened political views across countries

B. gave people direct access to authentic medical information and religious texts

C. shortened the time taken to produce books and pamphlets

D. enabled people to perform various tasks simultaneously

 

Q. 9 Steve Jobs predicted which one of the following with the introduction of the iPhone?

A. People would switch from reading on the internet to reading on their iPhones

B. People would lose interest in historical and traditional classics

C. Reading printed books would become a thing of the past

D. The production of e-books would eventually fall

 

Q. 10 “I’m still waiting to see if the iPhone can do what the printing press did for religion and democracy.” The author uses which one of the following to indicate his uncertainty?

A. The rise of religious groups in many parts of the world

B. The expansion in trolling and narcissism among the users of the internet

C. The continued suppression of free speech in closed societies.

D. The decline in reading habits among those who use the device.

 

Q. 11 The author attributes the French and American revolutions to the invention of the printing press because

A. maps enabled large numbers of Europeans to travel and settle in the American continent

B. the rapid spread of information exposed people to new ideas on freedom and democracy

C. It encouraged religious freedom and among the people by destroying the monoply of religious leaders on the scriptures

D. It made a available revolutionary strategies and opinions to the people

 

Q. 12 The main conclusion of the passage is that the new technology has 

A. some advantages but these are outweighed by its disadvantages

B. so far not proved as successful as the printing press in opening people’s minds

C. been disappointing because it has changed society too rapidly

D. been more wasteful than the printing press because people spend more time daydreaming or surfing

 

Questions: 13 – 18

This year alone, more than 8,600 stores could close, according to industry estimates, many of them the brand name anchor outlets that real estate developers once stumbled over themselves to court. Already there have been 5,300 retail closings this year… Sears Holdings — which owns Kmart — said in March that there’s “substantial doubt” it can stay in business altogether, and will close 300 stores this year. So far this year, nine national retail chains have filed for bankruptcy. Local jobs are a major casualty of what analysts are calling, with only a hint of hyperbole, the retail apocalypse. Since 2002, department stores have lost 448,000 jobs, a 25% decline, while the number of store closures this year in on pace to surpass the worst depths of the Great Recession. The growth of online retailers, meanwhile, has failed to offset those losses, with the ecommerce sector adding just 178,000 jobs over the past 15 years Some of those jobs can be found in the massive distribution centers Amazon has opened across the country, often not too far from malls the company helped shutter. But those are workplaces, not gathering places. And in the 61 years since the first enclosed one opened in Suburban Minneapolis, the shopping mall was home of first jobs and blind dates, the place for family photos and ear piercings, where goths and grandmothers could somehow walk through the same doors and find something they all liked. Sure the food was lousy for you and the oceans of parking lots encouraged car-heavy development, something now scorned by contemporary planners. Buts for better or worse, the mall has been America’s public square for the last 60 years. So what happens when it disappears? Think of your mall or think of the one you went to as a kid. Think of the perfume clouds in the department stores. The cinnamon wafting from the food court. As far back as Ancient Greece, societies have congregated around a central marketplace. In medieval Europe, they were outside cathedrals. For Half of the 20th century and almost 20 years into the new one, much of America has found their agora on the terrazzo between Orange Julius and Sbarro, Waldenbooks and the Gap, Sunglass Hut and Hot Topic. That mall was an ecosystem unto itself, a combination of community and commercialism peddling everything you needed and everything you didn’t Magic Eye posters, wind catchers, Air Jordans… A growing number of Americans, however, don’t see the need to go to any Macy’s at all. Our digital lives are friction less and ruthlessly efficient, with retail and romance available at a click. Malls were designed for leisure, abundance , ambling. Today, much of that time has been given over to busier lives and second jobs and apps that let you swipe right instead of haunt the food court. Mails, says Harvard business professor Leonard Schlesinger, “were built for patterns of social interaction that increasingly don’t exist.

 

Q. 13 The central idea of this passage is that

A. the closure of malls has affected the economic and social life of middle-class America

B. the advantages of malls outweigh their disadvantages

C. malls used to perform a social function that has been lost

D. malls are closing down because people have found alternate ways to shop

 

Q. 14 Why does the author say in paragraph 2, ‘the massive distribution centers Amazon has opened across the country, often not too far from malls the company helped shutter’?

A. To highlight the irony of the situation

B. To indicate that malls and distribution centres are located in the same area

C. To show that Amazon is helping certain brands go online

D. To indicate that the shopping habits of the American middle class have changed

 

Q. 15 The author calls the mall an ecosystem unto itself because

A. people of all ages and from all walks of life went there

B. people could shop as well as eat in one place

C. It was a commercial space as well as a gathering place

D. it sold things that were needed as well as those that were not

 

Q. 16 in paragraph 1, the phrase “real estate developers once stumbled over themselves to court” suggests that they

A. took brand-name anchor to court

B. no longer pursue brand-name anchor outlets

C. collaborated with one another to get brand-name anchor outlets

D. were eager to get brand-name anchor outlets to set up shop in their mall

 

Q. 17 Why does the author say that the mall has been America’s public square

A. Malls did not bar anybody from entering the space

B. Malls were a great place to shop for a huge section of the middle calss

C. Malls were a hangout place where families grew close to each other

D. Malls were a great place for everyone to gather and interact

 

Q. 18 The author describes “Perfume clouds in the department stores” in order to

A. evoke memories by painting a picture of malls

B. describe the smells and sights of malls

C. emphasise that all brands were available under one roof

D. show that malls smelt good because of the various stores and food court

 

Questions: 19 – 21

Scientists have long recognized the incredible diversity within species . But they thought it reflected evolutionary changes that unfolded imperceptibility, over millions of years. That diverges between populations within a species was enforced, according to Ernst Mayr, the great evolutionary biologist of the 1940s, when a population was separated from the rest of the species by a mountain range or a desert. Without the separation, gene flow was relentless. But as the separation persisted, the isolated population grew apart and speciation occurred. In the mid – 1960s, the biologist Paul Ehrlich – author of The Population Bomb(1968) – and his Stanford University Colleague Peter Raven challenged Mayr’s ideas about speciation. They had studied checkerspot butterflies living in the Jasper Ridge Biological Preserve in California, and it soon became clear that they were not examining a single population. Through years of capturing, marking and then recapturing the butterflies, they were able to prove that within the population spread over just 50 acres of suitable checkerspot habitat, there were three groups that rarely interacted despite their very close proximity. Among other ideas, Ehrlich and Raven argued in a now classic paper from 1969 that gene flow was not as predictable and ubiquitous as Mayr and his cohort maintained, and thus evolutionary divergence between neighboring groups in a population was probably common. They also asserted that isolation and gene flow were less important to evolutionary divergence than natural selection ( when factors such as mate choice, weather, disease or predation better- adapted individuals to survive and pass on their successful genetic traits). For example, Ehrlich and Raven suggested that, without the force of natural selection, an isolated population would remain unchanged and that, in other scenarios, natural selection could be strong enough to overpower gene flow.

 

Q. 19 Which of the following best sums up Ehrlich and Raven’s argument in their classic 1969 paper?

A. Ernst Mayr was wrong in identifying physical separation as the cause of species diversity

B. Checkerspot butterflies in the 50-acre Jasper Ridge Preserve formed three groups that rarely interacted with each other

C. While a factor, isolation was not as important to speciation as natural selection

D. Gene flow is less common and more erratic than Mayr and his colleagues claimed.

 

Q. 20 All of the following statements are true according to the passage EXCEPT

A. Gene flow contributes to evolutionary divergence

B. The Population Bomb questioned dominant ideas about species diversity

C. Evolutionary changes unfold imperceptibly over time

D. Checkerspot butterflies are known to exhibit speciation while living in close proximity

 

Q. 21 The author discusses Mayr, Ehrlich and Raven to demonstrate that

A. evolution is a sensitive and controversial topic

B. Ehrlich and Raven’s ideas about evolutionary divergence are widely accepted by

scientists.

C. the causes of speciation are debated by scientists

D. checkerspot butterflies offer the best example of Ehrlich and Raven’s ideas about

speciation

Questions: 22 – 24

Do sports mega events like the summer Olympic Games benefit the host city economically? It depends, but the prospects are less than rosy. The trick is converting… several billion dollars in operating costs during the 17-day fiesta of the Games into a basis for long-term economic returns. These days, the Summer Olympic Games themselves generate total revenue of $4 billion to $5 billion, the lion’s share of this goes to the International Olympics Committee. Any economic benefit would have to flow from the value of the Games as an advertisement for the city, the new transportation and communications infrastructure that were created for the Games, or the ongoing use of the new facilities. Evidence suggests that the advertising effect is far from certain. The infrastructure benefit depends on the initial condition of the city and the effectiveness of the planning. The facilities benefit is dubious at best for buildings such as velodromes or natatoriums and problematic for 100,000-seat Olympic stadiums. The latter require a conversion plan for future use. Hosting the summer games generally requires 30-plus sports venues and dozens of

training centers. Today, the Bird’s Nest in Beijing sits virtually empty, while the Olympic Stadium in Sydney costs some $30 million a year to operate. Part of the problem is that Olympics planning takes place in a frenzied and time-pressured atmosphere of intense competition with the other prospective host cities – not optimal conditions for contemplating the future shape of an urban landscape. Another part of the problem is that urban land is generally scarce and growing scarcer. Even if they have future use, are they the best use of precious urban real estate? Further, cities must consider the human cost. Residential areas often razed and citizens relocated. Life is made more hectic and congested. There are, after all, other productive uses that can be made of vanishing fiscal resources.

Q. 22 The central point in the first paragraph is that the economic benefits of the Olympic Games

A. are shared equally among the three organising committees

B. accrue mostly through revenue from advertising and ticket sales

C. accrue to host cities, if at all, only in the long term

D. are usually eroded by expenditure incurred by the host city

Q. 23 Sports facilities built for the Olympics are not fully utilised after the Games are over because

A. their scale and the costs of operating them are large

B. their location away from the city centre usually limits easy access

C. the authorities do not adapt them to local conditions

D. they become outdated having being built with little planning and under time pressure

Q. 24 The author feels that the Games place a burden on the host city for all of the following reasons EXCEPT that

A. they divert scarce urban land from more production uses

B. they involve the demolition of residential structures to accommodate sports facilities and infrastructure

C. the finances used to fund the Games could be better used for other purposes

D. the influx of visitors during the Games places a huge strain on the urban infrastructure

Question 25

To me, a “classic” means precisely the opposite of what my predecessors understood: a work is classical by reason of its resistance to contemporary and supposed universality, by reason of its capacity to indicate human particularity and difference in that past epoch. The classic is not what tells me about shared humanity – or, more truthfully put, what lets me recognize myself as already present in the past, what nourishes in me the illusion that everything has been like me and has existed only to prepare the way for me. Instead, the classic is what gives access to radically different dorms of human consciousness for any given generation of readers, and thereby expands for them the range of possibilities of what it means to be a human being. 

Q. 25 The passage given is followed by four summaries. Choose the option that best captures the author’s position:

A. A classic is able to focus on the contemporary human condition and a unified

experience of human consciouness

B. A classical work seeks to resist particularity and temporal difference even as it focuses on a common humanity

C. A classic is a work exploring the new, going beyond the universal, the contemporary, and the notion of a unified human consciousness

D. A classic is a work that provides access to a universal experience of the human race as opposed to radically different forms of human consciousness

Question 26

A translator of literary works needs a secure hold upon the two languages involved, supported by a good measure of familiarity with the two cultures. For an Indian translating works in an Indian language into English, finding satisfactory equivalents in a generalized western culture of practices and symbols in the original would be less difficult than gaining fluent control of contemporary English. When a westerner works on texts in Indian languages the interpolation of cultural elements will be the major challenge, rather than control over the grammar and essential vocabulary of the language concerned. It is much easier to remedy lapses in language in a text translated into English, than flaws of content Since it is easier for an Indian to learn the English language than it is for a Briton or American to comprehend Indian culture, translations of Indian texts is better left to Indians.

Q. 26 The given passage is followed by four summaries. Choose the option that best captures the author’s position.

A. While translating, the Indian and the westerner face the same challenges but they

have different skill profiles and the former has the advantage

B. As preserving cultural meanings is the essence of literary translation Indian’s

knowledge of the local culture outweighs the initial disadvantage of lower fluency in

English 

C. Indian translators should translate Indian texts into English as their work is less likely to pose cultural problems which are harder to address than the quality of language

D. Westerners might be good at gaining reasonable fluency in new languages, but as

understanding the culture reflected in literature is crucial, Indians remain better placed

Question 27

For each of the past three years, temperatures have hit peaks not seen since the birth of meteorology, and probably not for more than 100,000 years. The amount of CO2 in the air is at its highest level in 4 million years. This does not cause storms like Harvey – there have always been storms and hurricanes along the Gulf of Mexico – but it makes them wetter and more powerful. As the seas warm, they evaporate more easily and provide energy to storm fronts. As the air above them warms, it holds more water vapour. For every half a degree Celsius in warming, there is about 3% increase in atmospheric moisture content. Scientists call this the Clausius-Clapeyron equation. The storm surge was greater because sea levels have risen 20 cm as a result of more than 100 years of human related global warming which has melted glaciers and thermally expanded the volume of seawater.

Q. 27 The given passage is followed by four summaries. Choose the option that best captures the author’s position.

A. The storm Harvey is one of the regular, annual ones from the Gulf of Mexico, global warming and Harvey are unrelated phenomena.

B. Global warming does not breed storms but makes them destructive, the Clausius-

Clapeyron equation, though it predicts potential increase in atmospheric moisture

content, cannot predict the scale of damage storms might wreck.

C. Global warming melts glaciers, resulting in seawater volume expansion, the enables more water vapour to fill the air above faster. Thus, modern storms contain more destructive energy.

D. It is naive to think that rising sea levels and the force of tropical storms are unrelated; Harvey was destructive as global warming has armed it with more moisture content, but this may not be true of all storms.

Question 28

1. The process of handing down implies not a passive transfer, but some

contestation in defining what exactly is to be handed down.

2. Wherever Western scholars have worked on the Indian past, the selection is

even more apparent and the inventing of a tradition much more recognizable.

3. Every generation selects what it requires from the past and makes its innovations some more than others. 

4. it is now a truism to say that traditions are not handed down unchanged, but are invented.

5. just as life has death as its opposite, so is tradition by default the opposite of innovation.

Q. 28 The five sentences( labelled 1, 2, 3, 4, 5 ) given in this question when properly sequenced, form a coherent paragraph. Each sentence is labelled with a number. Decide on the proper order for the sentence of five numbers as you answer.

A. 54132

B. 21345

C. 31524

D. 51423

Question 29

1.Scientists have for the first time managed to edit genes in a human embryo to repair a genetic mutation fuelling hopes that such procedures may one day be available outside laboratory conditions.

2. The cardiac disease causes sudden death in otherwise healthy young athletes and affects about one in 500 people overall.

3. Correcting the mutation in the gene would not only ensure that the child is healthy but also prevents transmission of the mutation to future generations.

4. It is caused by a mutation in a particular gene and a child will suffer from the condition even if it inherits only one copy of the mutated gene.

5. In results announced in the Nature this week, scientists fixed a mutation that thickens the heart muscle, a condition called hypertrophic cardiomyopathy.

Q. 29 The five sentences ( labelled 1,2,3,4,5 ) given this question, when properly sequenced, form a coherent paragraph. Each sentence is labelled with a number. Decide on the proper order for the sentences and key in this sequence of five numbers as your answer. 

A. 42513

B. 15243

C. 25341

D. 15342

Question 30

1. The study suggests that the disease did not spread with such intensity, but it may have driven human migrations across Europe and Asia.

2. The oldest sample came from an individual who lived in southeast Russia about 5,000 years ago.

3. The ages of the skeletons correspond to a time of mass exodus from today’s Russia and Ukraine into western Europe and central Asia, suggesting that a pandemic could have driven those migrations.

4. In the analysis of fragments of DNA from 101 Bronze Age skeletons for sequences of Yersinia pestis, the bacterium that causes the disease, seven tested positive.

5. DNA from Bronze Age human skeletons indicate that the black plague could have emerged as early as 3,000 BCE, long before the epidemic that swept through Europe in the mid-1300s.

Q. 30 The five sentences ( labeled 1,2,3,4,5 ) given this question, when properly sequenced, form a coherent paragraph. Each sentence is labelled with a number. Decide on the proper order for the sentences and key in this sequence of five numbers as your answer. 

A. 12345

B. 15234

C. 54123

D. 32415

Question 31 

1.) This visual turn in social media has merely accentuated this announcing instinct of ours, enabling us with easy-to-create, easy-to-share and easy-toconsume platforms, gadgets and apps.

2.) There is absolutely nothing new about us framing the vision of who we are or what we want, visually or otherwise, in our Facebook page, for example.

3.) Turning the pages of most family albums, which belong to a period well before the digital dissemination of self-created and self-curated moments and images, would reconfirm the basic instinct of documenting our presence in a particular space, on a significant occasion, with others who matter. 

4.) We are empowered to book our faces and act as celebrities within the confinement of our respective friend lists, and communicate our activities, companionship and locations with minimal clicks and touches.

5.) What is unprecedented is not the desire to put out newsfeeds related to the self, but the ease with which this broadcast operation can now be executed, often provoking (un)anticipated responses from beyond’s one’s immediate location.

Q. 31 The five sentences ( labled 1,2,3,4,5 ) given this question, when properly sequenced, form a coherent paragraph. Each sentence is labelled with a number. Decide on the proper order for the sentences and key in this sequence of five numbers as your answer.

A. 32145

B. 41523

C. 51432

D. 12543

Question 32

1.) People who study children’s language spend a lot of time watching how babies react to the speech they hear around them.

2.) They make films of adults and babies interacting, and examine them very carefully to see whether the babies show any signs of understanding what the adults say.

3.) They believe that babies begin to react to language from the very moment they are born.

4.) Sometimes the signs are very subtle – slight movements of the baby’s eyes or the head or the hands.

5. ) You’d never notice them if you were sitting with the child, but by watching a recording over and over, you can spot them.

Q. 32 Five sentences related to a topic are given below. Four of them can be put together to form a meaningful and coherent short paragraph. Identify the odd one out.

A. 3

B. 2

C. 4

D. 5

Q. 33 Five sentences related to a topic are given below. Four of them can be put together to form a meaningful and coherent short paragraph. Identify the odd one out. 

A. Neuroscientists have just begun studying exercise’s impact within the brain cells – on the genes themselves.

B. Even there, in the roots of our biology, they’ve found signs of the body’s influence on the mind.

C. It turns out that moving our muscles produce proteins that travel through the

bloodstream and into the brain, where they play vital roles in the mechanisms of our

highest thought process.

D. In today’s technology-driven, plasma-screened-in world, it’s easy to forget that we are born movers – animals, in fact- because we’ve engineered movement right out of our lives.

E. Its only in the past few years that nueroscientists have begun to describe these factors and how they work , and each new discovery adds awe-inspiring depth to the

picture.

Q. 34 Five sentences related to a topic are given below. Four of them can be put together to form a meaningful and coherent short paragraph. Identify the odd one out. 

1. The water that made up ancient lakes and perhaps an ocean was lost.

2. Particles from the Sun collided with molecules in the atmosphere, knocking them into space or giving them an electric charge that caused them to be swept away by the solar wind.

3. Most of the planet’s remaining water is now frozen or buried, but clues over the past decade suggested that some liquid water, a presumed necessity for life, might survive in underground aquifers.

4. Data from NASA’s MAVEN orbiter show that solar storms stripped away most of Mars’s once-thick atmosphere.

5. A recent study reveals how Mars lost much of its early water, while another indicates that some liquid water remains.

A. 1

B. 2

C. 4

D. 5

Questions: 35 – 38

Healthy Bites is a fast food joint serving three items: burgers, fries and ice cream. It has two employees, Anish and Bani who prepare the items ordered by the clients. Preparation time is 10 minutes for a burger and 2 minutes for an order of ice cream. An employee can prepare only one of these items at a time. The fries are prepared in an automatic fryer which can prepare up to 3 portions of fries at a time, and takes 5 minutes irrespective of the number of portions. The fryer does not need an employee to constantly attend to it, and we can ignore the time taken by an employee to start and stop the fryer; thus, an employee can be engaged in preparing other items while the frying is on. However fries cannot be prepared in anticipation of future orders.

Healthy Bites wishes to serve the orders as early as possible. The individual items in any order are served as and when ready; however, the order is considered to be completely served only when all the items of the order are served. The given table shows the orders of three clients and the times at which they placed their orders.

Q. 35 Assume that only one client’s order can be processed at any given point of time. So, Anish or Bani cannot start preparing a new order while a previous order is being prepared. At what time is the order placed by Client 1 completely served?

A. 10:17

B. 10:10

C. 10:15

D. 10:20

Q. 36 Assume that only one client’s order can be processed at any given point of time. So, Anish or Bani cannot start preparing a new order while a previous order is being prepared. At what time is the order placed by Client 3 completely served?

A. 10:35

B. 10:22

C. 10:25

D. 10:17

Q. 37 Suppose the employees are allowed to process multiple orders at a time, but the preferences would be to finish orders of clients who placed their orders earlier. At what time is the order placed by Client 2 completely served?

A. 10:10

B. 10:12

C. 10:15

D. 10:17

Q. 38 Suppose the employees are allowed to process multiple orders at a time, but the preferences would be to finish orders of clients who places their orders earlier. Also assume that the fourth client came in only at 10:35. Between 10:00 and 10:50, for how many minutes is exactly one of the employees idle?

A. 7

B. 10

C. 15

D. 23

Questions: 39 – 42

A study to loo  at the early learning of rural kids was carried out in a number of villages spanning three states, chosen from the North East (NE), the West (W) and the South (S). 50 four-year old kids each were sampled from each of the 150 villages from NE, 250 villages from W and 200 villages from S. It was found that of the 30000 surveyed kids 55% studied in primary schools run by government (G), 37% in private schools (P) while the remaining 8% did not go to school (O). The kids surveyed were further divided into two groups based on whether their mothers dropped out of schools before completing primary education or not. The given table shows the number of kids in different types of schools for mothers who dropped out of school before completing primary education. It is also known that:

1. In S, 60% of the surveyed kids were in G. Moreover, in S, all surveyed kids whose mothers had completed primary education were in school.

2. In NE, among the O kids, 50% had mothers who had dropped out before completing primary education.

3. The number of kids in G in NE was the same as the number of kids in G in W.

G P O Total
NE 4200  500  300 5000
W 4200 1900 1200 7300
S 5100 300 300 5700
Total 13500 2700 1800 18000

 

Q. 39 What percentage of kids from S were studying in P?

A. 37%

B. 6%

C. 79%

D. 56%

Q. 40 Among the kids in W whose mothers had completely primary education, how many were not in school?

A. 300

B. 1200

C. 1050

D. 1500

Q. 41 In a follow up survey of the same kids two years later, it was found that all the kids were now in school. Of the kids who were not in school earlier, in one region, 25% were in G now, whereas the rest were enrolled in P; in the second region, all such kids were in G now, while in the third region, 50% of such kids had now joined G while the rest had joined P. As a result, in all three regions put together, 50% of the kids who were earlier out of school had joined G. It was also seen that no surveyed kid had changed schools. What number of the surveyed kids now ere in G in W? 

A. 6000

B. 5250

C. 6750

D. 6300

Q. 42  In a follow up survey of the same kids two years later, it was found that all the kids were now in school. Of the kids who were not in school earlier, in one region, 25% were in G now, whereas the rest were enrolled in P; in the second region, all such kids were in G now while in the third region, 50% of such kids had now joined G while the rest had joined P. As a result, in all three regions put together, 50% of the kids who were earlier out of school had joined G. It was also seen that no surveyed kid had changed schools. What percentage of the surveyed kids in S, whose mothers had dropped out before completing promary education, were in G now? 

A. 94.7%

B. 89.5%

C. 93.4%

D. Cannot be determined from the given information

Questions: 43 – 46

Applicants for the doctoral programmes of Ambi Institute of Engineering (AIE) and Bambi Institute of Engineering (BIE) have to appear for a Common Entrance Test (CET). The test has three sections: Physics (P), Chemistry (C), and Maths (M). Among those appearing for CET, those at or above the 80th percentile in at least two sections, and at or above the 90th percentile overall, are selected for Advance Entrance Test (AET) conducted by AIE. AET is used by AIE for final selection. For the 200 candidates who are at or above the 90th percentile overall based on CET, the following are known about their performance in CET:-

1. No one is below the 80th percentile in all 3 sections.

2. 150 are at or above the 80th percentile in exactly two sections.

3. The number of candidates at or above the 80th percentile only in P is is the

same as the number of candidates at or above the 80th percentile only in C. The same is the number of candidates at or above the 80th percentile only in M.

4. Number of candidates below 80th percentile in P : Number of candidates below 80th percentile in C : Number of candidates below 80th percentile in M = 4 : 2 : 1 BIE uses a different process for selection. If any candidate is appearing in the AET by AIE, BIE considers their AET score for final selection provided the candidate is at or above the 80th percentile at P. Any other candidate at or above the 80th percentile in P in CET, but who is not eligible for the AET, is required to appear in a separate test to be conducted by BIE for being considered for final selection. Altogether, there are 400 candidates this year who are at or above the 80th percentile in P. 

Q. 43 What best can be concluded about the number of candidates sitting for the separate test for BIE who were at or above the 90th percentile overall in CET?

A. 3 or 10

B. 10

C. 5

D. 7 or 10

Q. 44 If the number of candidates who are at or above 90th percentile overall and also at or above the 80th percentile in all three sections in CET is actually multiple of 5, what is the number of candidates who are at or above the 90th percentile overall at or above the 80th percentile in both P and M in CET?

A. 50

B. 60

C. 30

D. 49

Q. 45 If the number of candidates who are at or above the 90th percentile overall and also at or above the 80th percentile in all three sections in CET is actually multiple of 5, then how many candidates were shortlisted for the AET or AIE?

A. 180

B. 175

C. 170

D. 165

Q. 46 If the number of candidates who are at or above the 90th percentile overall and also at or above the 80th percentile in P in CET, is more than 100, how many candidates had to sit for the separate test for EIE?

A. 299

B. 310

C. 321

D. 330

Questions: 47 – 50

Simple Happiness Index (SHI) of a country is computed on the basis of three parameters: social support (S), freedom to life choices (F) and corruption perception (C). Each of these three parameters is measured on a scale of 0 to 8 (integers only). A country is then categorised based on the total score obtained by summing the scores of all the three parameters, as shown in the given table. (From the bar graph)

Further, the following are known:

1. Amda and Calla jointly have the lowest total score, 7, with identical scores in all

the three parameters.

2. Zooma has a total score of 17.

3. All the three countries, which are categorised as happy, have the highest score

in exactly one parameter.

 

Total Score 0-4 5-8 9-13 14-19 20-24
Category  Very Unhappy Unhappy Neutral Happy Very Happy

Following diagram depicts the frequency distribution of the scores in S, F and C of 10 countries – Amda, Benga, Calla, Delma, Eppa, Varsa, Wanna, Xanda, Yanga and Zooma:

Q. 47 What is Amda’s score in F?

A. 2

B. 1

C. 4

D. 3

Q. 48 What is Zooma’s score in S?

A. 6

B. 1

C. 2

D. 3

Q. 49 Benga and Delma, two countries categorized as happy, are tied with the same total score. What is the maximum score they can have?

A. 14

B. 15

C. 16

D. 17

Q. 50 If Benga scores 16 and Delma scores 15, then what is the maximum number of countries with a score of 13?

A. 0

B. 1

C. 2

D. 3

Questions: 51 – 54

There are 21 employees working in a division, out of whom 10 are special-skilled employees (SE) and the remaining are regular-skilled employees (RE). During the next five months, the division has to complete five projects every month. Out of the 25 projects, 5 projects are “challenging”, while the remaining ones are “standard”. Each of the challenging projects has to be completed in different months. Every month, five teams – T1, T2, T3, T4 and T5, work on one project each. T1, T2, T3, T4 and T5 are allotted the challenging project in the first, second, third, fourth and fifth month, respectively. The team assigned the challenging project has one more employee than the rest. In the first month, T1 has one more SE than T2, T2 has one more SE than T3, T3 has one more SE than T4, and T4 has one more SE than T5. Between two successive months, the composition of the teams changes as follows:-

a) The team allotted the challenging project, gets two SE from the team which was allotted the challenging project in the previous month. In exchange, one RE is shifted from the former team to the latter team.

b) After the above exchange, if T1 has any SE and T5 has any RE, then one SE is shifted from T1 to T5, and one RE is shifted from T5 to T1. Also, if T2 has any SE and T4 has any RE, then one SE is shifted from T2 to T4, and one RE is shifted from T4 to T2.

Each standard project has a total of 100 credit points, while each challenging project has 200 credit points. The credit points are equally shared between the employees included in that team.

Q. 51 The number of times in which the composition of team T2 and the number of times in which composition of team T4 remained unchanged in two successive months are: 

A. (2,1)

B. (1,0)

C. (0,0)

D. (1,1)

Q. 52 The number of SE in T1 and T5 for the projects in the third month are, respectively 

A. (0,2)

B. (0,3)

C. (1,2)

D. (1,3)

Q. 53 Which of the following CANNOT be the total credit points earned by any employee from the projects?

A. 140

B. 150

C. 170

D. 200

Q. 54 One of the employees named Aneek scored 185 points. Which of the following CANNOT be true?

A. Aneek worked only in teams T1,T2,T3 and T4

B. Aneek worked only in teams T1,T2,T5 and T4

C. Aneek worked only in teams T5,T2,T3 and T4

D. Aneek worked only in teams T1,T5,T3 and T4

Questions: 55 – 58

In a square layout of size 5 m x 5 m, 25 equal-sized square platforms of different heights are built. The heights (in metres) of individual platforms are as shown in the figure. Individuals (all of same height) are seated on these platforms. We say that individual A can reach an individual B if all the three following conditions are met:-

i) A and B are in the same row or column

ii) A is at a lower height than B

iii) If there is/are any individual(s) between A and B, such individual(s) must be at a height lower than that of A

Thus in the given table, consider the individual seated at height 8 on 3rd row and 2nd column. He can be reached by four individuals. He can be reached by the individual on his left at height 7, by the two individuals on his right at heights of 4 and 6 and by the individual above at height 5. Rows in the layout are numbered from top to bottom and columns are numbered from left to right.

Q. 55 How many individuals in this layout can be reached by just one individual?

A. 3

B. 5

C. 7

D. 8

Q. 56 Which of the following is true for any individual at a platform of height 1 m in this layout? 

A. They can be reached by all the individuals in their own row and column

B. They can be reached by at least 4 individuals

C. They can be reached by at least one individual

D. They cannot be reached by anyone

Q. 57 We can find two individuals who cannot be reached by anyone in

A. the last row

B. the fourth row

C. the fourth column

D. the middle column

Q. 58 Which of the following statements is true about this layout?

A. Each row has an individual who can be reached by 5 or more individuals

B. Each row has an individual who cannot be reached by anyone

C. Each row has at least two individuals who can be reached by an equal number of

individuals

D. All individuals at the height of 9 m can be reached by at least 5 individuals

Questions: 59 – 62

A new airlines company is planning to start operations in a country. The company has identified ten different cities which they plan to connect through their network to start with. The flight duration between any pair of cities will be less than one hour. To start operations, the company has to decide on a daily schedule.

The underlying principle that they are working on is the following:-

Any person staying in any of these 10 cities should be able to make a trip to any other city in the morning and should be able to return by the evening of the same day.

Q. 59 If the underlying principle is to be satisfied in such a way that the journey between any two cities can be performed using only direct ( non – stop) flights, then the minimum number of direct flights to be scheduled is:

A. 45

B. 90

C. 180

D. 135

Q. 60 Suppose three of the ten cities are to be developed as hubs. A hub is a city which is connected with every other city by direct flights each way, both in the morning as well as in the evening. The only direct flights which will be scheduled are originating and/or terminating in one of the hubs. Then the minimum number of direct flights that need to be scheduled so that the underlying principle of the airline to serve all the ten cities is met without visiting more than one hub during one trip is:

A. 54

B. 120

C. 96

D. 60

Q. 61 Suppose the 10 cities are divided into 4 distinct groups G1, G2, G3, G4 having 3, 3, 2 and 2 cities respectively and that G1 consists of cities named A, B and C. Further, suppose that direct flights are allowed only between two cities satisfying one of the following: 

1.) Both cities are in G1

2.) Between A and any city in G2

3.) Between B and any city in G3

4.) Between C and any city in G4

Then the minimum no. of direct flights that satisfies the underlying principle of the airline is:

A. 40

B. 30

C. 36

D. 46

Q. 62 Suppose the 10 cities are divided into 4 distinct groups G1, G2, G3, G4 having 3, 3, 2 and 2 cities respectively and that G1 consists of cities named A, B and C. Further, suppose that direct flights are allowed only between two cities satisfying one of the following:

1.) Both cities are in G1

2.) Between A and any city in G2

3.)Between B and any city in G3

4.)Between C and any city in G4 .

However, due to operational difficulties at A, it was later decided that the only flights that would operate at A would be those to and from B. Cities in G2 would have to be assigned to G3 or G4. What would be the maximum reduction in the number of direct flights as compared to the situation before the operational difficulties arose?

A. 1

B. 2

C. 3

D. 4

Questions: 63 – 66

Four cars need to travel from Akala (A) to Bakala (B). Two routes are available, one via Mamur (M) and the other via Nanur (N). The roads from A to M, and from N to B, are both short and narrow. In each case, one car takes 6 minutes to cover the distance, and each additional car increases the travel time per car by 3 minutes because of congestion. (For example, if two cars drive from A to M, each car takes 9 minutes.) On the road from A to N, one car takes 20 minutes, and each additional car increases the travel time per car by 1 minute. On the road from M to B, one car takes 20 minutes, and each additional car increases the travel time per car by 0.9 minute. The police department orders each car to take a particular route in such a manner that it is not possible for any car to reduce its travel time by not following the order, while the other cars are following the order.

Q. 63 How many cars would be asked to take the route A-N-B, that is Akala-Nanur-Bakula route, by the police department?

A. 1

B. 2

C. 3

D. 4

Q. 64 If all the cars follow the police order, what is the difference in travel time (in minutes) between a car which takes the route A-N-B and a car that takes the route A-M-B? 

A. 1

B. 0.1

C. 0.2

D. 09

Q. 65 A new one-way road is built from M to N. Possible routes for each car from A to B: A-M-B, AN- B and A-M-N-B. On the road from M to N, one car takes 7 mins and each additional car increases the travel time per car by 1 minute. Assume that any car taking the A-M-N-B routes travels the A-M portion at the same time as the other cars taking the A-M-B route, and the N-B portion at the same time as other cats taking the A-N-B route. How many cars would the police Dept. order to take the A-M-N-N route so that it is not possible for any car to reduce its travel time by not following the order while the other cars follow the order?

A. 1

B. 2

C. 3

D. 4

Q. 66 A new one-way road is built from M to N. Possible routes for each car from A to B: A-M-B, AN- B and A-M-N-B. On the road from M to N, one car takes 7 mins and each additional car increases the travel time per car by 1 minute. Assume that any car taking the A-M-N-B routes travels the A-M portion at the same time as the other cars taking the A-M-B route, and the N-B portion at the same time as other cats taking the A-N-B route. If all the cars follow the police order, what is minimum travel time from A to B?

A. 26

B. 32

C. 29.9

D. 30

Q. 67 Arun’s present age in years is 40% of Barun’s. In another few years, Arun’s age will be half of Barun’s. By what percentage will Barun’s age increase during this period? 

A. 20

B. 22

C. 18

D. 21

Q. 68 A person can complete a job in 120 days. He works alone on Day 1. On Day 2, he is joined by another person who also can complete the job in exactly 120 days. on Day 3, they are joined by another person of equal efficiency. Like this, everyday a new person with the same efficiency joins the work. How many days are required to complete the job? 

A. 20

B. 15

C. 18

D. 12

Q. 69 An elevator has a weight limit of 630 kg. It is carrying a group of people whom the heaviest weighs 57 kg and the lightest weighs 53 kg. What is the maximum possible number of people in the group?

A. 11

B. 12

C. 10

D. 13

Q. 70 A man leaves his home and walks at a speed of 12 kmph, reaching the railway station in 10 mins after the train had departed. if instead he had walked at a speed of 15 kmph, he would have reached the station in 10 minutes before the train’s departure. The distance (in km) from his home to the railway station is

A. 25

B. 15

C. 22

D. 20

Q. 71 Ravi invests 50% of his monthly savings in fixed deposits. 30% of the rest of his savings is invested in stocks and the rest into Ravi’s savings account. If the total amount deposited by him in the bank (for savings account and fixed deposits) is Rs 59500, then Ravi’s total monthly savings (in Rs) is

A. 56000

B. 63000

C. 60000

D. 70000

Q. 72 If a seller gives a discount of 15% on retail price, she still makes a profit of 2%. Which of the following ensures that she makes a profit of 20%?

A. Give a discount of 5% on retail price

B. Give a discount of 2% on retail price

C. Increase the retail price by 2%

D. Sell at retail price

Q. 73 A man travels by a motor boat down a river to his office and back. With the speed of the river unchanged, if he doubles the speed of his motor boat, then his total travel time gets reduced by 75%. The ratio of the original speed of the motor boat to the speed of the river is:

A. √6 : √2

B. √7 : 2

C. 2√5 : 3

D. 3 : 2

Q. 74 Suppose C1, C2, C3, C4 and C5 are five companies. The profits made by C1, C2 and C3 are in the ratio is 9 : 10 : 8 while the profits made by C2, C4 and C5 are in the ratio 18 : 19 : 20. If C5 has made a profit of Rs 19 crore more than C1, then the total profit (in Rs) made by all five companies is

A. 438 crore

B. 435 crore

C. 348 crore

D. 345 crore

Q. 75 The number of girls appearing for an admission test is twice the number of boys. If 30% of the girls and 45% of the boys get admission, the percentage of candidates who do not get admission is

A. 35

B. 50

C. 60

D. 65

Q. 76 A stall sells packets of popcorn and chips in three sizes: large, super and jumbo. The number of large, super and jumbo packets in its stock are in the ratio 7:17:16 and 6:15:14 for chips. If the total number of popcorn packets in its stock is the same as that of chips packets, then the number of jumbo popcorn packets and jumbo chips packets are in the ratio:

A. 1:1

B. 8:7

C. 4:3

D. 6:5

Q. 77 In a market, the price of medium quality mangoes is half that of good mangoes. A shopkeeper buys 80 kg good mangoes and 40 kg medium quality mangoes from the marker and then sells all these at a common price which is 10% less than the price at which he bought the good ones. His overall profit is

A. 6%

B. 8%

C. 10%

D. 12%

Q. 78 If Fatima sells 60 identical toys at a 40% discount on the printed price, then she makes 20% profit. Ten of these toys are destroyed in fire. While selling the rest, how much discount should be given on the printed price to that she can make the same amount of profit? 

A. 30%

B. 25%

C. 24%

D. 28%

Q. 79 If a and b are integers of opposite signs such that (a + 3)^2 : b^2 = 9 : 1 and (a – 1)^2 : (b – 1)^2 = 4 : 1, then the ratio a^2 : b^2 is

A. 9 : 4

B. 81 : 4

C. 1 : 4

D. 25 : 4

Q. 80 A class consists of 20 boys and 50 girls. In the mid-semester examination, the average score of the girls was 5 higher than that of the boys. In the final exam, however, the average score of the girls dropped by 3 while the average score of the entire class increased by 2. The increase in the average score of the boys is

A. 9.5

B. 10

C. 4.5

D. 6

Q. 81 The area of the closed region bounded by the equation | x | + | y | = 2 in the twodimensional plane is:

Note: Π represents pi

A. 4 Π

B. 4

C. 8

D. 2 Π

Q. 82 From a triangle ABC with sides of lengths 40 ft, 25 ft and 35 ft, a triangular portion GBC is cut off where G is the centroid of ABC. The area, in sq ft, of the remaining portion of triangle ABC is

A. 225√3

B. 500 / √3

C. 275 / √3

D. 250 / √3

Q. 83 Let ABC be a right-angled isosceles triangle with hypotenuse BC. Let BQC be a semi-circle, away from A, with diameter BC. Let BPC be an arc of a circle centered at A and lying between BC and BQC. If AB has length 6 cm then the area, in sq cm, of the region enclosed by BPC and BQC is

A. 9π – 18

B. 18

C. 9π

D. 9

Q. 84 A solid metallic cube is melted to form five solid cubes whose volumes are in the ratio 1 : 1 : 8 : 27 : 27. The percentage by which the sum of the surface areas of these five cubes exceeds the surface area of the original cube is nearest to

A. 10

B. 50

C. 60

D. 20

Q. 85 A ball of diameter 4 cm is kept on top of a hollow cylinder standing vertically. The height of the cylinder is 3 cm, while its volume is 9π cm^3. Then the vertical distance, in cm, of the topmost point of the ball from the base of the cylinder is

A. 6

B. 7

C. 8

D. 9

Q. 86 Let ABC be aright-angled triangle with BC as the hypotenuse. Lengths of AB and AC are 15 km and 20 km, respectively. The minimum possible time, in minutes, required to reach the hypotenuse from A at a speed of 30 kmph is

A. 20

B. 22

C. 24

D. 26

Q. 87 Suppose log3 (x) = log12 (y) = a, where x, y are positive numbers. If G is the geometric mean of x and y, and log6 (G) is equal to

A. √a

B. 2a

C. a / 2

D. a

Q. 88 If x + 1 = x^2 and x > 0, then 2 x^4 is

A. 6 + 4√5

B. 3 + 5√5

C. 5 + 3√5

D. 7 + 3√5

Q. 89 The value of log0.008 (√5) + log√3 ( 81) – 7 is equal to

A. 1/3

B. 2/3

C. 5/6

D. 7/6

Q. 90 If 9^(2x-1) – 81^(x-1) = 1944, then x is

A. 3

B. 9/4

C. 4/9

D. 1/3

Q. 91 The number of solutions (x, y, z) to the equation x – y – z = 25, where x, y and z are positive integers such that x <= 40, y <=12 and z <= 12 is

A. 101

B. 99

C. 87

D. 105

Q. 92 For how many integers n, will the inequality (n-5)(n-10) – 3(n-2) <=0 be satisfied?

A. 10

B. 11

C. 12

D. 13

Q. 93 If f1(x) = x^2 + 11x + n and f2(x) = x, then the largest positive integer n for which the equation f1(x) = f2(x) has two distinct real roots, is

A. 20

B. 21

C. 23

D. 24

Q. 94 If a, b, c and d are integers such that a + b + c + d = 30, then the minimum possible value of (a – b)^2 + (a – c)^2 + (a – d)^2 is

A. 1

B. 4

C. 12

D. 2

Q. 95 Let AB, CD, EF, GH and JK be five diameters of a circle with center at O. In how many ways can three points be chosen out of A, B, C, D, E, F, G, H, J, K and O so as to form a triangle? 

A. 120

B. 160

C. 180

D. 320

Q. 96 The shortest distance of the point (1/2 , 1) from the curve y = | x – 1 | + | x + 1 | is

A. 1

B. 0

C. √2

D. √(3/2)

Q. 97 If the square of the 7th term of an A.P. with positive common difference equals the product of the 3rd and 17th terms, then the ratio of the first term to the common difference is 

A. 2:3

B. 3:2

C. 3:4

D. 4:3

Q. 98 In how many ways can 7 identical erasers be distributed among 4 kids in such a way that each kid gets at least one eraser but nobody gets more than 3 erasers?

A. 16

B. 20

C. 14

D. 15

Q. 99 If f(x) = (5x + 2)/(3x – 5) and g(x) = x^2 – 2x – 1, then the value of g(f(f(3))) is?

A. 2

B. 1/3

C. 6

D. 2/3

Q. 100 Let a1, a2,… a3n be an arithmetic progression with a1 = 3 and a2 = 7. If a1 + a2 +…+ a3n = 1830, then what is the smallest positive integer m such that m(a1 + a2 + … + an ) > 1830?

A. 8

B. 9

C. 10

D. 11

Answer Sheet 
Question 1 2 3 4 5 6 7 8 9 10
Answer B B B C D A A D C C
Question 11 12 13 14 15 16 17 18 19 20
Answer B B C A C B D A C B
Question 21 22 23 24 25 26 27 28 29 30
Answer C C A D C C C A B C
Question 31 32 33 34 35 36 37 38 39 40
Answer A A D A B C A B A A
Question 41 42 43 44 45 46 47 48 49 50
Answer A A A B C A B A B B
Question 51 52 53 54 55 56 57 58 59 60
Answer B A B D C D C C C C
Question 61 62 63 64 65 66 67 68 69 70
Answer A D B B B B A B A D
Question 71 72 73 74 75 76 77 78 79 80
Answer D D B A D A B D D A
Question 81 82 83 84 85 86 87 88 89 90
Answer C B B B A C D D C B
Question 91 92 93 94 95 96 97 98 99 100
Answer B B D D B A A A A B
×

Hello!

Click one of our representatives below to chat on WhatsApp or send us an email to info@vidhyarthidarpan.com

×